Sunteți pe pagina 1din 294

CRIMINAL

LAW
REVIEW
BOOK 1
CASE
DIGEST

Submitted by:
GIANAN, SONNY II C.
2ND SEMESTER, SCHOOL YEAR
2014 2015
ATTY. NELSON SALVA
Professor

(FELONIES)
1

ROLLIE CALIMUTAN, Petitioner, vs. PEOPLE OF THE PHILIPPINES, ET AL.,


Respondents.
G.R. No. 152133 | February 9, 2006 (First Division)
CHICO-NAZARIO, J.
Facts: RTC charged petitioner Calimutan with the crime of homicide, alleging that on or
about February 4, 1996, in the morning thereof, at sitio Capsay, Barangay Panique,
Municipality of Aroroy, Province of Masbate, petitioner attack, assault and throw a stone
at Philip Cantre, hitting him at the back left portion of his body, resulting in laceration of
spleen due to impact which caused his death a day after.
The incident happened when the victim Cantre and witness Saano, together with two
other companions, had a drinking spree at a videoke bar. From the videoke bar, the
victim Cantre and witness Saano proceeded to go home to their respective houses,
but along the way, they crossed paths with petitioner Calimutan and a certain Michael
Bulalacao. Victim Cantre was harboring a grudge against Bulalacao, suspecting the
latter as the culprit responsible for throwing stones at the Cantres house on a previous
night. Thus, upon seeing Bulalacao, victim Cantre suddenly punched him. While
Bulalacao ran away, petitioner Calimutan dashed towards the backs of victim Cantre
and witness Saano. Petitioner Calimutan then picked up a stone, as big as a mans
fist, which he threw at victim Cantre, hitting him at the left side of his back. When hit by
the stone, victim Cantre stopped for a moment and held his back. Witness Saano put
himself between the victim Cantre and petitioner Calimutan, and attempted to pacify the
two, even convincing petitioner Calimutan to put down another stone he was already
holding. He also urged victim Cantre and petitioner Calimutan to just go home. Witness
Saano accompanied victim Cantre to the latters house, and on the way, victim Cantre
complained of the pain in the left side of his back hit by the stone. They arrived at the
Cantres house at around 12:00 noon, and witness Saano left victim Cantre to the care
of the latters mother, Belen.
RTC, in convicting petitioner of the crime charged, stated it its decision that it cannot be
legally contended that the throwing of the stone by the accused was in defense of his
companion, a stranger, because after the boxing Michael was able to run. While it
appears that the victim was the unlawful aggressor at the beginning, but the aggression
already ceased after Michael was able to run and there was no more need for throwing
a stone. The throwing of the stone to the victim which was a retaliatory act can be
considered unlawful, hence the accused can be held criminally liable under paragraph 1
of Art. 4 of the Revised Penal Code.
The act of throwing a stone from behind which hit the victim at his back on the left side
was a treacherous one and the accused committed a felony causing physical injuries to
the victim. The physical injury of hematoma as a result of the impact of the stone
resulted in the laceration of the spleen causing the death of the victim. The accused is
criminally liable for all the direct and natural consequences of this unlawful act even if
the ultimate result had not been intended.
2

Issue: Whether or not the conviction of Petitioner of the crime of Homicide is proper
Held: No Article 3 of the Revised Penal Code classifies felonies according to the means
by which they are committed, in particular: (1) intentional felonies, and (2) culpable
felonies. These two types of felonies are distinguished from each other by the existence
or absence of malicious intent of the offender
In intentional felonies, the act or omission of the offender is malicious. In the language
of Art. 3, the act is performed with deliberate intent (with malice). The offender, in
performing the act or in incurring the omission, has the intention to cause an injury to
another. In culpable felonies, the act or omission of the offender is not malicious. The
injury caused by the offender to another person is "unintentional, it being simply the
incident of another act performed without malice." (People vs. Sara, 55 Phil. 939). As
stated in Art. 3, the wrongful act results from imprudence, negligence, lack of foresight
or lack of skill.34
In the Petition at bar, this Court cannot, in good conscience, attribute to petitioner
Calimutan any malicious intent to injure, much less to kill, the victim Cantre; and in the
absence of such intent, this Court cannot sustain the conviction of petitioner Calimutan
for the intentional crime of homicide, as rendered by the RTC and affirmed by the Court
of Appeals. Instead, this Court finds petitioner Calimutan guilty beyond reasonable
doubt of the culpable felony of reckless imprudence resulting in homicide under Article
365 of the Revised Penal Code.
Article 365 of the Revised Penal Code expressly provides for the definition of reckless
imprudence
Reckless imprudence consists in voluntarily, but without malice, doing or failing to do an
act from which material damage results by reason of inexcusable lack of precaution on
the part of the person performing or failing to perform such act, taking into consideration
his employment or occupation, degree of intelligence, physical condition and other
circumstances regarding persons, time and place.
There are several circumstances, discussed in the succeeding paragraphs, that
demonstrate petitioner Calimutans lack of intent to kill the victim Cantre, and
conversely, that substantiate the view of this Court that the death of victim Cantre was a
result of petitioner Calimutans reckless imprudence. The RTC and the Court of Appeals
may have failed to appreciate, or had completely overlooked, the significance of such
circumstances.
It should be remembered that the meeting of the victim Cantre and witness Saano, on
the one hand, and petitioner Calimutan and his helper Bulalacao, on the other, was a
chance encounter as the two parties were on their way to different destinations. The
victim Cantre and witness Saano were on their way home from a drinking spree in
Crossing Capsay, while petitioner Calimutan and his helper Bulalacao were walking
3

from the market to Crossing Capsay. While the evidence on record suggests that a
running grudge existed between the victim Cantre and Bulalacao, it did not establish
that there was likewise an existing animosity between the victim Cantre and petitioner
Calimutan.
Granting that petitioner Calimutan was impelled by a lawful objective when he threw the
stone at the victim Cantre, his act was committed with inexcusable lack of precaution.
He failed to consider that a stone the size of a mans fist could inflict substantial injury
on someone. He also miscalculated his own strength, perhaps unaware, or even
completely disbelieving, that he could throw a stone with such force as to seriously
injure, or worse, kill someone, at a quite lengthy distance of ten meters.
Since it is irrefragable that the stone thrown by petitioner Calimutan at the victim Cantre
was the proximate cause of the latters death, despite being done with reckless
imprudence rather than with malicious intent, petitioner Calimutan remains civilly liable
for such death.
EDUARDO P. MANUEL, Petitioner, vs. PEOPLE OF THE PHILIPPINES,
Respondent.
G.R. No. 165842 | November 29, 2005 (Second Division)
CALLEJO, SR., J.
Facts: Petitioner was charged with bigamy on the ground that while he was legally
married to Rubylus Gaa and without the said marriage having been legally dissolved,
he contracted a second marriage with Tina Gandalera, herein complainant, who does
not know the existence of the first marriage.
The trial court ruled that the prosecution was able to prove beyond reasonable doubt all
the elements of bigamy under Article 349 of the Revised Penal Code. It declared that
Eduardos belief, that his first marriage had been dissolved because of his first wifes
20-year absence, even if true, did not exculpate him from liability for bigamy. Citing the
ruling of this Court in People v. Bitdu, the trial court further ruled that even if the private
complainant had known that Eduardo had been previously married, the latter would still
be criminally liable for bigamy.
Eduardo appealed the decision to the CA. He alleged that he was not criminally liable
for bigamy because when he married the private complainant, he did so in good faith
and without any malicious intent. He maintained that at the time that he married the
private complainant, he was of the honest belief that his first marriage no longer
subsisted. He insisted that conformably to Article 3 of the Revised Penal Code, there
must be malice for one to be criminally liable for a felony. He was not motivated by
malice in marrying the private complainant because he did so only out of his
overwhelming desire to have a fruitful marriage. He posited that the trial court should
have taken into account Article 390 of the New Civil Code.

Issue: Whether or not petitioner should be acquitted of the crime of bigamy on the
ground that presumptive death of Gaa had arisen by operation of law, as the two
requirements of Article 390 of the Civil Code are present
Held: No. As gleaned from the Information in the RTC, the petitioner is charged with
bigamy, a felony by dolo (deceit). Article 3, paragraph 2 of the Revised Penal Code
provides that there is deceit when the act is performed with deliberate intent. Indeed, a
felony cannot exist without intent. Since a felony by dolo is classified as an intentional
felony, it is deemed voluntary. Although the words "with malice" do not appear in Article
3 of the Revised Penal Code, such phrase is included in the word "voluntary."
Malice is a mental state or condition prompting the doing of an overt act without legal
excuse or justification from which another suffers injury. When the act or omission
defined by law as a felony is proved to have been done or committed by the accused,
the law presumes it to have been intentional. Indeed, it is a legal presumption of law
that every man intends the natural or probable consequence of his voluntary act in the
absence of proof to the contrary, and such presumption must prevail unless a
reasonable doubt exists from a consideration of the whole evidence.
For one to be criminally liable for a felony by dolo, there must be a confluence of both
an evil act and an evil intent. Actus non facit reum, nisi mens sit rea.
In the present case, the prosecution proved that the petitioner was married to Gaa in
1975, and such marriage was not judicially declared a nullity; hence, the marriage is
presumed to subsist. The prosecution also proved that the petitioner married the private
complainant in 1996, long after the effectivity of the Family Code.
The petitioner is presumed to have acted with malice or evil intent when he married the
private complainant. As a general rule, mistake of fact or good faith of the accused is a
valid defense in a prosecution for a felony by dolo; such defense negates malice or
criminal intent. However, ignorance of the law is not an excuse because everyone is
presumed to know the law. Ignorantia legis neminem excusat.
It was the burden of the petitioner to prove his defense that when he married the private
complainant in 1996, he was of the well-grounded belief that his first wife was already
dead, as he had not heard from her for more than 20 years since 1975. He should have
adduced in evidence a decision of a competent court declaring the presumptive death of
his first wife as required by Article 349 of the Revised Penal Code, in relation to Article
41 of the Family Code. Such judicial declaration also constitutes proof that the petitioner
acted in good faith, and would negate criminal intent on his part when he married the
private complainant and, as a consequence, he could not be held guilty of bigamy in
such case. The petitioner, however, failed to discharge his burden.
(MISTAKE OF FACT)
THE UNITED STATES, plaintiff-appellee, vs. AH CHONG, defendant-appellant.
G.R. No. L-5272 | March 19, 1910 (En Banc)
5

CARSON, J.
Facts: The accused, Ah Chong, was employed as a cook in Fort Mckinley and was
sharing the house with the deceased, Pascual Gualberto, who was employed as a
house boy. The door of the room they were occupying was not furnished with a
permanent lock, and as a measure of security, they fasten the door by propping a chair
against it. One evening, Ah Chong was suddenly awakened by someone trying to force
open the door of their room. The deceased and the accused had an understanding that
when either returned late at night, he should knock at the door and acquaint his
companion with his identity. Ah Chong sat up in bed and called out twice, Who is
there? but heard no answer. The room was quite dark, and as there had been recent
robberies in Fort McKinley, fearing that the intruder was a robber or a thief, he leaped to
his feet and called out. If you enter the room, I will kill you. Suddenly, he was struck by
the edge of the chair which had been placed against the door. Believing that he was
being attacked, he seized a common kitchen knife which he kept under his pillow and
wildly struck and fatally wounded the intruder who turned out to be his roommate,
Pascual.
Issue: Whether or not appellant was criminally liable
Held: No. To this question we think there can be but one answer, and we hold that
under such circumstances there is no criminal liability, provided always that the alleged
ignorance or mistake or fact was not due to negligence or bad faith.
In broader terms, ignorance or mistake of fact, if such ignorance or mistake of fact is
sufficient to negative a particular intent which under the law is a necessary ingredient of
the offense charged (e.g., in larcerny, animus furendi; in murder, malice; in crimes
intent) "cancels the presumption of intent," and works an acquittal; except in those
cases where the circumstances demand a conviction under the penal provisions
touching criminal negligence; and in cases where, under the provisions of article 1 of
the Penal Code one voluntarily committing a crime or misdeamor incurs criminal liability
for any wrongful act committed by him, even though it be different from that which he
intended to commit.
In the present case, the accused acted in good faith, without malice or criminal intent, in
the belief that he was doing no more than exercising his legitimate right of self-defense.
Had the facts been as he believed them to be, he would have been wholly exempt from
criminal liability on account of his act. Moreover, the accused cannot be said to have
been negligent or reckless as the facts as he saw them threatens his person and his
property. Under such circumstances, there is no criminal liability, as the ignorance or
mistake of fact was not due to negligence or bad faith.
THE PEOPLE OF THE PHILIPPINE ISLANDS, plaintiff-appellee, vs. FERNANDO DE
FERNANDO, defendant-appellant.
G.R. No. 24978 | March 27, 1926 (En Banc)
VILLA-REAL, J.
6

Facts: This appeal has been taken by the defendant Fernando de Fernando from the
judgment of the trial court, in which he was held guilty of the crime of murder committed
against Buenventura Paulino.
At the trial the following facts were proven beyond a reasonable doubt: The accused
Fernando de Fernando who, at that time, was a municipal policeman, when passing in
front of the house of one Remigio Delgado, was called by the latter's daughter
Paciencia Delgado, who stated that her father wished to see him. When the policeman
came up the house Remigio Delgado informed him that three unknown and suspicious
looking persons, dressed in blue, prowling around his house. The accused remained in
the said house talking with Paciencia Delgado, both being seated on a bench near the
window. While they were thus talking, at about 7 o'clock at night, there appeared in the
dark, at about 4 meters from the stairs, a person dressed in dark clothes, calling "Nong
Miong." At the time the accused nor Paciencia Delgado knew who was thus calling. The
accused inquired what he wanted but instead of answering he continued advancing with
bolo in hand. Upon seeing this Fernando de Fernando took out his revolver and fired a
shot in the air. As he saw that the unknown continued to ascend the staircase he fired at
him. The unknown disappeared and ran to the house of a neighbor Leon Torres, where,
after placing upon a table the bolos that he carried, he fell on the floor and expired.
Remigio Delgado, who was in the kitchen and had recognized the voice of the unknown,
on hearing the shots ran into the parlor, took hold of the arm of the defendant and asked
him why he had fired at Buenventura Paulino. Fernando de Fernando only said "Let me
go, that is a cross eyed person" and immediately repaired to the house of the teniente of
the barrio, Santiago Torres, from where he telephoned to the chief of police advising him
of what had happened. When the body was examined it was found that a bullet had
penetrated the base of the neck at the right, imbedding itself in the left side under the
skin.
Issue: Whether or not appellant is guilty of murder
Held: No. Taking into consideration the estate of mind of the accused at the time, and
the meaning that he gave to the attitude of the unknown person, in shooting the latter he
felt that he was performing his duty by defending the owners of the house against an
unexpected attack, and such act cannot constitute the crime of murder, but only that of
simple homicide. He cannot be held guilty, however, as principal with malicious intent,
because he though at the time that he was justified in acting as he did, and he is guilty
only because he failed to exercise the ordinary diligence which, under the
circumstances, he should have by investigating whether or not the unknown man was
really what he thought him to be. In firing the shot, without first exercising reasonable
diligence, he acted with reckless negligence.
The crime committed by the caused, therefore, is homicide through reckless negligence
defined and punished in article 568, in relation with article 404, of the Penal Code, the
penalty prescribed by law arresto mayor in its maximum degree to prision correcional in
its minimum degree.

In view of the foregoing and reversing the appealed judgment, the accused is held guilty
of the crime of homicide through reckless negligence, and he is sentenced to suffer one
year prision correcional, to pay the amount of P500 to the heirs of the deceased as an
indemnity, with subsidiary imprisonment in case of insolvency, the costs and with credit
of one-half of the preventive imprisonment already suffered. So ordered.
EDUARDO P. DIEGO, complainant, vs. JUDGE SILVERIO Q. CASTILLO,
REGIONAL TRIAL COURT, DAGUPAN CITY, BRANCH 43, respondent.
A.M. No. RTJ-02-1673 | August 11, 2004 (First Division)
AZCUNA, J.
Facts: This is an administrative complaint respondent judge for allegedly knowingly
rendering an unjust judgment in a criminal case and/or rendering judgment in gross
ignorance of the law.
Lucena Escoto contracted marriage with Jorge de Perio, Jr. In the marriage contract,
Lucena used and adopted the name Crescencia Escoto, with a civil status of single.
Subsequently, the same Crescencia Escoto contracted marriage with herein
complainants brother, Manuel P. Diego. The marriage contract shows that this time, the
accused used and adopted the name Lucena Escoto, again, with a civil status of single.
After trial of the criminal case for bigamy, respondent Judge promulgated a decision
acquitting Lucena. The decision states that the main basis for the acquittal was good
faith on the part of the accused. Respondent Judge gave credence to the defense of the
accused that she acted without any malicious intent. In his comment, respondent Judge
stated: "That the accused married Manuel P. Diego in the honest belief that she was
free to do so by virtue of the decree of divorce is a mistake of fact."
Issue: Whether or not respondent judge erred in ruling that the marriage of Lucena to
Manuel in the honest belief that she was free to do so by virtue of the decree of divorce
is a mistake of fact
Held: Yes. In his comment, respondent Judge stated: "That the accused married
Manuel P. Diego in the honest belief that she was free to do so by virtue of the decree of
divorce is a mistake of fact."
This Court, in People v. Bitdu, carefully distinguished between a mistake of fact, which
could be a basis for the defense of good faith in a bigamy case, from a mistake of law,
which does not excuse a person, even a lay person, from liability. Bitdu held that even if
the accused, who had obtained a divorce under the Mohammedan custom, honestly
believed that in contracting her second marriage she was not committing any violation
of the law, and that she had no criminal intent, the same does not justify her act. This
Court further stated therein that with respect to the contention that the accused acted in
good faith in contracting the second marriage, believing that she had been validly
divorced from her first husband, it is sufficient to say that everyone is presumed to know
8

the law, and the fact that one does not know that his act constitutes a violation of the
law does not exempt him from the consequences thereof.
Moreover, squarely applicable to the criminal case for bigamy is People v.
Schneckenburger, where it was held that the accused who secured a foreign divorce,
and later remarried in the Philippines, in the belief that the foreign divorce was valid, is
liable for bigamy.
(MALA INSE VS MALA PROHIBITA)
JOSEPH EJERCITO ESTRADA, petitioner, vs. SANDIGANBAYAN (Third Division)
and PEOPLE OF THE PHILIPPINES, respondents.
G.R. No. 148560 | November 19, 2001 (En Banc)
BELLOSILLO, J.
Facts: Section 2 of R.A. No. 7080 (An Act Defining and Penalizing the Crime of
Plunder) as amended by R.A. No. 7659 substantially provides that any public officer
who amasses, accumulates or acquires ill-gotten wealth through a combination or
series of overt or criminal acts in the aggregate amount or total value of at least fifty
million pesos (P50,000,000.00) shall be guilty of the crime of plunder. Petitioner Joseph
Ejercito Estrada, being prosecuted under the said Act, assailed its constitutionality,
arguing inter alia, that it abolishes the element of mens rea in crimes already punishable
under The Revised Penal Code; and as such, a violation of the fundamental rights of
the accused to due process and to be informed of the nature and cause of the
accusation against him.
Issue: Whether or not the crime of plunder as defined in R.A. No. 7080 is a malum
prohibitum
Held: No. Plunder is a malum in se which requires proof of criminal intent. As stated by
Justice Mendoza in his Concurring Opinion, because the constitutive crimes are mala in
se the element of mens rea must be proven in a prosecution for plunder. It is noteworthy
that the amended information alleges that the crime of plunder was committed "willfully,
unlawfully and criminally." It thus alleges guilty knowledge on the part of petitioner.
The legislative declaration in R.A. No. 7659 that plunder is a heinous offense implies
that it is a malum in se. For when the acts punished are inherently immoral or inherently
wrong, they are mala in se and it does not matter that such acts are punished in a
special law, especially since in the case of plunder the predicate crimes are mainly mala
in se. Indeed, it would be absurd to treat prosecutions for plunder as though they are
mere prosecutions for violations of the Bouncing Check Law (B.P. Blg. 22) or of an
ordinance against jaywalking, without regard to the inherent wrongness of the acts.
To clinch, petitioner likewise assails the validity of RA 7659, the amendatory law of RA
7080, on constitutional grounds. Suffice it to say however that it is now too late in the
9

day for him to resurrect this long dead issue, the same having been eternally consigned
by People v. Echegaray to the archives of jurisprudential history. The declaration of this
Court therein that RA 7659 is constitutionally valid stands as a declaration of the State,
and becomes, by necessary effect, assimilated in the Constitution now as an integral
part of it.
PEOPLE OF THE PHILIPPINES, plaintiff-appellee, vs. GO SHIU LING, MARY ONG,
TERESITA BAJAR y OBEDOZA, and TEODORO EVANGELISTA y DE LOS REYES,
accused. ANTONIO COMIA y QUINERY, accused-appellant.
G.R. No. 115156 | December 14, 1995 (Second Division)
MENDOZA, J
Facts: Accused-appellant Antonio Comia guilty of conspiring with four others to import
regulated drugs in violation of Art. III, 14 in relation to Art. IV, 21 of the Dangerous
Drugs Act (Rep. Act No. 6425, as amended).
In finding Comia guilty, the trial court held that he acted on his own in conspiracy with
unknown partners, by using Mary Ong's coded initials "VGMO" and thereby making it
appear they were part of Ong's shipment, since only the three parcels all addressed to
Comia and marked "VGMO" contained shabu. The trial court held that the fact that
Comia was persistent in his follow up showed that he knew the contents of the three
parcels.
The Solicitor General filed in lieu of an appellee's brief a Manifestation that, in his
opinion, the guilt of the accused has not been proven beyond reasonable doubt and, for
this reason, recommending the acquittal of accused-appellant Comia.
Issue: Whether or not appellant can be held criminally liable
Held: Yes. Accused-appellant and the Solicitor General contend that, instead of
showing that Comia knew that the packages contained shabu, the fact that Comia was
persistent in seeking the release of the packages shows that he did not know what the
parcels contained.
They contend that Comia knew that one parcel had been discovered to contain watches
in commercial quantity, not toys or gifts as stated in the customs declaration; that the
next step would be the seizure of the cargo; and that in following up the matter at the
ADC, his only purpose was to try to prevent the seizure of the goods. They contend that
if Comia knew that the packages contained shabu, he would have stopped going to the
ADC and instead would have gone into hiding.
This contention has no merit. It is clear from the evidence that Evangelista and his firm
had connections with the people inside the ADC and the Sta. Mesa Post Office,
because of which Evangelista and his firm were able to slip through customs
commercial quantities of highly dutiable goods. Accused-appellant himself states in his
brief that the fact that Emmanuel Laudit of the ADC allegedly warned Lydia Dizon that
the shipment was going to be discovered "indicates [the] existence of an alliance with
Laudit." Without such an "alliance," merely using dummy addressees with fictitious or
10

inaccurate addresses on the parcels handled by the TASCO would not be enough to
make the smuggling of goods and contraband possible.
Even granting that Comia acted in good faith, he cannot escape criminal responsibility.
The crime with which he is charged is a malum prohibitum. Lack of criminal intent and
good faith are not exempting circumstances. As held in People v. Lo Ho Wing:
Moreover, the act of transporting a prohibited drug is a "malum prohibitum" because it is
punished as an offense under a special law. It is a wrong because it is prohibited by law.
Without the law punishing the act, it cannot be considered a wrong. As such, the mere
commission of said act is what constitutes the offense punished and suffices to validly
charge and convict an individual caught committing the act so punished, regardless of
criminal intent.
Likewise, in People v. Bayona, it was held:
The rule is that in acts mala in se there must be a criminal intent, but in those mala
prohibita it is sufficient if the prohibited act was intentionally done. "Care must be
exercised in distinguishing the difference between the intent to commit the crime and
the intent to perpetrate to act."
Indeed, Comia cannot claim to have acted in good faith. Even assuming that he did not
know that the packages contained shabu, the fact is that he tried to facilitate the
importation of dutiable goods free of customs duties. It comes as a surprise therefore
that in moving for the acquittal of accused-appellant the Solicitor General should
downplay or minimize the role of accused-appellant by insisting that the latter was a
"mere employee" who did not have a choice "but to assent to whatever his employer
wanted to do," and that "besides, this practice of TASCO of using different addressed
for a client has been admitted by no less than its manager, Lydia Dizon herself."

THE PEOPLE OF THE PHILIPPINE ISLANDS, plaintiff-appellee, vs. CORNELIO


BAYONA, defendant-appellant.
G.R. No. L-42288 | February 16, 1935 (En Banc)
VICKERS, J.
Facts: This is an appeal from a decision the Court of First Instance of Capiz, finding the
defendant guilty of a violation of section 416 of the Election Law and sentencing him to
suffer imprisonment for thirty days and to pay a fine of P50, with subsidiary
imprisonment in case of insolvency, and to pay the costs.
11

Issue: Whether or not appellants intent is sufficient to convict him of the crime charged
Held: No. We cannot accept the reasons advanced by the Solicitor-General for the
acquittal of the defendant. The law which the defendant violated is a statutory provision,
and the intent with which he violated it is immaterial. It may be conceded that the
defendant did not intend to intimidate any elector or to violate the law in any other way,
but when he got out of his automobile and carried his revolver inside of the fence
surrounding the polling place, he committed the act complained of, and he committed it
willfully. The act prohibited by the Election Law was complete. The intention to intimidate
the voters or to interfere otherwise with the election is not made an essential element of
the offense. Unless such an offender actually makes use of his revolver, it would be
extremely difficult, if not impossible, to prove that he intended to intimidate the voters.
The rule is that in acts mala in se there must be a criminal intent, but in those mala
prohibita it is sufficient if the prohibited act was intentionally done. "Care must be
exercised in distinguishing the difference between the intent to commit the crime and
the intent to perpetrate the act. ..."
While it is true that, as a rule and on principles of abstract justice, men are not and
should not be held criminally responsible for acts committed by them without guilty
knowledge and criminal or at least evil intent, the courts have always recognized the
power of the legislature, on grounds of public policy and compelled by necessity, "the
great master of things", to forbid in a limited class of cases the doing of certain acts, and
to make their commission criminal without regard to the intent of the doer. In such cases
no judicial authority has the power to require, in the enforcement of the law, such
knowledge or motive to be shown.
If we were to adopt the specious reasoning that the appellant should be acquitted
because it was not proved that he tried to influence or intended to influence the mind of
any voter, anybody could sell intoxicating liquor or hold a cockfight or a horse race on
election day with impunity.

(CRIMINAL LIABILITY ART.4)


THE UNITED STATES, plaintiff-appellee, vs. MAXIMO MALLARI, defendantappellant.
G.R. No. L-10037 | December 23, 1914 (En Banc)
TORRES, J.
Facts: Defendant Maximo Mallari went to the house of the married couple, Vicente
Sunga and Canuta Flores, and from the shed outside asked Vicente Sunga to cure his
wife of a sickness from which she had been suffering for several days, and which he
thought was due to enchantment on the part of the said Vicente. As the latter refused,
12

averring that he was not a wizard and that he had not caused the illness of defendant's
wife, the former became enraged and insulted the said spouses. Threatening to kill
them, he ascended the stairway carrying in his hand a thin, sharp bolo. At his wife's
suggestion Vicente Sunga tried to get out to report the matter to the teniente of the
barrio, who lived at some paces from their house, but as he met the defendant on the
stairway, he immediately went back inside and jumped out of a window. He was
straightway pursued by the defendant and on arriving almost in front of the house of the
teniente saw that the defendant was following closely behind him. He therefore turned
to face his pursuer and defend himself as well as he could with his hands. Thereupon
the defendant with a single slash of the bolo wounded Vicente Sunga in the abdomen,
so that his intestines protruded therefrom. In this condition the victim sat down,
endeavoring with his hands to keep his intestines from falling out, while his assailant
took to flight.
Issue: Whether or not defendant-appellant is guilty of homicide
Held: Yes. The affirmations of witnesses cannot prevail against the testimony of two
impartial neighbors who came up at the cries of the deceased's wife calling for help and
who, along with the brother-in-law of the wounded man, witnessed the occurence and
rendered him aid, without having seen those two witnesses of the defense on the spot
or the alleged prior aggression of the deceased against the defendant.
Moreover, when the defendant was asked by the chief of police how he came to be
wounded in the rump, he replied that he did not know who had inflicted that wound, and
this statement of the chief was not impugned at the trial, so it may be that the defendant
in preparing his defense wounded himself with pocket-knife, for the wound was of a
trifling nature; and at the investigation held by the justice of the peace who went to the
defendant's house, the latter made no statement regarding the person who had inflicted
said wound, while the physician who examined him said that it must have been received
when he was standing still.
It does not therefore appear to be duly proven in the case that the defendant was
attacked and wounded with a pocket- knife by the deceased, and consequently that
there was any prior unlawful aggression on the part of the deceased, to justify the
finding that the defendant was compelled to wound him in the belly with a bolo in lawful
self-defense. The plea or circumstance of exemption from responsibility must be fully
proven in the same way as the principal fact, in order to hold that the perpetrator of the
crime is not responsible therefor.1awphil.net
With reference to the classification of the criminal act, it does not appear in the case that
this was erroneous, for in spite of the statement of the health officer that the deceased
might have been saved if the wound had been aseptically treated from the first, its
seriousness and fatal character being due to lack of antiseptics, still the person inflicting
it is responsible for all the consequences of his criminal action, and therefore for the
death that occured some days after the deceased received the wound.
13

SALUD VILLANUEVA VDA. DE BATACLAN and the minors NORMA, LUZVIMINDA,


ELENITA, OSCAR and ALFREDO BATACLAN, represented by their Natural
guardian, SALUD VILLANUEVA VDA. DE BATACLAN, plaintiffs-appellants, vs.
MARIANO MEDINA, defendant-appellant.
G.R. No. L-10126 | October 22, 1957 (En Banc)
MONTEMAYOR, J.
Facts: The deceased Juan Bataclan was among the passengers of Medina
Transportation, driven by Conrado Saylon and operated by Mariano Medina. On its way
from Cavite to Pasay, the front tires burst and the vehicle fell into a canal. Some
passengers were able to escape by themselves or with some help, while there were 4,
including Bataclan, who could not get out. Their cries were heard in the neighbourhood.
Then there came about 10 men, one of them carrying a torch. As they approached the
bus, it caught fire and the passengers died. The fire was due to gasoline leak and the
torch. Salud Villanueva Vda. de Bataclan, in her name and on behalf of her 5 minor
children, sought to claim damages from the bus company. The CFI favored the plaintiff,
and the Court of Appeals forwarded the case to the Supreme Court due to the amount
involved.
Issue: Whether or not the proximate cause of the death of Bataclan et al was their
burning by reason of the torches which ignited the gasoline
Held: No. The proximate cause was the overturning of the bus which was caused by the
negligence of the driver because he was speeding and also he was already advised by
Medina to change the tires yet he did not. Such negligence resulted to the overturning
of the bus. The torches carried by the would-be helpers are not to be blamed. It is just
but natural for the villagers to respond to the call for help from the passengers and since
it is a rural area which did not have flashlights, torches are the natural source of lighting.
Further, the smell of gas could have been all over the place yet the driver and the
conductor failed to provide warning about said fact to the villagers.
Proximate cause is that cause, which, in natural and continuous sequence, unbroken by
any efficient intervening cause, produces the injury, and without which the result would
not have occurred.
And more comprehensively, the proximate legal cause is that acting first and producing
the injury, either immediately or by setting other events in motion, all constituting a
natural and continuous chain of events, each having a close causal connection with its
immediate predecessor, the final event in the chain immediately effecting the injury as a
natural and probable result of the cause which first acted, under such circumstances
that the person responsible for the first event should, as an ordinary prudent and
intelligent person, have reasonable ground to expect at the moment of his act or default
that an injury to some person might probably result therefrom.
MELBA QUINTO, Petitioners, vs. DANTE ANDRES and RANDYVER PACHECO,
Respondents.
14

G.R. No. 155791 | March 16, 2005 (Second Division)


CALLEJO, SR., J.
Facts: Dante Andres and Randyven Pacheco invited Wilson Quinto and Edison Garcia
to go fishing with them inside a drainage culvert. However, only Quinto joined the two,
Garcia remains in a grassy area about two meters from the entrance of the drainage
system. After a while, Pacheco came out, went back again, and emerged again carrying
Wilson who was already dead. He laid the boys lifeless body down in the grassy area
and went to the house of Wilsons mother and informed her that her son had died. After
more than three months, the cadaver of Wilson was exhumed and the NBI performed
an autopsy thereon. An information was later filed with the RTC changing Andres and
Pacheco with homicide.
Issue: Whether or not the accused has criminal liability for the death of the victim
Held: Yes. A person committing a felony is criminally liable for all the natural and logical
consequences resulting therefrom although the wrongful act done be different from that
which he intended. "Natural" refers to an occurrence in the ordinary course of human
life or events, while "logical" means that there is a rational connection between the act
of the accused and the resulting injury or damage. The felony committed must be the
proximate cause of the resulting injury. Proximate cause is that cause which in natural
and continuous sequence, unbroken by an efficient intervening cause, produces the
injury, and without which the result would not have occurred. The proximate legal cause
is that acting first and producing the injury, either immediately, or by setting other events
in motion, all constituting a natural and continuous chain of events, each having a close
causal connection with its immediate predecessor.
There must be a relation of "cause and effect," the cause being the felonious act of the
offender, the effect being the resultant injuries and/or death of the victim. The "cause
and effect" relationship is not altered or changed because of the pre-existing conditions,
such as the pathological condition of the victim (las condiciones patologica del
lesionado); the predisposition of the offended party (la predisposicion del ofendido); the
physical condition of the offended party (la constitucion fisica del herido); or the
concomitant or concurrent conditions, such as the negligence or fault of the doctors (la
falta de medicos para sister al herido); or the conditions supervening the felonious act
such as tetanus, pulmonary infection or gangrene.
The felony committed is not the proximate cause of the resulting injury when:
(a) there is an active force that intervened between the felony committed and the
resulting injury, and the active force is a distinct act or fact absolutely foreign from the
felonious act of the accused; or
(b) the resulting injury is due to the intentional act of the victim.

15

PEOPLE OF THE PHILIPPINES, Appellee, vs. EDMAR AGUILOS, ODILON


LAGLIBA Y ABREGON and RENE GAYOT PILOLA, accused, RENE GAYOT
PILOLA, Appellant.
G.R. No. 121828 | June 27, 2003 (Second Division)
CALLEJO, SR., J.
Facts: Joselito Capa and Julian Azul were drinking in a store in Mandaluyong, while in
the midst of the drinking spree, Edmar Aguilos and Odilon Lagliba arrived and they were
invited to join the drinking. During the course of the drinking, Edmar had an argument
with the Julian that led to a commotion of the store. Edmar punched Julian in the face,
the latter and the former exchanged punches. Joselito intervened to stop the fight. But
the Odilon grabbed the Joselito the neck and stabbed the latter with a knife, Ronnie and
accused arrived at the scene and they saw their gang mate, Odilon, stabbing the victim,
they immediately pulled out their knives and subsequently stabbed the victim, Ronnie
bashed a hollow block and a bottle to the head of the victim, the three accused fled from
the scene of the crime, leaving the victim dead on the spot.
The appellant argues that the prosecution failed to prove that he conspired with Ronnie
and Odilon in stabbing the victim to death. He contends that for one to be a conspirator,
his participation in the criminal resolution of another must either precede or be
concurrent with the criminal acts. He asserts that even if it were true that he was
present at the situs criminis and that he stabbed the victim, it was Odilon who had
already decided, and in fact fatally stabbed the victim. He could not have conspired with
Odilon as the incident was only a chance encounter between the victim, the appellant
and his co-accused. In the absence of a conspiracy, the appellant cannot be held liable
as a principal by direct participation. Elisa could not categorically and positively assert
as to what part of the victims body was hit by whom, and how many times the victim
was stabbed by the appellant. He asserts that he is merely an accomplice and not a
principal by direct participation.
Issue: Whether or not appellant can be held guilty of in conspiracy with Ronnie and
Odilon in stabbing the victim to death
Held: Yes. There is conspiracy when two or more persons agree to commit a felony and
decide to commit it. Conspiracy as a mode of incurring criminal liability must be proved
separately from and with the same quantum of proof as the crime itself. Conspiracy
need not be proven by direct evidence. After all, secrecy and concealment are essential
features of a successful conspiracy. It may be inferred from the conduct of the accused
before, during and after the commission of the crime, showing that they had acted with
a common purpose and design. Conspiracy may be implied if it is proved that two or
more persons aimed by their acts towards the accomplishment of the same unlawful
object, each doing a part so that their combined acts, though apparently independent of
each other, were, in fact, connected and cooperative, indicating a closeness of personal
association and a concurrence of sentiment. There may be conspiracy even if an
offender does not know the identities of the other offenders, and even though he is not
aware of all the details of the plan of operation or was not in on the scheme from the
16

beginning. One need only to knowingly contribute his efforts in furtherance of it. One
who joins a criminal conspiracy in effect adopts as his own the criminal designs of his
co-conspirators. If conspiracy is established, all the conspirators are liable as coprincipals regardless of the manner and extent of their participation since in
contemplation of law, the act of one would be the act of all. Each of the conspirators is
the agent of all the others.
To hold an accused guilty as a co-principal by reason of conspiracy, he must be shown
to have performed an overt act in pursuance or furtherance of the conspiracy. The mere
presence of an accused at the situs of the crime will not suffice; mere knowledge,
acquiescence or approval of the act without cooperation or agreement to cooperate on
the part of the accused is not enough to make him a party to a conspiracy. There must
be intentional participation in the transaction with a view to the furtherance of the
common design and purpose. Conspiracy to exist does not require an agreement for an
appreciable period prior to the occurrence. From the legal standpoint, conspiracy exists
if, at the time of the commission of the offense, the accused had the same purpose and
were united in its execution. As a rule, the concurrence of wills, which is the essence of
conspiracy, may be deduced from the evidence of facts and circumstances, which taken
together, indicate that the parties cooperated and labored to the same end.
Even if two or more offenders do not conspire to commit homicide or murder, they may
be held criminally liable as principals by direct participation if they perform overt acts
which mediately or immediately cause or accelerate the death of the victim, applying
Article 4, paragraph 1 of the Revised Penal Code.

(IMPOSSIBLE CRIME)
THE PEOPLE OF THE PHILIPPINES, plaintiff-appellee, vs. RAFAEL BALMORES Y
CAYA, defendant-appellant.
G.R. No. L-1896 | February 16, 1950 (En Banc)
OZAETA, J.
Facts: Appellant, waiving the right to be assisted by counsel, pleaded guilty information
filed against, accusing him of attempted estafa through falsification of a security. From
that sentence he appealed to this court, contending (1) that the facts and (2) that the
trial court lacked jurisdiction to convict him on a plea of guilty because, being illiterate,
he was not assisted by counsel.
Issue: Whether or not the fact that appellant was illiterate deprives the trial court of
jurisdiction assisted by counsel
Held: No. It may be that appellant was either reckless or foolish in believing that a
falsification as patent as that which he admitted to have perpetrated would succeed; but
17

the recklessness and clumsiness of the falsification did not make the crime impossible
within the purview of paragraph 2, article 4, in relation to article 59, of the Revised Penal
Code. Examples of an impossible crime, which formerly was not punishable but is now
under article 59 of the Revised Penal Code, is the following: (1) When one tries to kill
another by putting in his soup a substance which he believes to be arsenic when in fact
it is common salt; and (2) when one tries to murder a corpse. (Guevara, Commentaries
on the Revised Penal Code, 4th ed., page 15; decision, Supreme Court of Spain,
November 26, 1879; 12 Jur. Crim., 343.) Judging from the appearance of the falsified
ticket in question, we are not prepared to say that it would have been impossible for the
appellant to consummate the crime of estafa thru falsification of said ticket if the clerk to
whom it was presented for the payment had not exercised due care.
SULPICIO INTOD, petitioner, vs. HONORABLE COURT OF APPEALS and PEOPLE
OF THE PHILIPPINES, respondents.
G.R. No. 103119 | October 21, 1992 (Second Division)
CAMPOS, JR., J.
Facts: In the morning of February 4, 1979, Intod, Pangasian, Tubio and Daligdig went to
Mandaya's house in Lopez Jaena, Misamis Occidental and asked him to go with them
to the house of Palangpangan. Thereafter, Mandaya and Intod, Pangasian, Tubio and
Daligdig had a meeting with Aniceto Dumalagan. He told Mandaya that he wanted
Palangpangan to be killed because of a land dispute between them and that Mandaya
should accompany the four men, otherwise, he would also be killed.
At about 10:00 o'clock in the evening of the same day, Petitioner, Mandaya, Pangasian,
Tubio and Daligdig, all armed with firearms, arrived at Palangpangan's house in
Katugasan, Lopez Jaena, Misamis Occidental. At the instance of his companions,
Mandaya pointed the location of Palangpangan's bedroom. Thereafter, Petitioner,
Pangasian, Tubio and Daligdig fired at said room. It turned out, however, that
Palangpangan was in another City and her home was then occupied by her son-in-law
and his family. No one was in the room when the accused fired the shots. No one was
hit by the gun fire.
Issue: Whether or not the act committed by petitioner constitutes an impossible crime
Held: Yes. The factual situation in the case at bar present a physical impossibility which
rendered the intended crime impossible of accomplishment and under Article 4,
paragraph 2 of the Revised Penal Code, such is sufficient to make the act an impossible
crime.
To be impossible under this clause, the act intended by the offender must be by its
nature one impossible of accomplishment. There must be either impossibility of
accomplishing the intended act in order to qualify the act an impossible crime. Legal
impossibility occurs where the intended acts, even if completed, would not amount to a
crime. Factual impossibility occurs when extraneous circumstances unknown to the
18

actor or beyond his control prevent the consummation of the intended crime. The case
at bar belongs to this category.
In our jurisdiction, impossible crimes are recognized. The impossibility of accomplishing
the criminal intent is not merely a defense, but an act penalized by itself. Furthermore,
the phrase "inherent impossibility" that is found in Article 4(2) of the Revised Penal
Code makes no distinction between factual or physical impossibility and legal
impossibility.
To uphold the contention of respondent that the offense was Attempted Murder because
the absence of Palangpangan was a supervening cause independent of the actor's will,
will render useless the provision in Article 4, which makes a person criminally liable for
an act "which would be an offense against persons or property, were it not for the
inherent impossibility of its accomplishment . . ." In that case all circumstances which
prevented the consummation of the offense will be treated as an accident independent
of the actor's will which is an element of attempted and frustrated felonies.
GEMMA T. JACINTO, Petitioner, vs. PEOPLE OF THE PHILIPPINES, Respondent.
G.R. No. 162540 | July 13, 2009 (Third Division)
PERALTA, J.
Facts: Petitioner was a collector for a company called Mega Foam Intl Inc. (Mega
Foam) and received a PhP10,000 check as payment from a Mega Foam customer.
However, instead of turning over the check to Mega Foam, the accused took the check
and had it deposited into her brother-in-laws bank account. It turns out that the check
was not funded. Both the regional trial court and the Court of Appeals ruled that the
accused was guilty of qualified theft.
Issue: Whether or not the crime of qualified theft was actually produced
Held: No. Intod v. Court of Appeals is highly instructive and applicable to the present
case. In Intod, the accused, intending to kill a person, peppered the latters bedroom
with bullets, but since the intended victim was not home at the time, no harm came to
him. The trial court and the CA held Intod guilty of attempted murder. But upon review
by this Court, he was adjudged guilty only of an impossible crime as defined and
penalized in paragraph 2, Article 4, in relation to Article 59, both of the Revised Penal
Code.
The requisites of an impossible crime are: (1) that the act performed would be an
offense against persons or property; (2) that the act was done with evil intent; and (3)
that its accomplishment was inherently impossible, or the means employed was either
inadequate or ineffectual. The aspect of the inherent impossibility of accomplishing the
intended crime under Article 4(2) of the Revised Penal Code was further explained by
the Court in Intod in this wise:

19

Under this article, the act performed by the offender cannot produce an offense against
persons or property because: (1) the commission of the offense is inherently impossible
of accomplishment; or (2) the means employed is either (a) inadequate or (b)
ineffectual.
That the offense cannot be produced because the commission of the offense is
inherently impossible of accomplishment is the focus of this petition. To be impossible
under this clause, the act intended by the offender must be by its nature one impossible
of accomplishment. There must be either (1) legal impossibility, or (2) physical
impossibility of accomplishing the intended act in order to qualify the act as an
impossible crime.
Legal impossibility occurs where the intended acts, even if completed, would not
amount to a crime.
Herein petitioner's case is closely akin to the above example of factual impossibility
given in Intod. In this case, petitioner performed all the acts to consummate the crime of
qualified theft, which is a crime against property. Petitioner's evil intent cannot be
denied, as the mere act of unlawfully taking the check meant for Mega Foam showed
her intent to gain or be unjustly enriched. Were it not for the fact that the check
bounced, she would have received the face value thereof, which was not rightfully hers.
Therefore, it was only due to the extraneous circumstance of the check being unfunded,
a fact unknown to petitioner at the time, that prevented the crime from being produced.
The thing unlawfully taken by petitioner turned out to be absolutely worthless, because
the check was eventually dishonored, and Mega Foam had received the cash to replace
the value of said dishonored check.
From the above discussion, there can be no question that as of the time that petitioner
took possession of the check meant for Mega Foam, she had performed all the acts to
consummate the crime of theft, had it not been impossible of accomplishment in this
case. The circumstance of petitioner receiving the P5,000.00 cash as supposed
replacement for the dishonored check was no longer necessary for the consummation
of the crime of qualified theft. Obviously, the plan to convince Baby Aquino to give cash
as replacement for the check was hatched only after the check had been dishonored by
the drawee bank. Since the crime of theft is not a continuing offense, petitioner's act of
receiving the cash replacement should not be considered as a continuation of the theft.
At most, the fact that petitioner was caught receiving the marked money was merely
corroborating evidence to strengthen proof of her intent to gain.
(STAGES OF EXECUTION)
ARISTOTEL VALENZUELA y NATIVIDAD, petitioner, vs. PEOPLE OF THE
PHILIPPINES and HON. COURT OF APPEALS NACHURA, respondents.
G. R. No. 160188 | June 21, 2007 (En Banc)
TINGA, J.
20

Facts: While a security guard was manning his post at the open parking area of a
supermarket, he saw the accused, Aristotel Valenzuela, hauling a push cart loaded with
cases of detergent and unloaded them where his co-accused, Jovy Calderon, was
waiting. Valenzuela then returned inside the supermarket, and later emerged with more
cartons of detergent. Thereafter, Valenzuela hailed a taxi and started loading the
cartons inside. As the taxi was about to leave, the security guard asked Valenzuela for
the receipt of the merchandize. The accused reacted by fleeing on foot, but were
subsequently apprehended at the scene. The trial court convicted both Valenzuela and
Calderon of the crime of consummated theft. Valenzuela appealed before the Court of
Appeals, arguing that he should only be convicted of frustrated theft since he was not
able to freely dispose of the articles stolen. The Court of Appeals affirmed the trial
courts decision, thus the Petition for Review was filed before the Supreme Court
Issue: Whether or not the crime of theft has a frustrated stage
Held: No. Article 6 of the Revised Penal Code provides that a felony is consummated
when all the elements necessary for its execution and accomplishment are present. In
the crime of theft, the following elements should be present: (1) that there be taking of
personal property; (2) that said property belongs to another; (3) that the taking be done
with intent to gain; (4) that the taking be done without the consent of the owner; and (5)
that the taking be accomplished without the use of violence against or intimidation of
persons or force upon things. The Court held that theft is produced when there is
deprivation of personal property by one with intent to gain. Thus, it is immaterial that the
offender is able or unable to freely dispose the property stolen since he has already
committed all the acts of execution and the deprivation from the owner has already
ensued from such acts. Therefore, theft cannot have a frustrated stage, and can only be
attempted or consummated.
RUJJERIC Z. PALAGANAS, petitioner, vs. PEOPLE OF THE PHILIPPINES,
respondent.
G.R. No. 165483 | September 12, 2006 (First Division)
CHICO-NAZARIO, J.
Facts: Brothers Servillano, Melton and Michael Ferrer were having their drinking spree
at their house but later decided to proceed to Tidbits Videoke Bar to continue their
drinking spree and to sing. Thereafter, Jaime Palaganas arrived together with Ferdinand
Palaganas (nephew) and Virgilio Bautista. When Jaime Palaganas
was singing,
Melton Ferrer sang with him. Jaime Palaganas got irritated and insulted. He felt that he
was being mocked by Melton Ferrer that caused him to go to the Ferrers table and
uttered statements which began the fight. Ferdinand sought help to Rujjeric Palaganas.
They went to the Bar and upon seeing the Ferrers outside; Ferdinand pointing at the
Ferrers instructed Rujjeric to shoot them. Rujjeric Palaganas shot Servillano, Melton
and Michael with the use of unlicensed firearm. As a result, Melton was killed, Servillano
was fatally wounded and Michael was shot in his right shoulder.

21

Issue: Whether or not guilty Petitioner is guilty of the crime of Frustrated Homicide as
regards to Michael
Held: No. Based on Article 6 of the Revised Penal Code, the distinctions between
frustrated and attempted felony are summarized as follows:
1.) In frustrated felony, the offender has performed all the acts of execution which
should produce the felony as a consequence; whereas in attempted felony, the offender
merely commences the commission of a felony directly by overt acts and does not
perform all the acts of execution.
2.) In frustrated felony, the reason for the non-accomplishment of the crime is some
cause independent of the will of the perpetrator; on the other hand, in attempted felony,
the reason for the non-fulfillment of the crime is a cause or accident other than the
offender's own spontaneous desistance.
In addition to these distinctions, we have ruled in several cases that when the accused
intended to kill his victim, as manifested by his use of a deadly weapon in his assault,
and his victim sustained fatal or mortal wound/s but did not die because of timely
medical assistance, the crime committed is frustrated murder or frustrated homicide
depending on whether or not any of the qualifying circumstances under Article 249 of
the Revised Penal Code are present. However, if the wound/s sustained by the victim in
such a case were not fatal or mortal, then the crime committed is only attempted murder
or attempted homicide.56 If there was no intent to kill on the part of the accused and the
wound/s sustained by the victim were not fatal, the crime committed may be serious,
less serious or slight physical injury.
Based on the medical certificate of Michael, as well as the testimony of the physician
who diagnosed and treated Michael, the latter was admitted and treated at the Dagupan
Doctors-Villaflor Memorial Hospital for a single gunshot wound in his right shoulder
caused by the shooting of petitioner. It was also stated in his medical certificate that he
was discharged on the same day he was admitted and that the treatment duration for
such wound would be for six to eight days only. Given these set of undisputed facts, it is
clear that the gunshot wound sustained by Michael in his right shoulder was not fatal or
mortal since the treatment period for his wound was short and he was discharged from
the hospital on the same day he was admitted therein. Therefore, petitioner is liable only
for the crime of attempted homicide as regards Michael in Criminal Case No. U-9609.
ESMERALDO RIVERA, ISMAEL RIVERA, EDGARDO RIVERA, Petitioners, vs.
PEOPLE OF THE PHILIPPINES, Respondent.
G.R. No. 166326 | January 25, 2006 (Third Division)
CALLEJO, SR., J.
Facts: When the victim, Ruben Rodil, went to a nearby store to buy food, accused
Edgardo Rivera mocked him for being jobless and dependent on his wife for support.
Ruben resented the rebuke and thereafter, a heated exchange of words ensued. In the
22

evening of the following day, when Ruben and his three-year-old daughter went to the
store to buy food, Edgardo, together with his brother Esmeraldo Rivera and Ismael
Rivera, emerged from their house and ganged up on him. Esmeraldo and Ismael
mauled Ruben with fist blows. And as he fell to the ground, Edgardo hit him three times
with a hollow block on the parietal area. Esmeraldo, Ismael and Edgardo fled to their
house only when the policemen arrived. Ruben sustained injuries and was brought to
the hospital. The doctor declared that the wounds were slight and superficial, though the
victim could have been killed had the police not promptly intervened. The trial court
found the accused guilty of the crime of frustrated murder. An appeal was made by the
accused, but the Court of Appeals affirmed the trial courts decision with modification,
changing the crime to attempted murder and imposed an indeterminate penalty of 2
years of prision correccional as minimum to 6 years and 1 day of prision mayor as
maximum.
Issue: Whether or not the Court of Appeals was correct in modifying the crime from
frustrated to attempted murder
Held: Yes. An essential element of murder and homicide, whether in their
consummated, frustrated or attempted stage, is intent of the offenders to kill the victim
immediately before or simultaneously with the infliction of injuries. Intent to kill is a
specific intent which the prosecution must prove by direct or circumstantial evidence,
while general criminal intent is presumed from the commission of a felony by dolo.
In the present case, the prosecution mustered the requisite quantum of evidence to
prove the intent of petitioners to kill Ruben. Esmeraldo and Ismael pummeled the victim
with fist blows. Even as Ruben fell to the ground, unable to defend himself against the
sudden and sustained assault of petitioners, Edgardo hit him three times with a hollow
block. Edgardo tried to hit Ruben on the head, missed, but still managed to hit the victim
only in the parietal area, resulting in a lacerated wound and cerebral contusions.
That the head wounds sustained by the victim were merely superficial and could not
have produced his death does not negate petitioners criminal liability for attempted
murder. Even if Edgardo did not hit the victim squarely on the head, petitioners are still
criminally liable for attempted murder.
The last paragraph of Article 6 of the Revised Penal Code defines an attempt to commit
a felony, thus:
There is an attempt when the offender commences the commission of a felony directly
by overt acts, and does not perform all the acts of execution which should produce the
felony by reason of some cause or accident other than his own spontaneous
desistance.
The essential elements of an attempted felony are as follows:
1. The offender commences the commission of the felony directly by overt acts;
23

2. He does not perform all the acts of execution which should produce the felony;
3. The offenders act be not stopped by his own spontaneous desistance;
4. The non-performance of all acts of execution was due to cause or accident other than
his spontaneous desistance.
The first requisite of an attempted felony consists of two elements, namely:
(1) That there be external acts;
(2) Such external acts have direct connection with the crime intended to be committed.
Petitioners, who acted in concert, commenced the felony of murder by mauling the
victim and hitting him three times with a hollow block; they narrowly missed hitting the
middle portion of his head. If Edgardo had done so, Ruben would surely have died.

PEOPLE OF THE PHILIPPINES, plaintiff-appellee, vs. HENRY ALMAZAN, accusedappellant.


G.R. Nos. 138943-44 | September 17, 2001 (Third Division)
BELLOSILLO, J.
Facts: The trial court declaed accused-appellant Henry Almazan guilty of murder and
frustrated murder committed against Noli S. Madriaga.
Apart from being positively identified by the prosecution witnesses as the person
responsible for the violence and the injuries inflicted, the trial court declared that the
theft of Henry's fighting cocks constituted sufficient motive for the killing and that as a
cockfight afficionado he must have found it imperative to exact vengeance on his
suspected culprits. The trial court held that the testimony of Johnald failed to create
reasonable doubt on the guilt of Henry since as a friend he was expected to extend
succor to a friend, especially one in need. Thus, the trial court held Henry Almazan
guilty of murder and frustrated murder as charged.
Issue: Whether or not the trial court erred in holding him guilty of frustrated murder as
the wound sustained by Noel Madriaga was not fatal that could have caused his death if
not for timely medical assistance
Held: Yes. For the charge of frustrated murder to flourish, the victim should sustain a
fatal wound that could have caused his death were it not for timely medical assistance.
This is not the case before us.
24

The court a quo anchored its ruling on the statement of Dr. Ticman on crossexamination that the wound of Noel could catch infection or lead to his death if not
timely and properly treated. However, in his direct testimony, Dr. Ticman declared that
the wound was a mere minor injury for which Noel, after undergoing treatment, was
immediately advised to go home. He even referred to the wound as a slight physical
injury that would heal within a week16 and for which the victim was in no danger of
dying. Clear as the statement is, coupled with the fact that Noel was indeed immediately
advised to go home as he was not in any danger of death, we have no reason to doubt
the meaning and implications of Dr. Ticman's statement. His statement that Noel could
catch infection was based on pure speculation rather than on the actual nature of the
wound which was a mere minor injury, hence, not fatal. According to jurisprudence, if
the victim was wounded with an injury that was not fatal, and could not cause his death,
the crime would only be attempted. The observation that the conviction should be for
slight physical injuries only is likewise improper as the accused-appellant was motivated
by the same impetus and intent, i.e., to exact vengeance and even kill, if necessary,
when he shot Noel Madriaga. The fact that the wound was merely a minor injury which
could heal in a week becomes inconsequential.
PEOPLE OF THE PHILIPPINES, plaintiff, vs. PRIMO CAMPUHAN Y BELLO
accused.
G.R. No. 129433 | March 30, 2000 (En Banc)
BELLOSILLO, J.
Facts: Ma. Corazon Pamintuan was downstairs busy preparing drinks for her two
daughters, she heard Crysthel, one of her daughters crying, Ayoko, Ayoko prompting
her to rush upstairs. Thereupon, she saw Primo inside her childrens room kneeling in
front of her four-year old daughter, whose pajamas were already removed, while his
short pants were down to his knees. Primo was apprehended and was charged with
statutory rape.
The trial court found him guilty and sentenced him to the extreme penalty of death. In
convicting the accused, the trial court relied quite heavily on the testimony of Corazon
that she saw Primo with his short pants down to his knees kneeling before Crysthel
whose pajamas and panty were supposedly already removed" and that Primo was
forcing his penis into Crysthels vagina.
Issue: Whether or not accused is guilty of consummated rape
Held: No. In People vs. Dela Pena, the SC clarified that the decisions finding a case for
rape even if the attackers penis merely touched the external portions of the female
genitalia were made in the context of the presence or existence of an erect penis
capable of full penetration. Where the accused failed to achieve an erection, had a limp
of flaccid penis, or an oversized penis which could not fit into the victims vagina, the
Court nonetheless held that rape was consummated on the basis of the victims
25

testimony that the accused repeatedly tried, but in vain, to insert his penis into her
vagina and in all likelihood reached the labia of her pudendum as the victim felt his
organ on the lips of her vulva, or that the penis of the accused touched the middle part
of her vagina.
Thus, touching when applied to rape cases does not simply mean mere epidermal
contact, stroking or grazing of organs, a slight brush or a scrape of the penis on the
external layer of the victim's vagina, or the mons pubis, as in this case.
There must be sufficient and convincing proof that the penis indeed touched the labias
or slid into the female organ, and not merely stroked the external surface thereof, for an
accused to be convicted of consummated rape. As the labias, which are required to be
touched by the penis, are by their natural situs or location beneath the mons pubis or
the vaginal surface, to touch them with the penis is to attain some degree of penetration
beneath the surface, hence, the conclusion that touching the labia majora or the labia
minora of the pudendum constitutes consummated rape.
Judicial depiction of consummated rape has not been confined to the oft-quoted
touching of the female organ, but has also progressed into being described as the
introduction of the male organ into the labia of the pudendum, or the bombardment of
the drawbridge. But to the SC's mind, the case at bar merely constitutes a "shelling of
the castle of orgasmic potency," or a "strafing of the citadel of passion."
Under Art. 6, in relation to Art. 335, of the Revised Penal Code, rape is attempted when
the offender commences the commission of rape directly by overt acts, and does not
perform all the acts of execution which should produce the crime of rape by reason of
some cause or accident other than his own spontaneous desistance. All the elements of
attempted rape and only of attempted rape are present in the instant case; hence,
the accused should be punished only for it.
(CONSPIRACY)
PEOPLE OF THE PHILIPPINES, Appellee, vs. ALBERTO ANTICAMARA y CABILLO
and FERNANDO CALAGUAS FERNANDEZ a.k.a. LANDO CALAGUAS
G.R. No. 178771 | June 8, 2011 (Second Division)
PERALTA, J.
Facts: Trial court found herein appellants guilty beyond reasonable doubt of the crime
of Murder committed against Sulpacio Abad and of the crime of Kidnapping and Serious
Illegal Detention of Sulpacio Abad and AAA. Appellant Al attempts to evade criminal
liability by alleging that he was only forced to participate in the commission of the crime
because he and his family were threatened to be killed.
Issue: Whether or not the trial court erred in holding that conspiracy existed between
and among the alleged perpetrators of the crime
26

Held: No. Under Article 8 of the Revised Penal Code, there is conspiracy when two or
more persons come to an agreement concerning a felony and decide to commit it. It
may be inferred from the acts of the accused before, during or after the commission of
the crime which, when taken together, would be enough to reveal a community of
criminal design, as the proof of conspiracy is frequently made by evidence of a chain of
circumstances. To be a conspirator, one need not participate in every detail of the
execution; he need not even take part in every act or need not even know the exact part
to be performed by the others in the execution of the conspiracy. Each conspirator may
be assigned separate and different tasks which may appear unrelated to one another
but, in fact, constitute a whole collective effort to achieve their common criminal
objective. Once conspiracy is shown, the act of one is the act of all the conspirators.
The precise extent or modality of participation of each of them becomes secondary,
since all the conspirators are principals.
In the present case, prior to the commission of the crime, the group met at the landing
field in Carmen, Pangasinan and discussed their plan to rob the house of the Estrellas
with the agreement that whoever comes their way will be eliminated. Appellant Al served
as a lookout by posting himself across the house of the Estrellas with the task of
reporting any movements outside. Fred then climbed the old unserviceable gate of the
Estrella compound and then opened the small door and the rest of the group entered
the house of the Estrellas through that opening. After almost an hour inside the house,
they left on board a vehicle with AAA and Sulpacio. AAA and Sulpacio were brought to
Sitio Rosalia, Brgy. San Bartolome, Rosales, Pangasinan. In that place, Sulpacio was
killed and AAA was brought to another place and deprived of her liberty. These
circumstances establish a community of criminal design between the malefactors in
committing the crime. Clearly, the group conspired to rob the house of the Estrellas and
kill any person who comes their way. The killing of Sulpacio was part of their conspiracy.
Further, Dick's act of arming himself with a gun constitutes direct evidence of a
deliberate plan to kill should the need arise.
There is nothing in the records to substantiate appellant Al's insistence that he was
under duress from his co-accused while participating in the crime that would suffice to
exempt him from incurring criminal liability. The evidence shows that Al was tasked to
act as a lookout and directed to station himself across the house of the Estrellas. Al was
there from 7:30 p.m. to 1:00 a.m. of the following day, while the rest of the group was
waiting in the landing field. Thus, while all alone, Al had every opportunity to escape
since he was no longer subjected to a real, imminent or reasonable fear. However, he
opted to stay across the house of the Estrellas for almost six (6) hours, and thereafter
returned to the landing field where the group was waiting for his report. Subsequently,
the group proceeded to the Estrellas house. When the group entered the house, Al
stayed for almost one (1) hour outside to wait for his companions. Later, when the group
left the house aboard a vehicle, Al rode with them in going to Sitio Rosalia, Brgy. San
Bartolome, Rosales, Pangasinan, bringing with them Sulpacio and AAA. Clearly,
appellant Al had ample opportunity to escape if he wished to, but he never did. Neither
did he request for assistance from the authorities or any person passing by the house of
27

the Estrellas during the period he was stationed there. Clearly, Al did not make any
effort to perform an overt act to dissociate or detach himself from the conspiracy to
commit the felony and prevent the commission thereof that would exempt himself from
criminal liability. Therefore, it is obvious that he willingly agreed to be a part of the
conspiracy.

PEOPLE OF THE PHILIPPINES, Plaintiff-Appellee, vs. KHADDAFY JANJALANI,


GAMAL B. BAHARAN a.k.a. Tapay, ANGELO TRINIDAD a.k.a. Abu Khalil, GAPPAL
BANNAH ASALI a.k.a. Maidan or Negro, JAINAL SALI a.k.a. Abu Solaiman,
ROHMAT ABDURROHIM a.k.a. Jackie or Zaky, and other JOHN and JANE DOES,
Accused, GAMAL B. BAHARAN a.k.a. Tapay, ANGELO TRINIDAD a.k.a. Abu Khalil,
and ROHMAT ABDURROHIM a.k.a. Abu Jackie or Zaky, Accused-Appellants.
G.R. No. 188314 | January 10, 2011 (Third Division)
SERENO, J.
Facts: On February 14 2005, a bus was going from Navotas to Alabang. Two men got
on the bus. Both seemed suspicious according to Elmer Andales, the conductor. The
two men alighted in Ayala Ave. and the bus exploded. After the explosion, the
spokesperson for Abu Sayyaff announced over radio that the explosion was a
valentines gift. Accused Asali, member of Abu Sayaff, gave a television interview,
confessing that he had supplied the explosive devices for the said bombing. The bus
conductor identified the accused Baharan and Trinidad, and confirmed that they were
the two men who had entered the RRCG bus on the evening of the said day. Asali
testified that he had given accused Baharan and Trinidad the TNT used in the bombing
incident in Makati City.
Issue: Whether or not participation of accused the existence of conspiracy involving
accused Baharan, Trinidad, and Rohmat was proven
Held: Yes. Conspiracy was clearly established from the "collective acts of the accusedappellants before, during and after the commission of the crime." As correctly declared
by the trial court in its Omnibus Decision:
Asalis clear and categorical testimony, which remains unrebutted on its major points,
coupled with the judicial admissions freely and voluntarily given by the two other
accused, are sufficient to prove the existence of a conspiracy hatched between and
among the four accused, all members of the terrorist group Abu Sayyaf, to wreak chaos
and mayhem in the metropolis by indiscriminately killing and injuring civilian victims by
utilizing bombs and other similar destructive explosive devices.
28

While said conspiracy involving the four malefactors has not been expressly admitted by
accused Baharan, Angelo Trinidad, and Rohmat, more specifically with respect to the
latters participation in the commission of the crimes, nonetheless it has been
established by virtue of the aforementioned evidence, which established the existence
of the conspiracy itself and the indispensable participation of accused Rohmat in seeing
to it that the conspirators criminal design would be realized.
It is well-established that conspiracy may be inferred from the acts of the accused,
which clearly manifests a concurrence of wills, a common intent or design to commit a
crime (People v. Lenantud, 352 SCRA 544). Hence, where acts of the accused
collectively and individually demonstrate the existence of a common design towards the
accomplishment of the same unlawful purpose, conspiracy is evident and all the
perpetrators will be held liable as principals (People v. Ellado, 353 SCRA 643).
In People v. Geronimo, the Court pronounced that it would be justified in concluding that
the defendants therein were engaged in a conspiracy "when the defendants by their
acts aimed at the same object, one performing one part and the other performing
another part so as to complete it, with a view to the attainment of the same object; and
their acts, though apparently independent, were in fact concerted and cooperative,
indicating closeness of personal association, concerted action and concurrence of
sentiments."
Accused contend that the testimony of Asali is inadmissible pursuant to Sec. 30, Rule
130 of the Rules of Court. It is true that under the rule, statements made by a
conspirator against a co-conspirator are admissible only when made during the
existence of the conspiracy. However, as the Court ruled in People v. Buntag, if the
declarant repeats the statement in court, his extrajudicial confession becomes a judicial
admission, making the testimony admissible as to both conspirators.
PEOPLE OF THE PHILIPPINES, Plaintiff-Appellee, vs. DIMA MONTANIR, RONALD
NORVA AND EDUARDO CHUA, Accused-Appellants.
G.R. No. 187534 | April 4, 2011 (Second Division)
PERALTA, J.
Facts: Josie Herrera, Robert Uy, Alicia "a.k.a. Alice" Buenaflor, together with appellants
Ronald Norva and Eduardo Chua, concocted a plan to kidnap Rafael Mendoza, and
after several days of conducting surveillance on their intended victim, they decided to
kidnap Rafael in Ali Mall, Cubao, Quezon City. However, the intended kidnapping failed,
because Rafael did not show up at the said place. A second attempt was made, but they
encountered an accident before they could even execute their original plan.
The trial court rendered judgment against the appellants for the crime of kidnapping. In
convicting the appellants, the trial court, based on the evidence presented, naturally
found the existence of conspiracy among the perpetrators.
29

Issue: Whether or not the trial court erred in finding the existence of conspiracy among
appellants to the commission of kidnapping
Held: No. Conspiracy exists when two or more persons come to an agreement
concerning the commission of a felony and decide to commit it. Verily, when conspiracy
is established, the responsibility of the conspirators is collective, not individual, that
render all of them equally liable regardless of the extent of their respective
participations, the act of one being deemed to be the act of the other or the others, in
the commission of the felony. Each conspirator is responsible for everything done by his
confederates which follows incidentally in the execution of a common design as one of
its probable and natural consequences even though it was not intended as part of the
original design. Responsibility of a conspirator is not confined to the accomplishment of
a particular purpose of conspiracy but extends to collateral acts and offenses incident to
and growing out of the purpose intended. Conspirators are held to have intended the
consequences of their acts and by purposely engaging in conspiracy which necessarily
and directly produces a prohibited result, they are, in contemplation of law, chargeable
with intending that result. Conspirators are necessarily liable for the acts of another
conspirator unless such act differs radically and substantively from that which they
intended to commit. As Judge Learned Hand put it in United States v. Andolscheck,
"when a conspirator embarks upon a criminal venture of indefinite outline, he takes his
chances as to its content and membership, so be it that they fall within the common
purposes as he understands them."
A scrutiny of the records show that the trial court did not err in finding conspiracy among
the appellants, as they each played a role in the commission of the crime. The trial court
correctly found the denial of appellant Dima that he had knowledge of the kidnapping,
unbelievable. The appellants bare denial is a weak defense that becomes even weaker
in the face of the prosecution witnesses positive identification of him. Jurisprudence
gives greater weight to the positive narration of prosecution witnesses than to the
negative testimonies of the defense.
PEOPLE OF THE PHILIPPINES, Appellee, vs. JUANITO P. QUIROL and MARIO P.
QUIROL, Appellants.
G.R. No. 149259 | October 20, 2005 (First Division)
AZCUNA, J.
Facts: In celebration of a fiesta in Apas, Lahug, Cebu City, a benefit disco dance was
held at the local UCMA Village. Appellants, Juanito and Mario Quirol, and the two
victims, Benjamin Silva and Roel Ngujo, attended. At the dance, Juanito, Mario and Jed
were together and drank all through the night with some friends. The dance ended just
prior to 4 a.m. and prosecution principal witness Wilson Cruz testified that it was about
that time when he was asked by Benjamin and Roel to accompany them in escorting
some ladies home.

30

Wilson, hidden behind a bush, said he could hear Benjamin plead for his life. A few
seconds later, Jed took out his .38 caliber service revolver and shot Benjamin at pointblank range on the head. As Benjamin fell, Roel was dragged down to his knees since
he was handcuffed to Benjamin. Mario then held Roel while Juanito started stabbing
him using a Batangas knife. Jed finished it by shooting Roel.
The RTC, finding that there is conspiracy among appellants, convicted Juanito Quirol
and Mario Quirol for double murder.
Issue: Whether or not the trial court erred in finding conspiracy among appellants
Held: No. Conspiracy need not be proven by direct evidence of prior agreement to
commit the crime. Neither is its necessary to show that all the conspirators actually hit
and killed the victim. What has to be shown is that all the participants performed specific
acts with such closeness and coordination as to unmistakably indicate a common
purpose and design. The conspiracy in the instant case was sufficiently proven by Jed
meeting with appellants at the old airport tower and walking together with them towards
the runway where appellants and Jed performed acts in unison with each other as to
unmistakably reveal a common purpose and design.
Anent Marios defense of alibi, despite corroboration from Exequiel Aranas, it is still an
inherently weak defense and cannot prevail over a positive identification from a witness
found credible by the trial court. Absent arbitrariness or oversight of some fact or
circumstance of significance and influence, we will not interfere with the credence given
to the testimony of Wilson over that of Mario and that of Exequiel, as assessments of
credibility are generally left to the trial court whose proximate contact with those who
take the witness stand places it in a more competent position to discriminate between
true and false testimony. Moreover, as correctly discussed by the Court of Appeals, the
distance between the scene of the crimes and where Mario claims he passed out is not
so far away as to prevent him from being physically present at the place of the crimes or
its immediate vicinity at the time the crimes were committed.
PEOPLE OF THE PHILIPPINES, appellee, vs. JOSE BULAN and ALLAN BULAN,
appellants.
G.R. No. 143404 | June 08, 2005 (Second Division)
CALLEJO, SR., J.
Facts: Jose Bulan and his sons, Allan and Estemson, were charged with murder
committed against Alberto Mariano in an Information. After trial, the trial court rendered
judgment convicting the accused of murder as accomplices. On appeal to the CA, the
appellate court rendered judgment affirming the decision of the RTC, with the
modification that the appellants and Estemson were guilty of murder as principals by
indispensable cooperation. The appellate court declared that as gleaned from the
evidence on record, the three of them conspired to kill the deceased.
Issue: Whether or not the appellants are guilty of the crime charged as principals by
direct participation as ruled by the CA
31

Held: Yes. Article 8 of the Revised Penal Code provides that there is conspiracy when
two or more persons agree to commit a crime and decide to commit it. Direct proof is
not essential to prove conspiracy; it may be established by acts of the accused before,
during and after the commission of the crime charged, from which it may be logically
inferred the existence of a common purpose to commit the same. The prosecution must
prove conspiracy by the same quantum of evidence as the felony charged itself. Indeed,
proof of previous agreement among the malefactors to commit the crime is not essential
to prove conspiracy. It is not necessary to show that all the conspirators actually hit and
killed the victim; what is primordial is that all the participants performed specific acts
with such closeness and coordination as to indicate a common purpose or design to
bring out the victims death. Once conspiracy is established, it is unnecessary to prove
who among the conspirators inflicted the fatal injury. If conspiracy is proved, all the
conspirators are criminally liable for the crime charged and proved. The act of one is the
act of all.
In this case, the appellants were waiting outside the dance hall near the gate when
Edwin Solo brought the victim towards them, onto the street. Jose held the victim by the
right shoulder, while Allan held him by the left. Estemson suddenly appeared from
behind the victim and stabbed the latter at the back with a small bolo. The appellants
continued holding the victim as Estemson stabbed him yet again. Even as Estemson
fled, the appellants dragged the victim from the gate, towards the store, where they
dropped the victims body and fled from the scene. Allan then left Catanduanes and hid
in Pasay City where he was arrested by the NBI on August 7, 1994.
PEOPLE OF THE PHILIPPINES, appellee, vs. ANTONIO COMADRE, GEORGE
COMADRE and DANILO LOZANO, appellants
G.R. No. 153559 | June 8, 2004 (En Banc)
Per Curiam
Facts: Appellants Antonio Comadre, George Comadre and Danilo Lozano were
charged with Murder with Multiple Frustrated Murder committed against Robert
Agbanlog in an information.
The undisputed facts show that when Antonio Comadre was in the act of throwing the
hand grenade, George Comadre and Danilo Lozano merely looked on without uttering a
single word of encouragement or performed any act to assist him. The trial court held
that the mere presence of George Comadre and Danilo Lozano provided
encouragement and a sense of security to Antonio Comadre, thus proving the existence
of conspiracy, as ruled by the trial court.
Issue: Whether or not the trial courts finding of conspiracy is correct
Held: No. Similar to the physical act constituting the crime itself, the elements of
conspiracy must be proven beyond reasonable doubt. Settled is the rule that to
32

establish conspiracy, evidence of actual cooperation rather than mere cognizance or


approval of an illegal act is required.
A conspiracy must be established by positive and conclusive evidence. It must be
shown to exist as clearly and convincingly as the commission of the crime itself. Mere
presence of a person at the scene of the crime does not make him a conspirator for
conspiracy transcends companionship.
The evidence shows that George Comadre and Danilo Lozano did not have any
participation in the commission of the crime and must therefore be set free. Their mere
presence at the scene of the crime as well as their close relationship with Antonio are
insufficient to establish conspiracy considering that they performed no positive act in
furtherance of the crime.
Neither was it proven that their act of running away with Antonio was an act of giving
moral assistance to his criminal act. The ratiocination of the trial court that "their
presence provided encouragement and sense of security to Antonio," is devoid of any
factual basis. Such finding is not supported by the evidence on record and cannot
therefore be a valid basis of a finding of conspiracy.
Time and again we have been guided by the principle that it would be better to set free
ten men who might be probably guilty of the crime charged than to convict one innocent
man for a crime he did not commit. There being no conspiracy, only Antonio Comadre
must answer for the crime.
PEOPLE OF THE PHILIPPINES, appellee, vs. NARCISO RAMOS y MATIAS,
RAMON SAN ROQUE y DELA CRUZ, EULALIA SAN ROQUE DE FRANCISCO y
DELA CRUZ alias LALING, WILLIAM RAMOS alias WILFREDO RAMOS,
(provisionally dismissed), and three (3) other John Does, accused. EULALIA SAN
ROQUE DE FRANCISCO y DELA CRUZ alias "LALING", appellant.
G.R. No. 135204 | April 14, 2004 (Third Division)
SANDOVAL-GUTIERREZ, J.
Facts: An information of murder was filed against appellant and her co-accused Narciso
Ramos y Matias alias "Narcing", Ramon San Roque y dela Cruz, Wilfredo Ramos and
three (3) other John Does. In convicting the appellant of murder, the trial court gave full
faith and credence to the testimony of prosecution lone eyewitness, Bernie Ambal.
In the instant case, appellant assails the sufficiency of evidence to establish the
existence of conspiracy among the accused. She vigorously contends that she did not
participate in the killing of the victim.
Issue: Whether or not the prosecution failed to adduce sufficient evidence to establish
the existence of conspiracy among the accused

33

Held: No. In determining the existence of conspiracy, it is not necessary to show that all
the conspirators actually hit and killed the victim. The presence of conspiracy among the
accused can be proven by their conduct before, during or after the commission of the
crime showing that they acted in unison with each other, evincing a common purpose or
design. There must be a showing that appellant cooperated in the commission of the
offense, either morally, through advice, encouragement or agreement or materially
through external acts indicating a manifest intent of supplying aid in the perpetration of
the crime in an efficacious way. In such case, the act of one becomes the act of all, and
each of the accused will thereby be deemed equally guilty of the crime committed.
The series of events in this case convincingly show that appellant and her co-accused
acted in unison and cooperated with each other in killing William Lomida. Appellant was
the one who opened the door and allowed the other accused to enter the house. She
joined them in bringing the victim to the residence of Narciso Ramos, her brother-in-law.
While her co-accused dragged the helpless victim, tied him to a santol tree, stabbed him
twice by a bladed knife, and shot him 5 to 7 times, appellant merely watched intensely.
She even "turned her back" as the lifeless body of the victim was being burned. And
after attaining their purpose, she fled with the other accused.
The above circumstances clearly show the common purpose and concerted efforts on
the part of appellant and her co-accused. We agree with the trial court in concluding that
their acts were indications of a criminal conspiracy to commit the crime of murder.
PEOPLE OF THE PHILIPPINES, plaintiff-appellee, vs. ASSEMBLYMAN ARTURO F.
PACIFICADOR (At Large), SGT. DOMINGO DALMACIO, SGT. ENRICO CABAERO,
C2C REYNALDO ALIPALA, PFC VICENTE VEGAFRIA, P/CPL. HECTOR FULLON,
and PAT/ LORENZO MINGOTE, accused, SSGT. DOMINGO DALMACIO, C2C
REYNALDO ALIPALA, PFC VICENTE VEGAFRIA, PCPL. HECTOR FULLON and
PAT. LORENZO MINGOTE, accused-appellants.
G.R. No. 126515 | February 6, 2002 (First Division)
PARDO, J.
Facts: Accused S/Sgt. Domingo Dalmacio, C2C Reynaldo Alipala, PFC Vicente
Vegafria, PCpl. Hector Fullon and Pat. Lorenzo Mingote, appeal from the decision of the
Regional Trial Court finding them guilty beyond reasonable doubt of multiple murder
committed against Clemente Samulde, Rhium Sanchez, Plaridel Sanchez IV, Aldrick
Sanchez, Mamerto Zaldivar, Jr., Armelito Tamboong and Abner Varon and frustrated
murder committed against Luna Sanchez.
Accused-appellants reiterated that the group of Luna Sanchez was the first to fire
against them and they merely fired back as an act of retaliation and in self-defense.
Hence, they claim that it was erroneous for the trial court to find all of them guilty of
multiple murder and frustrated murder and sentencing them accordingly. They anchor
their defense on the theory that the group of the victims staged the ambush, and that
they merely retaliated and fired back as an act of self-defense. In support of such
34

theory, they presented evidence that all the deceased on the side of the prosecution
were found positive for nitrates indicating that they fired guns, some of which are
armalite rifles, others revolvers or pistols.
Issue: Whether or not the trial court erred in ruling that sufficient evidence existed to
establish conspiracy
Held: No. Conspiracy need not be established by direct evidence, for it may be inferred
from the conduct of the accused, before, during, and after the commission of the crime,
which, if all taken together, would reasonably be strong enough to show a community of
criminal design.
Prosecution witness Luna Sanchez, was present at the scene of the crime, he being
one of the victims. He stated that during the election campaign trails on May 13, 1984,
the group of Pacificador, escorted by the accused-appellants overtook their Ford
Tamaraw vehicle several times, and the latter pointed their guns against them, making it
very apparent that accused-appellants were tailing the group of Luna Sanchez. And
ultimately, accused-appellants perpetrated their unlawful design against the group of
Luna Sanchez when they strategically positioned themselves at the southern end of the
single-lane Pangpang Bridge in Sibalom, Antique, parked their vehicle near the foot of
the bridge making sure that the group of Luna Sanchez would not be able pass through
and took cover in the nearby canal and waited for the arrival of their prey. The
simultaneous acts of leaving, waiting for their victims to come out, tailing and firing at
them continuously at close range, and escaping from the crime scene clearly establish a
conspiracy among the malefactors.
Hence, the trial court did not err when it ruled that conspiracy exists. Where the acts of
the accused collectively and individually demonstrate the existence of a common design
towards the accomplishment of the same unlawful purpose, conspiracy is evident, and
all the perpetrators will be liable as principals.
(SUPPLEMENTARY APPLICATION OF RPC PROVISIONS TO
SPECIAL PENAL LAWS ART.10)
EVANGELINE LADONGA, petitioner, vs. PEOPLE OF THE PHILIPPINES,
respondent.
G.R. No. 141066 | February 17, 2005 (Second Division)
AUSTRIA-MARTINEZ, J.
Facts: Three Informations for violation of B.P. Blg. 22 were filed with the RTC against
Ladonga spouses. RTC rendered a joint decision finding the Ladonga spouses guilty
beyond reasonable doubt of violating B.P. Blg. 22.
Petitioner brought the case to the Court of Appeals, arguing that the RTC erred in
finding her criminally liable for conspiring with her husband as the principle of
35

conspiracy is inapplicable to B.P. Blg. 22 which is a special law; moreover, she is not a
signatory of the checks and had no participation in the issuance thereof. She staunchly
insists that she cannot be held criminally liable for violation of B.P. Blg. 22 because she
had no participation in the drawing and issuance of the three checks subject of the three
criminal cases, a fact proven by the checks themselves. She contends that the Court of
Appeals gravely erred in applying the principle of conspiracy, as defined under the RPC,
to violations of B.P. Blg. 22. She posits that the application of the principle of conspiracy
would enlarge the scope of the statute and include situations not provided for or
intended by the lawmakers, such as penalizing a person, like petitioner, who had no
participation in the drawing or issuance of checks.
Issue: Whether or not CA is correct in applying the suppletory character of the Revised
Penal Code to Special Laws like B.P. Blg. 22
Held: Yes. Article 10 of the RPC reads as follows:
ART. 10. Offenses not subject to the provisions of this Code. Offenses which are or in
the future may be punishable under special laws are not subject to the provisions of this
Code. This Code shall be supplementary to such laws, unless the latter should specially
provide the contrary.
The article is composed of two clauses. The first provides that offenses which in the
future are made punishable under special laws are not subject to the provisions of the
RPC, while the second makes the RPC supplementary to such laws. While it seems
that the two clauses are contradictory, a sensible interpretation will show that they can
perfectly be reconciled.
The first clause should be understood to mean only that the special penal laws are
controlling with regard to offenses therein specifically punished. Said clause only
restates the elemental rule of statutory construction that special legal provisions prevail
over general ones. Lex specialis derogant generali. In fact, the clause can be
considered as a superfluity, and could have been eliminated altogether. The second
clause contains the soul of the article. The main idea and purpose of the article is
embodied in the provision that the "code shall be supplementary" to special laws, unless
the latter should specifically provide the contrary.
The appellate courts reliance on the cases of People vs. Parel, U.S. vs. Ponte, and
U.S. vs. Bruhez rests on a firm basis. These cases involved the suppletory application
of principles under the then Penal Code to special laws. People vs. Parel is concerned
with the application of Article 22 of the Code to violations of Act No. 3030, the Election
Law, with reference to the retroactive effect of penal laws if they favor the accused. U.S.
vs. Ponte involved the application of Article 17 of the same Penal Code, with reference
to the participation of principals in the commission of the crime of misappropriation of
public funds as defined and penalized by Act No. 1740. U.S. vs. Bruhez covered Article
4530 of the same Code, with reference to the confiscation of the instruments used in
violation of Act No. 1461, the Opium Law.
36

B.P. Blg. 22 does not expressly proscribe the suppletory application of the provisions of
the RPC. Thus, in the absence of contrary provision in B.P. Blg. 22, the general
provisions of the RPC which, by their nature, are necessarily applicable, may be applied
suppletorily. Indeed, in the recent case of Yu vs. People, the Court applied suppletorily
the provisions on subsidiary imprisonment under Article 39 of the RPC to B.P. Blg. 22.
The suppletory application of the principle of conspiracy in this case is analogous to the
application of the provision on principals under Article 17 in U.S. vs. Ponte. For once
conspiracy or action in concert to achieve a criminal design is shown, the act of one is
the act of all the conspirators, and the precise extent or modality of participation of each
of them becomes secondary, since all the conspirators are principals.
PEOPLE OF THE PHILIPPINES, appellee, vs. LUISITO D. BUSTINERA, appellant.
G. R. No. 148233 | June 8, 2004 (Third Division)
CARPIO MORALES, J.
Facts: RTC convicted herein appellant of qualified theft for the unlawful taking of a
Daewoo Racer GTE Taxi and sentencing him to suffer the penalty of reclusion perpetua.
The conviction of the said crime was based under Article 310 of the Revised Penal
Code, as amended for the unlawful taking of a motor vehicle. However, Article 310 has
been modified, with respect to certain vehicles,27 by Republic Act No. 6539, as
amended, otherwise known as "AN ACT PREVENTING AND PENALIZING
CARNAPPING."
Issue: Whether or not the trial court, having convicted appellant of qualified theft instead
of carnapping, erred in the imposition of penalty
Held: Yes. While the information alleges that the crime was attended with grave abuse
of confidence, the same cannot be appreciated as the suppletory effect of the Revised
Penal Code to special laws, as provided in Article 10 of said Code, cannot be invoked
when there is a legal impossibility of application, either by express provision or by
necessary implication.
Moreover, when the penalties under the special law are different from and are without
reference or relation to those under the Revised Penal Code, there can be no
suppletory effect of the rules, for the application of penalties under the said Code or by
other relevant statutory provisions are based on or applicable only to said rules for
felonies under the Code.
Thus, in People v. Panida which involved the crime of carnapping and the penalty
imposed was the indeterminate sentence of 14 years and 8 months, as minimum, to 17
years and 4 months, as maximum, this Court did not apply the provisions of the Revised
Penal Code suppletorily as the anti-carnapping law provides for its own penalties which
are distinct and without reference to the said Code.
37

The charge being simple carnapping, the imposable penalty is imprisonment for
not less than 14 years and 8 months and not more than 17 years and 4 months.
There can be no suppletory effect of the rules for the application of penalties
under the Revised Penal Code or by other relevant statutory provisions based
on, or applicable only to, the rules for felonies under the Code. While it is true
that the penalty of 14 years and 8 months to 17 years and 4 months is virtually
equivalent to the duration of the medium period of reclusion temporal, such
technical term under the Revised Penal Code is not given to that penalty for
carnapping. Besides, the other penalties for carnapping attended by the
qualifying circumstances stated in the law do not correspond to those in the
Code. The rules on penalties in the Code, therefore, cannot suppletorily apply to
Republic Act No. 6539 and special laws of the same formulation. For this reason,
we hold that the proper penalty to be imposed on each of accused-appellants is
an indeterminate sentence of 14 years and 8 months, as minimum, to 17 years
and 4 months, as maximum. (Emphasis and underscoring supplied; citations
omitted)
Appellant being then culpable for carnapping under the first clause of Section 14 of
Republic Act No. 6539, as amended, the imposable penalty is imprisonment for not less
than 14 years and 8 months, not more than 17 years and 4 months, for, as discussed
above, the provisions of the Revised Penal Code cannot be applied suppletorily and,
therefore, the alleged aggravating circumstance of grave abuse of confidence cannot be
appreciated.
SHARICA MARI L. GO-TAN, Petitioner vs. SPOUSES PERFECTO C. TAN and
JUANITA L. TAN, Respondents.*
G.R. No. 168852 | September 30, 2008 (Third Division)
YNARES-SANTIAGO, J.
Facts: Sharica Mari L. Go-Tan (petitioner) and Steven L. Tan (Steven) were married.
Petitioner filed a Petition with Prayer for the Issuance of a Temporary Protective Order
(TPO) against Steven and her parents-in-law, Spouses Perfecto C. Tan and Juanita L.
Tan (respondents) before the RTC, alleging that Steven, in conspiracy with
respondents, were causing verbal, psychological and economic abuses upon her in
violation of Republic Act (R.A.) No. 9262, otherwise known as the Anti-Violence Against
Women and Their Children Act of 2004.
RTC issued a Resolution dismissing the case as to respondents on the ground that,
being the parents-in-law of the petitioner, they were not included/covered as
respondents under R.A. No. 9262 under the well-known rule of law expressio unius est
exclusio alterius.

38

In this case, petitioner contends that R.A. No. 9262 must be understood in the light of
the provisions of Section 47 of R.A. No. 9262 which explicitly provides for the suppletory
application of the Revised Penal Code (RPC) and, accordingly, the provision on
conspiracy under Article 8 of the RPC can be suppletorily applied to R.A. No. 9262;
that Steven and respondents had community of design and purpose in tormenting her
by giving her insufficient financial support; harassing and pressuring her to be ejected
from the family home; and in repeatedly abusing her verbally, emotionally, mentally and
physically; that respondents should be included as indispensable or necessary parties
for complete resolution of the case.
Issue: Whether or not the provision on conspiracy under Article 8 of the RPC can be
suppletorily applied to R.A. No. 9262
Held: Yes. Section 3 of R.A. No. 9262 defines ''violence against women and their
children'' as any act or a series of acts committed by any person against a woman who
is his wife, former wife, or against a woman with whom the person has or had a sexual
or dating relationship, or with whom he has a common child, or against her child
whether legitimate or illegitimate, within or without the family abode, which result in or is
likely to result in physical, sexual, psychological harm or suffering, or economic abuse
including threats of such acts, battery, assault, coercion, harassment or arbitrary
deprivation of liberty.
While the said provision provides that the offender be related or connected to the victim
by marriage, former marriage, or a sexual or dating relationship, it does not preclude the
application of the principle of conspiracy under the RPC. Indeed, Section 47 of R.A. No.
9262 expressly provides for the suppletory application of the RPC.
Based on prevailing jurisprudence, the principle of conspiracy under Article 8 of the
RPC may be applied suppletorily to R.A. No. 9262 because of the express provision of
Section 47 that the RPC shall be supplementary to said law. Thus, general provisions
of the RPC, which by their nature, are necessarily applicable, may be applied
suppletorily. For once conspiracy or action in concert to achieve a criminal design is
shown, the act of one is the act of all the conspirators, and the precise extent or
modality of participation of each of them becomes secondary, since all the conspirators
are principals.
LEONILO SANCHEZ alias NILO, Appellant, vs. PEOPLE OF THE PHILIPPINES and
COURT OF APPEALS, Appellees.
G.R. No. 179090 | June 5, 2009 (Third Division)
NACHURA, J.
Facts: Appellant was convicted with the crime of Other Acts of Child Abuse. Applying
the Indeterminate Sentence Law, the RTC imposed upon appellant the penalty of six (6)
years of prision correccional, as minimum, to seven (7) years and four (4) months of
prision mayor, as maximum. The CA modified this by imposing upon appellant the
indeterminate penalty of six (6) years and one (1) day, as minimum, to eight (8) years,
39

as maximum, of prision mayor, postulating that since R.A. No. 7610 is a special law, the
RTC should have imposed on appellant an indeterminate sentence, "the maximum term
of which shall not exceed the maximum fixed by said law and the minimum shall not be
less than the minimum term prescribed by the same."
Issue: Whether or not the RTC erred in applying the Indeterminate Sentence Law
because R.A. No. 7610 is a special law
Held: No. Section 1 of the Indeterminate Sentence Law, as amended, provides:
SECTION 1. Hereafter, in imposing a prison sentence for an offense punished by
the Revised Penal Code, or its amendments, the court shall sentence the
accused to an indeterminate sentence the maximum term of which shall be that
which, in view of the attending circumstances, could be properly imposed under
the rules of the said Code, and the minimum of which shall be within the range of
the penalty next lower to that prescribed by the Code for the offense; and if the
offense is punished by any other law, the court shall sentence the accused to an
indeterminate sentence, the maximum term of which shall not exceed the
maximum fixed by said law and the minimum shall not be less than the minimum
term prescribed by the same.
To repeat, the penalty for Other Acts of Child Abuse is prision mayor in its minimum
period. This penalty is derived from, and defined in, the Revised Penal Code. Although
R.A. No. 7610 is a special law, the rules in the Revised Penal Code for graduating
penalties by degrees or determining the proper period should be applied. Thus, where
the special law adopted penalties from the Revised Penal Code, the Indeterminate
Sentence Law will apply just as it would in felonies. In People v. Simon, the Court
applied the first clause of Section 1 of the Indeterminate Sentence Law to cases of
illegal drugs. In Cadua v. Court of Appeals, the Court applied the same principle to
cases involving illegal possession of firearms. In those instances, the offenses were
also penalized under special laws. Finally, in Dulla v. Court of Appeals, a case involving
sexual abuse of a child as penalized under Section 5(b), Article III of R.A. No. 7610, the
Court likewise applied the same first clause of the Indeterminate Sentence Law. This
case should be no exception.
In the absence of any modifying circumstances, and because it is favorable to appellant,
we find the penalty of four (4) years, nine (9) months and eleven (11) days of prision
correccional, as minimum, to six (6) years, eight (8) months and one (1) day of prision
mayor, as maximum, proper.

(JUSTIFYING CIRCUMSTANCES ART. 11)


40

RODEL URBANO, Petitioner, vs. PEOPLE OF THE PHILIPPINES, Respondent.


G.R. No. 182750 | January 20, 2009 (Second Division)
VELASCO, JR., J.
Facts: In an Information filed before the RTC, petitioner was charged with Homicide. It
was alleged therein that the victim Brigido Tomelden and petitioner, having just arrived
from a picnic in the nearby town of Bugallon, Pangasinan, where, with some other coworkers, they drunk beer in a restaurant. While inside the compound, the two had a
heated altercation in the course of which Tomelden hurled insulting remarks at
petitioner. Reacting, petitioner asked why Tomelden, when drunk, has the penchant of
insulting petitioner.
The exchange of words led to an exchange of blows. Then petitioner delivered a "lucky
punch," as described by eyewitness Orje Salazar, on Tomeldens face, which made
Tomelden topple down. Tomelden was on the verge of hitting his head on the ground
had their companions not caught him and prevented the fall. The blow, however, caused
Tomeldens nose to bleed and rendered him unconscious.
RTC rendered judgment finding petitioner guilty as charged. On appeal, CA rendered a
decision, affirming the conviction of petitioner. Petitioner now urges the Court to set
aside the appealed decision, or at least modify it, maintaining that the appellate court
erred in not appreciating the mitigating circumstances of sufficient provocation on the
part of the victim and lack of intent to commit so grave a wrong in favor of the petitioner.
Issue: Whether or not the victims insulting remarks directed at the accused and uttered
immediately before the fist fight constitute sufficient provocation.
Held: Yes. When the law speaks of provocation either as a mitigating circumstance or
as an essential element of self-defense, the reference is to an unjust or improper
conduct of the offended party capable of exciting, inciting, or irritating anyone; it is not
enough that the provocative act be unreasonable or annoying; the provocation must be
sufficient to excite one to commit the wrongful act and should immediately precede the
act. This third requisite of self-defense is present: (1) when no provocation at all was
given to the aggressor; (2) when, even if provocation was given, it was not sufficient; (3)
when even if the provocation was sufficient, it was not given by the person defending
himself; or (4) when even if a provocation was given by the person defending himself, it
was not proximate and immediate to the act of aggression.
In the instant case, Tomeldens insulting remarks directed at petitioner and uttered
immediately before the fist fight constituted sufficient provocation. This is not to mention
other irritating statements made by the deceased while they were having beer in
Bugallon. Petitioner was the one provoked and challenged to a fist fight.
CELERINO SANCHEZ, petitioner, vs. PEOPLE OF THE PHILIPPINES, respondent.
G.R. No. 161007 | December 6, 2006 (Third Division)
TINGA, J.
41

Facts: Sanchez's account of the facts shows that he and Jamero were tenants of
adjacent lots located in San Jose, Mahayag, Zamboanga del Sur. At about 7:00 o'clock
in the morning of September 4, 1993, Sanchez saw Jamero destroying the dike which
served as the boundary between the two lots. Sanchez confronted Jamero and told the
latter that he was encroaching on his land. Jamero struck him with a shovel. The shovel
got stuck in the mud so Jamero resorted to throwing mud at Sanchez. Fighting back,
Sanchez hacked Jamero with a bolo, resulting in the latter's death. Sanchez then
proceeded to the municipal building to surrender upon the advice of his son-in-law.
According to the OSG, Jamero's attack on Sanchez was unsuccessful because the
latter was able to evade it and Jamero's shovel got stuck in the mud. Jamero fled
toward the ricefield when Sanchez unsheathed his bolo. Sanchez pursued him and
struck his head with a bolo. Jamero fell down but was able to stand up again. He ran
away but after a short distance, fell down again. Sanchez approached him and stabbed
him several times. Not satisfied, Sanchez pushed Jamero's face down into the kneedeep mud. After Jamero's aggression ceased when he fled and left his shovel stuck in
the mud, there was no longer any justification for Sanchez to go after him and hack him
to death.
Issue: Whether or not unlawful aggression, if not continuous, does not constitute
aggression warranting self-defense
Held: No. There can be no self-defense, complete or incomplete, unless the accused
proves the first essential requisiteunlawful aggression on the part of the victim.
Unlawful aggression presupposes an actual, sudden and unexpected or imminent
danger on the life and limb of a person a mere threatening or intimidating attitude is
not sufficient. There must be actual physical force or a threat to inflict physical injury. In
case of a threat, it must be offensive and positively strong so as to display a real, not
imagined, intent to cause injury. Aggression, if not continuous, does not constitute
aggression warranting self-defense.
In this case, the twin circumstances of Jamero's shovel getting stuck in the mud and his
running away from Sanchez convincingly indicate that there was no longer any danger
to the latter's life and limb which could have justified his pursuit of Jamero and
subsequent hacking and killing of the latter.
Sanchez's failure to prove unlawful aggression by Jamero and the prosecution's
evidence conclusively showing that it was Sanchez who was the unlawful aggressor
completely discounts Sanchez's claim of self-defense. Even incomplete self-defense by
its very nature and essence would always require the attendance of unlawful aggression
initiated by the victim which must clearly be shown.
PEOPLE OF THE PHILIPPINES, plaintiff-appellee, vs. HONORATO C. BELTRAN,
JR., accused-appellant.
G.R. No. 168051 | September 27, 2006 (First Division)
CHICO-NAZARIO, J.
42

Facts: Appellant was indicted in an Information for Murder allegedly committed as


follows: That on or about October 25, 1999 at around 10:00 oclock in the evening at
Velasquez Road, Brgy. Sta. Rita, Batangas City, Philippines and within the jurisdiction of
this Honorable Court, the above-named accused, while armed with a bolo, a deadly
weapon, with intent to kill and with the qualifiying circumstance of treachery, did then
and there, willfully, unlawfully and feloniously attack, assault and hack with the said
bolo, suddenly and without warning one Norman Conception y Habla while the latter
was unarmed and completely defenseless, thereby hitting him on the different parts of
his body, which directly caused the victims death. When arraigned on November 9,
1999, appellant pleaded Not Guilty to the charge therein. Thereafter, trial ensued.
Appellant, on his defense admitted that he hacked Norman with a bolo but insisted that
he did the same in self-defense. RTC rendered its Decision finding the accused
Honorato Beltran, Jr. guilty beyond reasonable doubt of the crime of murder.
Issue: Whether or not the appellant Honorato Beltran, Jr. is entitled to the justifying
circumstance of self-defense
Held: No. As an element of self-defense, unlawful aggression refers to an assault or
attack, or a threat thereof in an imminent and immediate manner, which places the
defendant's life in actual peril. It is an act positively strong showing the intent of the
aggressor and not merely a threatening or intimidating attitude. It is also described as a
sudden and unprovoked attack of immediate and imminent kind to the life, safety or
rights of the person attacked.
There is an unlawful aggression on the part of the victim when he puts in actual or
imminent peril the life, limb, or right of the person invoking self-defense. There must be
actual physical force or actual use of weapon. In order to constitute unlawful
aggression, the person attacked must be confronted by a real threat on his life and limb;
and the peril sought to be avoided is imminent and actual, not merely imaginary.
In the instant case, there was no unlawful aggression on the part of Norman that
justified the act of appellant in hacking him to death. There was no actual or imminent
danger on the life of appellant when he came face to face with Norman. As narrated by
Ever, Norman was just walking on the road and was not provoking appellant into a fight.
It was the appellant who approached and suddenly hacked Norman repeatedly even
when the latter was already fallen on the ground. In short, appellant was the unlawful
aggressor.
Even if this Court were to adopt the version of facts of appellant, the result or conclusion
would be the same.
Appellant alleged that he was resting inside his house when he heard Norman shouting
invectives against him and challenging him to a fight. When he went outside the house
to pacify Norman, the latter slapped the back of his head and brought out an ice-pick.
43

Appellant retreated and when Norman tried to follow him inside the house, he took a
bolo and repeatedly hacked Norman. The foregoing circumstances does not justify the
act of appellant in hacking Norman. Obviously, mere shouting of invectives and
challenging one to a fight does not put one's life in actual or imminent danger. In the
same vein, mere slapping of one's head does not place a person's life in serious danger
such that it compels him to use a bolo and hack the offender.
Time and again, we held that unlawful aggression is a sine qua non for upholding the
justifying circumstance of self-defense. It is an essential and indispensable requisite, for
without unlawful aggression on the part of the victim, there can be, in a jural sense, no
complete or incomplete self-defense. Without unlawful aggression, self-defense will not
have a leg to stand on and this justifying circumstance cannot and will not be
appreciated even if the other elements are present. To our mind, unlawful aggression is
clearly absent in the case at bar.
PEOPLE OF THE PHILIPPINES, Plaintiff-Appellee,vs. ROLANDO DAGANI y REYES
and OTELLO SANTIANO Y LEONIDA, Accused-Appellants.
G.R. No. 153875 | August 16, 2006 (First Division)
AUSTRIA-MARTINEZ, J.
Facts: Accused-appellants Otello Santiano y Leonida (Santiano) and Rolando Dagani y
Reyes (Dagani) were found guilty of the crime of Murder committed against Ernest
Felix.
Appellants testified that they were ordered by their desk officer to investigate a
commotion at the canteen. Upon reaching the place, Santiano ordered his co-accused,
Dagani, to enter, while the former waited outside. Dagani approached Javier who had
been striking a bottle of beer on the table. Javier then pulled out a .22 caliber revolver
and attempted to fire at Dagani, but the gun failed to go off. Then suddenly, while
outside the canteen, Santiano heard gunfire and, from his vantage point, he saw Javier
and Dagani grappling for a .22 caliber gun which belonged to Javier. During the course
of the struggle, the gun went off, forcing Santiano to fire a warning shot. He heard
Javiers gun fire again, so he decided to rush into the canteen. Santiano then shot
Javier from a distance of less than four meters.
Issue: Whether or not the justifying circumstances of self-defense and lawful
performance of official duty as PNR security officers can be invoked by appellants
Held: No. When self-defense is invoked, the burden of evidence shifts to the accused to
show that the killing was legally justified. Having owned the killing of the victim, the
accused should be able to prove to the satisfaction of the Court the elements of selfdefense in order to avail of this extenuating circumstance. He must discharge this
burden by clear and convincing evidence. When successful, an otherwise felonious
deed would be excused, mainly predicated on the lack of criminal intent of the accused.
Self-defense requires that there be (1) an unlawful aggression by the person injured or
44

killed by the offender, (2) reasonable necessity of the means employed to prevent or
repel that unlawful aggression, and (3) lack of sufficient provocation on the part of the
person defending himself. All these conditions must concur.
Unlawful aggression, a primordial element of self-defense, would presuppose an actual,
sudden and unexpected attack or imminent danger on the life and limb of a person not
a mere threatening or intimidating attitude but most importantly, at the time the
defensive action was taken against the aggressor. To invoke self-defense successfully,
there must have been an unlawful and unprovoked attack that endangered the life of the
accused, who was then forced to inflict severe wounds upon the assailant by employing
reasonable means to resist the attack.
In the instant case, the assertions that it was "quite probable" that Javier, during the
course of the struggle for the firearm, "could have easily killed" the appellants are
uncertain and speculative. There is aggression in contemplation of the law only when
the one attacked faces real and immediate threat to ones life. The peril sought to be
avoided must be imminent and actual, not just speculative.
The second element of self-defense demands that the means employed to neutralize
the unlawful aggression are reasonable and necessary. It is settled that reasonable
necessity of the means employed does not imply material commensurability between
the means of attack and defense. What the law requires is rational equivalence. The
circumstances in their entirety which surround the grappling of the firearm by Dagani
and Javier, such as the nature and number of gunshot wounds sustained by the victim
which amounted to two fatal wounds, that Dagani was able to restrain the hands of
Javier and push them away from his body, that Dagani was larger than Javier and had
finished Special Weapons and Tactics (SWAT) hand-to-hand combat training, and
Javier, as admitted by the appellants, was inebriated at the time of the incident, do not
justify appellant Santianos act of fatally shooting the victim twice.
All things considered, the appellants plea of self-defense is not corroborated by
competent evidence. The plea of self-defense cannot be justifiably entertained where it
is not only uncorroborated by any separate competent evidence but is in itself extremely
doubtful. Whether the accused acted in self-defense is a question of fact. Like alibi, the
affirmative defense of self-defense is inherently weak because, as experience has
demonstrated, it is easy to fabricate and difficult to disprove.
Appellants set up the defense that they were in the lawful performance of their official
duties. They specifically aver that they had been ordered by their desk officer to proceed
to the canteen in response to a telephone call stating that there was a group "creating
trouble;" that they were in the call of duty and exercising their functions and
responsibilities as members of the PNR Civil Security Office to preserve peace and
order and protect the lives and property in the PNR Compound; and that, invoking
jurisprudence, as security officers in the performance of duty, like the police, they must
stand their ground and overcome the opponent, and the force that may be exerted must
differ from that which ordinarily may be offered in self-defense.
45

Article 11 of the Revised Penal Code provides that a person who acts in the fulfillment
of a duty or in the lawful exercise of a right or office does not incur any criminal liability.
Two requisites must concur before this defense can prosper: 1) the accused must have
acted in the performance of a duty or in the lawful exercise of a right or office; and 2) the
injury caused or the offense committed should have been the necessary consequence
of such lawful exercise. These requisites are absent in the instant case.
Moreover, since it was not established that Javier fired his gun, the injury inflicted upon
him cannot be regarded as a necessary consequence of appellants due performance of
an official duty.
NOE TOLEDO y TAMBOONG, petitioner, vs. PEOPLE OF THE PHILIPPINES,
respondent.
G.R. No. 158057 | September 24, 2004 (Second Division)
CALLEJO, SR., J.
Facts: The accused Toledo was charged with homicide for the killing of one Ricky
Duarte. Toledo insisted that when he killed the victim, the same was purely accidental.
He claimed that the victim was so drunk that the same charged at the door of his house.
This prompted the accused to get his bolo and when he tried to prevent Ricky from
entering, he accidentally hit the latter whereby killing him.
The RTC and the CA found him guilty, prompting the accused to file the instant petition,
claiming that his actions were purely on self-defense. It was done when the victim
attacked him and in trying to defend himself, he accidentally killed Duarte.
Issue: Whether or not accused is entitled to the justifying circumstance of self-defense
Held: No. Self-defense under Article 11, paragraph 1 of the Revised Penal Code
necessarily implies a deliberate and positive overt act of the accused to prevent or repel
an unlawful aggression of another with the use of reasonable means. The accused has
freedom of action. He is aware of the consequences of his deliberate acts. The defense
is based on necessity which is the supreme and irresistible master of men of all human
affairs, and of the law. From necessity, and limited by it, proceeds the right of selfdefense. The right begins when necessity does, and ends where it ends. Although the
accused, in fact, injures or kills the victim, however, his act is in accordance with law so
much so that the accused is deemed not to have transgressed the law and is free from
both criminal and civil liabilities. On the other hand, the basis of exempting
circumstances under Article 12 of the Revised Penal Code is the complete absence of
intelligence, freedom of action, or intent, or the absence of negligence on the part of the
accused. The basis of the exemption in Article 12, paragraph 4 of the Revised Penal
Code is lack of negligence and intent. The accused does not commit either an
intentional or culpable felony. The accused commits a crime but there is no criminal
liability because of the complete absence of any of the conditions which constitute free
46

will or voluntariness of the act. An accident is a fortuitous circumstance, event or


happening; an event happening wholly or partly through human agency, an event which
under the circumstances is unusual or unexpected by the person to whom it happens.
Self-defense, under Article 11, paragraph 1, and accident, under Article 12, paragraph 4
of the Revised Penal Code, are affirmative defenses which the accused is burdened to
prove, with clear and convincing evidence. Such affirmative defenses involve questions
of facts adduced to the trial and appellate courts for resolution. By admitting killing the
victim in self-defense or by accident without fault or without intention of causing it, the
burden is shifted to the accused to prove such affirmative defenses. He should rely on
the strength of his own evidence and not on the weakness of that of the prosecution. If
the accused fails to prove his affirmative defense, he can no longer be acquitted.
To prove self-defense, the petitioner was burdened to prove the essential elements
thereof, namely: (1) unlawful aggression on the part of the victim; (2) lack of sufficient
provocation on the part of the petitioner; (3) employment by him of reasonable means to
prevent or repel the aggression. Unlawful aggression is a condition sine qua non for the
justifying circumstances of self-defense, whether complete or incomplete. Unlawful
aggression presupposes an actual, sudden, and unexpected attack, or imminent danger
thereof, and not merely a threatening or intimidating attitude.
PEOPLE OF THE PHILIPPINES, Appellee, vs. JOSEPH CAJURAO, Appellant.
G.R. No. 122767 | January 20, 2004 (Second Division)
CALLEJO, SR., J.
Facts: Appellant was convicted of murder committed against Santiago Betita. The trial
court rejected the appellants defense and concluded that he failed to present clear and
convincing evidence to prove that he killed Betita in self-defense. It gave credence and
full probative weight to the testimony of the witnesses for the prosecution, that the
appellant stabbed the defenseless victim. It also appreciated the presence of treachery,
qualifying the crime to murder.
Issue: Whether or not accused is entitled to the justifying circumstance of self-defense
Held: No. There can be no self-defense, complete or incomplete, unless there is clear
and convincing proof of unlawful aggression on the part of the victim. The unlawful
aggression, a constitutive element of self-defense, must be real or at least imminent and
not merely imaginary. A belief that a person is about to be attacked is not sufficient.
Even an intimidating or threatening attitude is by no means enough. Unlawful
aggression presupposes an actual or imminent danger on the life or limb of a person.
Mere shouting, an intimidating or threatening attitude of the victim does not constitute
unlawful aggression. Unlawful aggression refers to an attack that has actually broken
out or materialized or at the very least is clearly imminent; it cannot consist in oral
threats or merely a threatening stance or posture. The settled rule in jurisprudence is
that when unlawful aggression ceases, the defender no longer has the right to kill or
even wound the former aggressor. Retaliation is not a justifying circumstance. Upon the
47

cessation of the unlawful aggression and the danger or risk to life and limb, the
necessity for the person invoking self-defense to attack his adversary ceases. If he
persists in attacking his adversary, he can no longer invoke the justifying circumstance
of self-defense. Self-defense does not justify the unnecessary killing of an aggressor
who is retreating from the fray.
In this case, Pordios testified that the appellant stabbed Betita even as the latter moved
over to the next stall, still holding the lamp with the lighted wick which he took from her
stall to defend himself from the appellant. Betita had anticipated that the appellant would
assault him. Betitas fears proved to be well-founded, as the appellant rushed to where
he was and stabbed him on the right nipple. Pordios did not testify that before the
stabbing, Betita was about to throw the bottle at the appellant. In fine, Betita was in a
defensive position when he was stabbed. If, as claimed by the appellant, Betita was
about to throw the bottle at him, surely Betitas right hand would have been raised
above his head, his body and right hand arched backward, ready to throw the bottle at
the appellant. This was not the case.
Assuming that Betita did slap the appellant on the face, the appellants evidence shows,
however, that Betita anticipated that the appellant would retaliate and forthwith ran away
to the stall of Pacita and took hold of the knife. From that moment, the inceptive
unlawful aggression on the part of Betita had ceased to exist; there was no longer a
need for the appellant to still pursue the victim and kill him. In fine, when the appellant
stabbed the victim, he did so to retaliate.
PEOPLE OF THE PHILIPPINES, plaintiff-appellee, vs. JERRY ANTONIO Y
DIOLATA, accused-appellant.
G.R. No. 144933 | July 3, 2002 (First Division)
YNARES-SANTIAGO, J.
Facts: Accused-appellant was convicted of murder. The facts as presented by the
prosecution show that at 1:00 in the early morning of October 11, 1998, the victim,
Jomar Ephan, was engaged in a drinking session with Reynaldo Ephan and Roselito
Dacillo in front of a store in Barangay Pakna-an, Mandaue City. Accused-appellant
arrived and bought cigarettes. Then, he ordered Jomar, Reynaldo and Roselito to count
the cigarettes he bought, but the three told accused-appellant to let the storekeeper do
the counting. Rebuked, accused-appellant left the store. He returned minutes later and
suddenly stabbed the victim at the back, after which he immediately fled. The victim was
rushed by his companions to the hospital but died the following day.
On the other hand, the defense tried to prove that deceased attempted to hit accusedappellant but because the former was very drunk, he missed and fell on his belly. It was
at this point when accused-appellant got hold of a knife he saw under the table and
stabbed the deceased at the back.
Issue: Whether or not the trial court erred in rejecting accused-appellants self-defense
theory
48

Held: No.Where an accused invokes self-defense, he thereby admits authorship of the


crime. The burden of proof is thus shifted on him to prove all the elements of selfdefense, to wit: (1) unlawful aggression on the part of the victim; (2) reasonable
necessity of the means employed to repel the aggression; and (3) lack of sufficient
provocation on the part of the accused.
In the case at bar, even if we sustain the version of accused-appellant that the initial act
of aggression came from the group of the deceased, still we cannot uphold his plea of
self-defense. As testified by accused-appellant himself, the deceased who was at that
time very drunk tried to hit him but missed and fell on the ground. At that point, unlawful
aggression ceased and it was no longer necessary for him to stab the deceased. It was
accused-appellant, therefore, who became the aggressor when he, despite the
condition of the deceased, proceeded to stab the latter at the back. His act can no
longer be interpreted as an act of self-preservation but a perverse desire to kill. Hence,
he cannot successfully claim the benefit of self-defense. Furthermore, if it were true that
the companions of the deceased ganged up on him, his attack should have been
directed against them and not against the deceased who was already defenseless and
lying on the ground.
PEOPLE OF THE PHILIPPINES, plaintiff-appellee, vs. TEODORICO UBALDO,
accused-appellant.
G.R. No. 129389 | October 17, 2001 (Second Division)
QUISUMBING, J.
Facts: Appellant was found guilty of homicide. On the witness stand, appellant claimed
self-defense. According to him, Norberto Cabot, the victim, pulled a gun from his waist
when appellant accosted him. Norberto fired a shot at him, but missed as he was able
to duck. He then grappled with Norberto for possession of the gun and while they were
struggling, the gun went off three times, hitting Norberto each time. According to
appellant, it was impossible for Basilia Cabot to have witnessed the incident, since she
was not at the place where the shooting took place. On cross-examination, he clarified
that when they were struggling for possession of the firearm, he managed to hold the
gun so it pointed towards the deceased and that it was the victim who squeezed the
trigger three times.
Issue: Whether or not appellant acted in self-defense
Held: No. Generally, the burden of proof is upon the prosecution to prove the guilt of the
accused beyond reasonable doubt. Having invoked self-defense as a justifying
circumstance, however, appellant is deemed to have admitted having killed the victim,
and the burden of proof is shifted upon him to establish and prove his claim. To escape
liability, he must show the concurrent presence of all the elements of self-defense,
namely: (1) unlawful aggression on the part of the victim; (2) reasonable necessity of
the means employed to prevent or repel it; and (3) lack of sufficient provocation on the
part of the person defending himself.
49

In his memorandum, appellant contends that the testimonies of defense witnesses


Reynaldo Ventura, Anastacia Tapat, and appellant himself have clearly established all of
the foregoing elements of self-defense. Appellant submits that these testimonies have
satisfactorily shown that the sudden pointing and firing by Norberto of a gun at appellant
was an imminent and actual danger to the latter. To protect himself, appellant had to
wrestle for possession of the firearm, and in doing so, the gun went off and hit the
victim, causing his death.

PEOPLE OF THE PHILIPPINES, plaintiff-appellee,


vs. MAMERTO NARVAEZ, defendant-appellant.
G.R. Nos. L-33466-67

April 20, 1983|En Banc

MAKASIAR, J.

FACTS: In the afternoon of August 22, 1968, Graciano Juan, Jesus Verano and Cesar
Ibanez together with the two deceased Davis Fleischer and Flaviano Rubia, were
fencing the land of George Fleischer, father of deceased Davis Fleischer. The place was
in the boundary of the highway and the hacienda owned by George Fleischer. This is
located in the municipality of Maitum, South Cotabato. At the place of the fencing is the
house and rice drier of appellant Mamerto Narvaez.

At that time, appellant was taking his rest, but when he heard that the walls of his house
were being chiseled, he arose and there he saw the fencing going on. If the fencing
would go on, appellant would be prevented from getting into his house and the bodega
of his ricemill. So he addressed the group, saying 'Pare, if possible you stop destroying
my house and if possible we will talk it over what is good,' addressing the deceased
Rubia, who is appellant's compadre. The deceased Fleischer, however, answered: 'No,
gademit, proceed, go ahead.' Appellant apparently lost his equilibrium and he got his
gun and shot Fleischer, hitting him. As Fleischer fell down, Rubia ran towards the jeep,
and knowing there is a gun on the jeep, appellant fired at Rubia, likewise hitting him.
Both Fleischer and Rubia died as a result of the shooting'

50

CFI convicted the accused of murder qualified by treachery with the aggravating
circumstance of evident premeditation offset by the mitigating circumstance of voluntary
surrender. Hence, this appeal.

ISSUE: W/N the defendants act of killing the deceased falls under the justifying
circumstances of defense of property?

HELD: NO. The act of killing of the two deceased by appellant is not disputed.
Appellant admitted having shot them from the window of his house with the shotgun
which he surrendered to the police authorities. He claims, however, that he did so in
defense of his person and of his rights, and therefore he should be exempt from criminal
liability.

Defense of one's person or rights is treated as a justifying circumstance under Art. 11,
par. 1 of the Revised Penal Code, but in order for it to be appreciated, the following
requisites must occur:

First. Unlawful aggression;


Second. Reasonable necessity of the means employed to prevent or repel
it;
Third. Lack of sufficient provocation on the part of the person defending
himself (Art. 11, par. 1, Revised Penal Code, as amended).

The actuation of deceased Fleischer in angrily ordering the continuance of the fencing
would have resulted in the further chiselling of the walls of appellant's house as well as
the closure of the access to and from his house and rice mill-which were not only
imminent but were actually in progress. There is no question, therefore, that there was
aggression on the part of the victims: Fleischer was ordering, and Rubia was actually
51

participating in the fencing. This was indeed aggression, not on the person of appellant,
but on his property rights.

The following provisions of the Civil Code of the Philippines are in point:
Art. 536. In no case may possession be acquired through force or
intimidation as long as there is a possessor who objects thereto. He who
believes that he has an action or a right to deprive another of the holding
of a thing must invoke the aid of the competent court, if the holder should
refuse to deliver the thing.
Art. 539. Every possessor has a right to be respected in his possession;
and should he be disturbed therein he shall be protected in or restored to
said possession by the means established by the laws and the Rules of
Court (Articles 536 and 539, Civil Code of the Philippines).

Conformably to the foregoing provisions, the deceased had no right to destroy or cause
damage to appellant's house, nor to close his accessibility to the highway while he was
pleading with them to stop and talk things over with him. The assault on appellant's
property, therefore, amounts to unlawful aggression as contemplated by law.
Illegal aggression is equivalent to assault or at least threatened assault of
immediate and imminent kind (People vs. Encomiendas, 46 SCRA 522).

In the case at bar, there was an actual physical invasion of appellant's property which
he had the right to resist, pursuant to Art. 429 of the Civil Code of the Philippines which
provides:
Art. 429. The owner or lawful possessor of a thing has the right to exclude
any person from the enjoyment and disposal thereof. For this purpose, he
may use such force as may be reasonably necessary to repel or prevent
an actual or threatened unlawful physical invasion or usurpation of his
property (Emphasis supplied).

52

The reasonableness of the resistance is also a requirement of the justifying


circumstance of self-defense or defense of one's rights under paragraph 1 of Article 11,
Revised Penal Code. When the appellant fired his shotgun from his window, killing his
two victims, his resistance was disproportionate to the attack.

WE find, however, that the third element of defense of property is present, i.e., lack of
sufficient provocation on the part of appellant who was defending his property. As a
matter of fact, there was no provocation at all on his part, since he was asleep at first
and was only awakened by the noise produced by the victims and their laborers. His
plea for the deceased and their men to stop and talk things over with him was no
provocation at all.

Be that as it may, appellant's act in killing the deceased was not justifiable, since not all
the elements for justification are present. He should therefore be held responsible for
the death of his victims, but he could be credited with the special mitigating
circumstance of incomplete defense, pursuant to paragraph 6, Article 13 of the Revised
Penal Code.

(BATTERED WOMAN SYNDROME)


PEOPLE OF THE PHILIPPINES, appellee,
vs. MARIVIC GENOSA, appellant.
G.R. No. 135981|January 15, 2004|En Banc
PANGANIBAN, J.:
53

FACTS: On November 15, 1995, Marivic Genosa, who got home after work, got worried
that her husband Ben Genosa, who was not home yet might have gone gambling since
it was a payday. With her cousin Ecel Arao, appellant went to look for Ben at the
marketplace and taverns at Isabel, Leyte but did not find him there. They found Ben
drunk upon their return at the Genosas' house. Ecel went home despite appellant's
request for her to sleep in their house.

Then, Ben purportedly nagged appellant for following him, even challenging her to a
fight. She allegedly ignored him and instead attended to their children who were doing
their homework. Apparently disappointed with her reaction, Ben switched off the light
and, with the use of a chopping knife, cut the television antenna or wire to keep her from
watching television. According to appellant, Ben was about to attack her so she ran to
the bedroom, but he got hold of her hands and whirled her around. She fell on the side
of the bed and screamed for help. Ben left. At this point, appellant packed his clothes
because she wanted him to leave. Seeing his packed clothes upon his return home,
Ben allegedly flew into a rage, dragged appellant outside of the bedroom towards a
drawer holding her by the neck, and told her 'You might as well be killed so nobody
would nag me.' Appellant testified that she was aware that there was a gun inside the
drawer but since Ben did not have the key to it, he got a three-inch long blade cutter
from his wallet. She however, 'smashed' the arm of Ben with a pipe, causing him to drop
the blade and his wallet. Appellant then 'smashed' Ben at his nape with the pipe as he
was about to pick up the blade and his wallet. She thereafter ran inside the bedroom.
Appellant, however, insisted that she ended the life of her husband by shooting him.
She supposedly 'distorted' the drawer where the gun was and shot Ben. He did not die
on the spot, though, but in the bedroom.

RTC found the proffered theory of self-defense untenable, the court gave credence to
the prosecution evidence that appellant had killed the deceased while he was in bed
sleeping. Further, the trial court appreciated the generic aggravating circumstance of
treachery, because Ben Genosa was supposedly defenseless when he was killed -lying in bed asleep when Marivic smashed him with a pipe at the back of his head.

ISSUE: W/N the appellant acted in self-defense and in defense of her fetus on the
theory of the battered woman syndrome.
54

HELD: NO. Appellant admits killing Ben Genosa but, to avoid criminal liability, invokes
self-defense and/or defense of her unborn child. When the accused admits killing the
victim, it is incumbent upon her to prove any claimed justifying circumstance by clear
and convincing evidence. Well-settled is the rule that in criminal cases, self-defense
(and similarly, defense of a stranger or third person) shifts the burden of proof from the
prosecution to the defense.
The Battered Woman Syndrome
In claiming self-defense, appellant raises the novel theory of the battered woman
syndrome. While new in Philippine jurisprudence, the concept has been recognized in
foreign jurisdictions as a form of self-defense or, at the least, incomplete self-defense.
By appreciating evidence that a victim or defendant is afflicted with the syndrome,
foreign courts convey their "understanding of the justifiably fearful state of mind of a
person who has been cyclically abused and controlled over a period of time."
A battered woman has been defined as a woman "who is repeatedly subjected to any
forceful physical or psychological behavior by a man in order to coerce her to do
something he wants her to do without concern for her rights. Battered women include
wives or women in any form of intimate relationship with men. Furthermore, in order to
be classified as a battered woman, the couple must go through the battering cycle at
least twice. Any woman may find herself in an abusive relationship with a man once. If it
occurs a second time, and she remains in the situation, she is defined as a battered
woman."
Battered women exhibit common personality traits, such as low self-esteem, traditional
beliefs about the home, the family and the female sex role; emotional dependence upon
the dominant male; the tendency to accept responsibility for the batterer's actions; and
false hopes that the relationship will improve.
More graphically, the battered woman syndrome is characterized by the so-called "cycle
of violence," which has three phases: (1) the tension-building phase; (2) the acute
battering incident; and (3) the tranquil, loving (or, at least, nonviolent) phase.
During the tension-building phase, minor battering occurs -- it could be verbal or slight
physical abuse or another form of hostile behavior. The woman usually tries to pacify
the batterer through a show of kind, nurturing behavior; or by simply staying out of his
way. What actually happens is that she allows herself to be abused in ways that, to her,
are comparatively minor. All she wants is to prevent the escalation of the violence
exhibited by the batterer. This wish, however, proves to be double-edged, because her
"placatory" and passive behavior legitimizes his belief that he has the right to abuse her
in the first place.
However, the techniques adopted by the woman in her effort to placate him are not
usually successful, and the verbal and/or physical abuse worsens. Each partner senses
the imminent loss of control and the growing tension and despair. Exhausted from the
persistent stress, the battered woman soon withdraws emotionally. But the more she
becomes emotionally unavailable, the more the batterer becomes angry, oppressive
55

and abusive. Often, at some unpredictable point, the violence "spirals out of control"
and leads to an acute battering incident.
The acute battering incident is said to be characterized by brutality, destructiveness
and, sometimes, death. The battered woman deems this incident as unpredictable, yet
also inevitable. During this phase, she has no control; only the batterer may put an end
to the violence. Its nature can be as unpredictable as the time of its explosion, and so
are his reasons for ending it. The battered woman usually realizes that she cannot
reason with him, and that resistance would only exacerbate her condition.
At this stage, she has a sense of detachment from the attack and the terrible pain,
although she may later clearly remember every detail. Her apparent passivity in the face
of acute violence may be rationalized thus: the batterer is almost always much stronger
physically, and she knows from her past painful experience that it is futile to fight back.
Acute battering incidents are often very savage and out of control, such that innocent
bystanders or intervenors are likely to get hurt.
The final phase of the cycle of violence begins when the acute battering incident ends.
During this tranquil period, the couple experience profound relief. On the one hand,
the batterer may show a tender and nurturing behavior towards his partner. He knows
that he has been viciously cruel and tries to make up for it, begging for her forgiveness
and promising never to beat her again. On the other hand, the battered woman also
tries to convince herself that the battery will never happen again; that her partner will
change for the better; and that this "good, gentle and caring man" is the real person
whom she loves.
A battered woman usually believes that she is the sole anchor of the emotional stability
of the batterer. Sensing his isolation and despair, she feels responsible for his wellbeing. The truth, though, is that the chances of his reforming, or seeking or receiving
professional help, are very slim, especially if she remains with him. Generally, only after
she leaves him does he seek professional help as a way of getting her back. Yet, it is in
this phase of remorseful reconciliation that she is most thoroughly tormented
psychologically.
The illusion of absolute interdependency is well-entrenched in a battered woman's
psyche. In this phase, she and her batterer are indeed emotionally dependent on each
other -- she for his nurturant behavior, he for her forgiveness. Underneath this miserable
cycle of "tension, violence and forgiveness," each partner may believe that it is better to
die than to be separated. Neither one may really feel independent, capable of
functioning without the other.
In the instant case, we meticulously scoured the records for specific evidence
establishing that appellant, due to the repeated abuse she had suffered from her
spouse over a long period of time, became afflicted with the battered woman syndrome.
We, however, failed to find sufficient evidence that would support such a conclusion.
More specifically, we failed to find ample evidence that would confirm the presence of
the essential characteristics of BWS.
56

The defense fell short of proving all three phases of the "cycle of violence" supposedly
characterizing the relationship of Ben and Marivic Genosa. No doubt there were acute
battering incidents. In relating to the court a quo how the fatal incident that led to the
death of Ben started, Marivic perfectly described the tension-building phase of the cycle.
She was able to explain in adequate detail the typical characteristics of this stage.
However, that single incident does not prove the existence of the syndrome. In other
words, she failed to prove that in at least another battering episode in the past, she had
gone through a similar pattern.
The Supreme Court affirmed the conviction of appellant.
(DEFENSE OF RELATIVES)
RICARDO BALUNUECO, petitioner,vs.
COURT OF APPEALS AND THE PEOPLE OF THE PHILIPPINES, respondents.
G.R. No. 126968|April 9, 2003|Second Division
BELLOSILLO, J.:

FACTS: On the evening of May 2, 1982, accused Reynaldo, his father Juanito and
brothers Ricardo and Ramon, all surnamed Balunuexo, and one Armando Flores
chased Servando Iguico. With the five individuals inhot pursuit Servando scampered
into the safety of Amelias house, wife of his brother Senando.
Meanwhile, Senando went out of the house fully unaware of the commotion going on
outside. Upon seeing Senando, Reynaldo turned his attention on him and gave chase.
Senando instinctively fled towards the fields but he was met by Armando who hit him
with a stone, causing Senando to feel dizzy. Reynaldo, Ricardo, and Armando cornered
their quarry near a canal and ganged up on him. Armando placed a can on top of
Senandos head and Ricardo repeatedly struck Senando with an ax on the head,
shoulder, and hand. At one point, Ricardo lost his hold on the ax, but somebody tossed
him a bolo and then he continued hacking the victim who fell on his knees. To shield him
from further violence, Amelia put her arms around her husband but it was not enough to
detract Ricardo from his murderous frenzy. Amelia was also hit on the leg.
The trial court found Ricardo guilty of homicide and frustrated homicide. CA sustained
the conviction of accused Ricardo.

57

Petitioner argues that assuming he participated in the killing of Senando, he acted in


defense of his full-blood relatives: Reynaldo whom he personally witnessed being
boloed by the deceased in the arms, head and stomach; and Ramon who also became
a victim of the deceaseds fury after he was pushed by the deceased and had fallen to
the ground. Under such circumstances, the act of Senando in hacking him after he tried
to rescue his brothers, gave rise to a reasonable necessity for him to use a means to
prevent or repel the unlawful aggression. Considering further that there was lack of
sufficient provocation on his part, his acts were therefore justified under Art. 11, par. (2),
of The Revised Penal Code.

ISSUE: W/N petitioner acted in defense of his relatives.


HELD: NO. The essential elements of this justifying circumstance are the following: (a)
unlawful aggression; (b) reasonable necessity of the means employed to prevent or
repel it; and, (c) in case the provocation was given by the person attacked, the one
making the defense had no part therein.

Of the three (3) requisites of defense of relatives, unlawful aggression is a condition


sine qua non, for without it any defense is not possible or justified. In order to consider
that an unlawful aggression was actually committed, it is necessary that an attack or
material aggression, an offensive act positively determining the intent of the aggressor
to cause an injury shall have been made; a mere threatening or intimidating attitude is
not sufficient to justify the commission of an act which is punishable per se, and allow a
claim of exemption from liability on the ground that it was committed in self-defense or
defense of a relative. It has always been so recognized in the decisions of the courts, in
accordance with the provisions of the Penal Code.

Having admitted the killing of the victim, petitioner has the burden of proving these
elements by clear and convincing evidence. He must rely on the strength of his own
evidence and not on the weakness of that of the prosecution, for even if the prosecution
evidence is weak it cannot be disbelieved if the accused has admitted the killing.

In the case at bar, petitioner Ricardo utterly failed to adduce sufficient proof of the
existence of a positively strong act of real aggression on the part of the deceased
58

Senando. With the exception of his self-serving allegations, there is nothing on record
that would justify his killing of Senando.

First, Ricardos theory that when he reached the crime scene he found Senando
repeatedly hacking his brother Reynaldo who thereafter retaliated by smashing an axe
on the victims head is implausible in light of the seriousness of the wounds sustained
by the deceased as compared to the minor injuries inflicted upon petitioner and his two
(2) brothers. The fact that three (3) of the assailants suffered non-fatal injuries bolsters
the fact that Senando tried vainly to ward off the assaults of his assailants.

Second, Ricardo failed to present himself to the authorities. He may have accompanied
the injured Reynaldo to the hospital after the encounter but still he failed to present
himself to the authorities and report the matter to them. The natural impulse of any
person who has killed someone in defense of his person or relative is to bring himself to
the authorities and try to dispel any suspicion of guilt that the authorities might have
against him. This fact assumes a more special significance considering that his coaccused, Juanito and Armando, have remained at large.

Third, petitioner had a rather erratic recollection of people and events. He vividly
remembered how Reynaldo was injured by Senando but conveniently failed to recall the
events leading to the fatal wounding of the deceased. At another point, he testified that
Reynaldo axed Senando but later retracted his statement by declaring that it was in fact
Senando who hacked Reynaldo.15 We observe that the killing occurred within or near
the premises of the deceased. This proves per adventure the falsity of petitioners claim
that it was Senando, rather than he and his kin, who had initiated the unlawful
aggression.

The Supreme Court affirmed the decision of the Court of Appeals.


INTESTATE ESTATE OF MANOLITA GONZALES VDA. DE CARUNGCONG,
represented by MEDIATRIX CARUNGCONG, as Administratrix, Petitioner,
vs. PEOPLE OF THE PHILIPPINES and WILLIAM SATO, Respondents.
G.R. No. 181409 |February 11, 2010|Third Division
59

CORONA, J.:
FACTS: An information was filed against Sato for violation of Article 315 (a) of the
Revised Penal Code which was committed as follows:
That on or about the 24 th day of November, 1992, in Quezon City, Philippines, the
above-named accused, by means of deceit, did, then and there, wil[l]fully, unlawfully
and feloniously defraud MANOLITA GONZALES VDA. DE CARUNGCONG in the
following manner, to wit: the said accused induced said Manolita Gonzales Vda. De
Carungcong[,] who was already then blind and 79 years old[,] to sign and thumbmark a
special power of attorney dated November 24, 1992 in Savour of Wendy Mitsuko C.
Sato, daughter of said accused, making her believe that said document involved only
her taxes, accused knowing fully well that said document authorizes Wendy Mitsuko C.
Sato, then a minor, to sell, assign, transfer or otherwise dispose of to any person or
entity of her properties all located at Tagaytay City.
Sato moved for the quashal of the Information, claiming that under Article 332 of the
Revised Penal Code, his relationship to the person allegedly defrauded, the deceased
Manolita who was his mother-in-law, was an exempting circumstance.
The trial court granted the motion and dismissed the case. Upon appeal, CA sustained
the ruling of the trial court that the death of Zenaida did not extinguish the relationship
by affinity between her husband, private respondent Sato, and her mother Manolita, and
does not bar the application of the exempting circumstance under Article 332(1) of the
Revised Penal Code in Savour of private respondent Sato.
ISSUE: W/N death of the spouse dissolves the son-in-law and mother-in-law
relationship?
HELD: NO. Article 332 provides for an absolutory cause in the crimes of theft, estafa (or
swindling) and malicious mischief. It limits the responsibility of the offender to civil
liability and frees him from criminal liability by virtue of his relationship to the offended
party.
In connection with the relatives mentioned in the first paragraph, it has been held that
included in the exemptions are parents-in-law, stepparents and adopted children. By
virtue thereof, no criminal liability is incurred by the stepfather who commits malicious
mischief against his stepson; by the stepmother who commits theft against her stepson;
by the stepfather who steals something from his stepson; by the grandson who steals
from his grandfather; by the accused who swindles his sister-in-law living with him; and
by the son who steals a ring from his mother.
For purposes of Article 332(1) of the Revised Penal Code, we hold that the relationship
by affinity created between the surviving spouse and the blood relatives of the
deceased spouse survives the death of either party to the marriage which created the
60

affinity. (The same principle applies to the justifying circumstance of defense of ones
relatives under Article 11[2] of the Revised Penal Code, the mitigating circumstance of
immediate vindication of grave offense committed against ones relatives under Article
13[5] of the same Code and the absolutory cause of relationship in favor of accessories
under Article 20 also of the same Code.)

PEOPLE OF THE PHILIPPINES, plaintiff-appellee, vs. FLORENTINO EDUARTE @


Tinong, FREDESWINDO EDUARTE @ Indoy Acquitted, and JULIE EDUARTE
AT large. FLORENTINO EDUARTE @ Tinong, defendant-appellant.
G.R. No. 72976

|July 9, 1990|Third Division

GUTIERREZ, JR., J.:

FACTS: Florentino Eduarte, Fredeswindo Eduarte and Julie Eduarte are brothers and
residents of Dolores, Abra. Fredeswindo Eduarte married the sister of Roberto Trinidad.

On the night of December 4, 1984, Fredeswindo Eduarte, Roberto Trinidad and Sonny
Testado were in the house of Roberto Trinidad at Dolores, Abra drinking liquor.
Fredeswindo lost control of himself and suspecting that his wife had an illicit relation
with another man, he SavourSed with his wife. He became violent and made loud
outburst to the extent of getting a scissor inside the house. Seeing the intention of
Fredeswindo Eduarte, Roberto Trinidad pacified him. Instead of being appeased,
Fredeswindo Eduarte thrusted (sic) the scissor he was holding to his brother-in-law
Roberto Trinidad. Vicente Trinidad and his brothers went to the rescue of their brother
Roberto Trinidad and wrestled the scissor from Fredeswindo Eduarte. Thereafter,
Roberto Trinidad drove his jeep and together with Sonny Testado, went to the Municipal
Building of Dolores, Abra to seek the help of policemen. On their way back and while
approaching their house, Roberto Trinidad and Sonny Testado saw Fredeswindo
Eduarte lying flat on the road through the head lights of the jeep. Roberto Trinidad
stopped his jeep. Without putting off the light of the jeep, he alighted to help
Fredeswindo Eduarte. When Roberto Trinidad was in the act of extending assistance
and asking what happened to Fredeswindo Eduarte, Florentino Eduarte, who was
beside the road, shot him, which caused his death. Not long after, police authorities
arrived at the place and looked for Florentino Eduarte but failed to locate him.
61

RTC found Florentino guilty of the crime of murder, qualified by treachery. The accusedappellant maintains that he is innocent of the crime as charged invoking the justifying
circumstance of defense of relatives.
ISSUE: W/N the defense of relatives can be invoked by the accused in his Savour?
HELD: NO. Article II, subparagraph 2 of the Revised Penal Code provides that:
2. Anyone who acts in defense of the person or rights of his spouse, ascendants,
descendants, or legitimate, natural or adopted brothers or sisters, or of his relatives by
affinity in the same degrees and those by consanguinity within the fourth civil degree,
provided that the first and second requisites prescribed in the next preceding
circumstances are present, and the further requisite, in case the provocation was given
by the person attacked, that the one making defense had no part therein. (Emphasis
supplied)

The first and second requisites are unlawful aggression and reasonable necessity of the
means employed to prevent or repel the unlawful aggression. Hence, it was incumbent
upon the accused-appellant to prove the existence of the three essential requisites of
the justifying circumstance of defense of relatives namely: (1) unlawful aggression; (2)
reasonable necessity of the means employed to prevent or repel it; and (3) in case the
provocation was given by the person attacked, that the one making the defense had no
part therein.

We find no merit in the claim that the shooting of Roberto was done in defense of a
relative. For this justifying circumstance to prosper, the evidence adduced must be
persuasive. Although it is true that the accused-appellant took no part in the provocation
that led to the killing incident, his testimony that there was unlawful aggression on the
part of Roberto was self-serving and uncorroborated. The trial court discredited the
accused-appellants version and made the following observations:

The following circumstances render the pretension or claim of Florentino Eduarte


unreliable and unworthy of belief:

62

First, the position of the jeep in relation to Fredeswindo Eduarte. It is unrebutted that the
jeep was driven by Roberto Trinidad and stopped near Fredeswindo Eduarte who was
lying on the road. If Roberto Trinidad had any ill feelings towards his brother-in-law,
Fredeswindo Eduarte, he would not have stopped his jeep and alighted. It was not so
hard for him to accelerate the speed of his jeep and run over the body of Fredeswindo
Eduarte who was lying flat on the road, rather than alighting (sic) from his jeep and club
him.

Second, the headlights of the jeep remained lighted. If Roberto Trinidad had an intention
to club or to harm his inebriated and helpless brother-in-law lying on the road, he should
have put off the light of his jeep before alighting. His having alighted from his jeep
without putting off the light shows his intention of extending assistance to his brother-inlaw Fredeswindo Eduarte.

Third, failure of Florentino Eduarte to present himself to the authorities. It is clear that
immediately after having shot Roberto Trinidad, Florentino Eduarte ran away and never
presented himself to the authorities. If true that he shot Roberto Trinidad while in the act
of clubbing his brother Fredeswindo Eduarte, why did he not present himself to the
authorities and relate what happened? The natural instinct of any person who killed
someone in defense of his person or relative is to present himself to the authorities and
relate what happened to him and what he did.

Fourth, the immediate flee (sic) of Florentino Eduarte and Larry Baaga without
verifying the actual physical condition of Fredeswindo Eduarte. Larry Baaga and
Florentino Eduarte testified that they proceeded to the place of the incident to see
Fredeswindo Eduarte, that when they arrived, they saw Fredeswindo Eduardo lying on
the road. If it were true that they saw Roberto Trinidad club Fredeswindo Eduarte and
was in the act of clubbing when Florentino Eduarte shot him, why did they immediately
run away without extending any assistance to Fredeswindo Eduarte, or verifying his
actual condition? This action of Florentino Eduarte and Larry Baaga is strange and
demerits human experience. It is indicative of a guilty conscience.

63

Hence, for lack of a clear unlawful aggression on the part of the victim Roberto and of
the reasonable necessity of the means employed by the accused-appellant, the
justifying circumstance of defense of relative cannot be availed of.

(DEFENSE OF STRANGER)

PEOPLE OF THE PHILIPPINES, plaintiff-appellee,


vs. CRISPULO DIJAN Y MACAJIYA, accused-appellant.
G.R. No. 142682 |June 5, 2002|First Division
VITUG, J.:
FACTS: On the evening of April 11, 1998, Roderick Silvestre and Alvaro Hilario were at
a store to buy some cigarettes when they saw the group of Crispulo Dijan, Romualdo
Paglinawan and Oliver Lizardo, passing by the store. The two groups came to an
encounter when Romualdo Paglinawan suddenly confronted Alvaro Hilario for
purportedly giving him a "bad stare." Silvestre apologized to the group and, offering
them some cigarettes, explained that it was the natural way Hilario gazed at people.
Dijan, Paglinawan and Lizardo then left the place while Silvestre and Hilario who lived in
the same house proceeded home. While Silvestre and Hilario were walking, the three
accused, who apparently were waiting for the duo, suddenly ganged up on, and took
turns in stabbing, Hilario. At that point, Hilario, who was walking slightly ahead of
Silvestre, cried out and told the latter to flee. Silvestre ran away until he was able to
cling to a passing passenger jeepney.

Hilario was found to have sustained several stab wounds, punctured and incised
wounds, and abrasion in various parts of the body which caused his death. The medicolegal officer concluded that the wounds could have been inflicted by two assailants with
the use of two single-bladed weapons and an icepick.

In his defense, the accused contended that it was Hilario who initially stabbed
Paglinawan, and to help his friend, the accused then stabbed Hilario with the icepick.
RTC found the accused guilty beyond reasonable doubt of the crime of murder qualified
by treachery. Hence, this appeal.
ISSUE: W/N justifying circumstance of defense of stranger is applicable in this case.
64

HELD: NO. A party who invokes the justifying circumstance of "defense of a stranger"
has the burden of proving by clear and convincing evidence the exculpatory cause that
can save him from conviction. In order to successfully put up this defense an accused
must show (1) the existence of unlawful aggression on the part of the victim; (2) the
reasonable necessity of the means employed to prevent or repel it; and (3) that the
accused has not been induced by revenge, resentment, or other evil motive. The
unlawful aggression must be a continuing circumstance or must have been existing at
the time the defense is made. Once unlawful aggression is found to have ceased, the
one making the defense of a stranger would likewise cease to have any justification for
killing, or even just wounding, the former aggressor.
From the defense account, it would appear that Hilario was already disarmed and the
unlawful aggression by Hilario (if indeed he was the aggressor) to have by then been
abated, when accused-appellant still delivered the fatal thrusts on the victim.
Paglinawan himself testified:
"Q.
And because Crispulo Dijan was already able to take possession
of the weapon from Roderick Silvestre, you yourself was able to take
possession of the weapon from Hilario there was no more danger to you
as well as to Crispulo Dijan?
"A.
Yes, sir."
The number of wounds sustained by the victim would itself likewise negate accusedappellant's claim of defense of a stranger. The autopsy conducted on the corpse would
show that the deceased sustained fourteen injuries consisting of nine stab wounds,
three punctured wounds, an incised wound and an abrasion. Certainly, the nature and
number of wounds inflicted by an accused on the victim should be significant indicia in
determining the plausibility of the defense plea.
Supreme Court affirmed with modification the decision of the RTC. The accused is only
found guilty of homicide since it the evidence of the prosecution is found wanting in
respect to the qualifying circumstance of treachery.

PEOPLE OF THE PHILIPPINES, plaintiff-appellee,


vs. LUIS B. TORING DIOSDADO BERDON and CARMELO B. BERDON, accusedappellants.
G.R. No. L-56358 October 26, 1990|Third Division
FERNAN, C.J.

65

FACTS: In the evening of May 25, 1980, a benefit dance was held for the last
canvassing of votes for the candidates for princesses who would reign at the sitio fiesta.
Members of the kwaknit gang, headed by the accused Luis Toring, were present.
At around 10:45 PM, Samuel Augusto stepped out of the dancing area to answer the
call of nature. At the same time, Luis Toring and his group proceeded to the dark area
while whispering to each other. The accused then stabbed with the knife the right side of
Samuels abdomen. Upon being seen by a witness, the group then fled towards the
dark.
in his defense, Toring stated that it was Samuel who approached them and held one of
their members. Then Samuel thrust the butt of his shotgun on the chin of another
member, proceeded to another group who were also gangmates of Toring. Toring then
reacted to what he saw and got his kitchen knife, approached Samuel and stabbed him
twice.
RTC discredited the claim of Toring and found the accused guilty beyond reasonable
doubt of the crime of murder by direct participation as principal; Diosdado Berdon as
accomplice thereto and Carmelo Berdin as accessory after the fact.
Toring seeks his exoneration by contending that the assault was justified because he
acted in defense of his first cousin.
ISSUE: W/N the assault of the accused is justified since it was made in defense of his
relative.
HELD: NO. The appreciation of the justifying circumstance of defense of a relative,
however, hinges in this case on the presence of unlawful aggression on the part of the
victim. Corollarily, the claim of Toring that Samuel was, at the time of the assault,
carrying a shotgun to intimidate Toring's group must be proven.
The presence of unlawful aggression on the part of the victim and the lack of proof of
provocation on the part of Toring notwithstanding, full credence cannot be given, to
Toring's claim of defense of a relative. Toring himself admitted in court as well as in his
sworn statement that in 1979, he was shot with a .22 caliber revolver by Edgar Augusto,
Samuel's brother. It cannot be said, therefore, that in attacking Samuel, Toring was
impelled by pure compassion or beneficence or the lawful desire to avenge the
immediate wrong inflicted on his cousin. Rather, he was motivated by revenge,
resentment or evil motive because of a "running feud" between the Augusto and the
Toring brothers. As the defense itself claims, after the incident subject of the instant
case occurred, Toring's brother, Arsenio, was shot on the leg by Edgar Augusto. Indeed,
vendetta appears to have driven both camps to commit unlawful acts against each
other. Hence, under the circumstances, to justify Toring's act of assaulting Samuel
Augusto would give free rein to lawlessness.

66

The Supreme Court affirmed the decision of the lower court insofar as it convicts Luis
Toring as principal in the murder of Samuel Agusto and Diosdado Berdon as an
accomplice thereto.
(STATE OF NECESSITY)
VICKY C. TY, petitioner,
vs. PEOPLE OF THE PHILIPPINES, respondent.
G.R. No. 149275 |September 27, 2004|Second Division
TINGA, J.
FACTS: Petitioners mother Chua Lao So Un was confined at the Manila Doctors
Hospital (hospital) from 30 October 1990 until 4 June 1992. Being the patients
daughter, petitioner signed the "Acknowledgment of Responsibility for Payment" in the
Contract of Admission dated 30 October 1990. As of 4 June 1992, the Statement of
Account shows the total liability of the mother in the amount of P657,182.40. Petitioners
sister, Judy Chua, was also confined at the hospital from 13 May 1991 until 2 May 1992,
incurring hospital bills in the amount of P418,410.55. The total hospital bills of the two
patients amounted to P1,075,592.95.
On 5 June 1992, petitioner executed a promissory note wherein she assumed payment
of the obligation in installments. To assure payment of the obligation, she drew several
postdated checks against Metrobank payable to the hospital. The seven (7) checks,
each covering the amount of P30,000.00, were all deposited on their due dates. But
they were all dishonored by the drawee bank and returned unpaid to the hospital due to
insufficiency of funds, with the "Account Closed" advice. Soon thereafter, the
complainant hospital sent demand letters to Ty by registered mail. As the demand
letters were not heeded, complainant filed the seven (7) Informations subject of the
instant case.
The Court of Appeals affirmed the decision of the Regional Trial Court finding the
accused guilty of seven counts of violation of B.P. 22.
Petitioner interposed the defense that she issued the checks "under the impulse of an
uncontrollable fear of a greater injury or in avoidance of a greater evil or injury."
ISSUE: W/N the defense of uncontrollable fear is tenable to warrant exemption from
criminal liability.
HELD: NO. For this exempting circumstance to be invoked successfully, the following
requisites must concur: (1) existence of an uncontrollable fear; (2) the fear must be real
and imminent; and (3) the fear of an injury is greater than or at least equal to that
committed.
It must appear that the threat that caused the uncontrollable fear is of such gravity and
imminence that the ordinary man would have succumbed to it. It should be based on a
67

real, imminent or reasonable fear for ones life or limb. A mere threat of a future injury is
not enough. It should not be speculative, fanciful, or remote. A person invoking
uncontrollable fear must show therefore that the compulsion was such that it reduced
him to a mere instrument acting not only without will but against his will as well. It must
be of such character as to leave no opportunity to the accused for escape.
In this case, far from it, the fear, if any, harbored by Ty was not real and imminent. Ty
claims that she was compelled to issue the checks--a condition the hospital allegedly
demanded of her before her mother could be discharged--for fear that her mothers
health might deteriorate further due to the inhumane treatment of the hospital or worse,
her mother might commit suicide. This is speculative fear; it is not the uncontrollable
fear contemplated by law.
To begin with, there was no showing that the mothers illness was so life-threatening
such that her continued stay in the hospital suffering all its alleged unethical treatment
would induce a well-grounded apprehension of her death. Secondly, it is not the laws
intent to say that any fear exempts one from criminal liability much less petitioners
flimsy fear that her mother might commit suicide. In other words, the fear she invokes
was not impending or insuperable as to deprive her of all volition and to make her a
mere instrument without will, moved exclusively by the hospitals threats or demands.
Ty has also failed to convince the Court that she was left with no choice but to commit a
crime. She did not take advantage of the many opportunities available to her to avoid
committing one. By her very own words, she admitted that the collateral or security the
hospital required prior to the discharge of her mother may be in the form of postdated
checks or jewelry. And if indeed she was coerced to open an account with the bank and
issue the checks, she had all the opportunity to leave the scene to avoid involvement.
Moreover, petitioner had sufficient knowledge that the issuance of checks without funds
may result in a violation of B.P. 22. She even testified that her counsel advised her not
to open a current account nor issue postdated checks "because the moment I will not
have funds it will be a big problem." Besides, apart from petitioners bare assertion, the
record is bereft of any evidence to corroborate and bolster her claim that she was
compelled or coerced to cooperate with and give in to the hospitals demands.
The Supreme Court affirmed the decision of the Court of Appeals.

THE PEOPLE OF THE PHILIPPINES, plaintiff-appellee,


vs. PIO RICOHERMOSO, SEVERO PADERNAL, JUAN PADERNAL, ROSENDO
PERPEAN, MACARIO MONTEREY and RITO MONTEREY, defendants, JUAN
PADERNAL and SEVERO PADERNAL, defendants-appellants.
G.R. Nos. L-30527-28 March 29, 1974|Second Division
68

AQUINO, J.
FACTS: On January 30, 1965, Geminiano de Leon, who owned a parcel of land which
Pio Ricohermoso cultivated as kaingin, asked the latter about his share in the palay
harvest. Ricohermoso then instructed Geminiano to go to his house anytime and he
would give his share.
In the afternoon of the same day, Geminiano stopped by Ricohermosos place.
Geminiano sat on a sack beside Fabiana Rosales in front of the house while Marianito
de Leon, his son, stood behind. A .22 caliber rifle was slung on Marianitos right
shoulder. Ricohermoso stood near the door of his house while Severo Padernal was
stationed near the eaves of the house.
Geminiano asked Ricohermoso about the palay. The latter, no longer conciliatory and
evidently hostile, answered in a defiant tone: "Whatever happens, I will not give you
palay." Geminiano remonstrated: "Why did you tell us to pass by your house, if you were
not willing to give the palay?"
At that juncture, as if by pre-arrangement, Ricohermoso unsheathed his bolo and
approached Geminiano from the left, while Severo Padernal (Ricohermoso's father-inlaw) got an axe and approached Geminiano from the right. The latter looked up to the
sexagenarian Severo Padernal, with both hands raised and pleaded: "Mamay
(Grandpa), why will you do this to us. We will not fight you." While Geminiano was still
looking up to Severo Padernal on his right, Ricohermoso walked to Geminiano's left,
and, when about one meter from him, stabbed him on the neck with his bolo. Geminiano
fell face downward on the ground. While in that helpless position, he was hacked on the
back with an axe by Severo Padernal.
At that same place and time, while Severo Padernal and Ricohermoso were assaulting
Geminiano de Leon, another episode was taking place. Juan Padernal (Ricohermoso's
brother-in-law and the son of Severo) suddenly embraced Marianito de Leon from
behind, with his right arm locked around Marianito's neck and his left hand pressing
Marianito's left forearm. They grappled and rolled downhill towards a camote patch.
Marianito passed out. When he regained consciousness, his rifle was gone. He walked
uphill, saw his mortally wounded father Geminiano in his death throes, and embraced
him. He carried Geminiano for a short distance. The fifty-one year old Geminiano died
at two o'clock on that same day. The trial court found the Severo and Juan Padernal
guilty of the crime of murder.
ISSUE: W/N the act of preventing Marianito de Leon from shooting Ricohermoso and
Severo Padernal falls under the justifying circumstance of avoidance of a greater evil or
injury.
HELD: NO. His reliance on that justifying circumstance is erroneous. The act of Juan
Padernal in preventing Marianito de Leon from shooting Ricohermoso and Severo
69

Padernal, who were the aggressors, was designed to insure the killing of Geminiano de
Leon without any risk to his assailants.
Juan Padernal was not avoiding any evil when he sought to disable Marianito.
Padernal's malicious intention was to forestall any interference in the felonious assault
made by his father and brother-in-law on Geminiano. That situation is unarguably not
the case envisaged in paragraph 4 of article 11.
(FULFULLMENT OF DUTY/LAWFUL EXERCISE OF RIGHT)

RUPERTO A. AMBIL, JR., Petitioner,


vs. SANDIGANBAYAN and PEOPLE OF THE PHILIPPINES, Respondent.
G.R. No. 175457 |July 6, 2011|First Division
VILLARAMA, JR., J
x - - - - - - - - - - - - - - - - - - - - - - -x

ALEXANDRINO R. APELADO, SR., Petitioner,


vs. PEOPLE OF THE PHILIPPINES, Respondent.
G.R. No. 175482
FACTS: In an Information dated January 31, 2000, petitioners Ambil, Jr. and
Alexandrino R. Apelado, Sr. were charged with violation of Section 3(e) of R.A. No.
3019, together with SPO3 Felipe A. Balano. Upon reinvestigation, the Office of the
Ombudsman issued a Memorandum dated August 4, 2000, recommending the
dismissal of the complaint as regards Balano and the amendment of the Information to
include the charge of Delivering Prisoners from Jail under Article 156 of the Revised
Penal Code, as amended, (RPC) against the remaining accused. The Amended
Information reads:

That on or about the 6th day of September 1998, and for sometime prior [or]
subsequent thereto, [in] the Municipality of Borongan, Province of Eastern
Samar, Philippines, and within the jurisdiction of this Honorable Court, [the]
above-named accused, Ruperto A. Ambil, Jr.[,] being then the Provincial
70

Governor of Eastern Samar, and Alexandrino R. Apelado, being then the


Provincial Warden of Eastern Samar, both having been public officers, duly
elected, appointed and qualified as such, committing the offense in relation
to office, conniving and confederating together and mutually helping x x x
each other, with deliberate intent, manifest partiality and evident bad faith,
did then and there wilfully, unlawfully and criminally order and cause the
release from the Provincial Jail of detention prisoner Mayor Francisco
Adalim, accused in Criminal Case No. 10963, for Murder, by virtue of a
warrant of Arrest issued by Honorable Arnulfo P. Bugtas, Presiding Judge,
RTC-Branch 2, Borongan, Eastern Samar, and thereafter placed said
detention prisoner (Mayor Francisco Adalim) under accused RUPERTO A.
AMBIL, JR.s custody, by allowing said Mayor Adalim to stay at accused
Ambils residence for a period of Eighty-Five (85) days, more or less which
act was done without any court order, thus accused in the performance of
official functions had given unwarranted benefits and advantage to detainee
Mayor Francisco Adalim to the prejudice of the government.

Petitioner Ambil testified that it was upon the advice of Adalims lawyers that he directed
the transfer of Adalims detention to his home. He cites poor security in the provincial jail
as the primary reason for taking personal custody of Adalim considering that the latter
would be in the company of inmates who were put away by his sister and guards
identified with his political opponents.
The Sandiganbayan found petitioners guilty of violating Section 3(e) of RA 3019.
ISSUE: W/N petitioner is entitled to the justifying circumstance of fulfilment of a duty or
the lawful exercise of a right or office.
HELD: NO. Petitioner Ambil, Jr. invokes the justifying circumstance of fulfillment of duty
or lawful exercise of right or office. Under paragraph 5, Article 11 of the RPC, any
person who acts in the fulfillment of a duty or in the lawful exercise of a right or office
does not incur any criminal liability. In order for this justifying circumstance to apply, two
requisites must be satisfied: (1) the accused acted in the performance of a duty or in the
lawful exercise of a right or office; and (2) the injury caused or the offense committed be
the necessary consequence of the due performance of duty or the lawful exercise of
such right or office. Both requisites are lacking in petitioner Ambil, Jr.s case.
Section 3, Rule 114 of the Revised Rules of Criminal Procedure, as amended. Section
3, Rule 114 provides: SEC. 3. No release or transfer except on court order or bail.-No
person under detention by legal process shall be released or transferred except upon
order of the court or when he is admitted to bail.

71

Indubitably, the power to order the release or transfer of a person under detention by
legal process is vested in the court, not in the provincial government, much less the
governor. This was amply clarified by Asst. Sec. Ingeniero in his communication40
dated October 6, 1998 addressed to petitioner Ambil, Jr. Asst. Sec. Ingeniero.
As we have earlier determined, petitioner Ambil, Jr. exceeded his authority when he
ordered the transfer and detention of Adalim at his house. Needless to state, the
resulting violation of the Anti-Graft Law did not proceed from the due performance of his
duty or lawful exercise of his office.
The Supreme Court affirmed the decision of the Sandiganbayan.

RUFINO S. MAMANGUN, Petitioner,


vs. PEOPLE OF THE PHILIPPINES, Respondent.
G.R. No. 149152 February 2, 2007|First Division
GARCIA, J

FACTS: On July 31, 1992, a certain Liberty Contrerasa was heard shouting,
Magnanakaw... Magnanakaw. Several responded and thereupon chased the suspect
who entered the yard of Antonio Abacan and proceeded to the rooftop of the house.
Upon receiving a telephone call on the said robbery-holdup, three police officers,
including petitioner, were dispatched to the scene. They then went to the rooftop of the
house where the suspect was allegedly taking refuge. The three policemen, each armed
with a draw handgun, searched the rooftop and there they saw a man whom they
thought was the suspect, At the instance, petitioner, who was walking ahead of the
group, fired his handgun once, hitting the man, who was not the robbery suspect.The
Sandiganbayan found petitioner guilty of the crime of Homicide.
In his defense, petitioner insists that the shooting, which ultimately caused the demise
of Contreras, was justified because he was repelling Contreras unlawful attack on his
person, as Contreras was then about to strike him on the head with a steel pipe.
ISSUE: W/N petitioner is entitled to the justifying circumstance of fulfilment of a duty or
the lawful exercise of a right or office.
HELD: NO. The justifying circumstance of fulfillment of duty under paragraph 5, Article
II, of the Revised Penal Code may be invoked only after the defense successfully
proves that: (1) the accused acted in the performance of a duty; and (2) the injury
inflicted or offense committed is the necessary consequence of the due performance or
lawful exercise of such duty.
72

Concededly, the first requisite is present in this case. Petitioner, a police officer, was
responding to a robbery-holdup incident. His presence at the situs of the crime was in
accordance with the performance of his duty. However, proof that the shooting and
ultimate death of Contreras was a necessary consequence of the due performance of
his duty as a policeman is essential to exempt him from criminal liability.
As we see it, petitioners posturing that he shot Contreras because the latter tried to
strike him with a steel pipe was a mere afterthought to exempt him from criminal liability.
We see no plausible basis to depart from the Sandiganbayans findings that there was
no reason for the petitioner to shoot Contreras. The latter was unarmed and had already
uttered, "Hindi po ako, Hindi po ako" before the petitioner fatally shot him on the left
arm. Prosecution witness Ayson, who was then behind the petitioner when the latter
shot Contreras, testified that to the victims utterances, the petitioner even responded,
"Anong hindi ako," and immediately shot Contreras.8 As correctly observed by the
Sandiganbayan:
Besides being self-serving (with respect to the accused) and biased (with respect to his
co-policemen-witnesses), We find (1) the claim of the accused and his co-policemenwitnesses that the victim (Contreras) attacked the said accused and (2) their seemingly
"positive" identification of the stainless steel pipe (more of a rod) as his weapon, to be of
doubtful credibility.

EDUARDO L. BAXINELA, Petitioner-Appellant, vs.


THE PEOPLE OF THE PHILIPPINES, Respondent-Appellee.
G.R. No. 149652 l March 24, 2006 Second Division
AZCUNA, J

FACTS: In the evening of October 18, 1996, a minor altercation ensued between the
deceased Sgt. Lajo and another costumer at the pub where the two eventually able to
patch things up. Lajo was on his way out when petitioner Baxinela, a police officer,
followed the latter with his gun already drawn out. Then, from behind, Baxinela held
Lajos left arm and asked the latter if he was carrying a gun, to which Lajo responded
that he was an MIG. After that, an explosion was heard coming from Baxinelas gun.
Baxinela then got a gun from Lajos waist and handed it to Regimen. Afterwards
Baxinela held both of Lajos arms, who was still standing, and pushed him against the
wall and repeated his question. Lajo answered "Why did you shoot me? I am also a
military." At this point Lajo got out his wallet and gave it to Baxinela. Baxinela opened
the wallet and looked at an ID. Afterwards Baxinela and Regimen just left and did
nothing to aid Lajo. Alvarez and his fellow security guard, Rolando Gabriel, then picked
73

up Lajo and boarded him on a tricycle. RTC found petitioner Baxinela guilty of the crime
of homicide.
ISSUE: W/N petitioner is entitled to the justifying circumstance of fulfilment of a duty or
the lawful exercise of a right or office.
HELD: NO. In order to avail of this justifying circumstance it must be shown that: 1) the
accused acted in the performance of a duty or in the lawful exercise of a right or office;
and 2) the injury caused or the offense committed is the necessary consequence of the
due performance of duty or the lawful exercise of a right or office. While the first
condition is present, the second is clearly lacking. Baxinelas duty was to investigate the
reason why Lajo had a gun tucked behind his waist in a public place. This was what
Baxinela was doing when he confronted Lajo at the entrance, but perhaps through
anxiety, edginess or the desire to take no chances, Baxinela exceeded his duty by firing
upon Lajo who was not at all resisting. The shooting of Lajo cannot be considered due
performance of a duty if at that time Lajo posed no serious threat or harm to Baxinela or
to the civilians in the pub.
Essentially, Baxinela is trying to convince the Court that he should be absolved of
criminal liability by reason of a mistake of fact, a doctrine first enunciated in United
States v. Ah Chong. It was held in that case that a mistake of fact will exempt a person
from criminal liability so long as the alleged ignorance or mistake of fact was not due to
negligence or bad faith. In examining the circumstances attendant in the present case,
the Court finds that there was negligence on the part of Baxinela. Lajo, when he was
shot, was simply turning around to see who was accosting him. Moreover, he identified
himself saying "I am MIG." These circumstances alone would not lead a reasonable and
prudent person to believe that Baxinelas life was in peril. Thus, his act of shooting Lajo,
to the mind of this Court, constitutes clear negligence. But even if the Court assumes
that Lajos actions were aggressive enough to appear that he was going for his gun,
there were a number of procedures that could have been followed in order to avoid a
confrontation and take control of the situation. Baxinela, whom the Court assumes not
to be a rookie policeman, could have taken precautionary measures by simply
maintaining his hold on to Lajos shoulders, keeping Lajo facing away from him, forcing
Lajo to raise his hands and then take Lajos weapon. There was also Regimen who
should have assisted Baxinela in disabling and disarming Lajo. The events inside the
disco pub that unnecessarily cost the life of Lajo did not have to happen had Baxinela
not been negligent in performing his duty as a police officer.
The Court will, however, attribute to Baxinela the incomplete defense of fulfillment of a
duty as a privileged mitigating circumstance. In Lacanilao v. Court of Appeals, it was
held that if the first condition is fulfilled but the second is wanting, Article 69 of the
Revised Penal Code is applicable so that the penalty lower than one or two degrees
than that prescribed by law shall be imposed. Accordingly, the Court grants in favor of
Baxinela a privileged mitigating circumstance and lower his penalty by one degree. His
entitlement to the ordinary mitigating circumstance of voluntary surrender is also
recognized, thereby further reducing his penalty to its minimum.
74

The Court commiserates with our policemen who regularly thrust their lives in zones of
danger in order to maintain peace and order and acknowledges the apprehensions
faced by their families whenever they go on duty. But the use of unnecessary force or
wanton violence is not justified when the fulfillment of their duty as law enforcers can be
effected otherwise. A "shoot first, think later" attitude can never be countenanced in a
civilized society.
ROWENO POMOY, petitioner, vs. PEOPLE OF THE PHILIPPINES, respondent.
[G.R. No. 150647. September 29, 2004] |Third Division
PANGANIBAN, J
FACTS: On January 4, 1990, petitioner, a police officer, and Tomas Balboa, a suspect in
a robbery case, went to the investigation room for tactical interrogation. It was alleged
that when they reached the main building, Balboa attempted to grab the gun of
petitioner, two gun shots were heard. It was verified that petitioner was seen still holding
a .45 caliber pistol facing Balboa, who was lying in a pool of blood, about two feet away.
RTC found petitioner guilty of the crime of homicide. In his defense, petitioner contends
that the death of Balboa resulted from an accident.
ISSUE: W/N petitioner is entitled to the justifying circumstance of fulfilment of a duty or
the lawful exercise of a right or office.
HELD: YES. The elements of accident are as follows: 1) the accused was at the time
performing a lawful act with due care; 2) the resulting injury was caused by mere
accident; and 3) on the part of the accused, there was no fault or no intent to cause the
injury.1 From the facts, it is clear that all these elements were present. At the time of
the incident, petitioner was a member -- specifically, one of the investigators -- of the
Philippine National Police (PNP) stationed at the Iloilo Provincial Mobile Force
Company. Thus, it was in the lawful performance of his duties as investigating officer
that, under the instructions of his superior, he fetched the victim from the latters cell for
a routine interrogation.
Again, it was in the lawful performance of his duty as a law enforcer that petitioner tried
to defend his possession of the weapon when the victim suddenly tried to remove it
from his holster. As an enforcer of the law, petitioner was duty-bound to prevent the
snatching of his service weapon by anyone, especially by a detained person in his
custody. Such weapon was likely to be used to facilitate escape and to kill or maim
persons in the vicinity, including petitioner himself.
Petitioner cannot be faulted for negligence. He exercised all the necessary precautions
to prevent his service weapon from causing accidental harm to others. As he so
75

assiduously maintained, he had kept his service gun locked when he left his house; he
kept it inside its holster at all times, especially within the premises of his working area.
At no instance during his testimony did the accused admit to any intent to cause injury
to the deceased, much less kill him. Furthermore, Nicostrato Estepar, the guard in
charge of the detention of Balboa, did not testify to any behavior on the part of petitioner
that would indicate the intent to harm the victim while being fetched from the detention
cell.
The participation of petitioner, if any, in the victims death was limited only to acts
committed in the course of the lawful performance of his duties as an enforcer of the
law. The removal of the gun from its holster, the release of the safety lock, and the firing
of the two successive shots -- all of which led to the death of the victim -- were
sufficiently demonstrated to have been consequences of circumstances beyond the
control of petitioner. At the very least, these factual circumstances create serious doubt
on the latters culpability.

JOHN ANGCACO, petitioner,


vs. PEOPLE OF THE PHILIPPINES, respondent.
G.R. No. 146664 l February 28, 2002|Second Division
MENDOZA, J

FACTS: On September 25, 1980, petitioner and his co-accused were serving a warrant
of arrest on Restituto Bergante who was wanted in connection with a robbery case. It
was alleged that petitioner and his co-accused fired their guns and called out Restituto,
however Noe and his brother Noel Bergante answered that their father was not around.
When they went outside, Noe and his cousin, Freddie Ganancial were flanked by
petitioner. Then, shots rang out from the armalite of accused, as a result of which
Freddie turned around and dropped to the ground face down.
RTC found the petitioner guilty of the crime of murder. In his defense, petitioner claims
that the killing was done in fulfilment of a lawful duty.
ISSUE: W/N petitioner is entitled to the justifying circumstance of fulfilment of a duty or
the lawful exercise of a right or office.
HELD: NO. For this justifying circumstance to be appreciated, the following must be
established: (1) that the offender acted in the lawful exercise of a right or a duty; and (b)
that the injury or offense committed be the necessary consequence of the due
performance of such right or office.
76

In this case, the mission of petitioner and his colleagues was to effect the arrest of
Restituto Bergante. As Edep himself explained, the standard procedure in making an
arrest was, first, to identify themselves as police officers and to show the warrant to the
arrestee and to inform him of the charge against him, and, second, to take the arrestee
under custody. But, it was not shown here that the killing of Ganancial was in
furtherance of such duty. No evidence was presented by the defense to prove that
Ganancial attempted to prevent petitioner and his fellow officers from arresting Restituto
Bergante. There was in fact no clear evidence as to how Freddie Ganancial was shot.
Indeed, as already stated, any attempt by the victim to arrest the wanted person was
pointless as Restituto Bergante was not in his house. As regards the second requisite,
there can be no question that the killing of Freddie Ganancial was not a necessary
consequence of the arrest to be made on Restituto Bergante.
Reliance by the Court of Appeals on the case of People v. Oanis is misplaced. In Oanis,
the accused, who were police officers, shot and killed the victim under the erroneous
notion that the latter was the person they were charged to arrest. The Court held that
the first requisite - that the offenders acted in performance of a lawful duty - was present
because the offenders, though overzealous in the performance of their duty, thought
that they were in fact killing the man they have been ordered to take into custody dead
or alive. In this case, petitioner did not present evidence that he mistook Freddie
Ganancial for Restituto Bergante and, therefore, killed him (Ganancial) perhaps
because he placed the lives of the arresting officers in danger.
FRANCISCO T. SYCIP, JR., petitioner,
vs. COURT OF APPEALS and PEOPLE OF THE PHILIPPINES, respondents.
G.R. No. 125059|March 17, 2000| SecondDivision
QUISUMBING, J.

FACTS: Petitioner bought on instalment, a townhouse unit in Francel Realty


Corporation (FRC). Upon execution of the contract to sell, petitioner, as required, issued
to FRC 48 postdated checks, each in the amount of P9,304, covering 48 monthly
instalments.
After moving in his unit, petitioner complained to FRC regarding defects in the unit and
incomplete features of the townhouse project. Dissatisfied when FRC ignored his
complaint, petitioner served on FRC two notarial notices to the effect that he was
suspending his instalment payments on the payment pending compliance with the
project plans and specifications. The complaints were dismissed as to the defects, but
FRC was ordered by Housing and Land Use Regulatory Board (HLURB) to finish all
incomplete features of his townhouse. Petitioner appealed the dismissal.
77

Notwithstanding the notarial notices, FRC continued to present for encashment


petitioners postdated checks in its possession. Petitioner sent stop payment orders to
the bank. When FRC continued to present the other postdated checks to the bank as
the due date fell, the bank advised Sycip to close his checking account to avoid paying
bank charges every time he made a "stop payment" order on the forthcoming checks.
Due to the closure of petitioner's checking account, the drawee bank dishonored six
postdated checks. FRC filed a complaint against petitioner for violations of B.P. Blg. 22
involving said dishonored checks.
RTC found petitioner guilty beyond reasonable doubt of violating BP 22, the Bouncing
Checks Law. In his defense, petitioner argues that the court erred when it affirmed his
conviction considering that he had cause to stop payment of the checks issued to
respondent.
ISSUE: Whether or not the accused is criminally liable of the B.P. Blg. 22?

HELD: NO. The Bouncing Checks Law (B.P. No. 22), is violated when the following
elements are present: (1) the making, drawing and issuance of any check to apply for
account or for value; (2) the knowledge of the maker, drawer, or issuer that at the time
of issue he does not have sufficient funds in or credit with the drawee bank for the
payment of such check in full upon its presentment; and (3) the subsequent dishonor of
the check by the drawee bank for insufficiency of funds or credit or dishonor for the
same reason had not the drawer, without any valid cause, ordered the bank to stop
payment. In this case, although the first element of the offense exists, the other
elements have not been established beyond reasonable doubt. The second element
involves knowledge on the part of the issuer at the time of the checks issuance that he
did not have enough funds or credit in the bank for payment thereof upon its
presentment. B.P. No. 22 creates a presumption juris tantum that the second element
prima facie exists when the first and third elements of the offense are present. But such
evidence may be rebutted. If not rebutted or contradicted, it will suffice to sustain a
judgment in favor of the issue, which it supports. Such knowledge of the insufficiency of
petitioners funds is legally presumed from the dishonor of his checks for insufficiency
of funds. But such presumption cannot hold if there is evidence to the contrary. In this
case, the other party has presented evidence to contradict said presumption. Hence,
the prosecution is duty bound to prove every element of the offense charged, and not
merely rely on a rebuttable presumption.

78

What are involved in this case are postdated checks. Postdating simply means that on
the date indicated on its face, the check would be properly funded, not that the checks
should be deemed as issued only then. The checks were issued at the time of the
signing of the Contract to Sell in August 1989. However, there was no showing that at
the time said checks were issued, petitioner had knowledge that his deposit or credit in
the bank would be insufficient to cover them when presented for encashment. The
closure of petitioners Account No. 845515 with Citibank was not for insufficiency of
funds. It was made upon the advice of the drawee bank, to avoid payment of hefty bank
charges each time petitioner issued a stop payment order to prevent encashment of
postdated checks in private respondents possession. Said evidence contradicts the
prima facie presumption of knowledge of insufficiency of funds. But it establishes
petitioners state of mind at the time said checks were issued. Petitioner definitely had
no knowledge that his funds or credit would be insufficient when the checks would be
presented for encashment.

(OBEDIENCE TO AN ORDER)

RUPERTO A. AMBIL, JR., Petitioner,


vs. SANDIGANBAYAN and PEOPLE OF THE PHILIPPINES, Respondent.
G.R. No. 175457 |July 6, 2011|First Division
VILLARAMA, JR., J
x - - - - - - - - - - - - - - - - - - - - - - -x

ALEXANDRINO R. APELADO, SR., Petitioner,


vs. PEOPLE OF THE PHILIPPINES, Respondent.
79

G.R. No. 175482


FACTS: In an Information dated January 31, 2000, petitioners Ambil, Jr. and
Alexandrino R. Apelado, Sr. were charged with violation of Section 3(e) of R.A. No.
3019, together with SPO3 Felipe A. Balano. Upon reinvestigation, the Office of the
Ombudsman issued a Memorandum dated August 4, 2000, recommending the
dismissal of the complaint as regards Balano and the amendment of the Information to
include the charge of Delivering Prisoners from Jail under Article 156 of the Revised
Penal Code, as amended, (RPC) against the remaining accused. The Amended
Information reads:

That on or about the 6th day of September 1998, and for sometime prior [or]
subsequent thereto, [in] the Municipality of Borongan, Province of Eastern
Samar, Philippines, and within the jurisdiction of this Honorable Court, [the]
above-named accused, Ruperto A. Ambil, Jr.[,] being then the Provincial
Governor of Eastern Samar, and Alexandrino R. Apelado, being then the
Provincial Warden of Eastern Samar, both having been public officers, duly
elected, appointed and qualified as such, committing the offense in relation
to office, conniving and confederating together and mutually helping x x x
each other, with deliberate intent, manifest partiality and evident bad faith,
did then and there wilfully, unlawfully and criminally order and cause the
release from the Provincial Jail of detention prisoner Mayor Francisco
Adalim, accused in Criminal Case No. 10963, for Murder, by virtue of a
warrant of Arrest issued by Honorable Arnulfo P. Bugtas, Presiding Judge,
RTC-Branch 2, Borongan, Eastern Samar, and thereafter placed said
detention prisoner (Mayor Francisco Adalim) under accused RUPERTO A.
AMBIL, JR.s custody, by allowing said Mayor Adalim to stay at accused
Ambils residence for a period of Eighty-Five (85) days, more or less which
act was done without any court order, thus accused in the performance of
official functions had given unwarranted benefits and advantage to detainee
Mayor Francisco Adalim to the prejudice of the government.

Petitioner Ambil testified that it was upon the advice of Adalims lawyers that he directed
the transfer of Adalims detention to his home. He cites poor security in the provincial jail
as the primary reason for taking personal custody of Adalim considering that the latter
would be in the company of inmates who were put away by his sister and guards
identified with his political opponents. The Sandiganbayan found petitioners guilty of
violating Section 3(e) of RA 3019.

80

ISSUE: W/N petitioner is entitled to the justifying circumstance of obedience to an order


issued for some lawful purpose.
HELD: Under paragraph 6, Article 11 of the RPC, any person who acts in obedience to
an order issued by a superior for some lawful purpose does not incur any criminal
liability. For this justifying circumstance to apply, the following requisites must be
present: (1) an order has been issued by a superior; (2) such order must be for some
lawful purpose; and (3) the means used by the subordinate to carry out said order is
lawful. Only the first requisite is present in this case.
While the order for Adalims transfer emanated from petitioner Ambil, Jr., who was then
Governor, neither said order nor the means employed by petitioner Apelado, Sr. to carry
it out was lawful. In his capacity as the Provincial Jail Warden of Eastern Samar,
petitioner Apelado, Sr. fetched Mayor Adalim at the provincial jail and, unarmed with a
court order, transported him to the house of petitioner Ambil, Jr. This makes him liable
as a principal by direct participation under Article 17(1) of the RPC.
LUIS A. TABUENA, petitioner, vs. HONORABLE SANDIGANBAYAN, and THE
PEOPLE OF THE PHILIPPINES, respondents.
[G.R. No. 103501-03. February 17, 1997] En Banc
FRANCISCO, J
ADOLFO M. PERALTA, petitioner, vs. HON. SANDIGANBAYAN (First Division), and
THE PEOPLE OF THE PHILIPPINES, represented by the OFFICE OF THE SPECIAL
PROSECUTOR, respondents.
[G.R. No. 103507. February 17, 1997]
FACTS: Then President Marcos instructed petitioner Tabuena over the phone to pay
directly to the presidents office and in cash what the MIAA owes the Philippine National
Construction Corporation (PNCC), to which Tabuena replied, Yes, sir, will do it. About a
week later, Tabuena received from the private secretary of Marcos, a Presidential
Memorandum dated January 8, 1986, directing Tabuena to pay immediately the PNCC
the sum of P55 Million in cash as partial payment of MIAAs account, signed by then
president Marcos.
In a memo dated January 7, 1986 from Minister Ongpin for the President, the former
requested for the partial deferment of PNCCs advances for the MIA Development
Project. At the same time, PNCC has potential escalation claims amounting to P99
Million, however there has been no funding allocation for any of the escalation claims
due to budgetary constraints.
The MIA Project has been completed and operational as far back as 1982 and yet the
residual amounts due to PNCC have not yet been paid, resulting in undue burden to
PNCC due to additional cost of money to service of its obligations for this contract.

81

In obedience to President Marcos verbal instruction and memo, Tabuena, with the help
of Dabao and Peralta, caused the release of P55 Million of MIAA funds by means of
three withdrawals.
The disbursement of the P55 Million was, as described by Tabuena and Peralta
themselves, out of the ordinary and not based on the normal procedure. Not only
were there no vouchers prepared to support the disbursement, the P55 Million was paid
in cold cash. Also, no PNCC receipt for the P55 Million was presented.
The Sandiganbayan convicted petitioner of the crime of malversation under Article 17 of
the Revised Penal Code. The defense of Tabuena was that they acted in good faith.
Tabuena claimed that he was merely complying with the MARCOS Memorandum which
ordered him to forward immediately to the Office ofthe President P55 million in cash as
partial payment of MIAAs obligations to PNCC, and that he was of the belief that MIAA
indeed had liabilities to PNCC.
ISSUE: W/N petitioner is entitled to the justifying circumstance of obedience to an order
issued for some lawful purpose.
HELD: YES. First. Tabuena had no other choice but to make the withdrawals, for that
was what the MARCOS Memorandum required him to do. He could not be faulted if he
had to obey and strictly comply with the presidential directive, and to argue otherwise is
something easier said than done. Marcos was undeniably Tabuenas superior the
former being then the President of the Republic who unquestionably exercised control
over government agencies such as the MIAA and PNCC. In other words, Marcos had a
say in matters involving inter-government agency affairs and transactions, such as for
instance, directing payment of liability of one entity to another and the manner in which
it should be carried out. And as a recipient of such kind of a directive coming from the
highest official of the land no less, good faith should be read on Tabuenas compliance,
without hesitation nor any question, with the MARCOS Memorandum. Tabuena
therefore is entitled to the justifying circumstance of Any person who acts in obedience
to an order issued by a superior for some lawful purpose. The subordinate-superior
relationship between Tabuena and Marcos is clear. And so too, is the lawfulness of the
order contained in the MARCOS Memorandum, as it has for its purpose partial payment
of the liability of one government agency (MIAA) to another (PNCC).

THE PEOPLE OF THE PHILIPPINES, plaintiff-appellee,


vs. MANUEL BERONILLA, FILIPINO VELASCO, POLICARPIO PACULDO, and
JACINTO ADRIATICO, defendants-appellants.
G.R. No. L-4445 | February 28, 1955| En Banc
REYES, J.B.L., J
82

FACTS: Arsenio Borjal served as mayor during the Japanese occupation. Manuel
Beronilla, after being appointed as Military Mayor of La Paz, Abra, received a copy of a
memorandum issued by Lt. Col. Arnold authorizing all military mayors to appoint a hury
of 12 bolomen to try persons accused of treason, espionage or the aiding of the enemy.
He also received a list of all puppet officials of the province of Abra (Borjal included) with
a memorandum instructing all Military Mayors to investigate said persons and gather
against them complaints from people of the municipality.
Beronilla, upon the return of Borjal who left La Paz because of an attempt on his life,
placed Borjal under custody. Pursuant to his instructions, complaints were gathered, a
12-man jury was appointed, prosecutors and a clerk of the jury were assigned.
The trial lasted for 19 days and the jury found Borjal guilty on all counts (espionage,
aiding the enemy, abuse of authority). Death penalty was imposed. Beronilla forwarded
the records of the case to the Headquarters of the 15th Infantry for review.
The records were returned by Lt. Col. Arnold adding that the matter was best handled
by the La Paz Government and whatever disposition taken was approved.
Upon receipt of the letter, Beronilla then ordered the execution of Borjal. The execution
was reported to Col. Arnold and Beronilla received compliments based on the reply of
his superior. Two years after, those who were involved were indicted in the CFI of Abra
for murder for allegedly conspiring and confederating in the execution of Borjal. The
defendants were convicted, thus this appeal.
ISSUE: W/N defendant is entitled to the justifying circumstance of obedience to an
order issued for some lawful purpose.
HELD: YES. The records are ample to sustain the claim of the defense that the arrest,
prosecution and trial of Borjal were done pursuant to express orders of the 15th Infantry
HQ.
Actually, the conduct of the appellants does not dispose that these appellants were
impelled by malice (dolo). The arrest and trial of Borjal were made upon express orders
of the higher command; the appellants allowed Borjal to be defended by counsel, one of
them (attorney Jovito Barreras) chosen by Borjal's sister; the trial lasted nineteen (19)
days; it was suspended when doubts arose about its legality, and it was not resumed
until headquarters (then in Langangilang, Abra) authorized its resumption and sent an
observer (Esteban Cabanos, of the S-5) to the proceedings, and whose suggestions on
procedure were followed; and when the verdict of guilty was rendered and death
sentence imposed, the records were sent to Arnold's headquarters for review, and Borjal
was not punished until the records were returned eight days later with the statement of
Arnold that "whatever disposition you make of the case is hereby approved" (Exhibit 8),
which on its face was an assent to the verdict and the sentence. The lower Court, after
finding that the late Arsenio Borjal had really committed treasonable acts, (causing
soldiers and civilians to be tortured, and hidden American officers to be captured by the
83

Japanese) expressly declared that "the Court is convinced that it was not for political or
personal reason that the accused decided to kill Arsenio Borjal" (Decision, p. 9; Record,
p. 727).
It appearing that the charge is the heinous crime of murder, and that the accusedappellants acted upon orders, of a superior officers that they, as military subordinates,
could not question, and obeyed in good faith, without being aware of their illegality,
without any fault or negligence on their part, we can not say that criminal intent has
been established (U. S. vs. Catolico, 18 Phil., 507; Peo. vs. Pacana, 47 Phil., 48; Sent.
of the Tribunal Supremo of Spain, 3 July 1886; 7 January 1901; 24 March 1900; 21 Feb.
1921; 25 March 1929). Actus non facit reum nisi mens si rea.
To constitute a crime, the act must, except in certain crimes made such by
statute, be accompanied by a criminal intent, or by such negligence or
indifference to duty or to consequence, as, in law, is equivalent to criminal intent.
The maxim is, actus non facit reum, nisi mens rea-a crime is not committed if the
minds of the person performing the act complained of be innocent. (U. S. vs.
Catolico, 18 Phil., 507).
EXEMPTIING CIRCUMSTANCES
INSANITY/IMBECILITY
PEOPLE OF THE PHILIPPINES, Plaintiff-Appellee,
vs. ANICETO BULAGAO, Accused-Appellant.
G.R. No. 184757

October 5, 2011|First Division

LEONARDO-DE CASTRO, J

FACTS: AAA, an adopted child, was residing in the house of her sister at Lolomboy,
Bocaue, Bulacan. On June 17, 2000, at around 8:00 PM, AAA and her sister FFF were
sleeping in a room without a door. AAA was suddenly awakened when she felt
somebody enter the room. She recognized the accused-appellant as the intruder, and
saw that he was holding a knife. Accused-appellant poked the knife at AAAs neck,
causing her to freeze in fear. Accused-appellant removed AAAs clothes, and then his
own. Both AAA and accused-appellant were wearing t-shirt and shorts before the
undressing. Accused-appellant kissed her neck and inserted his penis into her vagina.
FFF woke up at this moment, but accused-appellant did not stop and continued raping
AAA for one hour.

84

On June 29, 2000, AAA was residing in the house of her sister, also located in
Lolomboy, Bocaue, Bulacan. At around 11:00 p.m. on that day, AAA was sleeping in the
second floor of the house, where there are no rooms. AAA was roused from her sleep
when accused-appellant was already undressing her. Accused-appellant removed his
shorts and inserted his penis into her vagina. AAA tried to resist, but accused-appellant
held her hands. Accused-appellant then touched her breasts and kissed her. Accusedappellant remained on top of her for half an hour.

AAA told her mother and her brother about the rape incidents but they did not believed
her. AAA filed a complaint against the accused. RTC found the accused guilty of the
crime of rape. The accused raised as a defense that he was suffering from mental
retardation.
ISSUE: W/N defendant is entitled to the exempting circumstance of insanity?
HELD: NO. We agree with the finding of the trial court that there was no proof that the
mental condition accused-appellant allegedly exhibited when he was examined by
Yolanda Palma was already present at the time of the rape incidents. Anyone who
pleads the exempting circumstance of insanity bears the burden of proving it with clear
and convincing evidence. Besides, this Court observes that neither the acts of the
accused-appellant proven before the court, nor his answers in his testimony, show a
complete deprivation of intelligence or free will. Insanity presupposes that the accused
was completely deprived of reason or discernment and freedom of will at the time of the
commission of the crime. Only when there is a complete deprivation of intelligence at
the time of the commission of the crime should the exempting circumstance of insanity
be considered.

PEOPLE OF THE PHILIPPINES, plaintiff-appellee,


vs. ENRICO A. VALLEDOR, accused-appellant.
G.R. No. 129291

July 3, 2002 |First Division

YNARES-SANTIAGO, J

FACTS: On March 6, 1991, Roger Cabiguen was in his house while working on a
lettering job inside his bedroom together with his first cousin, Elsa Rodriguez, and his
85

friends. Roger was working at his table and seated on his bed while Elsa was across
the table. Antonio was on the left side, while Simplicio was seated near the door, on the
right side of Roger.
All of a sudden, the accused entered the room, uttered Rogers nickname and
immediately attached him with a knife, but the former was able to parry the thrust and
was stabbed on the right forearm. Accused then stabbed Elsa on the chest and said that
Ako akabales den, Elsa (I had my revenge, Elsa). Thereafter, the accused fled. It was
later revealed that the accused also stabbed one of their neighbors. Elsa was declared
dead on arrival.
The accused raised the defense of insanity. RTC convicted the accused of the crimes
of murder, attempted murder and frustrated murder.
ISSUE: W/N defendant is entitled to the exempting circumstance of insanity?
HELD: In considering a plea of insanity as a defense, the starting premise is that the
law presumes all persons to be of sound mind. Otherwise stated, the law presumes all
acts to be voluntary, and it is improper to presume that acts were done unconsciously.

In People v. Estrada, it was held that:


In the eyes of the law, insanity exists when there is a complete deprivation of
intelligence in committing the act. Mere abnormality of the mental faculties will
not exclude imputability. The accused must be "so insane as to be incapable of
entertaining a criminal intent." He must be deprived of reason and act without the
least discernment because there is a complete absence of the power to discern
or a total deprivation of freedom of the will.

Since the presumption is always in favor of sanity, he who invokes insanity as an


exempting circumstance must prove it by clear and positive evidence. And the
evidence on this point must refer to the time preceding the act under prosecution
or to the very moment of its execution.

Insanity is evinced by a deranged and perverted condition of the mental faculties which
is manifested in language and conduct. An insane person has no full and clear
understanding of the nature and consequences of his acts. Hence, insanity may be
shown by the surrounding circumstances fairly throwing light on the subject, such as
86

evidence of the alleged deranged person's general conduct and appearance, his acts
and conduct consistent with his previous character and habits, his irrational acts and
beliefs, as well as his improvident bargains. The vagaries of the mind can only be
known by outward acts, by means of which we read thoughts, motives and emotions of
a person, and through which we determine whether the acts conform to the practice of
people of sound mind.

In the case at bar, accused-appellant failed to discharge the burden of overcoming the
presumption of sanity at the time of the commission of the crime. The following
circumstances clearly and unmistakably show that accused-appellant was not legally
insane when he perpetrated the acts for which he was charged: 1) Simplicio Yayen was
positioned nearest to accused-appellant but the latter chose to stab Roger and Elsa; 2)
Accused-appellant called out the nickname of Roger before stabbing him; 3) Simplicio
Yayen and Antonio Magbanua who were likewise inside the room were left unharmed;
4) Accused-appellant, a spurned suitor of Elsa, uttered the words, "Ako akabales den,
Elsa." (I had my revenge, Elsa) after stabbing her; and 5) Accused-appellant hurriedly
left the room after stabbing the victims.

Evidently, the foregoing acts could hardly be said to be performed by one who was in a
state of a complete absence of the power to discern. Judging from his acts, accusedappellant was clearly aware and in control of what he was doing as he in fact purposely
chose to stab only the two victims. Two other people were also inside the room, one of
them was nearest to the door where accused-appellant emerged, but the latter went for
the victims. His obvious motive of revenge against the victims was accentuated by
calling out their names and uttering the words, "I had my revenge" after stabbing them.
Finally, his act of immediately fleeing from the scene after the incident indicates that he
was aware of the wrong he has done and the consequence thereof.

Accused-appellant's acts prior to the stabbing incident to wit: crying; swimming in the
river with his clothes on; and jumping off the jeepney; were not sufficient to prove that
he was indeed insane at the time of the commission of the crime. As consistently held
by this Court, "A man may act crazy but it does not necessarily and conclusively prove
that he is legally so." Then, too, the medical findings showing that accused-appellant
was suffering from a mental disorder after the commission of the crime, has no bearing
on his liability. What is decisive is his mental condition at the time of the perpetration of
87

the offense. Failing to discharge the burden of proving that he was legally insane when
he stabbed the victims, he should be held liable for his felonious acts.

(MINORITY)
PEOPLE OF THE PHILIPPINES, Plaintiff-Appellee,
vs. HERMIE M. JACINTO, Accused-Appellant.
G.R. No. 182239|March 16, 2011|First Division
PEREZ, J
FACTS: On January 28, 2003, the victims father sent his daughter CCC to the store to
buy cigarette and the victim, AAA followed her older sister but did not return with the
latter. The father thought that she was left behind to watch television at another house.
A witness saw appellant with the victim later on, at the store where the latter was seated
in his lap. The victim testified that when she left the store with the accused, he had
carnal knowledge of her. She went home crying after the incident. The victims father
confronted the appellant and called the police.
RTC found the appellant guilty beyond reasonable doubt of the crime of rape committed
upon a 5-year old girl. The defense moved to reopen the trial for reception of newly
discovered evidence stating that appellant was apparently only seventeen years old
when the crime was committed.
ISSUE: W/N appellant is entitled to the exempting circumstance of minority.
HELD: NO. In the determination of the imposable penalty, the Court of Appeals correctly
considered Republic Act No. 9344 (Juvenile Justice and Welfare Act of 2006) despite
the commission of the crime three (3) years before it was enacted on 28 April 2006.
We recognize its retroactive application following the rationale elucidated in People v.
Sarcia:
[Sec. 68 of Republic Act No. 9344] allows the retroactive application of the Act to those
who have been convicted and are serving sentence at the time of the effectivity of this
said Act, and who were below the age of 18 years at the time of the commission of the
offense. With more reason, the Act should apply to this case wherein the
conviction by the lower court is still under review. (Emphasis supplied.)
Sec. 6 of Republic Act No. 9344 exempts a child above fifteen (15) years but below
eighteen (18) years of age from criminal liability, unless the child is found to have acted
with discernment, in which case, "the appropriate proceedings" in accordance with the
Act shall be observed.
88

We determine discernment in this wise:


Discernment is that mental capacity of a minor to fully appreciate the consequences of
his unlawful act. Such capacity may be known and should be determined by taking into
consideration all the facts and circumstances afforded by the records in each case.
xxx The surrounding circumstances must demonstrate that the minor knew what he was
doing and that it was wrong. Such circumstance includes the gruesome nature of the
crime and the minors cunning and shrewdness.
In the present case, we agree with the Court of Appeals that: "(1) choosing an isolated
and dark place to perpetrate the crime, to prevent detection[;] and (2) boxing the victim
xxx, to weaken her defense" are indicative of then seventeen (17) year-old appellants
mental capacity to fully understand the consequences of his unlawful action.

PEOPLE OF THE PHILIPPINES, Plaintiff-Appellee,


vs. HENRY ARPON y JUNTILLA, Accused-Appellant.
G.R. No. 183563

December 14, 2011|First Division

LEONARDO-DE CASTRO, J

FACTS: The prosecution presented the lone testimony of AAA, victim, to prove the
charges against the accused. AAA testified that she was born on November 1, 1987. In
one afternoon when she was only eight years old, she stated that the accused-appellant
raped her inside their house. She could not remember, though, the exact month and
date of the incident. The accused-appellant stripped off her shorts, panties and shirt and
went on top of her. The accused had carnal knowledge with the victim. AAA also
testified that the accused raped her again for five times in different nights in July 1999.
AAA further related that the accused raped her again twice in August 1999. AAA said
that she did not tell anybody about the rapes because the accused threatened to kill her
mother if she did. She only filed a complaint when he proceeded to also rape her
younger sister.
RTC and the Court of Appeals found the accused guilty beyond reasonable of one count
of statutory rape and seven counts of rape and was sentenced to suffer death penalty.
In his appeal, the accused insists that the court a quo erred in imposing the supreme
penalty of death considering that he was only 13 years old when the first incident of
rape happened.
ISSUE: W/N appellant is entitled to the exempting circumstance of minority on the first
incident of rape.
89

HELD: YES. SEC. 6. Minimum Age of Criminal Responsibility. A child fifteen (15)
years of age or under at the time of the commission of the offense shall be exempt from
criminal liability. However, the child shall be subjected to an intervention program
pursuant to Section 20 of the Act.

A child above fifteen (15) years but below eighteen (18) years of age shall likewise be
exempt from criminal liability and be subjected to an intervention program, unless
he/she has acted with discernment, in which case, such child shall be subjected to the
appropriate proceedings in accordance with this Act.
The exemption from criminal liability herein established does not include exemption
from civil liability, which shall be enforced in accordance with existing laws. (Emphases
ours.)

As held in Sierra, the above provision effectively modified the minimum age limit of
criminal irresponsibility in paragraphs 2 and 3 of the Revised Penal Code, as amended,
"i.e., from under nine years of age and above nine years of age and under fifteen (who
acted without discernment) - to fifteen years old or under and above fifteen but below
18 (who acted without discernment) in determining exemption from criminal liability."

Accordingly, for the first count of rape, which in the information in Criminal Case No.
2000-01-46 was allegedly committed in 1995, the testimony of the accused-appellant
sufficiently established that he was only 13 years old at that time. In view of the failure
of the prosecution to prove the exact date and year of the first incident of rape, i.e.,
whether the same occurred in 1995 or in 1998 as previously discussed, any doubt
therein "should be resolved in favor of the accused, it being more beneficial to the
latter." The Court, thus, exempts the accused-appellant from criminal liability for the first
count of rape pursuant to the first paragraph of Section 6 of Republic Act No. 9344. The
accused-appellant, nevertheless, remains civilly liable therefor.

NIEL F. LLAVE, Petitioner,


vs. PEOPLE OF THE PHILIPPINES, Respondent.
90

G.R. No. 166040

April 26, 2006|First Division


CALLEJO, SR., J

FACTS: On September 24, 2002, Debbielyn was ordered by her mother to bring home
the container with the unsold quail eggs in her mothers store. Debbielyn did as told and
went on her way. As she neared a vacant house, she saw petitioner, who suddenly
pulled her behind a pile of hollow blocks which was in front of the vacant house. She
resisted but to no avail. Petitioner ordered her to lie down on the cement. Petrified, she
complied. He removed her shorts and underwear then removed his own. He got on top
of her. She felt his penis being inserted into her vagina. He kissed her. She felt pain and
cried. She was sure there were passersby on the street near the vacant house at the
time.
It was then that Teofisto came out of their house and heard the girls cries. He rushed to
the place and saw petitioner on top of Debbielyn, naked from the waist down. Teofisto
shouted at petitioner, and the latter fled from the scene. Teofisto told Debbielyn to inform
her parents about what happened. She told her father about the incident. Her parents
later reported what happened to the police authorities. Debbielyn told the police that
petitioner was a bad boy because he was a rapist.

In his defense, petitioner testified and declared that he was a freshman at the Pasay
City South High School. He had been one of the three outstanding students in grade
school and received awards such as Best in Mathematics. He also finished a computer
course and received a Certificate of Completion from the Philippine Air Force
Management Information Center. He denied having raped the private complainant.

The Court found the Child in Conflict with the Law (CICL) Llava, guilty beyond
reasonable doubt, and crediting him with the special mitigating circumstance of minority.
The trial court declared that petitioner acted with discernment. In his appeal, petitioner
argues that he was only 12 years old at the time of the alleged rape incident; he is
presumed to have acted without discernment under paragraph 3 of Article 12 of the
Revised Penal Code.
ISSUE: W/N the accused who was a minor above 9 years but below 15 years of age at
the time of the crime acted with discernment.
HELD: YES. Discernment, as used in Article 12(3) of the Revised Penal Code is defined
as follows: "the discernment that constitutes an exception to the exemption from
91

criminal liability of a minor under fifteen (15) years of age but over nine (9), who
commits an act prohibited by law, is his mental capacity to understand the difference
between right and wrong" (People v. Doquena, 68 Phil. 580 [1939]). For a minor above
nine but below fifteen years of age, he must discern the rightness or wrongness of the
effects of his act (Guevarra v. Almodova, G.R. No. 75256, January 26, 1989).

Professor Ambrocio Padilla, in his annotation of Criminal Law (p. 375, 1998 Ed.), writes
that "discernment is more than the mere understanding between right and wrong.
Rather, it means the mental capacity of a minor between 9 and 15 years of age to fully
appreciate the consequences of his unlawful act" (People v. Navarro, [CA] [51 O.G.
4062]). Hence, in judging whether a minor accused acted with discernment, his mental
capacity to understand the difference between right and wrong, which may be known
and should be determined by considering all the circumstances disclosed by the record
of the case, his appearance, his attitude and his behavior and conduct, not only before
and during the commission of the act, but also after and even during the trial should be
taken into consideration (People v. Doquena, supra).

In the instant case, petitioners actuations during and after the rape incident, as well as
his behavior during the trial showed that he acted with discernment.

The fact appears undisputed that immediately after being discovered by the
prosecutions witness, Teofisto Bucud, petitioner immediately stood up and ran away.
Shortly thereafter, when his parents became aware of the charges against him and that
private complainants father was looking for him, petitioner went into hiding. It was not
until the Barangay Tanod came to arrest him in his grandmothers house that petitioner
came out in the open to face the charges against him. His flight as well as his act of
going into hiding clearly conveys the idea that he was fully aware of the moral depravity
of his act and that he knew he committed something wrong. Otherwise, if he was indeed
innocent or if he was not least aware of the moral consequences of his acts, he would
have immediately confronted private complainant and her parents and denied having
sexually abused their daughter.

During the trial, petitioner submitted documentary evidence to show that he was a
consistent honor student and has, in fact, garnered several academic awards. This
92

allegation further bolstered that he acted with discernment, with full knowledge and
intelligence. The fact that petitioner was a recipient of several academic awards and
was an honor student further reinforces the finding that he was possessed of
intelligence well beyond his years and thus was able to distinguish, better than other
minors of his age could, which conduct is right and which is morally reprehensible.
Hence, although appellant was still a minor of twelve years of age, he possessed
intelligence far beyond his age. It cannot then be denied that he had the mental capacity
to understand the difference between right and wrong. This is important in cases where
the accused is minor. It is worthy to note that the basic reason behind the enactment of
the exempting circumstances under Article 12 of the Revised Penal Code is the
complete absence of intelligence, freedom of action, or intent on the part of the
accused. In expounding on intelligence as the second element of dolus, the Supreme
Court has stated: "The second element of dolus is intelligence; without this power,
necessary to determine the morality of human acts to distinguish a licit from an illicit act,
no crime can exist, and because the infant has no intelligence, the law exempts (him)
from criminal liability" (Guevarra v. Aldomovar, 169 SCRA 476 [1989], at page 482).

ALVIN JOSE, petitioner,


vs. PEOPLE OF THE PHILIPPINES, respondent.
G.R. No. 162052, January 13, 2005| Second Division
CALLEJO, SR., J
FACTS: On November 14, 1995, P/Supt. Joseph Castro received an information from
an unnamed information that a big time group of drug pushers from Greenhills will
deliver 100 grams of shabu at Chowking Restaurant in Laguna. Acting on such report,
SPO1 Guevarra was assigned as poseur-buyer and SPO2 Manglo and Luna, other
members of the team Manglo and Luna arrived at the restaurant and position their cars
at the parking area where they had a commanding view of people going in and out.
Later in the afternoon, a car arrived with Sonny Zarraga as the driver and petitioner
Alvin Jose. The unnamed informant approached and talked to Zarraga. Then, the
informant called SPO1 Guevarra and informed the latter that Zarraga had with him 100
grams of shabu. SPO1 Guevarra offered to buy the shabu. Zarraga asked SPO1
Guevarra if he had the money to buy 100 grams of shabu. Guevarra responded in the
affirmative. He showed the aforecited bundle of "money bills." Zarraga then asked Alvin
Jose to bring out the shabu and handover (sic) to SPO1 Guevarra; the latter, in turn,
handed the bundle of "money bills."
93

Guevarra scratched his head, the pre-arranged signal to signify that the transaction was
consummated Immediately thereafter, William Manglo and Wilfredo Luna approached
and introduced themselves as Narcom Operatives. They arrested Zarraga and
petitioner Alvin Jose. The buy-bust bundle of "money bills" and the shabu were
recovered. The two were brought to Camp Vicente Lim for investigation. Edgar Groyon
conducted the investigation.
RTC convicted the accused therein of violation of Section 21(b), Article VI in relation to
Section 29, Article of RA 6425, as amended.
ISSUE: W/N the accused who was a minor above 9 years but below 15 years of age at
the time of the crime acted with discernment.
HELD: NO. Under Article 12(3) of the Revised Penal Code, a minor over nine years of
age and under fifteen is exempt from criminal liability if charged with a felony. The law
applies even if such minor is charged with a crime defined and penalized by a special
penal law. In such case, it is the burden of the minor to prove his age in order for him to
be exempt from criminal liability. The reason for the exemption is that a minor of such
age is presumed lacking the mental element of a crime the capacity to know what is
wrong as distinguished from what is right or to determine the morality of human acts;
wrong in the sense in which the term is used in moral wrong. However, such
presumption is rebuttable. For a minor at such an age to be criminally liable, the
prosecution is burdened to prove beyond reasonable doubt, by direct or circumstantial
evidence, that he acted with discernment, meaning that he knew what he was doing and
that it was wrong. Such circumstantial evidence may include the utterances of the
minor; his overt acts before, during and after the commission of the crime relative
thereto; the nature of the weapon used in the commission of the crime; his attempt to
silence a witness; his disposal of evidence or his hiding the corpus delicti.

In the present case, the prosecution failed to prove beyond reasonable doubt that the
petitioner, who was thirteen (13) years of age when the crime charged was committed,
acted with discernment relative to the sale of shabu to the poseur-buyer. Moreover, its
cross-examination of Rene did not, in any way, attempt to show his discernment. He
was merely asked about what he knew of the incident that transpired on 16 April 1991
and whether he participated therein. Accordingly, even if he was, indeed, a coconspirator, he would still be exempt from criminal liability as the prosecution failed to
rebut the presumption of non-discernment on his part by virtue of his age. The crossexamination of Rene could have provided the prosecution a good occasion to extract
from him positive indicators of his capacity to discern. But, in this regard, the
government miserably squandered the opportunity to incriminate him.
(ACCIDENT)
94

VIRGILIO TALAMPAS y MATIC, Petitioner,


vs. PEOPLE OF THE PHILIPPINES, Respondent.
G.R. No. 180219| November 23, 2011|First Division
BERSAMIN, J

FACTS: On July 5, 1995, Jose Sevillo together with Eduardo Matic (Eduardo) and
Ernesto Matic (Ernesto) were in front of his house along the road, repairing his tricycle
when he noticed the appellant who was riding on a bicycle passed by and stopped. The
latter alighted at about three (3) meters away from him, walked a few steps and brought
out a short gun, a revolver, and poked the same to Eduardo and fired it hitting Eduardo
who took refuge behind Ernesto. The appellant again fired his gun three (3) times, one
shot hitting Ernesto at the right portion of his back causing him (Ernesto) to fall on the
ground with his face down. Another shot hit Eduardo on his nape and fell down on his
back (patihaya). Thereafter, the appellant ran away, while he (Jose) and his neighbors
brought the victims to the hospital. On June 6, 1995, Jose executed a Sworn Statement
at the Bian Police Station.
On his part, petitioner interposed self-defense and accident. RTC found petitioner guilty
beyond reasonable doubt of the crime of homicide, which was affirmed by the Court of
Appeals.
ISSUE: W/N defendant is entitled to the exempting circumstance of accidental death?
HELD: NO. Talampas could not relieve himself of criminal liability by invoking accident
as a defense. Article 12(4) of the Revised Penal Code,the legal provision pertinent to
accident, contemplates a situation where a person is in fact in the act of doing
something legal, exercising due care, diligence and prudence, but in the process
produces harm or injury to someone or to something not in the least in the mind of the
actor an accidental result flowing out of a legal act. Indeed, accident is an event that
happens outside the sway of our will, and although it comes about through some act of
our will, it lies beyond the bounds of humanly foreseeable consequences. In short,
accident presupposes the lack of intention to commit the wrong done.

The records eliminate the intervention of accident. Talampas brandished and poked his
revolver at Eduardo and fired it, hitting Eduardo, who quickly rushed to seek refuge
behind Ernesto. At that point, Talampas fired his revolver thrice. One shot hit Ernesto at
95

the right portion of his back and caused Ernesto to fall face down to the ground. Another
shot hit Eduardo on the nape, causing Eduardo to fall on his back. Certainly, Talampas
acts were by no means lawful, being a criminal assault with his revolver against both
Eduardo and Ernesto.

NOE TOLEDO y TAMBOONG, petitioner,


vs. PEOPLE OF THE PHILIPPINES, respondent.
G.R. No. 158057|September 24, 2004| Second Division
CALLEJO, SR., J.:
FACTS: On September 16, 1995, appellant went to a black-smith who made the design
of his bolo. When he went home to Tuburan, Odiongan, Romblon late in the afternoon,
appellant saw the group of Lani Famero, Michael Fosana, Rex Cortez and Ricky Guarte
drinking gin at the house of the Spouses Manuel and Eliza Guarte, Rickys parents.
Appellants house is about five (5) meters away from the house of Spouses Guarte.
Appellant requested the group of Ricky to refrain from making any noise. Thereupon,
appellant proceeded inside his house and went to sleep.
Around 9:00 p.m., Gerardo Faminia, Eliza Guartes brother went home accompanied by
Ricky. Gerardos home is about twelve (12) meters away from the Guarte home.
Minutes later, Ricky came back and together with Lani, Rex and Michael, went to sleep
at the Guarte house. They had not laid down for long when they heard stones being
hurled at the roof of the house. The stoning was made three (3) times. Ricky rose from
bed and peeped through a window. He saw appellant stoning their house. Ricky went
out of the house and proceeded to appellants house. Ricky asked appellant, his uncle,
why he was stoning their house. Appellant did not answer but met Ricky at the doorstep
of his (appellants) house and, without any warning, stabbed Ricky on the abdomen with
a bolo. Eliza had followed his son Ricky and upon seeing that Ricky was stabbed,
shouted for help. Lani heard Elizas cry for help and immediately rushed outside the
house. Lani saw Ricky leaning on the ground and supporting his body with his hands.
Lani helped Ricky stand up and brought him to the main road. Lani asked Ricky who
stabbed him and Ricky replied that it was appellant who stabbed him. Then Docloy
Cortez arrived at the scene on board his tricycle. Accordingly, Ricky was put on the
tricycle and taken to the hospital.
RTC convicted the petitioner of homicide, which was affirmed on appeal in the Court of
Appeals. Petitioner invoked self-defense and accidental death.
ISSUE: W/N defendant is entitled to the exempting circumstance of accidental death?
HELD: NO. It is an aberration for the petitioner to invoke the two defenses at the same
time because the said defenses are intrinsically antithetical. There is no such defense
as accidental self-defense in the realm of criminal law.

96

Self-defense under Article 11, paragraph 1 of the Revised Penal Code necessarily
implies a deliberate and positive overt act of the accused to prevent or repel an unlawful
aggression of another with the use of reasonable means. The accused has freedom of
action. He is aware of the consequences of his deliberate acts. The defense is based on
necessity which is the supreme and irresistible master of men of all human affairs, and
of the law. From necessity, and limited by it, proceeds the right of self-defense. The right
begins when necessity does, and ends where it ends. Although the accused, in fact,
injures or kills the victim, however, his act is in accordance with law so much so that the
accused is deemed not to have transgressed the law and is free from both criminal and
civil liabilities.On the other hand, the basis of exempting circumstances under Article 12
of the Revised Penal Code is the complete absence of intelligence, freedom of action,
or intent, or the absence of negligence on the part of the accused. The basis of the
exemption in Article 12, paragraph 4 of the Revised Penal Code is lack of negligence
and intent. The accused does not commit either an intentional or culpable felony. The
accused commits a crime but there is no criminal liability because of the complete
absence of any of the conditions which constitute free will or voluntariness of the act. An
accident is a fortuitous circumstance, event or happening; an event happening wholly or
partly through human agency, an event which under the circumstances is unusual or
unexpected by the person to whom it happens.
Self-defense, under Article 11, paragraph 1, and accident, under Article 12, paragraph 4
of the Revised Penal Code, are affirmative defenses which the accused is burdened to
prove, with clear and convincing evidence. Such affirmative defenses involve questions
of facts adduced to the trial and appellate courts for resolution. By admitting killing the
victim in self-defense or by accident without fault or without intention of causing it, the
burden is shifted to the accused to prove such affirmative defenses. He should rely on
the strength of his own evidence and not on the weakness of that of the prosecution. If
the accused fails to prove his affirmative defense, he can no longer be acquitted.
First, if the testimony of the petitioner is to be believed, the force of the struggle
between him and the victim would have caused the door to fall on the petitioner.
However, the petitioner failed to adduce real evidence that the door of his house was
destroyed and that he sustained any physical injuries, considering that he was only five
inches away from the door. Second, if the door fell to the sala of the house of the
petitioner, the victim must have fallen on top of the door. It is incredible that the bolo of
the petitioner could have hit the stomach of the victim. The claim of the petitioner that he
managed to step aside and avoid being crushed by the door belies his claim that the
bolo accidentally hit the victim on the stomach. Third, when he surrendered to the
barangay captain and to the policemen, he failed to relate to them that his bolo
accidentally hit the stomach of the victim. Fourth, there is no evidence that the petitioner
surrendered either the bolo that accidentally hit the victim or the balisong held by the
deceased to the barangay captain or the police authorities. Such failure of the petitioner
negates his claim that his bolo accidentally hit the stomach of the victim and that he
acted in self-defense.

97

PEOPLE OF THE PHILIPPINES, plaintiff-appellee,


vs. SPO1 RODOLFO CONCEPCION y PERALTA, accused-appellant.
G.R. No. 136844 August 1, 2002| Second Division
QUISUMBING, J.:

FACTS: On November 24, 1997, Lorenzo Galang got involved in a quarrel at the town
plaza. He was brought to the barangay hall for questioning by Barangay Captain
Remigio Capitli.

Shortly after, appellant Rodolfo Concepcion arrived and fired his rifle twice or thrice past
the ears of Lorenzo, who was then sitting, but without injuring him. After that, however,
appellant thrust the barrel of the gun against the abdomen of Lorenzo. Then there was
an explosion. Lorenzo was shot in the thigh. At least three more shots were fired, hitting
Lorenzo in the chest. According to Sison and Yarte, appellant shot Lorenzo deliberately.
Lorenzo died instantly.

In his defense, appellant RODOLFO CONCEPCION claimed that the shooting was only
accidental. According to him, he was investigating Lorenzo for the latters disorderly
behavior at the town plaza when it happened. He said Lorenzo appeared drunk and
unruly, and even verbally challenged him to fight. At this juncture, according to
appellant, he fired two shots in the air, but Lorenzo grabbed the barrel of his gun. The
gun accidentally fired and Lorenzo was hit. RTC found appellant guilty of murder and
sentenced him to reclusion perpetua.
ISSUE: W/N defendant is entitled to the exempting circumstance of accidental death
while performing a lawful act?
HELD: NO. Under Article 12 (4) of the Revised Penal Code, among those exempted
from criminal liability is: Any person who, while performing a lawful act with due care,
causes an injury by mere accident without fault or intention of causing it.

Well settled is the rule in criminal cases, that the prosecution has the burden of proof to
establish the guilt of the accused. However, once the defendant admits the commission
of the offense charged, but raises an exempting circumstance as a defense, the burden
of proof is shifted to him. By invoking mere accident as a defense, appellant now has
98

the burden of proving that he is entitled to that exempting circumstance under Article 12
(4) of the Code.

The existence of accident must be proved by the appellant to the satisfaction of the
court. For this to be properly appreciated in appellants favor, the following requisites
must concur: (1) that the accused was performing a lawful act with due care; (2) that the
injury is caused by mere accident; and (3) that there was no fault or intent on his part to
cause the injury. Appellant must convincingly prove the presence of these elements in
order to benefit from the exempting circumstance of accident. However, his defense
utterly failed to discharge this burden. Thus, we find no reversible error in the judgment
of the trial court.
By appellants own testimony, the victim was unarmed. In contrast, appellant had an
armalite and a handgun. It is highly inconceivable that an unarmed man could pose
bodily harm to another who is heavily armed.

(IRRESISTABLE FORCE/UNCONTROLLABLE CAUSE)


VICKY C. TY, petitioner,
vs. PEOPLE OF THE PHILIPPINES, respondent.
G.R. No. 149275 |September 27, 2004|Second Division
TINGA, J.
FACTS: Petitioners mother Chua Lao So Un was confined at the Manila Doctors
Hospital (hospital) from 30 October 1990 until 4 June 1992. Being the patients
daughter, petitioner signed the "Acknowledgment of Responsibility for Payment" in the
Contract of Admission dated 30 October 1990. As of 4 June 1992, the Statement of
Account shows the total liability of the mother in the amount of P657,182.40. Petitioners
sister, Judy Chua, was also confined at the hospital from 13 May 1991 until 2 May 1992,
incurring hospital bills in the amount of P418,410.55. The total hospital bills of the two
patients amounted to P1,075,592.95.
On 5 June 1992, petitioner executed a promissory note wherein she assumed payment
of the obligation in installments. To assure payment of the obligation, she drew several
postdated checks against Metrobank payable to the hospital. The seven (7) checks,
each covering the amount of P30,000.00, were all deposited on their due dates. But
they were all dishonored by the drawee bank and returned unpaid to the hospital due to
insufficiency of funds, with the "Account Closed" advice. Soon thereafter, the
complainant hospital sent demand letters to Ty by registered mail. As the demand
99

letters were not heeded, complainant filed the seven (7) Informations subject of the
instant case.
RTC found the petitioner guilty of seven counts of violation of BP 22 otherwise known as
the Bouncing Checks Law. The decision was affirmed by the Court of Appeals, hence
this petition for review.
Petitioner interposed the defense that she issued the checks "under the impulse of an
uncontrollable fear of a greater injury or in avoidance of a greater evil or injury."
ISSUE: W/N the defense of uncontrollable fear is tenable to warrant exemption from
criminal liability.
HELD: NO. For this exempting circumstance to be invoked successfully, the following
requisites must concur: (1) existence of an uncontrollable fear; (2) the fear must be real
and imminent; and (3) the fear of an injury is greater than or at least equal to that
committed.
It must appear that the threat that caused the uncontrollable fear is of such gravity and
imminence that the ordinary man would have succumbed to it. It should be based on a
real, imminent or reasonable fear for ones life or limb. A mere threat of a future injury is
not enough. It should not be speculative, fanciful, or remote. A person invoking
uncontrollable fear must show therefore that the compulsion was such that it reduced
him to a mere instrument acting not only without will but against his will as well. It must
be of such character as to leave no opportunity to the accused for escape.
In this case, far from it, the fear, if any, harbored by Ty was not real and imminent. Ty
claims that she was compelled to issue the checks--a condition the hospital allegedly
demanded of her before her mother could be discharged--for fear that her mothers
health might deteriorate further due to the inhumane treatment of the hospital or worse,
her mother might commit suicide. This is speculative fear; it is not the uncontrollable
fear contemplated by law.
To begin with, there was no showing that the mothers illness was so life-threatening
such that her continued stay in the hospital suffering all its alleged unethical treatment
would induce a well-grounded apprehension of her death. Secondly, it is not the laws
intent to say that any fear exempts one from criminal liability much less petitioners
flimsy fear that her mother might commit suicide. In other words, the fear she invokes
was not impending or insuperable as to deprive her of all volition and to make her a
mere instrument without will, moved exclusively by the hospitals threats or demands.
Ty has also failed to convince the Court that she was left with no choice but to commit a
crime. She did not take advantage of the many opportunities available to her to avoid
committing one. By her very own words, she admitted that the collateral or security the
hospital required prior to the discharge of her mother may be in the form of postdated
100

checks or jewelry. And if indeed she was coerced to open an account with the bank and
issue the checks, she had all the opportunity to leave the scene to avoid involvement.
Moreover, petitioner had sufficient knowledge that the issuance of checks without funds
may result in a violation of B.P. 22. She even testified that her counsel advised her not
to open a current account nor issue postdated checks "because the moment I will not
have funds it will be a big problem." Besides, apart from petitioners bare assertion, the
record is bereft of any evidence to corroborate and bolster her claim that she was
compelled or coerced to cooperate with and give in to the hospitals demands.
The Supreme Court affirmed the decision of the Court of Appeals.

PEOPLE OF THE PHILIPPINES, Plaintiff-Appellee,


vs. NELIDA DEQUINA Y DIMAPANAN, JOSELITO JUNDOC Y JAPITANA & NORA
JINGABO Y CRUZ, Accused-Appellants.
G.R. No. 177570| January 19, 2011|First Division
LEONARDO-DE CASTRO, J.:

FACTS: On September 29, 1999, he and SPO1 Anthony Blanco were instructed by their
superior, Chief Inspector Romulo Sapitula to proceed at the corner of Juan Luna and
Raxabago Sts., Tondo, Manila, where, according to the report given by the informant,
three persons a male and two female[s] would be coming from Baguio City to deliver
unknown quantity of marijuana.

At around 9:00 a.m., they noticed a taxi cab coming from Yuseco St. heading towards
the direction of the pier. From the position of their patrol car, a taxi stopped, from it
emerged three passengers a man and two women each one of them carrying a
black travelling bag, who fitted the descriptions given to them by Inspector Sapitula,
they intently watched and monitored their movements.

About one or two minutes later, as the trio started walking towards the western portion
of Raxabago St., they drove and trailed them. As the patrol car got closer behind them,
[Dequina] noticed its presence. She started walking in a more hurried pace as if she
wanted to run away. SPO1 Blanco alighted from the car and chased [Dequina] while
101

PO3 Masanggue, who was behind the wheels also alighted and restrained [Jundoc] and
[Jingabo]. While thus trying to get away, [Dequina] dropped the bag she was carrying.
As a result, the zipper of the bag gave way. Bundles of dried leaves wrapped in
transparent plastic bags case into view. Suspecting the stuffs to be marijuana, they
further inspected the other two bags in the possession of [Jingabo] and [Jundoc] and
found out that they had the same contents. They boarded the three accused, along with
their bags in their patrol car and proceeded to the hospital for physical examination
before bringing them to their headquarters.
RTC found the accused-appelllants guilty of the crime of illegal transport of
marijuana. In his defense, Dequina contends that she transported the marijuana under
the compulsion of an irresistible fear.
ISSUE: W/N defendant is entitled to the exempting circumstance of compulsion of an
irresistible fear.
HELD: NO. A person who acts under the compulsion of an irresistible force, like one
who acts under the impulse of an uncontrollable fear of equal or greater injury, is
exempt from criminal liability because he does not act with freedom. Actus me invito
factus non est meus actus. An act done by me against my will is not my act. The force
contemplated must be so formidable as to reduce the actor to a mere instrument who
acts not only without will but against his will. The duress, force, fear or intimidation must
be present, imminent and impending, and of such nature as to induce a well-grounded
apprehension of death or serious bodily harm if the act be done. A threat of future injury
is not enough. The compulsion must be of such a character as to leave no opportunity
for the accused for escape or self-defense in equal combat Here, Dequinas version of
events that culminated with her and Jundoc and Jingabos arrests on September 29,
1999 is implausible. Equally far-fetched is Jundoc and Jingabos assertion of blind trust
in Dequina and total ignorance of the transportation of marijuana. We agree with the
Court of Appeals when it observed that:

While [Dequina] wants us to believe that she acted under compulsion and that a
certain Sally called all the shots, she nevertheless admitted that their
accommodations when they reached Manila was with her aunt in Guadalupe. On
cross examination, she said that it was she who told Sally that they were going to
stay with her aunt. More importantly, the alleged threat on her daughter was
unclear. At one point in her testimony, she claimed that her daughter was to be
under the custody of Sally while she was away. However, during the trial her
lawyer manifested that her daughter was in fact in Manila and in the court room
attending the hearing.1wphi1 Moreover, accused-appellants themselves picture
a very precise and elaborate scheme in the transport of the huge shipment of
marijuana. With this, it is simply contrary to human experience that the people
behind the shipment would entrust the same to an unknowing and uncertain
102

person such as [Dequina] and her two stooges, unless they themselves were in
on it. Furthermore, the scheme or transport of the marijuana shipment was so
exact that [Jundoc] and [Jingabo] only had enough time to rest in the house of
[Dequinas] aunt in Guadalupe from the time they arrived in Manila in the
morning to the time they had to go to provincial bus station in the afternoon,
negating their purported desire to see Manila. Clearly, the defense story is
riddled with holes.

ENTRAPMENT V. INSTIGATION
PEOPLE OF THE PHILIPPINES, Appellee,
vs. ROMEO DANSICO y MONAY a.k.a. "Lamyak" and AUGUSTO CUADRA y
ENRIQUEZ, Appellants.
G.R. No. 178060|February 23, 2011| Third Division
BRION, J.:

FACTS: On September 7, 1998, the buy-bust team went to May-Anao, Tigaon where
they briefed the local Tigaon Police at their station of the impending buy-bust operation.
The buy-bust team afterwards proceeded to the nipa hut owned by appellant Dansico.
Paz and the confidential informant met with the appellants; the confidential informant
informed the appellants that Paz wanted to buy P5,000.00 worth of marijuana. Paz
handed the buy-bust money to the appellants who left in a motorcycle to get the
marijuana.
After three hours, more or less, the appellants returned with a brick, allegedly
marijuana, wrapped in a newspaper. Appellant Dansico took the brick from appellant
Cuadra and gave it to Paz. At this point, Paz gave the pre-arranged signal for P/Insp.
Vargas and the buy-bust team to approach. The team immediately apprehended
appellant Dansico, while appellant Cuadra resisted by throwing stones at and grappling
with P/Insp. Vargas. Paz turned the seized marijuana to P/Insp. Vargas and the group
proceeded to the Tigaon Police Station.
RTC found appellant guilty beyond reasonable doubt of illegal sale of marijuana under
Section 4, Article II of RA 6425. In their appeal, defendants argue that they were
instigated into selling marijuana.
ISSUE: W/N defendant was instigated into selling marijuana.

103

HELD: NO. To determine whether there is instigation or entrapment, we held in People


v. Doria that the conduct of the apprehending officers and the predisposition of the
accused to commit the crime must be examined:

[I]n buy-bust operations demands that the details of the purported transaction must be
clearly and adequately shown. This must start from the initial contact between the
poseur-buyer and the pusher, the offer to purchase, the promise or payment of the
consideration until the consummation of the sale by the delivery of the illegal drug
subject of the sale. The manner by which the initial contact was made, whether or not
through an informant, the offer to purchase the drug, the payment of the "buy-bust"
money, and the delivery of the illegal drug, whether to the informant alone or the police
officer, must be the subject of strict scrutiny by courts to insure that law-abiding citizens
are not unlawfully induced to commit an offense. Criminals must be caught but not at all
cost. At the same time, however, examining the conduct of the police should not disable
courts into ignoring the accused's predisposition to commit the crime. If there is
overwhelming evidence of habitual delinquency, recidivism or plain criminal proclivity,
then this must also be considered. Courts should look at all factors to determine the
predisposition of an accused to commit an offense in so far as they are relevant to
determine the validity of the defense of inducement.
In the present case, Paz testified to his initial contact with the confidential informant, on
one hand, and with the appellants, on the other. Acting as the poseur-buyer, Paz asked
the appellants if they had P5,000.00 worth of marijuana which the appellants told him
was equivalent to one (1) kilo. Paz and the appellants initially haggled over the price
before the appellants left to get the marijuana after receiving payment. The appellants
were immediately arrested by the group of P/Insp. Vargas after the marijuana was
handed to Paz.

In addition to this testimony, appellant Dansico admitted that his brother-in-law sells
marijuana in Naga City. All these circumstances, collectively considered, fully support
the conclusion that the appellants, by their own volition, sold marijuana to Paz.

THE PEOPLE OF THE PHILIPPINES, Appellee,


vs. RAFAEL STA. MARIA y INDON, Appellant.
104

G.R. No. 171019| February 23, 2007|First DIvision


GARCIA, J.:

FACTS: On November 27, 2002, P/Chief Insp. Noli Pacheco, Chief of the Provincial
Drug Enforcement Group of received an intelligence report about the illegal drug
activities of a certain "Fael," who later turned out to be appellant Rafael Sta. Maria.
P/Chief Insp. Pacheco formed a surveillance team to look for a police asset to negotiate
a drug deal with appellant. In the morning of November 29, 2002, the surveillance team
reported to P/Chief Insp. Pacheco that a confidential asset found by the team had
already negotiated a drug deal for the purchase of P200 worth of shabu from appellant
at the latters house at No. 123 Sitio Gulod, Barangay Pantubig, San Rafael, Bulacan
between 7:00 and 7:30 in the evening of November 29, 2002. The surveillance team
then prepared for a buy-bust operation, with PO3 Enrique Rullan as team leader, and
PO1 Rhoel Ventura, who was provided with two (2) marked P100-bills, as poseur-buyer.
At the appointed time and place, PO1 Ventura and the confidential informant proceeded
to appellants house and knocked at the door. Appellant opened the door and the
confidential informant introduced to him PO1 Ventura as a prospective buyer. PO1
Ventura later handed the two (2) marked P100-bills to appellant who, in turn, gave him a
plastic sachet of shabu. Thereupon, PO1 Ventura sparked his cigarette lighter, which
was the pre-arranged signal to the other members of the buy-bust team that the sale
was consummated. Appellant was arrested and the two marked P100-bills recovered
from him. Also arrested on that occasion was one Zedric dela Cruz who was allegedly
sniffing shabu inside appellants house and from whom drug paraphernalia were
recovered. Upon laboratory examination of the item bought from appellant, the same
yielded positive for methylampetamine hydrochloride or shabu weighing 0.041 gram.
RTC found appellant guilty beyond reasonable doubt of the crime of violation of
Section 5, Article II of Republic Act No. 9165, otherwise known as the Comprehensive
Dangerous Drugs Act of 2002. Appellant contends that what transpired on that fateful
evening of November 29, 2002 was instigation and not a valid buy-bust operation.
ISSUE: W/N defendant was instigated into selling marijuana.
HELD: NO. In entrapment, the entrapper resorts to ways and means to trap and capture
a lawbreaker while executing his criminal plan. In instigation, the instigator practically
induces the would-be-defendant into committing the offense, and himself becomes a coprincipal. In entrapment, the means originates from the mind of the criminal. The idea
and the resolve to commit the crime come from him. In instigation, the law enforcer
conceives the commission of the crime and suggests to the accused who adopts the
idea and carries it into execution. The legal effects of entrapment do not exempt the
criminal from liability. Instigation does.
Here, the mere fact that the agreement between appellant and the police informant for
the purchase and sale of illegal drugs was made on November 27, 2002, while the buy105

bust operation was conducted on November 29, 2002, is of no moment. Without more, it
does not prove that said informant instigated appellant into committing the offense. If at
all, the earlier agreement and the subsequent actual sale suggest that appellant was
habitually dealing in illegal drugs.
ROBERTO E. CHANG and PACIFICO D. SAN MATEO, petitioners,
vs. PEOPLE OF THE PHILIPPINES, respondent.
G.R. No. 165111 | July 21, 2006 | Third Division
CARPIO MORALES, J.:

FACTS: Roberto Chang (Chang) was the Municipal Treasurer of Makati, while Pacifico
San Mateo (San Mateo) was the Chief of the Operations of the Makati Treasurers
Office. Edgar Leoncito Feraren (Feraren), on the other hand, was a driver-clerk at the
same office. They work hand in hand to collect from Group Developers Incorporated
(GDI), through its employee Mario Magat, an amount of P 125, 000.
In exchange for the said sum, they issue to the GDI a Certificate of Examination stating
that the company had no tax liability for a particular period. The GDI reported the said
incident
to National Bureau
of
Investigation
(NBI)
as
part
of
an
entrapment operation against the said individuals. The Sandiganbayan convicted Chang
and San Mateo while Feraren was acquitted. Chang and Mateo maintain that the
alleged entrapment operation by the authorities was actually an instigation; which is
an absolutory cause under criminallaw, and therefore not punishable.
ISSUE: W/N the incident can be counted as instigation.
HELD: NO. There is entrapment when law officers employ ruses and schemes to
ensure the apprehension of the criminal while in the actual commission of the crime.
There is instigation when the accused is induced to commit the crime. The difference in
the nature of the two lies in the origin of the criminal intent. In entrapment, the mens
reaoriginates from the mind of the criminal. The idea and the resolve to commit the
crime comes from him. In instigation, the law officer conceives the commission of the
crime and suggests to the accused who adopts the idea and carries it into execution.

From the evidence for the prosecution, it was clearly established that the criminal intent
originated from the minds of petitioners. Even before the June 19, 1991 meeting took
place, petitioners already made known to Magat that GDI only had two options to
106

prevent the closure of the company, either to pay the assessed amount
ofP494,601.11 to the Municipality, or pay the amount of P125,000 to them.

Respecting the failure of Chang to testify, it bears noting that the evidence for the
prosecution did establish beyond reasonable doubt the presence of conspiracy as it did
his and San Mateo's guilt. The burden of the evidence having shifted to him, it was
incumbent for him to present evidence to controvert the prosecution evidence. He opted
not to, however. He is thus deemed to have waived his right to present evidence in his
defense.

(MITIGATING CIRCUMSTANCES)
INCOMPLETE JUSTIFYING/INC. EXEMPTING CIRCUMSTANCES
PEOPLE OF THE PHILIPPINES, petitioner,
vs. COURT OF APPEALS and ELADIO C. TANGAN, respondents.
G.R. No. 103613

February 23, 2001

x------------------x
ELADIO C. TANGAN, petitioner,
vs. PEOPLE OF THE PHILIPPINES and COURT OF APPEALS, respondents.
G.R. No. 105830

February 23, 2001|First Division

YNARES-SANTIAGO, J.:

FACTS: On December 1, 1984, accused was driving alone on Roxas Boulevard


heading south. He had just come from Buendia Avenue on an intelligence operation. At
the same time, Generoso Miranda, was driving his car in the same direction along
Roxas Boulevard with his uncle, Manuel Miranda, after coming from the Ramada Hotel.
Generoso was moving ahead of Tangan. Suddenly, firecrackers were thrown in
Generoso's way, causing him to swerve to the right and cut Tangan's path. Tangan blew
his horn several times. Generoso, slowed down to let Tangan pass. Tangan accelerated
and overtook Generoso, but when he got in front, Tangan reduced speed. Generoso
tried four or five times to overtake on the right lane but Tangan kept blocking his lane.
As he approached Airport Road, Tangan slowed down to make a U-tum. Generoso
passed him, pulled over and got out of the car with his uncle. Tangan also stopped his
car and got out. As the Mirandas got near Tangan's car, Generoso loudly retorted,
107

" Putang ina mo, bakit mo ginigitgit ang sasakyan ko?" Generoso and Tangan then
exchanged expletives. Tangari pointed his hand to Generoso and the latter slapped it,
saying, "Huwag mo akong dinuduro! Sino ka ba, ano ba ang pinagmamalaki
mo?" Tangan countered, "Ikaw, ano ang gusto mo?" With this, Tangan went to his car
and got his .38 caliber handgun on the front seat.

According to the prosecution witnesses, particularly, Mary Ann Borromeo, Rosalia Cruz
and Manuel Miranda, the accused pointed his gun at Generoso Miranda and when
Manuel Miranda tried to intervene, the accused pointed his gun at Manuel Miranda, and
after that the accused pointed again the gun to Generoso Miranda, the accused shot
Generoso Miranda at a distance of about a meter but because the arm of the accused
was extended, the muzzle of the gun reached to about more or less one foot away from
the body of Generoso Miranda. The shot hit the stomach of Generoso Miranda causing
the latter to fall and while still conscious, Generoso Miranda told Manuel Miranda, his
uncle, to get the gun. Manuel Miranda grappled for the possession of the gun and
during their grappling, Rosalia Cruz intervened and took hold of the gun and after
Rosalia Cruz has taken hold of the gun, a man wearing a red T-shirt took the gun from
her. The man in T-shirt was chased by Manuel Miranda who was able to get the gun
where the man in red T-shirt placed it. RTC found the defendant guilty of the crime of
homicide.

ISSUE: W/N the privilege mitigating circumstance of incomplete self-defense must be


appreciated in favour of the defendant?
HELD: NO. The element of unlawful aggression in self-defense must not come from the
person defending himself but from the victim.
A mere threatening or intimidating attitude is not sufficient. Likewise, the exchange of
insulting words and invectives between Tangan and Generoso Miranda, no matter how
objectionable, could not be considered as unlawful aggression, except when coupled
with physical assault. There being no lawful aggression on the part of either
antagonists, the claim of incomplete self-defense falls. Tangan undoubtedly had
possession of the gun, but the Mirandas tried to wrestle the gun from him. It may be
said that the former had no intention of killing the victim but simply to retain possession
of his gun. However, the fact that the victim subsequently died as a result of the gunshot
wound, though the shooter may not have the intention to kill, does not absolve him from
culpability. Having caused the fatal wound, Tangan is responsible for all the
consequences of his felonious act. He brought out the gun, wrestled with the Mirandas
but anticipating that the gun may be taken from him, he fired and fled.

108

(NO INTENTION TO COMMIT SO GRAVE A WRONG)


PEOPLE OF THE PHILIPPINES, plaintiff-appellee,
vs. ELBERT CALLET y SABANAL, accused-appellant.
G.R. No. 135701|May 9, 2002|First Division
PUNO, J.:

FACTS: On September 15, 1996, the victim, Alfredo Senador, his 12-year old son,
Lecpoy Senador, and Eduardo Perater were at the flea market. There were many
people in the vicinity. Some were playing cara y cruz while others were playing
volleyball. Alfredo, Lecpoy and Eduardo were beside each other as they watched a cara
y cruz game. Alfredo sat close to the ground, with his buttocks resting on his right foot.
Lecpoy and Eduardo sat on a piece of wood and on a stone, respectively.
Out of nowhere, the accused, Elbert Callet, appeared behind Alfredo and stabbed the
latter on the left shoulder near the base of the neck with a 9-inch hunting knife.
Instinctively, Alfredo stood up and managed to walk a few meters. When he fell on the
ground, Lecpoy and Eduardo rushed to help him but to no avail. Alfredo died shortly
thereafter.
ISSUE: W/N defendant is entitled to mitigating circumstance of lack of intention to
commit so grave a wrong.
HELD: NO. The lack of "intent" to commit a wrong so grave is an internal state. It is
weighed based on the weapon used, the part of the body injured, the injury inflicted and
the manner it is inflicted. The fact that the accused used a 9-inch hunting knife in
attacking the victim from behind, without giving him an opportunity to defend himself,
clearly shows that he intended to do what he actually did, and he must be held
responsible therefor, without the benefit of this mitigating circumstance.

PEOPLE OF THE PHILIPPINES, Appellee,


vs. NOEL T. SALES, Appellant.
G.R. No. 177218|October 3, 2011|First Division
DEL CASTILLO, J.
109

FACTS: On September 19, 2002, brothers Noemar and Junior, then nine and eight
years old, respectively, left their home to attend the fluvial procession of Our Lady of
Peafrancia without the permission of their parents. They did not return home that night.
When their mother, Maria Litan Sales (Maria), looked for them the next day, she found
them in the nearby Barangay of Magsaysay. Afraid of their fathers rage, Noemar and
Junior initially refused to return home but their mother prevailed upon them. When the
two kids reached home in the evening of September 20, 2002, a furious appellant
confronted them. Appellant then whipped them with a stick which was later broken so
that he brought his kids outside their house. With Noemars and Juniors hands and feet
tied to a coconut tree, appellant continued beating them with a thick piece of wood.
During the beating Maria stayed inside the house and did not do anything as she feared
for her life.
When the beating finally stopped, the three walked back to the house with appellant
assisting Noemar as the latter was staggering, while Junior fearfully followed. Maria
noticed a crack in Noemars head and injuries in his legs. She also saw injuries in the
right portion of the head, the left cheek, and legs of Junior. Shortly thereafter, Noemar
collapsed and lost consciousness. Maria tried to revive him and when Noemar remained
motionless despite her efforts, she told appellant that their son was already dead.
However, appellant refused to believe her. Maria then told appellant to call a quack
doctor. He left and returned with one, who told them that they have to bring Noemar to a
hospital. Appellant thus proceeded to take the unconscious Noemar to the junction and
waited for a vehicle to take them to a hospital. As there was no vehicle and because
another quack doctor they met at the junction told them that Noemar is already dead,
appellant brought his son back to their house. RTC convicted the appellant for the
crimes of parricide and slight physical injuries.
ISSUE: W/N the defendant is entitled to mitigating circumstance of lack of intention to
commit so grave a wrong.
HELD: NO. There was error in appreciating the mitigating circumstance of lack of
intention to commit so grave a wrong. Appellant adopted means to ensure the success
of the savage battering of his sons. He tied their wrists to a coconut tree to prevent their
escape while they were battered with a stick to inflict as much pain as possible. Noemar
suffered injuries in his face, head and legs that immediately caused his death. "The
mitigating circumstance of lack of intent to commit so grave a wrong as that actually
perpetrated cannot be appreciated where the acts employed by the accused were
reasonably sufficient to produce and did actually produce the death of the victim.
(PROVOCATION OR THREATS)
ARTURO ROMERA, petitioner,
vs. PEOPLE OF THE PHILIPPINES, respondent.
110

G.R. No. 151978|July 14, 2004| SECOND DIVISION


QUISUMBING, J.
FACTS: In the afternoon of October 4, 1998, petitioner Arturo Romera was with the
victim, Roy Mangaya-ay, and five other men namely, Eligario "Beboy" Acenas, Dennis
"Bobong" Mangaya-ay, Ric Mangaya-ay, Bebing Zulueta and Franklin Generol. They
were all headed for Biasong to play volleyball. When they reached Biasong, it was
raining, so they decided to while away time at the house of Ciriaca Capil. Franklin
Generol hung a string made of cigarette foil on Bebing Zuluetas pants and said,
"Theres a monkey among us." Everybody laughed except Roy Mangaya-ay, who got
angry and chided Franklin Generol to stop lest he make enemies. Bebing Zulueta also
got angry and pointed a finger at Franklin Generol and said, "Even if you are stronger
and older, if you will be hit by my fist, you will crawl." Petitioner then stood up and
warned everyone, "You all watch out in Balaguan." He pulled Franklin Generol to join
him and said, "Lets go, there are many boastful people here." Thereafter, petitioner and
Franklin left the group.
At six oclock in the evening, Roy and his companions arrived in Balaguan. On their way
home, they passed by the house of one Antonio Mangaya-ay. In said house, which is
about one kilometer away from petitioners own, they saw petitioner already carrying a
bolo waiting for them.
Suddenly, raising the bolo with his right hand, petitioner uttered, "Here are the brave
ones." Roy and his companions ran away but Roy slipped on the muddy ground.
Petitioner approached Roy and said, "Come here, brave one." He held Roy up by the
collar and stabbed him in the stomach. Roy fell unconscious. When he woke up, he
found himself at the provincial hospital where he underwent surgery and stayed for
more than three weeks.
After the stabbing incident, petitioner voluntarily surrendered to a certain Tibo Ramoso
of the Citizens Armed Force Geographical Unit (CAFGU). Ramoso accompanied
petitioner to the Balingasay police station. RTC convicted petitioner of frustrated
homicide.
ISSUE: W/N mitigating circumstances of provocation and passion or obfuscation
present in this case.
HELD: NO. Thrusting his bolo at petitioner, threatening to kill him, and hacking the
bamboo walls of his house are, in our view, sufficient provocation to enrage any man, or
stir his rage and obfuscate his thinking, more so when the lives of his wife and children
are in danger. Petitioner stabbed the victim as a result of those provocations, and while
petitioner was still in a fit of rage. In our view, there was sufficient provocation and the
circumstance of passion or obfuscation attended the commission of the offense.

111

But, we must stress that provocation and passion or obfuscation are not two separate
mitigating circumstances. Well-settled is the rule that if these two circumstances are
based on the same facts, they should be treated together as one mitigating
circumstance. From the facts established in this case, it is clear that both circumstances
arose from the same set of facts aforementioned. Hence, they should not be treated as
two separate mitigating circumstances.

PEOPLE OF THE PHILIPPINES, appellee,


vs. DENNIS TORPIO y ESTRERA, appellant.
G.R. No. 138984| June 4, 2004| SECOND DIVISION
CALLEJO, SR., J.:

FACTS: On October 11, 1997, Manuel Torpio and wife included, together with an old
woman visitor named Fausta Mariaca, were taking their supper. Anthony Rapas
knocked and asked for Dennis Torpio who, after eating, went and left home with
Anthony upon the latters invitation for a drinking spree. They have some round of drinks
at a nearby store together with another companion. Not contented, they left and
proceeded to the seashore where in a cottage there were people also drinking. Joining
the group, Anthony and Dennis again drank. Later, the two and their companion
transferred to another cottage and there they again drank now with gin liquor except
Dennis who did not anymore drink.
For one reason or another, because Dennis did not drink, Anthony got angry and he
then bathed Dennis with gin, and boxed or mauled him and tried to stab him with
a batangas knife but failed to hit Dennis as the latter was crawling under the table. He
got up and ran towards home. His family was awakened, his mother shouted as Dennis
was taking a knife and appearing bloodied. Manuel Torpio woke up and tried to take the
knife from Dennis but failed and, in the process, wounded or cut himself in his left hand.
Dennis left with the knife, passed by another route towards the seashore and upon
reaching the cottage where Anthony and their companion Porboy Perez were, looked
for Anthony. Anthony upon seeing Dennis sensed danger and he fled by taking the
seashore. But Dennis, being accustomed to the place was able to meet and block
Anthony. Upon seeing the shining knife of Dennis, Anthony tried to evade by turning to
his left and Dennis thus hit the back portion of Anthony. Anthony ran farther but he was
caught in a fishing net across the small creek and he fell on his back. It is at this
juncture Dennis mounted on Anthony and continued stabbing the latter. He left the place
but did not proceed to home, instead, he went to the grassy meadow near the camp and
112

there slept until morning. He then went to a certain police officer to whom he voluntarily
surrendered and together they went to the police headquarters.
RTC found the appellant guilty beyond reasonable doubt of murder. In his appeal,
defendant contends that the act of stabbing Anthony was preceded by a quarrel
between them; hence, the victim had been forewarned of the danger to his life and limb.
ISSUE: W/N the defendant is entitled to mitigating circumstance of provocation or
threats.
HELD: NO. The mitigating circumstance of having acted in the immediate vindication of
a grave offense was, likewise, properly appreciated. The appellant was humiliated,
mauled and almost stabbed by the deceased. Although the unlawful aggression had
ceased when the appellant stabbed Anthony, it was nonetheless a grave offense for
which the appellant may be given the benefit of a mitigating circumstance. But the
mitigating circumstance of sufficient provocation cannot be considered apart from the
circumstance of vindication of a grave offense. These two circumstances arose from
one and the same incident, i.e., the attack on the appellant by Anthony, so that they
should be considered as only one mitigating circumstance.

Under Article 249 of the Revised Penal Code, homicide is punishable by reclusion
temporal. However, considering that there are two mitigating circumstances and no
aggravating circumstance attendant to the crime, the imposable penalty, following
Article 64(5) of the Revised Penal Code, is prision mayor, the penalty next lower to that
prescribed by law, in the period that the court may deem applicable. Applying the
Indeterminate Sentence Law, the maximum penalty to be imposed shall be taken from
the medium period of prision mayor, while the minimum shall be taken from within the
range of the penalty next lower in degree, which is prision correccional. Hence, the
imposable penalty on the appellant is imprisonment from six (6) years of prision
correccional, as minimum, to eight (8) years and one (1) day of prision mayor, as
maximum.

(VINDICATION OF A GRAVE OFFENSE)

PEOPLE OF THE PHILIPPINES, appellee,


vs. DENNIS TORPIO y ESTRERA, appellant.
G.R. No. 138984| June 4, 2004| SECOND DIVISION
113

CALLEJO, SR., J.:

FACTS: On October 11, 1997, Manuel Torpio and wife included, together with an old
woman visitor named Fausta Mariaca, were taking their supper. Anthony Rapas
knocked and asked for Dennis Torpio who, after eating, went and left home with
Anthony upon the latters invitation for a drinking spree. They have some round of drinks
at a nearby store together with another companion. Not contented, they left and
proceeded to the seashore where in a cottage there were people also drinking. Joining
the group, Anthony and Dennis again drank. Later, the two and their companion
transferred to another cottage and there they again drank now with gin liquor except
Dennis who did not anymore drink.
For one reason or another, because Dennis did not drink, Anthony got angry and he
then bathed Dennis with gin, and boxed or mauled him and tried to stab him with
a batangas knife but failed to hit Dennis as the latter was crawling under the table. He
got up and ran towards home. His family was awakened, his mother shouted as Dennis
was taking a knife and appearing bloodied. Manuel Torpio woke up and tried to take the
knife from Dennis but failed and, in the process, wounded or cut himself in his left hand.
Dennis left with the knife, passed by another route towards the seashore and upon
reaching the cottage where Anthony and their companion Porboy Perez were, looked
for Anthony. Anthony upon seeing Dennis sensed danger and he fled by taking the
seashore. But Dennis, being accustomed to the place was able to meet and block
Anthony. Upon seeing the shining knife of Dennis, Anthony tried to evade by turning to
his left and Dennis thus hit the back portion of Anthony. Anthony ran farther but he was
caught in a fishing net across the small creek and he fell on his back. It is at this
juncture Dennis mounted on Anthony and continued stabbing the latter. He left the place
but did not proceed to home, instead, he went to the grassy meadow near the camp and
there slept until morning. He then went to a certain police officer to whom he voluntarily
surrendered and together they went to the police headquarters.
RTC found the appellant guilty beyond reasonable doubt of murder. In his appeal,
defendant contends that the act of stabbing Anthony was preceded by a quarrel
between them; hence, the victim had been forewarned of the danger to his life and limb.
ISSUE: W/N the defendant is entitled to mitigating circumstance of vindication of a
grave offense.
HELD: YES. The mitigating circumstance of having acted in the immediate vindication
of a grave offense was, likewise, properly appreciated. The appellant was humiliated,
mauled and almost stabbed by the deceased. Although the unlawful aggression had
ceased when the appellant stabbed Anthony, it was nonetheless a grave offense for
which the appellant may be given the benefit of a mitigating circumstance. But the
mitigating circumstance of sufficient provocation cannot be considered apart from the
circumstance of vindication of a grave offense. These two circumstances arose from
114

one and the same incident, i.e., the attack on the appellant by Anthony, so that they
should be considered as only one mitigating circumstance.

Under Article 249 of the Revised Penal Code, homicide is punishable by reclusion
temporal. However, considering that there are two mitigating circumstances and no
aggravating circumstance attendant to the crime, the imposable penalty, following
Article 64(5) of the Revised Penal Code, is prision mayor, the penalty next lower to that
prescribed by law, in the period that the court may deem applicable. Applying the
Indeterminate Sentence Law, the maximum penalty to be imposed shall be taken from
the medium period of prision mayor, while the minimum shall be taken from within the
range of the penalty next lower in degree, which is prision correccional. Hence, the
imposable penalty on the appellant is imprisonment from six (6) years of prision
correccional, as minimum, to eight (8) years and one (1) day of prision mayor, as
maximum.

(PASSION OR OBFUSCATION)
PEOPLE OF THE PHILIPPINES, Plaintiff-Appellee,
vs. PFC FLORO MALEJANA, Accused-Appellant.
G.R. No. 145002|January 24, 2006| Second Division
AZCUNA, J.:

FACTS: On July 28, 1990, one of the witnesses narrated that around 7:15 p.m., while
he was seated in front of his jeep parked at the side of the road at Marisfoque, Pilar,
Sorsogon in the company of Roces, Sy, Andrade, Bernarda Sy, Jose Belmonte and
Ernesto Francisco, he saw appellant at about 30 meters away heading towards their
direction. Upon reaching their group, appellant asked them where Roces was. When he
noticed Roces who was sitting at a distance of 1 meter beside Madrid, appellant
brandished an armalite rifle and fired a shot into the air. Then he pointed the barrel of
115

the gun at Roces and fired five (5) times, hitting Roces thrice. After the victim fell to the
ground, appellant left the scene of the incident and went to his house about 150 meters
away. Madrid and his other companions tried to assist Roces but discovered that the
latter was already dead, presumably from the gunshot wounds that were inflicted upon
him by appellant. RTC found defendant guilty beyond reasonable doubt of the crime of
murder.
ISSUE: W/N mitigating circumstance of passion or obfuscation can be appreciated in
favour of defendant.
HELD: NO. Passion and obfuscation similarly cannot be appreciated in favor of
appellant. To be entitled to this mitigating circumstance, the following elements must be
present: (1) There should be an act both unlawful and sufficient to produce such
condition of mind; (2) the act that produced the obfuscation was not far removed from
the commission of the crime by a considerable length of time, during which the
perpetrator might recover his normal equanimity.

The bare assertion that the victim and appellant had an argument does not provide
justifiable basis for applying to him this mitigating circumstance. The cause that
produced the passion and obfuscation has not been established nor proven by clear
and convincing evidence. The defense advances mere speculations and conjectures to
gloss over the fact that there is lack of proof of the cause. Courts are not permitted to
render judgments upon guesses or surmises. Suspicion, it has been said, cannot give
probative force to testimony which in itself is insufficient to establish or justify an
inference of a particular fact.

PEOPLE OF THE PHILIPPINES, appellee,


vs. MARIVIC GENOSA, appellant.
G.R. No. 135981|January 15, 2004|En Banc
PANGANIBAN, J.:
116

FACTS: On November 15, 1995, Marivic Genosa, who got home after work, got worried
that her husband Ben Genosa, who was not home yet might have gone gambling since
it was a payday. With her cousin Ecel Arao, appellant went to look for Ben at the
marketplace and taverns at Isabel, Leyte but did not find him there. They found Ben
drunk upon their return at the Genosas' house. Ecel went home despite appellant's
request for her to sleep in their house.

Then, Ben purportedly nagged appellant for following him, even challenging her to a
fight. She allegedly ignored him and instead attended to their children who were doing
their homework. Apparently disappointed with her reaction, Ben switched off the light
and, with the use of a chopping knife, cut the television antenna or wire to keep her from
watching television. According to appellant, Ben was about to attack her so she ran to
the bedroom, but he got hold of her hands and whirled her around. She fell on the side
of the bed and screamed for help. Ben left. At this point, appellant packed his clothes
because she wanted him to leave. Seeing his packed clothes upon his return home,
Ben allegedly flew into a rage, dragged appellant outside of the bedroom towards a
drawer holding her by the neck, and told her 'You might as well be killed so nobody
would nag me.' Appellant testified that she was aware that there was a gun inside the
drawer but since Ben did not have the key to it, he got a three-inch long blade cutter
from his wallet. She however, 'smashed' the arm of Ben with a pipe, causing him to drop
the blade and his wallet. Appellant then 'smashed' Ben at his nape with the pipe as he
was about to pick up the blade and his wallet. She thereafter ran inside the bedroom.

Appellant, however, insisted that she ended the life of her husband by shooting him.
She supposedly 'distorted' the drawer where the gun was and shot Ben. He did not die
on the spot, though, but in the bedroom.

RTC found the proffered theory of self-defense untenable, the court gave credence to
the prosecution evidence that appellant had killed the deceased while he was in bed
sleeping. Further, the trial court appreciated the generic aggravating circumstance of
treachery, because Ben Genosa was supposedly defenseless when he was killed -lying in bed asleep when Marivic smashed him with a pipe at the back of his head.

117

ISSUE: W/N mitigating circumstance of passion or obfuscation can be appreciated in


favour of defendant.
HELD: YES. The cyclical nature and the severity of the violence inflicted upon appellant
resulted in "cumulative provocation which broke down her psychological resistance and
natural self-control," "psychological paralysis," and "difficulty in concentrating or
impairment of memory."
Based on the explanations of the expert witnesses, such manifestations were
analogous to an illness that diminished the exercise by appellant of her will power
without, however, depriving her of consciousness of her acts. There was, thus, a
resulting diminution of her freedom of action, intelligence or intent. Pursuant to
paragraphs 9 and 10 of Article 13 of the Revised Penal Code, this circumstance should
be taken in her favor and considered as a mitigating factor.

In addition, we also find in favor of appellant the extenuating circumstance of having


acted upon an impulse so powerful as to have naturally produced passion and
obfuscation. It has been held that this state of mind is present when a crime is
committed as a result of an uncontrollable burst of passion provoked by prior unjust or
improper acts or by a legitimate stimulus so powerful as to overcome reason. To
appreciate this circumstance, the following requisites should concur: (1) there is an act,
both unlawful and sufficient to produce such a condition of mind; and (2) this act is not
far removed from the commission of the crime by a considerable length of time, during
which the accused might recover her normal equanimity.

Here, an acute battering incident, wherein Ben Genosa was the unlawful aggressor,
preceded his being killed by Marivic. He had further threatened to kill her while dragging
her by the neck towards a cabinet in which he had kept a gun. It should also be recalled
that she was eight months pregnant at the time. The attempt on her life was likewise on
that of her fetus. His abusive and violent acts, an aggression which was directed at the
lives of both Marivic and her unborn child, naturally produced passion and obfuscation
overcoming her reason. Even though she was able to retreat to a separate room, her
emotional and mental state continued. According to her, she felt her blood pressure rise;
she was filled with feelings of self-pity and of fear that she and her baby were about to
die. In a fit of indignation, she pried open the cabinet drawer where Ben kept a gun,
then she took the weapon and used it to shoot him.

118

The confluence of these events brings us to the conclusion that there was no
considerable period of time within which Marivic could have recovered her normal
equanimity. Helpful is Dr. Pajarillo's testimony that with "neurotic anxiety" -- a
psychological effect on a victim of "overwhelming brutality [or] trauma" -- the victim
relives the beating or trauma as if it were real, although she is not actually being beaten
at the time. She cannot control "re-experiencing the whole thing, the most vicious and
the trauma that she suffered." She thinks "of nothing but the suffering." Such reliving
which is beyond the control of a person under similar circumstances, must have been
what Marivic experienced during the brief time interval and prevented her from
recovering her normal equanimity. Accordingly, she should further be credited with the
mitigating circumstance of passion and obfuscation.

PEOPLE OF THE PHILIPPINES, Plaintiff-Appellee


vs. MARCELO BATES, Accused-Appellant.
G. R. No.13990728 March 2003
J. Austria
FACTS: On November 28, 1995, Edgar Fuentes, Simon Fuentes and Jose Boholst left
Barangay Esperanza, Ormoc City to deliver copra to a certain Fely Rodado at Barangay
Green Valley, Ormoc City. After delivering copra around 5:00 in the afternoon, the three
men headed back to Barangay Esperanza. While they were along a trail leading to the
house of Carlito Bates, the latter suddenly emerged from the thick banana plantation
surrounding the trail, aiming his firearm at Jose Boholst who was then walking ahead of
his companions. Jose grabbed Carlitos right hand and elbow and tried to wrest
possession of the firearm. While the two were grappling for possession, the gun fired,
hitting Carlito who immediately fell to the ground. At that instant, Marcelo Bates and his
son Marcelo Bates, Jr., brother and nephew of Carlito, respectively, emerged from the
banana plantation each brandishing a bolo. They immediately attacked Jose hacking
him several times. Jose fell to the ground and rolled but Marcelo and his son kept on
hacking him. Marcelo, then, turned to Simon and Edgar and shouted huwes de
kutsilyo. Upon hearing the same, Simon and Edgar ran.
ISSUE: Whether or not Marcelo could validly invoke the mitigating circumstance of
passion and obfuscation?
119

HELD: Passion and obfuscation may not be properly appreciated in favor of appellant.
To be considered as a mitigating circumstance, passion or obfuscation must
arise from lawful sentiments and not from a spirit of lawlessness or revenge or
from anger and resentment. In the present case, clearly, Marcelo was infuriated upon
seeing is brother, Carlito, shot by Jose. However, a distinction must be made between
the first time that Marcelo hacked Jose and the second time that the former hacked the
latter. When Marcelo hacked Jose right after seeing the latter shoot at Carlito, and if
appellant refrained from doing anything else after that, he could have validly invoked the
mitigating circumstance of passion and obfuscation. But when, upon seeing his brother
Carlito dead, Marcelo went back to Jose, who by then was already prostrate on the
ground and hardly moving, hacking Jose again was a clear case of someone acting out
of anger in the spirit of revenge.

(VOLUNTARY SURRENDER)
PEOPLE OF THE PHILIPPINES, Appellee
vs. REX NIMUAN Y CACHO, Appellant.
G. R. No. 18245821 March 2011
(Third Division), J. Brion
Facts: In the afternoon of July 22, 2004, while Alfredo was talking with friends, he saw
the victim, the appellant and a certain Boy Nieva drinking in a neighborhood store in
Barangay San Eugenio, Aringay, La Union. Later that afternoon, as Alfredo was walking
home along a path inside a mango plantation in the barangay, he spotted the appellant
and the victim about 30 meters ahead of him, walking in the same trail leading to their
respective houses. Unaware of his presence, the appellant who was walking a meter
behind the victim suddenly hacked the latter with a bolo. Alfredo ran away to seek
help when he saw the victim fall to the ground after the attack.

The postmortem report revealed that the victim died from massive loss of blood due to
multiple hack wounds on his right forearm, face and head.

120

The appellant, interposing alibi, claimed that between 3:00 and 5:00 p.m. of July 22,
2004, he was watching television at the house of his uncle, Manuel Dulay, at San Benito
Sur when a certain Barangay Captain Cario, along with a barangay kagawad, arrived
and informed him that he was a suspect in the death of the victim. The appellant and his
mother went with the barangay officials to the police station of Aringay, La Union, where
he was detained

RTC found appellant guilty of murder and appreciated the mitigating circumstance of
voluntary surrender. On appeal, CA ruled that RTC erred in appreciating the mitigating
circumstance.

Issue: Whether or not the mitigating circumstance of voluntary surrender may be


appreciated

Held: The Court finds no reason to disturb the findings of the RTC, as affirmed by the
CA. The records are replete with evidence establishing the appellant's guilt beyond
reasonable doubt. Alfredos eyewitness account was corroborated by the post mortem
report on the location and severity of the wounds sustained by the victim. Both the RTC
and the CA correctly appreciated the qualifying circumstance of treachery because the
attack was deliberate, sudden and unexpected, affording the hapless, unarmed and
unsuspecting victim no opportunity to resist or to defend himself. The appellant was
correctly sentenced to suffer the penalty of reclusion perpetua since the mitigating
circumstance of voluntary surrender cannot be appreciated in his favor; the records
indicate that the appellant did not intend to assume responsibility for the death of the
victim when he and his mother went with the barangay officials to the police station.

PEOPLE OF THE PHILIPPINES, Plaintiff-Appellee,


vs. JAY MANDY MAGLIAN Y REYES, Accused-Appellant.
G. R. No. 18983430 March 2011[First Division]
J. Velasco Jr.

121

Facts: On January 4, 2000, the accused and Mary Jay were having dinner at their
home in Dasmarias, Cavite when they got into an argument. The accused did not
want Mary Jay to attend a party, causing them to fight. Incensed, the accused collected
the clothes that Mary Joy had given him for Christmas and told her he would burn them
all and started pouring kerosene on the clothes. Mary tried to wrestle the can of
kerosene from him and, at the same time, warned him not to pour it on her. Despite his
wife's plea, the accused still poured gas on her, thus setting both the clothes and his
wife on fire. RTC and CA both found accused guilty of the crime charged.

To avail himself of this mitigating circumstance, accused-appellant claims that he


voluntarily yielded to the police authorities on October 14, 2002, or before the
commencement of the criminal proceedings against him. He avers that this claim is
backed by the records of the case and a certification made by the Das Marias Police
Station. He contends that both the RTC and the CA inexplicably did not appreciate this
mitigating circumstance in his favor.

Issue: Whether or not the mitigating circumstance of voluntary surrender may be


appreciated

Held: An accused may enjoy the mitigating circumstance of voluntary surrender if the
following requisites are present: "1) the offender has not been actually arrested; 2) the
offender surrendered himself to a person in authority or the latter's agent; and 3) the
surrender was voluntary." We explained, "The essence of voluntary surrender is
spontaneity and the intent of the accused to give himself up and submit himself
to the authorities either because he acknowledges his guilt or he wishes to save
the authorities the trouble and expense that may be incurred for his search and
capture."

We find that in the case of accused-appellant, all the elements for a valid voluntary
surrender were present. Accused-appellant at the time of his surrender had not actually
122

been arrested. He surrendered to the police authorities. His surrender was voluntary,
as borne by the certification issued by the police. There is, thus, merit to the claim of
accused-appellant that he is entitled to the mitigating circumstance of voluntary
surrender.

PEOPLE OF THE PHILIPPINES, Appellee


vs. EDGAR CONCILLADO, Appellant.
GR No.18120428 November 2011 [First Division]
J. Del Castillo

Facts: On August 24, 2002, Diosdado Pido was shot, stabbed and hacked with 26
wounds. On the same night, Edgar Concillado surrendered himself to the police. He
was implicated along with his wife Dolores and his cousin Erlito due to a witness
testimony claiming that all accused jointly acted to commit murder. The defense
contends that Dolores and Erlito were not involved while Edgar only acted out of self
defense (when the deceased hacked him while he was urinating by a fence).

The Regional Trial court found all the accused guilty of murder and rejected the pleas
for self-defense due to the fact that the wounds inflicted makes the claim doubtful. Upon
appeal, the Court of Appeals found fault in the witness testimony and acquitted both
Erlito and Dolores. Edgar was held liable only for homicide and was granted a lower
penalty due to voluntary surrender. The Supreme Court affirmed the ruling from the
Court of Appeals.

123

Issue: Whether or not the appellant qualifies for the mitigating circumstance of
voluntary surrender
Held: YES. "For the mitigating circumstance of voluntary surrender to be appreciated,
the surrender must be spontaneous and in a manner that shows that the accused made
an unconditional surrender to the authorities, either based on recognition of guilt or from
the desire to save the authorities from the trouble and expenses that would be involved
in the accused search and capture. Moreover, it is imperative that the accused was not
actually arrested, the surrender is before a person in authority or an agent of a person in
authority, and the surrender was voluntary."
In the instant case, records show that Edgars surrender was spontaneous. He
presented himself to the police authorities even before the latter had knowledge of the
killing. He also unconditionally admitted before them that he killed Diosdado.

(CONFESSION OF GUILT)
PEOPLE OF THE PHILIPPINES, Plaintiff
vs. EDGAR DAWATON, Accused.
G.R. No.14624717 September 2002
J. Bellosillo

Facts: Edgar Dawaton was found guilty by the trial court of murder qualified by
treachery and was sentenced to death. On 20 September 1998, Leonidas Lavares and
several companions, including Dawaton were drinking in the house of the accuseds
uncle. Already drunk, Leonidas Lavares decided to sleep while the accused and his
companions continued drinking. Dawaton awakened Lavares by stabbing him at the
base of the neck. Dawaton continued stabbing Lavares until the victim died. Dawaton
then ran away to the house of his other relative, where he was later on arrested by the
police.
Issue: Whether or not the penalty of death imposed by the trial court upon the accused
was correct?
Held: No. The Supreme Court held that the trial court erred in not considering the
alternative circumstance of intoxication in favor of the accused. Under Art. 15 of The
Revised Penal Code, intoxication of the offender shall be considered as a mitigating
circumstance when the offender commits a felony in a state of intoxication, if the same
124

is not habitual or subsequent to the plan to commit said felony. Otherwise, when
habitual or intentional, it shall be considered as an aggravating circumstance. The
allegation that the accused was drunk when he committed the crime was corroborated
by the prosecution witnesses. The accused and his drinking companions had consumed
four (4) bottles of gin at the house of Esmeraldo Cortez, each one drinking at least a
bottle. It was also attested that while the four (4) shared another bottle of gin at the
house of Amado Dawaton, it was the accused who drank most of its contents. The
Court further stated that Under Art. 63, par. 3, of The Revised Penal Code, in all cases
in which the law prescribes a penalty composed of two (2) indivisible penalties, such as
in this case, when the commission of the act is attended by a mitigating circumstance
and there is no aggravating circumstance, the lesser penalty shall be applied. Since no
aggravating circumstance attended the killing but there existed the mitigating
circumstance of intoxication, the accused should be sentenced only to the lesser
penalty of reclusion perpetua.

PEOPLE OF THE PHILIPPINES, Plaintiff-Appellee


vs. WILLIAM MUYCO MONTINOLA, Accused-Appellant
GR No.14624717 September 2002
J.Bellosillo

Facts: On 18 November 1996, William Montinola, armed with an unlicensed Cal .380
Pistol Llama deliberately, willfully and criminally with violence against or intimidation of
persons, with intent of gain, take and carry away cash amount of P67,500.00 belonging
to Jose Eduardo Reteracion. Montinola shot the victim on the neck, killing Reteracion.
Two criminal cases were filed against Montinola and he was later on sentenced to
reclusion perpetua for robbery with homicide and death for illegal possession of firearm.
Issue: Whether the use of an unlicensed firearm on the killing perpetrated by reason or
on occasion of the robbery may be treated as a separate offense or as an aggravating
circumstance in the crime of robbery with homicide?

Held: Sec. 1 of P.D.1866 provides that if homicide or murder is committed with the use
of an unlicensed firearm, the penalty of death shall be imposed. Said Presidential
Decree was however, amended by R.A. 8294, while Montinolas case was still pending.
R.A. 8294 provides that if homicide or murder is committed with the use of an
unlicensed firearm, such use of an unlicensed firearm shall be considered as an
aggravating circumstance. The Court held In recent cases, we ruled that there could be
no separate conviction for illegal possession of firearm if homicide or murder is
125

committed with the use of an unlicensed firearm; instead, such use shall be considered
merely as an aggravating circumstance in the homicide or murder committed. Hence,
insofar as the new law will be advantageous to WILLIAM as it will spare him from a
separate conviction for illegal possession of firearm, it shall be given retroactive effect.
Pursuant to the third paragraph of Section 1 of P.D. No. 1866, as amended by R.A. No.
8294, use of an unlicensed firearm is a special aggravating circumstance in the
homicide or murder committed. At any rate, even assuming that the aggravating
circumstances present in the commission of homicide or murder may be counted in the
determination of the penalty for robbery with homicide, we cannot appreciate in this
case the special aggravating circumstance of use of an unlicensed firearm mentioned in
the third paragraph of Section 1 of P.D. No. 1866, as amended by R.A. No. 8294. Such
law was not yet enacted when the crime was committed by WILLIAM; it cannot,
therefore, be given retroactive effect for being unfavorable to him.
The Court further held Under Article 294 of the Revised Penal Code, as amended by
R.A. No. 7659, robbery with homicide is punishable by reclusion perpetua to death,
which are both indivisible penalties. Article 63 of the same Code provides that in all
cases in which the law prescribes a penalty composed of two indivisible penalties, the
greater penalty shall be applied when the commission of the deed is attended by one
aggravating circumstance. If we would apply retroactively the special aggravating
circumstance of use of unlicensed firearm under Section 1 of P.D. No. 1866, as
amended by R.A. No. 8294, the imposable penalty would be death. Conformably with
our ruling in People v. Valdez, insofar as the new law would aggravate the crime of
robbery with homicide and increase the penalty from reclusion perpetua to death, it
would not be given retroactive application, lest it would acquire the character of an ex
post facto law. Hence, we shall not appreciate that special aggravating circumstance.
There being no modifying circumstances, the lesser penalty of reclusion perpetua shall
be imposed upon accused-appellant WILLIAM.

126

(SIMILAR AND ANALOGOUS CIRCUMSTANCES)

Exuperancio Canta, Petitioner,


vs. People of the Philippines, Respondent
Gr No. 140937 l 28 February 2001[Second Division]
J. Mendoza

Facts: Narciso Gabriel owns a cow that was passed on from one person to another and
each person was responsible for the care and custody of the said cow. At the time the
cow got lost, it was under the care and custody of Gardenio Agapay. Agapay took the
cow in the mountain of Pilipogan, 40 meters away from his hut, at around 5:00 in the
afternoon. When he came back to get the cow at past 9 in the evening, the cow was
gone. However, Aagapay saw footprints that led to the house of Filomeno Vallejos.
Vallejos told Agapay that Exuperancio Canta took the cow.
Agapay and Maria were instructed by Narciso to get the cow and on their way to
Florenitno Cantas house, they saw Exuperancio. The latter told them that if it was
really Narciso who was the owner of the cow, he should get it himself. Exuperancia
accompanied the two to his fathers house and both recognized the cow but Florentino
was not home. Exuperancio told Maria and Agapay that he would call them the next
day to talk about the matter with his father. Exuperancio never called. The matter was
reported to the police and Narciso and Exuperancio were called for investigation.
Exuperancio admitted taking the cow but claims that he was the real owner of the cow
and that it was lost on December 3, 1985. However, Narciso presented a certificate of
ownership issued on March 9, 1986, signed by the municipal treasurer, in which the cow
was described as two years old and female. Then, the petitioner also presented a
Certificate of Ownership of Large Cattle dated February 27, 1985 and a statement
executed by Franklin Telen, who was the janitor at the treasurer's office of the
municipality, that he executed the certificate of ownership in favor of Exuperancio. The
trial court rendered its decision finding petitioner guilty of the offense charged.
Exuperancio filed a Motion for reconsideration but was denied by the Court of Appeals
and affirmed the trial court's decision.
Issue: Whether or not the lower courts were correct in sentencing Exuperancio to ten
(10) years and one (1) day of prision mayor, as minimum, to twelve (12) years, five (5)
127

months, and eleven (11) days of reclusion temporal medium, as maximum, and to pay
the costs?
Decision: No. The Supreme Court held that the trial court correctly found petitioner
guilty of violation of 2(c) of P. D. No. 533, otherwise known as the Anti-Cattle Rustling
Law of 1974. However, it erred in imposing the penalty of 10 years and 1 day of prision
mayor, as minimum, to 12 years, 5 months and 11 days of reclusion temporal medium,
as maximum. The trial court apparently considered P. D. No. 533 as a special law and
applied 1 of the Indeterminate Sentence Law, which provides that "if the offense is
punished by any other law, the court shall sentence the accused to an indeterminate
sentence, the maximum term of which shall not exceed the maximum fixed by said law
and the minimum shall not be less than the minimum term prescribed by the same."
However, as held in People v. Macatanda,P. D. No. 533 is not a special law. The penalty
for its violation is in terms of the classification and duration of penalties prescribed in the
Revised Penal Code, thus indicating that the intent of the lawmaker was to amend the
Revised Penal Code with respect to the offense of theft of large cattle. In fact, 10 of the
law provides:
The provisions of Articles 309 and 310 of Act No. 3815, otherwise known as the
Revised Penal Code, as amended, pertinent provisions of the Revised
Administrative Code, as amended, all laws, decrees, orders, instructions, rules and
regulations which are inconsistent with this Decree are hereby repealed or
modified accordingly.
There being one mitigating circumstance and no aggravating circumstance in the
commission of the crime, the penalty to be imposed in this case should be fixed in its
minimum period. Applying the Indeterminate Sentence Law, in relation to Art. 64 of the
Revised Penal Code, petitioner should be sentenced to an indeterminate penalty, the
minimum of which is within the range of the penalty next lower in degree, i. e.,prision
correccional maximum to prision mayor medium, and the maximum of which is prision
mayor in its maximum period.
PEOPLE OF THE PHILIPPINES, Plaintiff vs. TOMAS NAVASCA, FLORENCIO
GERALDES, LORENZO SOBERANO AND MANUEL MARQUEZ, Defendants.
GR No. L-28107 l 15 March 1977 [Second Division]

Facts: On or about March 17, 1959, accused, all armed with deadly weapons,
committed robbery and took away the sum of (P1,800.00) belonging to Go So alias
OWA (victim) and on the same occasion thereof killed the said victim. The court
convicted all accused to the crime of robbery with homicide and to suffer the penalty of
death as to accused Florencio Geraldes and Manuel Marquez, Tomas Navasca was
128

committed to the National Mental Hospital while accused Lorenzo Soberano is to suffer
Reclusion Perpetua, on the basis of testifying for the prosecution and revealing
voluntarily in open court, the sordid details of the crime.
Issue: Whether or not the fact of testifying for the prosecution without being discharged
from the information is considered an analogous circumstance which could mitigate
liability
Held: Yes. Considering however accused Lorenzo Soberano's act of testifying for the
prosecution and revealing voluntarily in Open Court, the sordid details of this crime, the
Court considers this a mitigating circumstance in his favor similar to a plea of guilty, that
is a mitigating circumstance of "similar analogous nature", and, hence, he is hereby
sentenced to suffer the penalty of RECLUSION PERPETUA.
The penalty for the crime of robbery with homicide is reclusion perpetua to death. The
crime having been committed by a band, all the members thereof are liable for each of
the assaults committed, 4 unless any one of them attempted to prevent their
commission. No evidence was adduced by any of the accused that he had attempted to
prevent the commission of the crime, hence all must be held liable for the crime of
robbery with homicide. There being one aggravating circumstance and no mitigating
circumstance to offset the same in the case of Florencio Geraldes and Manuel
Marquez, the penalty of death was correctly imposed on them. The act of testifying for
the prosecution, without previous discharge, by Lorenzo Soberano should be
considered in his favor as a mitigating circumstance analogous to a plea of guilty,
hence there exist in the case of Lorenzo Soberano, one mitigating circumstance
and one aggravating circumstance which offset each other. Consequently,
Lorenzo Soberano was properly sentenced to reclusion perpetua.
People of the Philippines, Plaintiff-Appelle
vs. Jimmy Monaga, Jesus Barrido and Benhur Banaban, Accused, Benhur
Banaban Accused-Appellant.
GR No. L-39528 l 19 November 1982 [Second Division]
J. Concepcion Jr.
Facts: The spouses Herminio Balderas and Marina Balderas were tenants of Juanita
Barrido in her riceland. Juanita told the spouses that she would give them P800.00 if
they vacate the land and her house since land has been found neglected. Herminio
Balderas refused to vacate and continue to occupy the land despite force and
intimidation. Benhur Banaban, also a tenant of Juanita Barrido, lost his female carabao
and suspected Herminio Balderas to have stolen it. When they confronted Herminio, he
told them that they would get the carabao if they pay him P50.00. Matter was escalated
to the Barrio Captain and then to the Mayor of town. Later on, the carabao was found
dead. Herminio promised to pay the carabao for P400.00 but for some reasons, failed to
do so. At about 8:45 in the evening of February 8, 1972, while Herminio Balderas was
129

walking along the road with his child on his right shoulder, followed by his wife Marina
Balderas, he was ambushed, shot and wounded by Jesus Barrido, Benhur Banaban,
Danilo Banaban, Jimmy Monaga, and two other unidentified men, near the bodega of
Mr. Blancaflor in Piliwan and died as a consequence thereof. The trial convicted the
accused for the crime charged. Only Benhur Banaban appealed. The defense of the
appellant Benhur Banaban according to the trial court is an alibi and anchored on the
testimony of his son Danilo Banaban that he (Danilo) was the one who shot Herminio
Balderas on the night of February 8, 1972, resulting in the latter's death, and that the
accused Jimmy Monaga, Jesus Barrido, and Benhur Banaban had nothing to do with
the death of the said Herminio Balderas. Hence, the appeal.
Issue: Whether or not the court erred in finding the appellant guilty of the crime
charged and whether or not a mitigating circumstance of analogous circumstance be
appreciated.
Held: It results that the trial court did not err in finding the appellant Benhur Banaban
guilty of the crime of Murder. The appellant, however, is entitled to the mitigating
circumstance analogous to, if not the same as, vindication of a grave offense
committed by the deceased when the latter took away the carabao of the
appellant and held it for ransom, and thereafter, failed to fulfill his promise to pay
its value after the carabao had died.
(AGGRAVATING CIRCUMSTANCES- ART. 14)
RUJJERIC PALAGANAS, Petitioner
vs. PEOPLE OF THE PHILIPPINES,Respondent.
GR No. 165483 l 12 September 2006 [(First Division]
J. Chico-Nazario
Facts: On January 16, 1998 brothers Servillano and Michael Ferrer went to Tidbits
Videoke bar singing and drinking beer. On the same evening Jaime Palaganas and
Ferdinand Palaganas and Virgilio Bautista arrived. The two groups occupied separate
tables. After the Ferrers singing Jaime Palaganas started singing and was joined by
Tony Ferrer who sang loudly and in mocking manner. This insulted Jaime and soon a
fight ensued between Ferrers and Palaganas. Ferdinand ran towards his house and
sought help from his brother Fuijeric, the latter went outside however he was stoned by
the Ferrer brothers. As they were continuously stoned the appellant Ferdinand suddenly
pulled the trigger with the gun in his hands. The trial court rendered a decision finding
the petitioner guilty of the crime of Homicide and Frustrated homicide but not guilty of
violation of COMELEC RES. 2958.
Issue: Whether or not violation of COMELEC RES. 2958 may be considered as Special
aggravating circumstances which will negate consideration of mitigating circumstances
of voluntary surrender?
130

Held: With the passage of Republic Act. No. 8294 on 6 June 1997, the use of an
unlicensed firearm in murder or homicide is now considered as a SPECIAL
aggravating circumstance and not a generic aggravating circumstance. Republic
Act No. 8294 applies to the instant case since it took effect before the commission of
the crimes in 21 April 1998. Therefore, the use of an unlicensed firearm by the petitioner
in the instant case should be designated and appreciated as a SPECIAL aggravating
circumstance and not merely a generic aggravating circumstance.

People of the Philippines, Appellee vs. GERARDO GERRY EVINA Y PADUAL,


Appellant, GR Nos. 124830-31 l 27 June 2003 (Second Division]
J. Callejo, Sr.
Facts: Gerardo Gavina was served sentence of Reclusion Perpetua for raping certain
Ms. Maritess Catcharo. Based on the given facts, Gerardo took advantage of the time
when the victims mother was not around. He would likely forced Maritess to have
carnal knowledge against her will and even poked a knife at her while doing the deed in
the victims dwelling and threatened the victim to kill her family should she tell her
parents what happened. On November 13, 1991 when the appellant arrived at the
Catcharro residence he proceeded inside the bedroom of Maritess, the latter ran out of
the bedroom and told her mother not to leave her because her Papa Gerry might rape
her again. Surprised by what he heard, the following day Maritess was brought to
Tacloban City Medical Center for a check-up and found to have lacerations to the
victims genitalia. Contrary to the facts above, appellant claimed that the night of the
incident he was working as porter until 10 PM, thus it cannot be said that he committed
the crime accused of him. Based on the information submitted, aggravating
circumstances of use of weapon and dwelling were not alleged.
Issue: Whether or not aggravating circumstances proved during trial but was not
alleged in the information may be considered?
Held: The Supreme Court held in the negative. Although the special aggravating
circumstance of the use of a weapon and the aggravating circumstance of
dwelling were proven, these aggravating circumstances cannot be considered in
fixing the penalty because they were not alleged in the information as mandated
by Rule 110, Sections 8 and 9 of the Revised Rules of Criminal Procedure.
Although the crimes charged were committed before the effectivity of the said rule,
nevertheless, the same should be applied retroactively being favorable to the appellant.
PEOPLE OF THE PHILIPPINES, Plaintiff-Appellee vs.
EFREN MENDOZA Y SALVADOR, Accused-Appellant
GR No. 133382 l 9 March 2000 [Third Division]
J. Panganiban
131

Facts: Efren Mendoza was charged with the crime of murder for killing Anchito Nano. In
this case Efren alleged that Anchito Nano arrived at their house and upon arrival it
started to destroy the house and that her wife was shouting for help. Efren immediately
look for something to protect his family but found a bolo. He approached Anchito but
the latter tried to hack him but he was able to hack him first on the right side of his neck
resulting to the death of the victim. Thereafter Mendoza went to Municipal Hall of Vinzon
and voluntarily surrendered to the police. He claimed that it was self defense. The
autopsy revealed that location of the wounds found on the body of the victim came from
the back of the victims body. The court ruled rejecting appellants self defense. This
court finds that the accused was not in imminent danger of death or great bodily harm,
an attempt to defend himself by means which appeared unreasonable by using a long
bolo is unjustifiable. Hence this appeal.
Issue: Whether or not voluntary surrender was offset by the aggravating circumstances
of treachery?
Held: The Supreme Court held in the negative. A qualifying circumstance changes
the nature of the crime. A generic aggravating circumstance, on the other hand,
does not affect the designation of the crime; it merely provides for the imposition
of the prescribed penalty in its maximum period. Thus, while a generic
aggravating circumstance may be offset by a mitigating circumstance, a
qualifying circumstance may not. 32Treachery in the present case is a qualifying, not
a generic aggravating circumstance. Its presence served to characterize the killing as
murder; it cannot at the same time be considered as a generic aggravating
circumstance to warrant the imposition of the maximum penalty. Thus, it cannot offset
voluntary surrender.
(ABUSE OF PUBLIC POSITION)
People of the Philippines, Plaintiff-Appellee vs. PO3 Renato Villamor and
Jessie JoyMaghilom (At large), Accused, PO3 Renato Villamor,
Accused-Appelant, GR Nos. 140407-08 l 15 January 2002
(En Banc) J. Ynares-Santiago
Facts: On November 25, 1995, brothers Jerry Velez and Jelord Velez were on their way
home on board a motorcycle. A motorcycle was speeding behind them and as they
were about to cross the bridge, they heard gun shots firing behind them. As they turned
around, Jerry identified PO3 Renato Villamor and Jessie Maghilom riding the
motorcycle behind them. Shots were fired at them and Jerry sustained wounds on the
abdomen and elbow while Jelord died on the spot. The trial proceeded against PO3
Villamor while Maghilom was still at large. During trial, the Trial Court found the PO3
Renato Villamor guilty of having commited Murder aggravated by the circumstance of
taking advantage of his public position.

132

Issue: Whether or not the Trial Court properly applied the aggravating circumstance of
taking advantage of public position?
Decision: The Supreme Court ruled that the aggravating circumstance of taking
advantage of public position under paragraph 1 of Article 14 of the Revised Penal Code
was improperly applied.
A public officer must use the influence that is vested in his office as a means to
realize the purpose of the crime to be appreciated as an aggravating
circumstance. The question Did the accused abuse his office to commit the
crime must be asked in order to appreciate this circumstance as an aggravating
circumstance.
No proof was shown that Villamor took advantage of his position of being a policeman
when he shot Jelord Velez. Neither was his influence, prestige or ascendancy used in
killing Velez. Even without occupying a public position, the accused could have
committed the crime.

Ricardo Fortuna Y Gragasin, Petitioner vs. People of the Philippines,


Respondent, GR Nos. 135784 l 15 December 2000
(Second Division) J. Bellosillo
Facts: On July 21, 1992, siblings Diosdada Montecillo and Mario Montecillo were
standing at the corner of Mabini and Harrison Streets. A mobile patrol car stopped in
front of them and a policeman alighted. The policeman frisked Mario and took Marios
belt. He motioned Mario to enter the car. Mario obeyed and was followed by Diosdada.
While inside the car, the policemen told Mario that he would be brought to the Bicutan
police station where he would be interrogated, mauled and heckled for carrying a deadly
weapon. They told the Montecillos that the bailbond for carrying a deadly weapon was
P12,000. The Montecillos were asked how much they had and then Diosdada was
asked to alight from the car. The driver followed her, took P1,500 from her wallet and
instructed her to tell the others that she only had P3,500. Inside the car, they were told
to put all her money on the box. The Montecillos were told to get off at Harrison Plaza.
From there, they went home. The 3 policemen, Fortuna, Garcia, and Pablo, were
charged with robbery and were found guilty of having conspired in committing the crime
with intimidation of persons.
Issue: Whether or not abuse of public position should be taken as an aggravating
circumstance by the mere fact that the accused were police officers?
Held: The Supreme Courted held that the lower courts failed to appreciate the
aggravating circumstance of abuse of public position. The mere fact that the three
(3) accused were all police officers at the time of the robbery placed them in a
position to perpetrate the offense. If they were not police officers they could not
have terrified the Montecillos into boarding the mobile patrol car and forced them
133

to hand over their money. Precisely it was on account of their authority that the
Montecillos believed that Mario had in fact committed a crime and would be
brought to the police station for investigation unless they gave them what they
demanded.

(In Contempt or with Insult to Public Authorities)


People of the Philippines, Plaintiff-Appellee vs. Hernando De Mesa and
Two John Does, Accused, Hernando De Mesa, Accused-Appellant
GR Nos. 137036 l 14 March 2001
(First Division) J. Puno
Facts: Barangay Chairman Patricio Motas of Sta. Cruz Putol, San Pablo City was
pronounced dead on arrival on October 15, 1996 at San Pablo City District Hospital.
The autopsy report showed that the cause of death was shock and hemorrhage due to
gunshot wounds at the back of the victim.
Hernando De Mesa was found guilty beyond reasonable doubt for the crime of murder
by the Regional Trial Court of San Pablo City. He was sentenced to suffer the penalty of
Reclusion Perpetua, pay the costs and to indemnify the heirs of the victim. Treachery,
nighttime, in contempt of or with assault to public authorities, were appreciated by the
trial court as aggravating circumstances attending the case thereby qualifying the crime
committed to murder.
Issue: Whether or not the trial court erred in determining the nature of the crime
committed and the corresponding penalty to be imposed?
Held: Yes. The trial court erred in appreciating the aggravating circumstance that
the commission of the crime was in contempt of or with assault to public
authorities. The requisites of this circumstance are: (1) the public authority is
engaged in the discharge of his duties and (2) he is not the person against whom
the crime is committed.[31] None of these circumstances are present in this
case. In the first place, the crime was committed against the barangay chairman
himself. At the time that he was killed, he was not engaged in the discharge of his
duties as he was in fact playing a card game with his neighbors.
Absent any qualifying aggravating circumstance, the crime committed by accusedappellant is only homicide, for which the imposable penalty under the Revised Penal
Code is reclusion temporal. Applying the indeterminate sentence law and considering
that there is neither aggravating nor mitigating circumstance present in this case, the
penalty that may be imposed on accused.
134

People of the Philippines, Plaintiff-Appellee vs. Renato Tac-an Y Hipos,


Accused-Appellant, GR Nos. 76338-39 l 26 February 1990
(Third Division) J. Feliciano
Facts: Renato Tac-an and Francis Escano were close friends being classmates in high
school and members of the local Bronx gang. Francis withdrew from the gang on the
advice of his mother who saw that Renato carried a handgun on his visits to their home.
Things started turning sour between the two, and came to a head on Dec 14, 1984.
After an earlier altercation on that day, Renato went home and got his gun. He entered
the Mathematics class under Mr. Damaso Pasilbas in Rm15 and shouted for Francis.
After locating the victim he fired at him but missed. He was later able to hit him in the
head as he was running to the door with his classmates to escape. After this, Renato
paced outside in the hallway. A teacher unknowing that Renato was the culprit, asked
him for help unwittingly informing him that Francis was still alive. Renato immediately reentered the room and saying "So, he is still alive. Where is his chest?" Standing over
Francis sprawled face down on the classroom floor, Renato aimed at the chest of
Francis and fired once more. The bullet entered Francis' back below the right shoulder,
and exited on his front chest just above the right nipple.
Tac-an was charged with illegal possession of firearms under P.D. No. 1866. An
amended information for murder was subsequently filed aggravated by the use of illegal
possession of firearms.
Issue: Whether or not the crime was committed in contempt of or with insult to the
public authorities?
Held: No. We believe the trial court erred in so finding the presence of a generic
aggravating circumstance. Article 152 of the Revised Penal Code, as amended by
Republic Act No. 1978 and Presidential Decree No. 299, provides as follows:
Art. 152. Persons in authority and agents of persons in authority. Who shall be
deemed as such. In applying the provisions of the preceding and other articles of this
Code, any person directly vested with jurisdiction, whether as an individual or as a
member of some court or government corporation, board, or commission, shall be
deemed a person in authority. A barrio captain and a barangay chairman shall also be
deemed a person in authority.
135

A person who by direct provision of law or by election or by appointment by competent


authority, is charged with the maintenance of public order and the protection and
security of life and property, such as a barrio councilman, barrio policeman and
barangay leader and any person who comes to the aid of persons in authority, shall be
deemed an agent of a person in authority.
In applying the provisions of Articles 148 and 151 of this Code, teachers,
professors and persons charged with the supervision of public or duly
recognized private schools, colleges and universities, and lawyers in the actual
performance of their professional duties or on the occasion of such
performance, shall be deemed persons in authority. (As amended by P.D. No. 299,
September 19, 1973 and Batas Pambansa Blg. 873, June 12, 1985).
Careful reading of the last paragraph of Article 152 will show that while a teacher
or professor of a public or recognized private school is deemed to be a "person in
authority," such teacher or professor is so deemed only for purposes of
application of Articles 148 (direct assault upon a person in authority), and 151
(resistance and disobedience to a person in authority or the agents of such
person) of the Revised Penal Code. In marked contrast, the first paragraph of Article
152 does not identify specific articles of the Revised Penal Code for the application of
which any person "directly vested with jurisdiction, etc." is deemed "a person in
authority." Because a penal statute is not to be given a longer reach and broader scope
than is called for by the ordinary meaning of the ordinary words used by such statute, to
the disadvantage of an accused, we do not believe that a teacher or professor of a
public or recognized private school may be regarded as a "public authority" within the
meaning of paragraph 2 of Article 14 of the Revised Penal Code, 31 the provision the
trial court applied in the case at bar.
(Disregard of Rank, Age, Sex)
People of the Philippines, Appellee vs. Antonio Reyesy Magano, Appellant,
GR No. 153119 l 13 April 2004
(EnBanc) J. Callejo Sr.
Facts: On the 11th of July 1998, a seventy four year-old lady was found bloody dead
sprawled sideways on the floor opposite the sink near the kitchen. The fact that the
crime was committed inside the victims dwelling without the permission of the latter is
indeed an aggravating robbery with homicide. The appellant; Antonio Reyes said that he
was not aware of what he was doing while he was committing robbery because he was
drunk and killing Ms. Lagrada is not his intention because she started shouting upon
seeing him. However the accused is already on the bus terminal when the police caught
him.
136

Issue: Whether or not Reyes is guilty of the crime of consummated robbery with
homicide regardless of age and sex of the deceased
Held: Robbery with homicide is essentially a felony against property. [45] The
aggravating circumstance of disregard of the victims age is applied only to
crimes against persons and honor.[46] The bare fact that the victim is a woman
does not per se constitute disregard of sex. For this circumstance to be properly
considered, the prosecution must adduce evidence that in the commission of the
crime, the accused had particularly intended to insult or commit disrespect to the
sex of the victim.[47] In this case, the appellant killed the victim because the latter
started to shout. There was no intent to insult nor commit disrespect to the victim on
account of the latters sex.

People of the Philippines, Plaintiff-Appellee vs. William Montinola,


Accused-Appellant
GR No.131856-579 July 2001
C.J. Davide Jr.

Facts: On 18 November 1996, William Montinola, armed with an unlicensed Cal .380
Pistol Llama deliberately, willfully and criminally with violence against or intimidation of
persons, with intent of gain, take and carry away cash amount of P67,500.00 belonging
to Jose Eduardo Reteracion. Montinola shot the victim on the neck, killing Reteracion.
Two criminal cases were filed against Montinola and he was later on sentenced to
reclusion perpetua for robbery with homicide and death for illegal possession of firearm.
Issue: Whether or not the aggravating circumstance of disregard of age and sex of the
deceased be appreciated
Held: No.In People v. Capillas,[22] People v. Ang,[23] and People v. Punzalan,[24] we held
that when the killing is committed by reason or on the occasion of the robbery, the
qualifying circumstances attendant to the killing would be considered as generic
aggravating circumstances; thus, in all these three cases the circumstance of abuse of
superior strength[25] served to aggravate the crime. In the third case, evident
premeditation was also considered as aggravating. However, in these three cases, as
well as in People v. Ponciano,[26] we said that disregard of age, sex or rank is not
aggravating in robbery with homicide, which is primarily a crime against property,
as the homicide is regarded as merely incidental to the robbery.
137

People of the Philippines, Plaintiff-Appellee vs.


Edralin Taboga, Accused-Appellant
GR No.144086-876 February 2002
(En Banc) J. Ynares-Santiago

Facts: On April 1, 1998, Cirilo Urayani woke up to the sound of loud explosions. He
went out of the house to fetch water, and he saw the house of Francisca Tubon on fire.
Marites Ceria, a niece of Francisca Tubon, was also awakened by the explosions. She
rushed to her aunts house and, seeing it on fire, shouted for help. She called out the
name of her aunt but there was no response. Barangay officials and residents helped in
dousing out the fire. When they entered the burned house, they discovered the charred
remains of Francisca Tubon. They examined the body and found stab wounds on the
chest of the deceased. Later that morning, the incident was reported to the police
authorities. Upon investigation, several items and money were found few meters from
the burned house.

The deceaseds former farm workers were rounded up, namely, Mario Ceria, Edwin
Ceria, Tante Dumadag and Edralin Taboga. Brgy. Capt. Pagao noticed fresh blood
stains on the short pants of Taboga. He confronted Taboga, and the latter readily
admitted that he killed Francisca Tubon and set the flue-cured tobacco stored inside her
house on fire, causing the whole house, including the dead body of the old woman, to
be burned.

Accused-appellant Edralin Taboga raised the defense of denial and alibi. He alleged
that he was in the house of the parents of his live-in partner, some 70m away. Accusedappellant further claimed that he was maltreated by the policemen and forced to admit
the crime. The trial court found the accused guilty of the crime of robbery with homicide
and arson.

138

Issue: Whether or not the aggravating circumstance of disregard of rand age or sex
was attendant in the commission of the crime.

Held: In any event, the aggravating circumstance alleged attended the killing. The
immutable fact remains that the crime of homicide was committed without regard to her
age and sex.

Anent the circumstance of age, there must be a showing that the


malefactor deliberately intended to offend or insult the age of the victim. Neither could
disregard of respect due to sex be appreciated if the offender did not manifest
any intention to offend or disregard the sex of the victim. In other words, killing a woman
is not attended by the aggravating circumstance if the offender did not manifest any
specific insult or disrespect towards the offended partys sex. In the case at bar, there
is absolutely no showing that accused-appellant deliberately intended to offend
or insult the victim. However, even if disrespect or disregard of age or sex were
not appreciated, the four circumstances enumerated in Article 14, paragraph 3 of
the Revised Penal Code, as amended, can be considered singly or together.

139

(Dwelling)

People of the Philippines, Plaintiff-Appellee vs. Edgar Evangelio y Gallo, Joseph


Evangelio, Atilano Agaton y Obico and Noel Malpas y Gracia, Accused,
Joseph Evangelio, Accused-Appellant, GR No.18190231 August 2011
(Third Division) J. Peralta

Facts: On 3rd of October 2001, in the City of Tacloban, all accused, armed with a
handgun and deadly/bladed weapons forcibly enter the inhabited house/residence of
BBB and while inside, take, and carry away from said residence some personal
properties amounting to PhP336,000.00. On the occasion of the said robbery, accused
feloniously have carnal knowledge of AAA, [2] a 17-year-old minor, against her will and
consent and at the time when the latter lost consciousness after her head was banged
on the bathroom floor. Trial court convicted accused for the crime charged and
appreciated the aggravating circumstances of nighttime, committed by a band, dwelling
and unlawful entry. On appeal, the CA affirmed the decision with modification. CA did
not consider the aggravating circumstances of nighttime and unlawful entry. Hence, this
petition.
Issue: Whether or not trial court erred in appreciating the aggravating circumstance of
dwelling in the commission of the crime
Held: No. The aggravating circumstance of dwelling was attendant in the present case.
Dwelling aggravates a felony where the crime is committed in the dwelling of the
offended party provided that the latter has not given provocation therefor. [39] In this
case, robbery with violence was committed in the house of the victims without
provocation on their part. In robbery with violence and intimidation against
persons, dwelling is aggravating because in this class of robbery, the crime may
be committed without the necessity of trespassing the sanctity of the offended
party's house.[40]It is considered an aggravating circumstance primarily because
of the sanctity of privacy that the law accords to the human abode. [41] He who
goes to anothers house to hurt him or do him wrong is more guilty than he who
offends him elsewhere.[42]

People of the Philippines, Plaintiff-Appellee vs. Arnold Agcanas,


Accused-Appellant, GR No.17447611 October 2011
140

(En Banc) J. Sereno

Facts: On 4 May 2000, at about 9:00pm, while the victim Warlito Raguirag was having
dinner at home, herein accused Arnold Agcanas entered the former's house through the
kitchen door. The accused pointed a gun at the back of the left ear of the victim and
shot him point-blank. Beatriz Raguirag, the victims wife, shouted, We were invaded
[sinerrek] by Arnold Agcanas.[3] Under the 50-watt light bulb and with only a meter
between them, the wife was able to identify the accused, who was the son of her
cousin.
The trial court found the accused guilty beyond reasonable doubt of the crime of
murder, qualified by treachery and attended by the aggravating circumstances of
dwelling, given that the crime was committed in the kitchen of the house of the victim
and the use of an illegally possessed firearm. On appeal, CA affirmed the decision with
some modifications based on prevailing jurisprudence. Hence, this petition.
Issue: Whether or not trial court erred in finding that the crime was aggravated by
dwelling
Held: No. The trial court was also correct in ruling that dwelling was an aggravating
circumstance. It has been held in a long line of cases that dwelling is aggravating
because of the sanctity of privacy which the law accords to human abode. He
who goes to another's house to hurt him or do him wrong is more guilty than he
who offends him elsewhere.

People of the Philippines, Appellee vs. Alejandro Calongui y Lopez,


Appellant, GR No.1705663 March 2006
(First Division) J. Ynares-Santiago

Facts: Calonqui was found guilty for two counts of rape. On January 1, 1998 about 2
am in Tagbong, Camarines Sur, Calonqui was able to rape the 13 year old girl Maricel in
the latters house. On September 26, 1998 at about three in the morning, the accused
again raped the victim. Both rape incidents were witnessed by the brother of Maricel.

141

Issue: Whether or not the aggravating circumstance of dwelling be appreciated against


Calonqui.

Held: No. A review of the records shows that there are no aggravating circumstances
present in the case at bar. Dwelling cannot be appreciated because Marinel and the
appellant lived in the same house at the time of the rape incidents.[38] As a result,
the rationale for considering dwelling as an aggravating circumstance, i.e., the
violation by the offender of the sanctity of the home of the victim by trespassing
therein to commit a crime,[39] is absent. His conviction is nonetheless affirmed.

People of the Philippines, Appellee vs. Catalino Melendres, Jr.,


Bernardino Kirit, Teodulo Kitay, Appellants, GR No.13494030 April 2003
(Second Division) J. Austria-Martinez

Facts: In the evening of July 23, 1992, first cousins, Rodrigo Hungoy and Mardie
Balasabas together with Mardies brothers, Syrel and Exor, were resting inside the
house of one Pacifico Gualingco. The four companions are cultivating the farmland
owned by Pacifico and were staying at the house built on the middle of the farm. The
house has two floors, the second floor being two and a half feet above the ground and
accessible through a stair inside the sala of the house. About 9:00 of the same
evening, the barking of the dog awakened Rodrigo and Mardie. They peeped through
the porch and looked at the front yard where they saw three persons in the yard who
they did not immediately recognize because it was dark. The three persons proceeded
to the upper portion of the house and pushed the main door which was not
locked. Syrel and Exor remained asleep while Rodrigo and Mardie jumped out and hid
at the back portion of the house which was not illuminated. They again peeped through
a hole in the bamboo walling of the house which was already dilapidated. There,
Rodrigo and Mardie recognized the faces of the three appellants because the room
where Syrel and Exor were sleeping was illuminated by a tingkaro, a kerosene
lamp. The three went near Syrel and Exor. With the use of a bolo, Catalino and
Bernardino proceeded to hack Syrel while Teodulo hacked Exor. After witnessing the
hacking of Syrel and Exor, Rodrigo and Mardie went straight to the house of Rodrigo
142

which is about 150 meters away from the house where the incident happened. Rodrigo
went directly inside the house ahead of Mardie. Mardie, on the other hand, went to
another house which is approximately ten meters away from Rodrigos house, occupied
by his parents. He informed Rodrigos father and one Ricardo Palomar about the
hacking of Syrel and Exor. Rodrigos father then advised Mardie to go inside the house
where Rodrigo was and to close the door.The following morning Rodrigo and Mardie
informed the latters mother, Lita Balasabas about the incident that happened the
previous night. Together with some relatives and neighbors they then went to the house
where the incident occurred and found the dead bodies of Syrel and Exor. They placed
the cadavers in coffins and brought them to the poblacion and had them
autopsied. After securing the death certificates of her sons, Lita, together with other
companions went to the police to report the incident. They then buried the bodies on
the same day.
The principal defense of appellants is alibi. The trial court found the appellants guilty
beyond reasonable doubt of double murder and imposed a penalty of two (2) reclusion
perpetua.
Issue: Whether or not the aggravating circumstance of treachery and dwelling should
be appreciated in this case.
Held: The trial court erred in not taking into consideration the aggravating
circumstances of treachery and dwelling that attended the killing of Syrel and Exor.
The aggravating circumstance of dwelling is present in the instant case. Under Article
14, paragraph 3 of the Revised Penal Code, dwelling is aggravating if the crime is
committed in the dwelling of the offended party and the latter has not given
provocation. As contemplated under the law, dwelling may mean temporary
dwelling. Moreover, dwelling may be aggravating even though the victim was not
the owner of the house where the crime was committed. In the present case, it is
not disputed that the house owned by Pacifico Gualingco where Syrel and Exor were
killed was used by the victims as dwelling.

(Abuse of Confidence or Obvious Ungratefulness)

People of the Philippines, Plaintiff-Appellee vs. Salvador Arrojado,


Accused-Appellant, GR No.13049231 January 2001
(Second Division) J. Mendoza
143

Facts: Accused-appellant Salvador Arrojado and the victim Mary Ann Arrojado are first
cousins. Starting February 15, 1996, accused-appellant lived with the victim and her
father. He helped care for the victim's father, for which he was paid a P1,000.00 monthly
salary.
In the early morning of June 1, 1996, accused-appellant went to the house cousin,
Erlinda Arrojado Magdaluyo, and reported that the victim had committed suicide. In
response, Erlinda, together with her husband Romulo Magdaluyo and her father
Teodorico Arrojado, went with accused-appellant to the house where they found the
victim dead. The victim, who was bloodied, was lying on her left side facing the
bedroom door with her hands clasped together. On her bed was a rosary and a cruifix.
Near her was a knife. Erlinda recognized it to be the knife kept in the kitchen. Erlinda
also noticed that the electric fan was turned on full blast, while all the windows were
closed except the window on the east side which was slightly open. As he went to the
other room, where the victim's father stayed, accused-appellant told Erlinda that he was
afraid he might be suspected as the one responsible for the victim's death. The matter
was reported to the police which noticed that the victim's room "was very neat as if
nothing happened." The police saw no signs of forcible entry.
Issue: Whether or not the aggravating circumstance of abuse of confidence or obvious
ungratefulness is present.
Held: The aggravating circumstance of abuse of confidence, however, is present
in this case. For this aggravating circumstance to exist, it is essential to show
that the confidence between the parties must be immediate and personal such as
would give the accused some advantage or make it easier for him to commit the
criminal act. The confidence must be a means of facilitating the commission of
the crime, the culprit taking advantage of the offended party's belied that the
former would not abuse said confidence. In this case, while the victim may have
intimated her fear for her safety for which reason she entrusted her jewelry and bank
book to Erlinda Arrojado Magdaluyo, her fears were subsequently allayed as shown by
the fact that she took back her personal effects from Erlinda. Thinking that accusedappellant would not do her any harm, because he was after all her first cousin, the
victim allowed accused-appellant to sleep in the same room with her father and left the
bedroom doors unlocked.

(Nighttime, Uninhabited Place or Band)


144

People of the Philippines, Appellee vs. William Ancheta, Edgardo Areola


Antos Dacanay, Lito Dela Cruz, Felipe Ulep @ Boy Ulep and Ely Calcala, Accused,
Felipe Ulep @ Boy Ulep, Accused-Appellant, GR No.1439354 June 2004
(Third Division) J. Corona

Facts: On March 20, 1987, Alfredo Roca he was in his farm to thresh palay. Later on,
an owner-type jeep with trailer stopped at a spot not far from his hut. He recognized the
occupants as accused Antos Dacanay, Edgardo "Liling" Areola, William Ancheta, Lito de
la Cruz, Ely Calacala and appellant Felipe "Boy" Ulep who all alighted from the jeep.
Dacanay, Areola and Ancheta stood on one side of the irrigation canal facing Marjun
Roca who was standing on the other side. Alfredo saw Dacanay suddenly pull out a
gun and shoot Marjun on the head, causing the latter to fall to the ground. As he lay on
the ground, Marjun was again shot, this time by Areola and Ancheta. Thereafter, Ulep,
de la Cruz and Calacala started firing at Alfredos hut. Alfredo was not hit, however,
because he was able to get out of the hut and dive into the irrigation canal in the nick of
time. However, Benita and Febe were fatally hit by the initial volley of gunfire. The
assailants fired at Alfredo in the canal but they did not hit him. Ancheta then hurled a
grenade which exploded near the hut. When the group ran out of bullets, Alfredo
emerged from the canal and hid inside his hut. He saw the group load onto the trailer 35
sacks of palay, each containing an average of 50 kilos valued at P4.50 per kilo. Alfredo
owned the stolen palay. Appellant Ulep and his companions then boarded their jeep and
left.
Issue: Whether or not the aggravating circumstances of treachery and with a band are
present.
Held: Of the aggravating circumstances alleged in the information, only treachery and
band were established.
The offense was proven to have been executed by a band. A crime is committed by a
band when at least four armed malefactors act together in the commission
thereof. In this case, all six accused were armed with guns which they used on
their victims. Clearly, all the armed assailants, including appellant, took direct
part in the execution of the robbery with homicide.
145

People of the Philippines, Plaintiff-Appellee vs. Edgar Evangelio y Gallo, Joseph


Evangelio, Atilano Agaton y Obico and Noel Malpas y Gracia, Accused,
Joseph Evangelio, Accused-Appellant, GR No.18190231 August 2011
(Third Division) J. Peralta

Facts: On 3rd of October 2001, in the City of Tacloban, all accused, armed with a
handgun and deadly/bladed weapons forcibly enter the inhabited house/residence of
BBB and while inside, take, and carry away from said residence some personal
properties amounting to PhP336,000.00. On the occasion of the said robbery, accused
feloniously have carnal knowledge of AAA, [2] a 17-year-old minor, against her will and
consent and at the time when the latter lost consciousness after her head was banged
on the bathroom floor. Trial court convicted accused for the crime charged and
appreciated the aggravating circumstances of nighttime, committed by a band, dwelling
and unlawful entry. On appeal, the CA affirmed the decision with modification. CA did
not consider the aggravating circumstances of nighttime and unlawful entry. Hence, this
petition.
Issue: Whether or not trial court erred in appreciating the aggravating circumstance of
nighttime in the commission of the crime
Held: Yes. The CA correctly ruled in not considering the aggravating circumstances of nighttime
and unlawful entry.

x x x [T]he aggravating circumstances of nighttime and unlawful entry cannot be


considered. Under the law, specifically Sections 8 and 9, Rule 110 of the Revised
Rules on Criminal Procedure, as well as jurisprudence, it is required that
qualifying as well as aggravating circumstances must be expressly and
specifically alleged in the Complaint or Information; otherwise, the same will not
be considered by the court against the appellant, even if proved during the trial.
And, this principle is applicable to all criminal cases.
The information merely stated that the crime took place on or about the 3 rd day of
October 2001, without specifying the time of its commission. Also nighttime is
considered an aggravating circumstance only when it is deliberately sought to
prevent the accused from being recognized or to ensure escape. There must be
proof that this was intentionally sought to ensure the commission of the crime,
and that the accused took advantage of it to insure his immunity from captivity.
Here, there is a paucity of evidence that nighttime was purposely, deliberately, and
146

especially sought by the accused. The mere fact that the offense was committed at
night will not suffice to sustain a finding of nocturnity.
The trial court and the CA however correctly appreciated the aggravating circumstance of the
commission of a crime by a band. In the crime of robbery with rape, band is considered as
an aggravating circumstance. The prosecution established that one of the accused was
armed with a handgun, while the other three had knives when they committed the crime.

People of the Philippines, Appellee vs. Alvin Villanueva, Appellant,


GR No.13917711 August 2003

(En Banc) J. Corona

Facts: On November 15, 1996, at around 12 midnight, Marife Brabante was attending
to her duties as cashier a store owned by her mother, Rita Binay-an. She was assisted
by Cheryl Dapiaoen and George Bautista. They were about to close the store when
appellant, together with a certain Jerry, Teddy and an unidentified person, arrived. They
occupied one of the tables and started drinking the liquor which they brought with them.
Appellant then asked Marife if they could stay until 5:00 a.m. but Marife refused since
they were unruly. When the group was about to leave, Cheryl asked appellant to pay the
amount of P35 as cover charge. At this point, appellant became angry and threw two
bottles of gin on the floor. Marifes brother, Orland, who was sleeping in one of the
rooms of the store, was awakened by the noise. He went out and told appellant not to
create trouble. But appellant shoved him and left with a warning that he would return to
kill somebody. Appellant got on his tricycle and bumped the door of the store while his
companions threw rocks at it.
Meanwhile, Marifes other brother, Otoleo, got up from his bed and asked Cheryl to go
with him to buy balut at the nearby store, which was only eight meters away from their
store. After 30 minutes, appellant returned to Highlander Store with a knife. He walked
past Marife and told her that she was not the one he was going to kill. Appellant went
toward the Seven Star Store where Otoleo and Cheryl were then buying balut. Upon
reaching the store, appellant suddenly stabbed Otoleo at the back. The victim turned to
face appellant but the latter again stabbed him twice on the left armpit. Otoleo fell to the
147

ground and appellant ran away. Marife, who was outside the Highlander Store, rushed
to the bloodied body of her brother and hugged him. She brought the victim to the
Rosario District Hospital in Rosario, La Union where he was declared dead on arrival.
The appellant escaped from prison before the prosecution had completed the
presentation of its evidence. Hence, he was deemed to have waived his right to present
his evidence to dispute the charge. The trial court convicted the appellant of the offense
charged.
Issue: Whether or not the qualifying circumstance of nighttime is correctly appreciated
by the lower court in convicting the accused-appellant.
Held: The trial court erred in appreciating nighttime as an aggravating circumstance. At
the outset, it should be noted that the circumstance of nighttime was not alleged
in the information as mandated by law. And even if alleged, nighttime cannot
properly be considered in this case because, although the crime was committed
late at night, there was no evidence that nighttime facilitated the commission of
the crime, or that it was specially sought by the offender to ensure the
commission thereof, or that the offender took advantage of it for impunity.29 The
record does not show that appellant deliberately sought the cover of darkness when he
assaulted Otoleo Brabante. The prosecution established no more than the simple fact
that the crime was committed at night.

People of the Philippines, Plaintiff-Appellee vs. Resty Silva, Rodolfo Sadango,


alias Dupong and Jun-Jun Flores, Accused-Appellants,
GR No.1408718 August 2002

(En Banc) Per Curiam

Facts: On September 3, 1996, Edmundo Ceriales, passed by his brother Manuels


house. There, he saw six other men. Some of these men were playing tong-its while the
others were merely watching the game. Suddenly three men arrived. One pointed an
armalite gun to all those present and ordered them to lie on their stomach. Another
ordered brothers Edmundo and Manuel to get out of the house.
As soon as they were out, they were made to lie face down. Upon the order of the
appellant, Edmundo and Manuel were tied together by two armed men (Rodolfo
Sandangao and Junjun Flores). Edmundo recognized Sandangao through the light
148

coming from a vehicle. They continued walking side by side as they were tied. Edmundo
whispered Manuel that he recognized the assailant.
Upon reaching Lucing Guerreros coconut plantation, they were made to sit on a hollow
block. It was at this point where appellant Resty Silva focused a flashlight on himself.
He then asked the brothers if they knew their abductors. When he and Manuel positively
answered, appellant Silva retorted Papano yan Dupong, kilala pala tayo, obligado na
nating patayin.
Edmundo was thereafter separated from Manuel and was brought some 20m away.
Sandangao tied the feet of Edmundo. He then tried to free himself jumping away from
where he was until he fell into a hole. While there, he tried to untie his hands and feet till
he heard the scream of Manuel followed by sound akin to the cutting of a tree. When
Edmundo was already untying his feet he was given a warning to come out or they
would kill Manuel if he failed. He slowly stuck out his head from the hole and failing to
see the three men, he ran away fast until he reached the place of his godfather Andres
Macatiag. He spent the night at the house of Macatiag, upon the latters offer. The next
day, Macatiag went to Edmundos house to check the latters family. Later that day, the
headless body of Manuel was found. Macatiag proceeded were the body was found. He
saw that the feet were still tied. Edmundo was able to identify the body because of the
clothes he was wearing. Several days after the beheading incident the missing head of
the victim Manuel Ceriales was found at Baler, Aurora.
Issue: Whether nighttime attended the commission of the crime.
Held: The facts as related by Edmundo, who was a direct witness to the crime, being a
victim himself, and as corroborated by the other witnesses, clearly established the
crimes of murder and attempted murder. Qualifying circumstance of evident
premeditation and aggravating circumstances of treachery and nighttime are present.
By and of itself, nighttime is not an aggravating circumstance, however, it
becomes aggravating only when: (1) it is especially sought by the offender; or (2)
it is taken advantage of by him; or (3) it facilitates the commission of the crime by
ensuring the offenders immunity from capture.[33] In this case, the trial court
correctly appreciated nighttime as aggravating considering that nighttime facilitated the
abduction of the Ceriales brothers, the killing of Manuel Ceriales and the attempt to kill
Edmundo Ceriales. Evidence shows that accused-appellants took advantage of the
darkness to successfully consummate their plans. The fact that they brought with them
a flashlight clearly shows that they intended to commit the crime in darkness.

People of the Philippines, Plaintiff-Appellee vs. Raelito Librando,


Larry Surdillas and Eddie Purisima, Accused-Appellants,
149

GR No.1322516 July 2000

(Second Division) J.De Leon Jr.

Facts: On December 11, 1996, Edwin Labandero brought his 8-yr old daughter Aileen
to market. On their way home, Edwin, Aileen and a relative, Fernando de los
Santos, traversed a hilly portion of the trail when they met accused-appellants Raelito
Librando, Larry Surdillas and Eddie Purisima. Raelito inquired from Edwin the
whereabouts of Fernando and without any warning hit Edwin with a piece of wood.
Eddie Purisima followed suit and delivered another blow to Edwin. Edwin ran but he
was chased by Raelito. Thereafter, the three men took turns hitting Edwin with pieces of
wood until the latter fell and died. Although it was already dark at that time, Aileen had
no trouble identifying the accused-appellants since Edwin was carrying a lighted torch.
While the men took turns in mauling the deceased, Edwin, Fernando took Aileen with
him and ran to report the incident to the Barangay.
On December 12, 1996, Raelito voluntarily surrendered himself to the police who came
looking for him at his fathers house while Eddie and Larry were invited for questioning
at the police headquarters. The three men were asked to participate in a police line up
during which Aileen was asked to identify her fathers assailants. According to PO2
Dencing, Aileen positively identified Raelito Librando, Larry Surdillas and Eddie
Purisima as her father's assailants. After the line up, however, only Raelito was
detained while Larry and Eddie were released.
The trial court convicted them beyond reasonable doubt of the crime of murder, qualified
by abuse of superior strength and taking into consideration the aggravating
circumstances of nighttime and uninhabited place, considered only as one, and the
mitigating circumstance of voluntary surrender in favor of all the accused, they are
sentenced to suffer the penalty of RECLUSION PERPETUA.
Issue: Was it proper to consider nighttime and uninhabited place as just one
aggravating circumstance?
Held: The trial court did not err in considering the nighttime and uninhabited place as
just one aggravating circumstance. In the case of People vs. Santos it has been
held that if the aggravating circumstances of nighttime, uninhabited place or
band concur in the commission of the crime, all will constitute one aggravating
circumstance only as a general rule although they can be considered separately if
their elements are distinctly perceived and can subsist independently, revealing a
greater degree of perversity.
150

(Recidivism)

People of the Philippines, Appellee vs. Francisco Dacillo alias Dodoy and
Joselito Pacot y Ibarra, Accused, Francisco Dacillo alias Dodoy, Appellant,
GR No.14936814 April 2004, (En Banc) J. Corona

Facts: Appellant Dacillo together with Joselito Pacot were indicted for murder in an
information and that the commission of the foregoing offense was attended by the
aggravating circumstance of abuse of superior strength.

The case against appellants co-accused, Joselito Pacot, was provisionally dismissed
for lack of sufficient evidence to identify him with certainty.Appellant was arraigned on
February 21, 2001 and, assisted by counsel, pleaded not guilty. Pre-trial was conducted
on March 1, 2001 and trial ensued thereafter.

When the body was discovered in the evening of February 11, 2000, appellant
immediately left for Cebu City, arriving there the next day, February 12, 2000. He stayed
in Cebu City until his arrest the following year.

On May 31, 2001, the trial court rendered judgment finding appellant guilty of murder
and imposed upon him the supreme penalty of death. The Court finds the accused
Francisco Dacillo, guilty beyond reasonable doubt of the crime of murder for the death
of Rosemarie Tallada, as defined and penalized under Art. 248 of the Revised Penal
Code, as amended. Considering the aggravating circumstance of recidivism with no
mitigating circumstance to offset the same, he is hereby sentenced to the extreme
penalty of death.

151

Issue: Whether or not it is necessary, in recidivism as an aggravating circumstance, to


be alleged in the information?

Held: The Court, however, finds that the trial court erred in imposing the death penalty
on the ground that appellant admitted during re-cross examination that he had a prior
conviction for the death of his former live-in partner. The fact that appellant was a
recidivist was appreciated by the trial court as a generic aggravating circumstance
which increased the imposable penalty from reclusion perpetua to death.

In order to appreciate recidivism as an aggravating circumstance, it is necessary


to allege it in the information and to attach certified true copies of the sentences
previously meted out to the accused. This is in accord with Rule 110, Section 8 of
the Revised Rules of Criminal Procedure which states:

SEC. 8. Designation of the offense. - The complaint or information shall


state the designation of the offense given by the statute, aver the acts or
omissions constituting the offense, and specify its qualifying and
aggravating circumstances. If there is no designation of the offense,
reference shall be made to the section or subsection of the statute
punishing it.

(By means of Inundation, Fire, etc.)

People of the Philippines, Appellee vs. Edna Malngan y Mayo, Appellant


GR No.17047026 September 2006

(En Banc) J. Chico-Nazario

152

Facts: On January 2, 2001, when Remigio Bernardo and his tanods saw the accusedappellant Edna, one hired as a housemaid by Roberto Separa Sr., with her head turning
in different directions, hurriedly leaving the house of her employer. She was seen to
have boarded a pedicab which was driven by a person later identified as Rolando
Gruta. She was heard by the pedicab driver to have instructed that she be brought to
Nipa Street, but upon her arrival there, she changed her mind and asked that she be
brought instead to Balasan Street where she finally alighted, after paying her fare.
Thirty minutes later, it was discovered that a fire gutted the house of the employer of the
housemaid. Later on, it was reported that a woman (the same housemaid) who was
acting strangely and suspiciously. The accused was later on apprehended and brought
to the barangay hall for investigation. Upon inspection, a disposable lighter was found
inside accused-appellants bag.
Thereafter, accused-appellant Edna confessed to the barangay chairman in the
presence of multitudes of angry residents outside the barangay hall that she set her
employers house on fire because she had not been paid her salary for about a year
and that she wanted to go home to her province but her employer told her to just ride a
broomstick in going home. The accused was charged and convicted with the crime of
arson with multiple homicide.
Issue: Whether or not Edna Malngan was guilty of the crime of destructive arson or
simple arson?
Held: The crime committed by the accused-appellant is Simple Arson and not Arson
with Multiple Homicide. The Supreme Court ruled that there is no complex crime of
Arson with Multiple Homicide.
The case falls under simple arson since from a reading of the body of the information it
can be seen that it states that the accused, with intent to cause damage, deliberately
set fire upon the two-storey residential house, that by reason and on the occasion of the
said fire, which were the direct cause of their death. It is clear that her intent was
merely to destroy her employers house through the use of fire.
When fire is used with the intent to kill a particular person who may be in a house
and that objective is attained by burning the house, the crime is murder only.
When the Penal Code declares that killing committed by means of fire is murder,
it intends that fire should be purposely adopted as a means to that end. There can
be no murder without a design to take life. In other words, if the main object of
the offender is to kill by means of fire, the offense is murder. But if the main
objective is the burning of the building, the resulting homicide may be absorbed
by the crime of arson. The latter being the applicable one in this case.

153

People of the Philippines, Appellee vs. Antonio Comadre, George Comadre, and Danilo
Lozano, Appellants
GR No.1535598 June 2004

(En Banc) Per Curiam

Facts: At around 7:00 oclock in the evening of August 6, 1995, Robert Agbanlog,
Jimmy Wabe, Gerry Bullanday, Rey Camat and Lorenzo Eugenio were having a
drinking spree on the terrace of the house of Roberts father, Jaime Agbanlog. Jaime
was seated on the banister of the terrace listening to the conversation of the
companions of his son.

As the drinking session went on, Robert and the others noticed appellants Antonio
Comadre, George Comadre and Danilo Lozano walking. The three stopped in front of
the house. While his companions looked on, Antonio suddenly lobbed an object which
fell on the roof of the terrace. Appellants immediately fled by scaling the fence of a
nearby school.

The object, which turned out to be a hand grenade, exploded ripping a hole in the roof
of the house. Robber Agbanlog and his companions were hit by shrapnel and slumped
unconscious on the floor. They were all rushed to the hospital for medical treatment.
However, Robert Agbanlog died before reaching the hospital for wounds sustained
which the grenade explosion inflicted. Roberts companions sustained shrapnel injuries.

The appellants were arrested the following day but denied any participation in the
incident, claimed they were elsewhere when the incident occurred and that they had no
animosity towards the victims whatsoever.

After trial, the court a quo convicted appellants of the complex crime of Murder with
Multiple Attempted Murder for having conspiring, confederating and mutually helping
one another, with intent to kill and by means of treachery and with the use of an
explosive.
154

Issue: Whether or not the use of explosive qualifies the crime to murder?
Held: Yes, the killing by means of explosives qualifies the crime to murder. The
information alleges that both treachery and the use of explosive attended the crime.
Since both circumstances can qualify the killing to murder under Article 248 of
the Revised Penal Code, the Supreme Court held that when the killing is
perpetrated with treachery and by means of explosives, the latter shall be
considered as a qualifying circumstance. Not only does jurisprudence support
this view but also, since the use of explosives is the principal mode of attack,
reason dictates that this attendant circumstance should qualify the offense
instead of treachery which will then be relegated merely as a generic aggravating
circumstance.

(Evident Premeditation)

People of the Philippines, Plaintiff-Appellee vs. Alex Paling, Ernie Vilbar @ Dodong (at
large) and Roy Vilbar, Accused, Alex Paling, Accused-Appellant
GR No.18539016 March 2011

(First Division) J. Velasco Jr.

Facts: In the evening of July 1, 1996, Richard, Jojo, and Rolly were in the house of
Paling watching television. At around 9:15 p.m., the group left the said house and
decided to proceed to the other house of Paling situated in the latters farm. En route,
Jojo and Rolly, along with the victim, Walter Nolasco (Walter), were invited by Paling,
Ernie, and Barangay Kagawad Rene Mondejar to a drinking spree at the house of the
latter. Jojo, Rolly, and Walter accepted the invitation, while Richard just waited for them
outside the house of Paling.6
About 15 minutes later, Richard went back to his companions and told them that they
had to go home since they still have to go to school the following morning. The three
acceded, but Ernie convinced Walter to stay with them a little longer. Thus, Richard,
Jojo, and Rolly went ahead, while Walter stayed behind. 7
155

At around 10:00 p.m., Francisco, the uncle-in-law of Walter, 8 was roused from his sleep
by the barking of his dogs. When he went out to find out why the dogs were barking, he
saw Vilbar and Ernie walking beside Walter. They were heading towards Brgy.
Greenhills where Palings farmhouse was located.9
At around 10:30 p.m. that same night, Richard, who was already asleep in the
farmhouse of Paling, was awakened when he heard Jeniline Paling-Bernesto, the
daughter of Paling, shout, "Kill him in a distance. Dont kill him here, kill him away from
here." When Richard went outside to find out what was happening, he saw Paling,
Vilbar, and Ernie assaulting Walter. Vilbar was holding Walter, while Paling and Ernie
were stabbing him. The three left, bringing with them the cadaver of Walter. The
following day, July 2, 1996, at 10:00 a.m., Walters cadaver was found in the farm of one
Jonathan Policarpio.
RTC convicted the accused for murder. On appeal, CA affirmed the trial courts decision
in toto. In convicting Paling the RTC appreciated the aggravating circumstance of
evident premeditation.
Issue: Whether or not the trial court erred in appreciating the aggravating circumstance
of evident premeditation
Held: We disagree. The killing of Walter was not attended by evident premeditation. In
this regard, it is worth noting that "qualifying circumstances cannot be presumed, but
must be established by clear and convincing evidence as conclusively as the killing
itself."

The qualifying circumstance of evident premeditation cannot be considered since there


was neither proof that Paling and the other accused indeed planned or determined to kill
Walter nor was there any proof that the perpetrators had sufficient lapse of time
between the determination and the execution to allow them to reflect. In People v.
Dadivo, this Court enumerated the requirements to prove evident premeditation, to wit:
x x x The requirements to prove evident premeditation are the following: (1) the
time when the offender determined to commit the crime; (2) an act manifestly
indicating that the culprit has clung to his determination; and (3) sufficient lapse
of time between the determination and execution to allow him to reflect upon the
consequences of his act.52

156

Despite the foregoing disquisition, the crime committed by Paling is still murder and not
homicide, since the killing of Walter is qualified by taking advantage of superior strength.

People of the Philippines, Plaintiff-Appellee vs. Amador Segobre y Quijano Appellant, GR


No.1698778 June 2004
(Second Division) J. Quisumbing

Facts: Villafaa testified that on March 15, 1997, at around 5:00 p.m., he was walking
along Crisostomo Street, Antipolo City, when he saw appellant standing at the nearby
electric post. Two minutes later, he saw appellant block the victim Roberto Crescini, who
was coming from Sumulong Highway on a bicycle. Appellant then grabbed Crescinis
right shoulder with his left hand and stabbed Crescini on the right chest. After the
incident, appellant ran away. Thereafter, Villafaa left. The following day, he learned that
Crescini had died. An information was filed charging appellant with the crime of murder.
The lower court convicted appellant of the crime of murder and found the circumstances
of treachery and evident premeditation to have attended the killing. The Court of
Appeals affirmed the lower court, but appreciated the circumstance of treachery only.
Hence this appeal.
Issue: Whether or not evident premeditation attended the commission of the crime.
Held: The circumstance of evident premeditation could not be appreciated in
connection with the killing of Crescini. For evident premeditation to be
appreciated, the following requisites must be shown: (1) the time when the
accused determined to commit the crime; (2) an act manifestly indicating that the
accused has clung to his determination; and (3) a sufficient lapse of time between
such a determination and the actual execution to allow the accused time to reflect
upon the consequences of his act. None of these requisites have been shown from
the facts of this case. The records do not show the time and date when appellant
resolved to commit the crime. Absent this first requisite, the lapse of time as stated in
the third requisite cannot be proved. The second element cannot likewise be proved,
absent any showing that appellant performed acts manifestly indicating that he clung to
his determination of killing Crescini.

(Craft, Fraud or Disguise)


157

People of the Philippines, Plaintiff-Appellee vs. Vivencio Labuguen @ Dencio, AccusedAppellant, GR No.1278499 August 2000
(En Banc) Per Curiam

Facts: The deceased Bonifacio Angeles was engaged in buying cows and selling them
to the public market. One day, the accused Vivencio Labuguen went to him and told him
that he knows of three big cows for sale and that the place where they are is near.
Believing on such declaration, he took money from his cabinet at his house amounting
to P40,000 and then drove in his motorcycle with the accused to see the cows. On their
way to see the cows, they have been seen together by several witnesses who later on
identified them in court as the victim and the accused respectively. The accused
according to the witness was wearing a jacket and with a handkerchief tied on his
forehead. One of the witnesses, a driver of a minibus testified that while driving on his
way to his destination, he saw a man behind the talahibs and he noticed that he was
wiping something from his head and right face. It was the same man whom his
conductor identified as the one who stopped their bus and rode on it. His conductor
testified further that he noticed that the mans jacket was soaked with blood including
his pants and that he did not talk when asked where he was headed to and instead just
gave his fare. The conductor even noticed that there was a lot of money on the breast
side pocket of his jacket and that one bill was even falling. The man then alighted from
the minibus after reaching his destination without saying any word. Later that afternoon,
a news broke out that a mans body was found dead in the middle of the ricefield. He
was later on identified as Bonifacio Angeles. Appellant placed reliance on his defense of
denial and alibi. Vehemently denying the charge against him, he asseverated that he
could not have committed the crime on October 27, 1994 because he left for
Maconacon, Isabela on October 17, 1994, to manage the logging operation of a certain
Orlando Ramos and stayed there until December 20, 1994. Based on the strength of
the testimony of the witnesses, complaint and information were filed against Vivencio
and the Regional Trial Court found him guilty of the crime of Robbery with Homicide and
sentenced with the penalty of death. The case was brought to the Supreme Court for
automatic review.
Issue: Whether or not the court has correctly appreciated the employment of generic
aggravating circumstance of fraud and craft in the commission of the crime even if not
alleged in the information?
Held: Though not alleged in the Information, the generic aggravating circumstances of
fraud and craft were properly appreciated by the trial court. Craft involves intellectual
trickery and cunning on the part of the offender. When there is a direct
inducement by insidious words or machinations, fraud is present. By saying that
he would accompany the victim to see the cows which the latter intended to buy,
appellant was able to lure the victim to go with him.
158

(Abuse of Superior Strength)

People of the Philippines, Plaintiff-Appellee vs. Conrado Laog y Ramin,


Accused-Appellant, GR No.1783215 October 2011
(First Division) J. Villarama Jr.

Facts: AAA testified that at around 6pm of June 6, 2000, she and her friend, Jennifer
Patawaran-Rosal, were walking along the rice paddies on their way to apply for work at
a canteen. Suddenly, appellant, who was holding an ice pick and a lead pipe, waylaid
them and forcibly brought them to a grassy area at the back of a concrete wall. Without
warning, appellant struck AAA in the head with the lead pipe causing her to feel dizzy
and to fall down. When Jennifer saw this, she cried out for help but appellant also hit
her on the head with the lead pipe, knocking her down. Appellant stabbed Jennifer
several times with the ice pick and thereafter covered her body with thick grass.
[5]
Appellant then turned to AAA. He hit AAA in the head several times more with the
lead pipe and stabbed her on the face. While AAA was in such defenseless position,
appellant pulled down her jogging pants, removed her panty, and pulled up her blouse
and bra. He then went on top of her, sucked her breasts and inserted his penis into her
vagina. After raping AAA, appellant also covered her with grass. At that point, AAA
passed out. The trial court found accused guilty beyond reasonable doubt for both
crimes (murder and rape). Likewise, the trial court appreciated the aggravating
circumstance of abuse of superior strength. On appeal, CA affirmed with some
modification. Hence, this petition.
Issue: Whether or not the court erred in applying the aggravating circumstance of
abuse of superior strength
Held: The aggravating circumstance of abuse of superior strength is considered
whenever there is notorious inequality of forces between the victim and the
aggressor that is plainly and obviously advantageous to the aggressor and
purposely selected or taken advantage of to facilitate the commission of the
crime.[50] It is taken into account whenever the aggressor purposely used
excessive force that is out of proportion to the means of defense available to the
person attacked.[51]
In this case, as personally witnessed by AAA, appellant struck Jennifer in the head with
a lead pipe then stabbed her repeatedly until she was dead. Clearly, the manner by
which appellant had brutally slain Jennifer with a lethal weapon, by first hitting her in the
head with a lead pipe to render her defenseless and vulnerable before stabbing her
repeatedly, unmistakably showed that appellant intentionally used excessive force out of
159

proportion to the means of defense available to his unarmed victim. As aptly observed
by the appellate court:
It has long been established that an attack made by a man with a deadly weapon upon
an unarmed and defenseless woman constitutes the circumstance of abuse of that
superiority which his sex and the weapon used in the act afforded him, and from which
the woman was unable to defend herself. Unlike in treachery, where the victim is not
given the opportunity to defend himself or repel the aggression, taking advantage of
superior strength does not mean that the victim was completely defenseless. Abuse of
superiority is determined by the excess of the aggressors natural strength over that of
the victim, considering the momentary position of both and the employment of means
weakening the defense, although not annulling it. By deliberately employing deadly
weapons, an ice pick and a lead pipe, [a]ccused-[a]ppellant clearly took advantage of
the superiority which his strength, sex and weapon gave him over his unarmed
victim. The accused-appellants sudden attack caught the victim off-guard rendering her
defenseless.

People of the Philippines, Appellee vs. Ben Ambrocio, Benigno Ambrocio Sr.,
Benigno Ambrocio Jr (at large) Joseph Andrade and Carlito Francisco (at large),
Accused, Ben Ambrocio, Benigno Ambrocio Sr. and Joseph Andrade,
Accused-Appellant, GR No.14026729 June 2004
(Second Division) J. Quisumbing

Facts: At around 1:30 p.m. of February 24, 1998, a certain Roger Domingo reported
that he found coconut lumber which could have been felled without permit, in a
construction site in Sitio Nasunog, Dalipdip, Altavas, Aklan. [4] As barangay captain of
Dalipdip, part of Masangyas duties was to issue permits to cut coco lumber. He
investigated the report and instructed Elienito Gervacio, a member of the Lupong
Tagapamayapa, to verify Domingos report and to wait for him at Sitio Nasunog.
At around 2:00 p.m., Masangya recalled, he was accompanied by the victim, Roberto
Sanchez, to Sitio Nasunog. There they saw the pile of coconut lumber beside the road
160

near the construction site. He noticed that in the house under construction, there was a
drinking spree going on amongst the five accused Carlito Francisco, Joseph Andrade,
Benigno Ambrocio, and his sons Ben and Benny. They alighted from the
motorcycle. Ben Ambrocio walked towards the two until Ben was six meters away from
them with only the construction sites bamboo fence separating them. He heard Ben
ask, What are you doing there, captain? He noticed that Ben and his companions
were already drunk. He replied, Who are you to question me? I am
the barangay captain here. Upon hearing this reply, the other four accused came down
from the house, and all five jumped over the fence. With their bolos drawn, the five
approached them. Sanchez suggested that they talk things over calmly, saying, We
will just talk peacefully. Without warning, Ben suddenly hacked Sanchez at the back. [10]
Masangya testified that at that point, he shouted to the victim, Berto, you run
away. He himself sped off, but Benigno Ambrocio, Francisco and Andrade pursued
him. They failed to overtake him.[11]Sanchez, however, was not so lucky. Before he
could run away, he suffered another blow. This time Benny Ambrocio struck him at the
back with his bolo. The victim fell. All the five accused then gathered around Sanchez
and continued hacking him to death. Afterwards, they carried his body to the area
where it was later found.[12]Trial court found accused guilty for the crime of murder.
Issue: Whether or not the aggravating circumstance of abuse of superior strength be
appreciated in the case

Held: But while we agree that treachery might not have attended the killing of Sanchez,
we rule that there was abuse of superior strength that should be appreciated as an
aggravating circumstance on the part of appellants. Abuse of superior strength is
present when the aggressors purposely use excessive force out of proportion to
the means of defense available to the person attacked. [73]
Superiority in number does not necessarily amount to the aggravating
circumstance of taking advantage of superior strength. But in this case, it has
been shown that the aggressors cooperated in such a way as to secure the
advantage of their numerical strength and advantage. There is proof of the relative
numerical strength of the aggressors and the assaulted, a ratio of 5 to 2. There is also
proof that the aggressors simultaneously assaulted the deceased. When all five
accused, armed with bolos, joined forces to attack and pursue Sanchez and Masangya,
in a concerted effort, they definitely abused their superiority in number and in
arms. Since this aggravating circumstance was alleged in the information and duly
proved, it qualifies properly the killing to murder.
161

People of the Philippines, Appellee vs. Francisco Calpito, alias Francis


Appellant, GR No.12329827 November 2003
(First Division) J. Azcuna

Facts: That on or about the 21 st day of November, 1990, appellant Francisco Calpito
armed with a deadly weapon, with intent to gain did, then and there willfully, unlawfully
and feloniously by means of violence and intimidation on the person of Florentina Villas
rob, take and carry away a shoulder bag containing cash in the amount of P15,000 and
jewelries amounting to P30,000 belonging to Florentina Villas.
The appellant
attack and stab with the said weapon Florentina Villas and
Israel Montilla inflicting wounds on Florentina Villas which caused her death and a
wound on Israel Montilla which necessitated medical attendance on him for a period of
5-7 days and which incapacitated him from performing his usual work for the same
length of time.
However, appellant entered a plea of not guilty and waived pre-trial. But on June 15,
1993, appellant was re-arraigned and after being appraised of the consequences of the
nature of his offense, he changed his plea to one of guilty.
The court a quofinding the charge of Robbery with Homicide unsubstantiated by
evidence, convicted appellant of the crime of murder.
Appellant, thereafter, filed a Motion for Reconsideration arguing that the trial court erred
in convicting him of Murder instead of Homicide and in failing to apply the mitigating
circumstance of minority.
The court denied the motion and affirmed appellants conviction for murder.
Issue: Whether or not the abuse of superior strength qualified the killing to murder?
Held: Yes. A perusal of the facts of the case readily reveals that abuse of superior
strength attended the crime. In several cases, this Court has ruled that this
circumstance depends on the age, size and strength of the parties. It is
considered whenever there is a notorious inequality of forces between the victim
and the aggressor, assessing a superiority of strength notoriously advantageous
for the aggressor which the latter selected or took advantage of in the
commission of the crime. In a recent case, it was held that an attack made by a
162

man with a deadly weapon upon an unarmed and defenseless woman constitutes
an abuse of the aggressors superior strength. The circumstance must apply with
more reason in the present case, where the abuse of superior strength is evident
from the notorious disparity between the relative strength of the victim, a 74-yearold unarmed woman, and the assailant, a young man armed with a knife.

(Treachery)

People of the Philippines, Appellee vs. Angelino Yanson, Appellant,


GR No.1791953 October 2011
(First Division) J. Del Castillo

Facts: In the afternoon of May 12, 1991, Elmo Galfo (Galfo) and the victim, Carlito Magan (Magan),
were drinking whisky in the store of a certain Lorna Tamson (Tamson). [6] After a while, they were joined
by appellant and Salcedo.[7] They finished drinking at around 8:45 in the evening[8] after which Galfo and
Magan walked home together.[9]

After traversing a distance of about half a kilometer, Galfo noticed two persons following them, [10] one of
whom suddenly stabbed Magan at the back.[11] Galfo positively identified the appellant as the person
who stabbed Magan. Galfo tried to approach the victim but appellant and his companion, Salcedo,
rushed towards him thus prompting him to run away for safety. While running, however, he managed to
look back and saw appellant and Salcedo stab the victim some more.[12]

According to Dr. Edgardo Jabasa, the Provincial Health Officer of Guimaras,[13] the victim suffered eight
stab wounds,[14] two of which were fatal and were inflicted at the back.[15]

The trial court found the testimony of Galfo that he personally saw appellant stab Magan at the
back as credible because he was positioned only five arms length away from the victim. The
trial court also appreciated the qualifying circumstance of treachery considering the
suddenness and the surreptitiousness of the attack on the victim.[23] However, it did not lend
163

credence to Galfo's testimony that he also saw Salcedo stab the victim. According to the trial
court, it would be highly improbable for Galfo to look back and witness the stabbing by Salcedo
while running at a fast pace. Thus, it exonerated Salcedo of any participation in the crime.[24]
On appeal, CA affirmed the trial courts finding that it was indeed appellant who stabbed
Magan. Hence, this petition.
Issue: Whether or not the qualifying circumstance of treachery attended the commission of
crime
Held: Yes. Both the trial court and the CA correctly appreciated the qualifying aggravating
circumstance of treachery. There is treachery when the offender commits any of the
crimes against persons, employing means, methods, or forms in the execution, which
tend directly and specially to insure its execution, without risk to the offender arising
from the defense which the offended party might make. The essence of treachery is
that the attack comes without a warning and in a swift, deliberate, and unexpected
manner, affording the hapless, unarmed, and unsuspecting victim no chance to resist
or escape. For treachery to be considered, two elements must concur: (1) the
employment of means of execution that gives the persons attacked no opportunity to
defend themselves or retaliate; and (2) the means of execution were deliberately or
consciously adopted.[44]
The prosecution established that appellant suddenly stabbed the victim from behind
thereby giving him no opportunity to resist the attack or defend himself. As correctly observed
by the appellate court:
It is apparent that there was treachery in the killing of [Magan]. As surely
testified by [Galfo], [appellant] followed the unsuspecting victim when he was
going home and thereafter, deliberately stabbed him in the back which resulted
in the falling of [Magan] to the ground and rendering him defenseless to
[appellant's] further attacks. Verily, [appellant] employed means which insured
the killing of [Magan] and such means assured him from the risk of [Magan's]
defense had he made any. It must also be noted that [Magan] was stabbed four
times in the back and two of these wounds were the proximate cause of his
death. Stabbing from behind is a good indication of treachery x x x

164

People of the Philippines, Appellee vs. Eugenio Piliin y Garcia, Appellant,


GR No.1729668 February 2007
(Second Division) J. Tinga

Facts: On 19 November 1997, Rodrigo arrived at the gate of his house aboard on an
owner-type jeep he was driving. His wife, Norma Zayenis (Norma), who was inside the
house at the time, went out to open the gate. When Rodrigo was about to park his jeep,
a man, later identified as Piliin, suddenly approached him, poked his gun, and fired at
him, hitting the left side of his neck. Rodrigo fell unconscious and the man quickly ran
away. By reason of the gunshot wound, the victim thereafter died. Piliin confesses killing
Rodrigo and implicated Yu and Caballes as his co-perpetrators. However, After trial,
appellant was found guilty for murder. The two other accused, Yu and Caballes were
acquitted for insufficiency of evidence.
During the stage of appeal, Piliin argues that the prosecution failed to establish the
existence of treachery. According to him, the witness failed to see the inception of the
attack because she was in the act of opening the gate for her husband when the latter
was shot. She lacked knowledge of the attending circumstances prior to the shooting
incident. Hence, the trial courts finding of treachery becomes speculative.
Issue: Whether or not treachery must be appreciated as an aggravating circumstance?
Held: Yes. There is treachery when the offender commits any of the crimes
against persons, employing means, methods or forms in their execution, without
risk to himself arising from the defenses which the offended party might make. To
establish treachery, two elements must concur: (1) that at the time of the attack,
the victim was not in a position to defend himself, and (2) that the offender
consciously adopted the particular means of attack employed. The essence of
treachery is the unexpected and sudden attack on the victim which renders the
latter unable and unprepared to defend himself by reason of the suddenness and
severity of the attack. Appellants wife witnessed the incident from its inception up to
its consummation.
In this case, the victim was about to park his car when appellant suddenly appeared and
shot him without any warning. The attack was so sudden that the latter had no
opportunity to repel it or defend himself. It can readily be inferred that the manner of the
attack adopted by appellant manifested treachery. Furthermore, as correctly observed
by the Solicitor General, the weapon used and the nature of the injury inflicted, which
pertained to the lone gunshot fatally wounding appellant, established that appellant
deliberately and consciously adopted the particular mode of attack to ensure the
commission of the offense with impunity.
165

PEOPLE OF THE PHILIPPINES, Plaintiff-Appellee,


- versus GILBERTO VILLARICO, SR. @ BERTING, GILBERTO VILLARICO, JR., JERRY
RAMENTOS, and RICKY VILLARICO,Accused-Appellants.
G.R. No. 158362, April 4, 2011
Third Division, Bersamin, J.
FACTS: At around 7:50 p.m. on August 8, 1999, Haide was busy preparing dinner in the
kitchen of his familys residence in Bolinsong, Bonifacio, Misamis Occidental. At that
time, Haides sister-in-law Remedios Cagatan was attending to her child who was
answering the call of nature near the toilet. From where she was, Remedios saw all the
accused as they stood at the rear of the kitchen aiming their firearms at the door Ricky
Villarico was at the left side, and Gilberto, Jr. stood behind him, while Gilberto, Sr.
was at the right side, with Ramentos behind him. When Gilberto, Jr. noticed Remedios,
he pointed his gun at her, prompting Remedios to drop to the ground and to shout to
Lolita Cagatan, her mother-in-law and Haides mother: Nay, Nay tawo Nay (Mother,
mother, there are people outside, mother). At that instant, Remedios heard three
gunshots.
Francisco Cagatan, the father of Haide, also heard the gunshots just as he was coming
out of the toilet, making him instinctively jump into a hole, from where he was able to
see and recognize Gilberto, Sr., Gilberto, Jr. and Ricky who were then standing by the
kitchen door. They were aiming their guns upward, and soon after left together with
Ramentos.The accused denied the accusations and each proffered an alibi.
ISSUE: Whether or not qualifying circumstance of treachery can be appreciated.
HELD: YES. The essence of treachery is in the mode of attack, not in the relative
position of the victim and the assailant.
The RTC ruled out the attendance of treachery due to its persuasion that the victim
must have been facing his assailants at the time of the assault and was thus not taken
by surprise. The CA differed from the RTC, however, and stressed that regardless of the
position of the victim, the essence of treachery was the element of surprise that the
assailants purposely adopted to ensure that the victim was not able to defend himself.
There is treachery when: (a) at the time of the attack, the victim was not in a position to
defend himself; and (b) the accused consciously and deliberately adopted the particular
means, methods, or forms of attack employed by him. The essence of treachery lies in
the suddenness of the attack that leaves the victim unable to defend himself, thereby
ensuring the commission of the offense. It is the suddenness of the attack coupled with
the inability of the victim to defend himself or to retaliate that brings about treachery;
consequently, treachery may still be appreciated even if the victim was facing the
assailant.
166

Here, the elements of treachery were present. His assailants gunned Haide down while
he was preoccupied in the kitchen of his own abode with getting dinner ready for the
household. He was absolutely unaware of the imminent deadly assault from outside the
kitchen, and was for that reason in no position to defend himself or to repel his
assailants.
The testimonies of Remedios and Francisco on how and where the four accused had
deliberately and strategically positioned themselves could not but reveal their deliberate
design to thereby ensure the accomplishment of their design to kill Haide without any
possibility of his escape or of any retaliation from him.
PEOPLE OF THE PHILIPPINES, Plaintiff-Appellee,
vs.
CHRISTOPHER AVILES, Accused-Appellant.
G.R. No. 172967
December 19, 2007
Third Division, Chico-Nazario, J.
FACTS: On 19 June 2002 at around 7:30 p.m., Novelito Contapay (Contapay) was
driving his passenger jeep along Alexander Street, Poblacion, Urdaneta City, at less
than ten kilometers per hour due to heavy traffic in front of Magic Mall. His lone
passenger, the deceased Danilo Arenas, was seated beside him. Arenas suddenly
shouted apaya. Contapay turned his head and saw Christopher Aviles stabbing Arenas.
Aviles upper body was already inside the jeep with one foot on the running board.
Contapay halted the jeep and tried to help Arenas by holding the hand of Aviles, but the
latter stabbed Contapay on his left knee. Contapay pushed Aviles who ran away.
Contapay alighted from the jeepney, but he was not able to chase Aviles because of his
bleeding left knee. Contapay noticed that Arenas was already unconscious, and he
brought the latter to the Urdaneta Sacred Heart Hospital.
ISSUE: Whether or not qualifying circumstance of treachery attended the commission of
the crime.
HELD: NO. There is treachery when the following requisites are present: (1) the
employment of means, methods, or manner of execution to ensure the safety of the
malefactor from defensive or retaliatory action on the part of the victim and (2) the
deliberate or conscious adoption of such means, method or manner of execution.
The Court of Appeals ruled that the fact that Arenas shouted "Apaya" (perhaps a
shortened form of apay aya, which is more accurately translated in Filipino as bakit ba)
showed that he was probably surprised to see Aviles trying to get inside the jeepney
which was moving slowly because of heavy traffic. The testimony of Contapay that after
hearing Arenas shout "Apaya," he saw Aviles already stabbing Arenas, showed that the
attack was sudden and unexpected.

167

Although Contapay testified that he turned around immediately when the deceased
shouted "Apaya," he did not testify as to how the attack was initiated. Also, considering
that he was driving the jeepney when Arenas was attacked, he could not even have
known how the attack was initiated.
For treachery to be appreciated, it must be present at the inception of the attack. If the
attack is continuous and treachery was present only at a subsequent stage and not at
the inception of the attack, it cannot be considered. Rather than being an expression of
surprise at the presence of Aviles as held by the Court of Appeals, the shout "Apaya" or
"Apay aya," when translated as "Bakit ba," connotes confusion as to why the person to
whom it is spoken is acting the way he is acting. This implies the lapse of several
moments between the commencement of the attack and Arenas shouting.
Qualifying circumstances must be proven beyond reasonable doubt as the crime
itself. It cannot be considered on the strength of evidence which merely tends to show
that the victim was probably surprised to see the assailant trying to get inside the
jeepney. As discussed above, Arenas shout can be interpreted in different ways. In fact,
prosecution witness Dr. Ramon Gonzales even testified that it was possible that Aviles
and Arenas were having a fight.

PEOPLE OF THE PHILIPPINES, plaintiff-appellee,


vs.
JUAN GONZALES ESCOTE, JR. @ Jun Mantika of Sta. Lucia, Angat, Bulacan and
VICTOR ACUYAN y OCHOVILLOS @ Vic Arroyo of Sto. Nio, Poblacion, Bustos,
Bulacan, accused-appellants.
G.R. No. 140756
April 4, 2003
En banc, Callejo, Sr., J.
FACTS: On September 28, 1996 at past midnight, Rodolfo Cacatian, the regular driver
of Five Star Passenger Bus, drove the bus from its terminal at Pasay City to its
destination in Bolinao, Pangasinan. Also on board was Romulo Digap, the regular
conductor of the bus, as well as some passengers. At Camachile, Balintawak, six
passengers boarded the bus, including Victor Acuyan and Juan Gonzales Escote, Jr.
who were wearing maong pants, rubber shoes, hats and jackets. Juan seated himself
on the third seat near the aisle, in the middle row of the passengers' seats, while Victor
stood by the door in the mid-portion of the bus beside Romulo. Another passenger,
SPO1 Jose C. Manio, Jr., was seated at the rear portion of the bus on his way home to
Angeles City. Tucked on his waist was his service gun bearing Serial Number 769806.
Every now and then, Rodolfo looked at the side view mirror as well as the rear view and
center mirrors installed atop the driver's seat to monitor any incoming and overtaking
vehicles and to observe the passengers of the bus.
When the bus was travelling along the highway in Plaridel, Bulacan, Juan and Victor
suddenly stood up, whipped out their handguns and announced a holdup. Petrified,
168

Rodolfo glanced at the center mirror towards the passengers' seat and saw Juan and
Victor armed with handguns. Juan fired his gun upward to awaken and scare off the
passengers. Victor followed suit and fired his gun upward. Juan and Victor then
accosted the passengers and divested them of their money and valuables. Juan
divested Romulo of the fares he had collected from the passengers. The felons then
went to the place Manio, Jr. was seated and demanded that he show them his
identification card and wallet. Manio, Jr. brought out his identification card bearing No.
00898. Juan and Victor took the identification card of the police officer as well as his
service gun and told him: "Pasensya ka na Pare, papatayin ka namin, baril mo rin and
papatay sa iyo." The police officer pleaded for mercy: "Pare maawa ka sa akin. May
pamilya ako." However, Victor and Juan ignored the plea of the police officer and shot
him on the mouth, right ear, chest and right side of his body. Manio, Jr. sustained six
entrance wounds. He fell to the floor of the bus. Victor and Juan then moved towards
the driver Rodolfo, seated themselves beside him and ordered the latter to maintain the
speed of the bus. Rodolfo heard one of the felons saying: "Ganyan lang ang pumatay
ng tao. Parang pumapatay ng manok." The other said: "Ayos na naman tayo pare.
Malaki-laki ito." Victor and Juan further told Rodolfo that after they (Victor and Juan)
shall have alighted from the bus, he (Rodolfo) should continue driving the bus and not
report the incident along the way. The robbers assured Rodolfo that if the latter will
follow their instructions, he will not be harmed. Victor and Juan ordered Rodolfo to stop
the bus along the overpass in Mexico, Pampanga where they alighted from the bus. The
robbery was over in 25 minutes.
ISSUE: Whether treachery may be appreciated against Juan and Victor.
HELD: NO. Robbery with homicide is a crime against property and hence treachery
which is appreciated only to crimes against persons should not be appreciated as a
generic aggravating circumstance. It held in another case that treachery is not
appreciated in robbery with rape precisely because robbery with rape is a crime against
property. These rulings of the Court find support in case law that in robbery with
homicide or robbery with rape, homicide or rape are merely incidents of the robbery,
with robbery being the main purpose and object of the criminal. Indeed, in People vs.
Cando, two distinguished members of this Court advocated a review of the doctrine that
treachery is a generic aggravating circumstance in robbery with homicide. They opined
that treachery is applicable only to crimes against persons. After all, in People vs.
Bariquit, this Court in a per curiam decision promulgated in year 2000 declared that
treachery is applicable only to crimes against persons. However, this Court held
in People vs. Cando that treachery is a generic aggravating circumstance in robbery
with homicide, citing its prior rulings that in robbery with homicide, treachery is a generic
aggravating circumstance when the victim of homicide is killed with treachery. This
Court opted not to apply its ruling earlier that year inPeople vs. Bariquit.

PEOPLE OF THE PHILIPPINES, plaintiff-appellee,


vs.
CLAUDIO TEEHANKEE, JR., accused-appellant.
169

G.R. Nos. 111206-08 October 6, 1995


Second Division, Puno, J.
FACTS: On July 12, 1991, Jussi Olavi Leino invited Roland Chapman, Maureen
Hultman and other friends for a party at his house in Forbes Park, Makati. After a while,
Maureen requested Leino to take her home at Campanilla Street, Dasmarias Village,
Makati. Chapman tagged along. When they entered the village, Maureen asked Leino to
stop along Mahogany Street, about a block away from her house in Campanilla Street.
She wanted to walk the rest of the way for she did not like to create too much noise in
going back to her house. She did not want her parents to know that she was going
home that late. Leino offered to walk with her while Chapman stayed in the car and
listened to the radio.
Leino and Maureen started walking on the sidewalk along Mahogany Street. When they
reached the corner of Caballero and Mahogany Streets, a light-colored Mitsubishi boxtype Lancer car, driven by accused Claudio Teehankee, Jr., came up from behind them
and stopped on the middle of the road. Accused alighted from his car, approached
them, and asked: "Who are you? (Show me your) I.D." Leino thought accused only
wanted to check their identities. He reached into his pocket, took out his plastic wallet,
and handed to accused his Asian Development Bank (ADB) I.D. Accused did not bother
to look at his I.D. as he just grabbed Leino's wallet and pocketed it.
Chapman saw the incident. All of a sudden, he manifested from behind Leino and
inquired what was going on. He stepped down on the sidewalk and asked accused:
"Why are you bothering us?" Accused pushed Chapman, dug into his shirt, pulled out a
gun and fired at him. Chapman felt his upper body, staggered for a moment, and asked:
"Why did you shoot me?" Chapman crumpled on the sidewalk. Leino knelt beside
Chapman to assist him but accused ordered him to get up and leave Chapman alone.
All the while, accused was pointing his gun to and from Leino to Maureen, warning the
latter to shut up. Accused ordered Leino to sit down on the sidewalk. Leino obeyed and
made no attempt to move away. Accused stood 2-3 meters away from him. He knew he
could not run far without being shot by accused.
Maureen continued to be hysterical. She could not stay still. She strayed to the side of
accused's car. Accused tried but failed to grab her. Maureen circled around accused's
car, trying to put some distance between them. The short chase lasted for a minute or
two. Eventually, accused caught Maureen and repeatedly enjoined her to shut up and
sit down beside Leino.
Maureen finally sat beside Leino on the sidewalk. Two (2) meters away and directly in
front of them stood accused. For a moment, accused turned his back from the two. He
faced them again and shot Leino. Leino was hit on the upper jaw, fell backwards on the
sidewalk, but did not lose consciousness. Leino heard another shot and saw Maureen
fall beside him. He lifted his head to see what was happening and saw accused return
to his car and drive away.
170

ISSUE: Whether the killing of Chapman and Hultman and the shooting of Leino was
attended by treachery.
HELD: For the killing of Chapman NO. For the killing of Hultman and the shooting of
Leino, YES.
Concededly, the shooting of Chapman was carried out swiftly and left him with no
chance to defend himself. Even then, there is no evidence on record to prove that
appellant consciously and deliberately adopted his mode of attack to insure the
accomplishment of his criminal design without risk to himself. It appears to us that
appellant acted on the spur of the moment. Their meeting was by chance. They were
strangers to each other. The time between the initial encounter and the shooting was
short and unbroken. The shooting of Chapman was thus the result of a rash and
impetuous impulse on the part of appellant rather than a deliberate act of will. We have
consistently ruled that mere suddenness of the attack on the victim would not, by itself,
constitute treachery. Hence, absent any qualifying circumstance, appellant should only
be held liable for Homicide for the shooting and killing of Chapman.
As to the wounding of Jussi Leino and the killing of Maureen Hultman, we hold that
treachery clearly attended the commission of the crimes. The evidence shows that after
shooting Chapman in cold blood, appellant ordered Leino to sit on the pavement.
Maureen became hysterical and wandered to the side of appellant's car. When
appellant went after her, Maureen moved around his car and tried to put some distance
between them. After a minute or two, appellant got to Maureen and ordered her to sit
beside Leino on the pavement. While seated, unarmed and begging for mercy, the two
were gunned down by appellant. Clearly, appellant purposely placed his two victims in a
completely defenseless position before shooting them. There was an appreciable lapse
of time between the killing of Chapman and the shooting of Leino and Hultman a
period which appellant used to prepare for a mode of attack which ensured the
execution of the crime without risk to himself. Treachery was thus correctly appreciated
by the trial court against appellant insofar as the killing of Hultman and the wounding of
Leino are concerned.

(IGNOMINY)
PEOPLE OF THE PHILIPPINES, appellee, vs.
DOMINADOR CACHOLA y SALAZAR, ERNESTO AMAY y PASCUA, NESTOR
MARQUEZ y MANUEL, BENJAMIN LAEGEN y CAMADO, RODOLFO SAGUN y
171

JIMENEZ, RODEMIR GUERZO y LATAOAN, MELLKE IGNACIO y SALVADOR, and


NELSON C. ECHABARIA, appellants.
G.R. Nos. 148712-15
January 21, 2004
En banc, Per curiam
FACTS: In just an instant, 12-year-old Jessie E. Barnachea lost his mother, an elder
brother, an uncle, and a cousin as a result of the carnage that took place at around 6:00
p.m. of 28 December 1999 right inside their house in Barangay Calumbaya, Bauang, La
Union. Their horrible death was attributed to herein appellants, who, however, pleaded
not guilty to the four separate informations for murder.
Jessie was about to leave their house to watch cartoons in his uncles house next door
when two armed men suddenly entered the front door of their house. The two ordered
Jessie to drop to the floor, and then hit him in the back with the butt of a long gun.
Without much ado, the intruders shot to death Jessies uncle, Victorino V. Lolarga, who
was then in the living room. Jessie forthwith crawled and hid under a bed, from where
he saw the feet of a third man who had also entered the house. The men entered the
kitchen and continued shooting. When the rampage was over and after the malefactors
had already departed, Jessie came out of his hiding place and proceeded to the kitchen.
There he saw his mother, Carmelita Barnachea; his brother Felix Barnachea, Jr.; and
his cousin Rubenson Abance - all slaughtered.
ISSUE: Whether or not ignominy is present in the given case.
HELD: NO. For ignominy to be appreciated, it is required that the offense be committed
in a manner that tends to make its effect more humiliating, thus adding to the victims
moral suffering. Where the victim was already dead when his body or a part thereof was
dismembered, ignominy cannot be taken against the accused. In this case, the
information states that Victorinos sexual organ was severed after he was shot and there
is no allegation that it was done to add ignominy to the natural effects of the act. We
cannot, therefore, consider ignominy as an aggravating circumstance.

PEOPLE OF THE PHILIPPINES, plaintiff-appellee, vs.


BALIWANG BUMIDANG, accused-appellant.
G.R. No. 130630
December 4, 2000
En banc, Per curiam

172

FACTS: On September 29, 1996, at around 2:00 a.m. while father and daughter,
namely, Melencio and Gloria Imbat, were already asleep in their house, the accused
Baliwang Bumidang y Baohan aged 19 years and half-naked, loudly called Melencio
Imbat and asked the latter to open the door. Melencio was aroused from his sleep and
he opened the door downstairs because Bumidang threatened to kill them if the door
was not opened. Accused Bumidang entered and asked the old man to bring him
upstairs. Pointing the weapon at him in a threatening manner, the accused ordered him
to lie in a prone position which he obeyed because he was afraid. Then Bumidang
asked the room of his unmarried daughter, Gloria, aged 56. Melencio, 80 years old,
pointed the room of his daughter which was in the same room but separated by
an aparador. Bumidang went to Gloria's room, still carrying the spear. Suddenly, Gloria
screamed for help, but the octogenarian remained in a prone position as Bumidang
threatened to kill him if he would help his daughter. The accused ordered her to stand
up and removed her pajama, with the panty going along with it. While the accused was
removing her clothes, she sat and struggled. Bumidang then removed his shortpants
and became completely naked. He used the flashlight to examine her genital. He placed
the spear beside her and whenever she attempted to move, he would point the spear at
her. The accused then went on top of her, inserted his penis into her pudenda. At this
instant, Gloria shouted to her father but the accused pointed the spear at her, and told
her, "can you see this?" The accused then made a pumping motion. After he was sated,
having satisfied his lust, the accused held her breast and kissed her lips. After resting
on top of her, he went to the door and left. Melencio helplessly saw the accused on top
of her daughter but he could not move because he was too afraid and weak. He did not
see how the accused consummated his beastly desire because he was too ashamed to
look at what he was doing to her daughter. Before the accused left, he made the
following threat: "If you will report to the authorities, I will come back and kill all of you."
ISSUE: Whether ignominy attended the commission of the crime.
HELD: YES. The aggravating circumstance of ignominy shall be taken into account if
means are employed or circumstances brought about which add ignominy to the natural
effects of the offense; or if the crime was committed in a manner that tends to make its
effects more humiliating to the victim, that is, add to her moral suffering. It was
established that BALIWANG used the flashlight and examined the genital of Gloria
before he ravished her. He committed his bestial deed in the presence of Gloria's old
father. These facts clearly show that BALIWANG deliberately wanted to further humiliate
Gloria, thereby aggravating and compounding her moral sufferings. Ignominy was
appreciated in a case where a woman was raped in the presence of her betrothed, or of
her husband, or was made to exhibit to the rapists her complete nakedness before they
raped her.
THE PEOPLE OF THE PHILIPPINES, plaintiff-appellee, vs.
JAIME JOSE Y GOMEZ, ET AL., defendants. JAIME JOSE Y GOMEZ, BASILIO
PINEDA, JR., alias "BOY," EDGARDO AQUINO Y PAYUMO and ROGELIO CAAL Y
SEVILLA, defendants-appellants.
G.R. No. L-28232 February 6, 1971
173

En banc, Per curiam

FACTS: Four malefactors (pineda, Aquino, Joce and Caal) abducted Miss De la Riva
about 4:30 o'clock in the morning of June 26, 1967. Miss De la Riva was brought at the
Swanky Hotel in Pasay City.
Inside the room Miss De la Riva was made to sit on a bed. Her blindfold was removed.
She saw Pineda and Aquino standing in front of her, and Jose and Caal sitting beside
her, all of them smiling meaningfully. Pineda told the complainant: "Magburlesque ka
para sa amin." The other three expressed their approval and ordered Miss De la Riva to
disrobe. The complainant ignored the command. One of the appellants suggested
putting off the light so that the complainant would not be ashamed. The idea, however,
was rejected by the others, who said that it would be more pleasurable for them if the
light was on. Miss De la Riva was told to remove her stocking in order, according to
them, to make the proceedings more exciting. Reluctantly, she did as directed, but so
slowly did she proceed with the assigned task that the appellants cursed her and
threatened her again with the Thompson and the acid. They started pushing Miss De la
Riva around. One of them pulled down the zipper of her dress; another unhooked her
brassiere. She held on tightly to her dress to prevent it from being pulled down, but her
efforts were in vain: her dress, together with her brassiere, fell on the floor.
The complainant was now completely naked before the four men, who were kneeling in
front of her and feasting their eyes on her private parts. This ordeal lasted for about ten
minutes, during which the complainant, in all her nakedness, was asked twice or thrice
to turn around. Then Pineda picked up her clothes and left the room with his other
companions. The complainant tried to look for a blanket with which to cover herself, but
she could not find one. One after another, the four malefactors enter the room and
forcibly performed sexual carnage upon Miss De la Riva. After the appellants had been
through with the sexual carnage, they gave Miss De la Riva her clothes, told her to get
dressed and put on her stockings, and to wash her face and comb her hair, to give the
impression that nothing had happened to her.
ISSUE: Whether or not ignominy attended the commission of the crime.
HELD: YES. Ignominy, since the appellants in ordering the complainant to exhibit to
them her complete nakedness for about ten minutes, before raping her, brought about a
circumstance which tended to make the effects of the crime more humiliating.

PEOPLE OF THE PHILIPPINES, appellee, vs.


RUFINO MALLARI y ILAG, appellant.
G.R. No. 145993. June 17, 2003
En banc, Davide, Jr. C.J.

174

FACTS: Liza Galang testified that on 7 July 1996 at around 4:00 p.m., her common-law
husband Joseph admonished Rufino and his brothers Ino and Felix Mallari not to drive
fast while passing by Josephs house. Rufino and his brothers, who were then hottempered, challenged Joseph to a fight. The latter just ignored the challenge; and,
instead he and his own brothers Radi and Manny asked apology from Rufino.
Later that afternoon, while Joseph and Liza were watching a basketball game at the
barangay basketball court, Rufino and his brothers, who were then carrying bladed
weapons, arrived and attempted to stab Joseph; but Joseph was able to run away.
When they were not able to catch up with him, Rufino boarded and drove the truck
parked near the basketball court and continued chasing Joseph until the truck ran over
the latter, which caused his instantaneous death.
ISSUE: Whether or not the use of motor vehicle can be appreciated as qualifying
circumstance in the given case.
HELD: YES.
The antecedent events show that, indeed, Rufino deliberately ran over Joseph. At
around 4:00 p.m. of 7 July 1996, when Rufino passed by Josephs house while driving
the truck, he got angry when Joseph admonished him not to drive at high speed in front
of Josephs house. Rufino, already in a fighting mood, challenged Joseph to a fight, but
the latter just ignored it. To put an end to the argument, Joseph and his brothers
apologized to Rufino.
Apparently, Rufino was not appeased by the apology and continued to harbor ill-feelings
against Joseph. Rufino got the chance to vent his anger not long thereafter. At around
5:30 p.m., while Joseph was watching a basketball game at the basketball court located
beside Rufinos house, Rufino and his brothers Ino and Felix, carrying with them bladed
weapons, attempted to stab Joseph. But before they could do it, Joseph was able to run
away. They chased Joseph, but were unable to catch up with him. Instead of giving up
on his evil design, Rufino went back to the basketball court, boarded the truck parked
nearby, and resumed his pursuit of Joseph. Upon seeing Joseph on the road, Rufino hit
him with the truck.

PEOPLE OF THE PHILIPPINES, plaintiff-appellee, vs.


THADEOS ENGUITO, defendant-appellant.
G.R. No. 128812
February 28, 2000
Third Division, Gonzaga-Reyes, J.
FACTS: About 3:00 o'clock dawn of September 22, 1991, Felipe Requerme, a motorela
driver who while driving his motorela with his wife on board, from Lapasan towards
175

Poblacion, Cagayan de Oro City, picked up a passenger near the Nazareno church. The
passenger was later identified as the deceased, Engr. Wilfredo Achumbre. Achumbre
asked him to bring him across the Marcos bridge towards his home. After travelling a
distance of 300 meters more or less and near the Sacred Heart of Jesus Montessori
School, Requerme's motorela was bumped by a white motor vehicle. The vehicle kept
pushing the motorela causing it to run very fast for the next 400 meters until it reached
the area in front of Wheels Marketing. Because of the violent push the motorela turned
around facing the direction from where it came from and fell on its right
side.1wphi1.nt
A tamaraw pick-up stopped near them and he immediately informed that they were
intentionally hit by the white vehicle. Requerme and his wife were brought to the
Operation Kahusay ug Kalinaw (OKK). At the OKK the driver of the white service pickup who bumped his motorela arrived. Requerme identified the driver as Thadeos
Enguito whom he pointed inside the courtroom.
Mrs. Requerme shouted and waved her hand signalling the driver to stop but the driver
kept pushing the motorela violently. The push was so fast and strong that the motorela
was already uncontrolled and running very fast. Their passenger jumped out when they
were already at the Marcos bridge near the Wheels Marketing. Then the motorela made
a 180 degrees turn facing the direction where they came from and fell on its right side.
Struggling out of the motorela she noticed that the white vehicle went up the elevated
catwalk or pathway pursuing Achumbre who was hit when he was already at the railing
(barandilla). Then she observed that the white vehicle drove across the bridge towards
Iligan City. At the OKK she saw the accused brought by policemen and she asked him
why he bumped them and the accused answered "I have to do it Manang because look
at me now" (TSN, Dec. 16, 1991).
ISSUE: Whether or not the instant case was a Crime of Murder with the use of motor
vehicle.
HELD: YES.The indictment against accused-appellant is murder attended by the use of
motor vehicle. The use of a motor vehicle qualifies the killing to murder if the same was
perpetrated by means thereof. Appellant's claim that he merely used the motor vehicle,
Kia Ceres van, to stop the victim from escaping is belied by his actuations. By his own
admission, he testified that there was a police mobile patrol near the crossing. Accusedappellant could have easily sought the assistance of the police instead of taking the law
into his own hands. Moreover, accused-appellant already noticed the deceased trying to
jump out of the motorela but he still continued his pursuit. He did not stop the vehicle
after hitting the deceased who was hit when he (Achumbre) was at the railing of the
Marcos bridge. Accused-appellant further used the vehicle in his attempt to escape. He
was already more than one (1) kilometer away from the place of the incident that he
stopped his vehicle upon seeing the police mobile patrol which was following him.

176

MOISES SIMANGAN, Petitioner, vs.


PEOPLE OF THE PHILIPPINES, Respondent.
G.R. NO. 157984. July 8, 2004
Second Division, Callejo, Sr. J.
FACTS: At 8:00 p.m. on February 10, 1980, the petitioner, Loreto Bergado, Bening
Gumabong and two other male persons arrived at the store of the spouses Ernesto
Flores and Sofronia Saquing in Barangay Maasin, Solana, Cagayan. The petitioner
asked Ernesto to go with them to serve as a guide. The petitioner warned Ernesto and
Sofronia not to tell anyone that they had been to the store. cralawred
The next morning, Romeo Balunggaya arrived at Sofronias house and told her that
Ernesto was dead, and that his body had been found about three hundred (300) meters
away.
On February 18, 1980, Fernando Saquing attended his classes in civil engineering at
the St. Louis University in Tuguegarao, Cagayan. He noticed his seatmate and close
friend, petitioner Moises Simangan, writing on a piece of paper. He grabbed the paper,
read it, and saw that the petitioner had written the following: Andres Buena alias Ka
Ren, Cely Pea alias Ka Laarni, Moises Simangan alias Ka Ronie Ledesma. The
petitioner warned Fernando not to divulge his secret to anybody. cralawred
On February 24, 1980, Fernando and the petitioner were on their way home from their
ROTC classes at the St. Louis University. The petitioner then narrated to Fernando that
at about 7:00 p.m. on February 10, 1980, after buying cigarettes from a store, the storeowner agreed to go with him and his four companions. The petitioner revealed that they
brought the victim over to the place where twenty of his other comrades were waiting.
He also told Fernando that he and his companions stabbed the victim over and over
again, and tasted the latters blood so that they would not get sick. The petitioner warned
that if Fernando divulged to anyone what he had just revealed, he (the petitioner), would
drink his blood, too. cralawred
The petitioner did not know that Fernando was the first cousin of Sofronia, the widow of
Ernesto Flores, who was, in turn, the store-owner referred to by Moises. Fernando
immediately told Sofronia what the petitioner had told him.
ISSUE: Whether or not the appellate court erred in appreciating against the petitioner
the aggravating circumstances of cruelty.
HELD: NO. The crime is not aggravated by cruelty simply because the victim sustained
ten stab wounds, three of which were fatal. For cruelty to be considered as an
aggravating circumstance, there must be proof that, in inflicting several stab wounds on
the victim, the perpetrator intended to exacerbate the pain and suffering of the
victim. The number of wounds inflicted on the victim is not proof of cruelty.
THE PEOPLE OF THE PHILIPPINES, plaintiff-appellee, vs.
ORLANDO M. GUERRERO, SR., and ORLANDO A. GUERRERO,
177

JR., accused, ORLANDO A. GUERRERO, JR., accused-appellant.


G.R. No. 134759. September 19, 2002
Second Division, Quisumbing J.
FACTS: Responding police officers arrived at the scene and saw the victim Ernesto
Ocampo lying dead, his head having been separated from the body while his penis was
completely severed and placed on his abdomen. Witness Jacalne said he was informed
that one Dino Guerrero was inside the house nearby. Dino Guerrero came out with his
hands extended forward. SPO1 Emilio Taracatac immediately frisked and handcuffed
him. Before Dino was handcuffed, according to the witness, he said that it was his son
who had killed the victim. Thereafter, Dino was brought to the police station for custodial
investigation.
Jacalne testified that appellant Orlando Guerrero, Jr., was not at the scene of the crime
during their investigation. But upon their return to the police station, appellant was
already there. Appellant admitted killing the victim, according to Jacalne, by clubbing the
victim first with the wooden stick, and then cutting his head and his penis with a knife.
Dr. Sanglay said that the victim had been beheaded, and his penis was cut off totally.
Two wounds were found on his head, one on the front, and another a little bit backwards
on the right side of the head. There were other wounds, she said, in different parts of
Ernestos body. But of these injuries, she added that decapitation or separation of the
head from the body was the fatal one. However, she could not ascertain the order in
which the wounds were inflicted. She could not likewise determine whether they were
inflicted while the victim was standing up or lying down.
ISSUE: Whether or not the court a quo gravely erred in appreciating the qualifying
circumstance of cruelty.
HELD: NO.As established by the testimony of witnesses, appellant first severed the
victims head before his penis was cut-off. This being the sequence of events, cruelty
has to be ruled out for it connotes an act of deliberately and sadistically augmenting the
wrong by causing another wrong not necessary for its commission, or inhumanely
increasing the victim's suffering. As testified to by Dr. Sanglay, and reflected in her
medical certificate, Ernesto in fact died as a result of his head being severed. No
cruelty is to be appreciated where the act constituting the alleged cruelty in the killing
was perpetrated when the victim was already dead.
PEOPLE OF THE PHILIPPINES, plaintiff-appellee, vs.
STEPHEN MARK WHISENHUNT, accused-appellant.
G.R. No. 123819
November 14, 2001
First Division, Ynares-Santiago, J.
FACTS: While Demetrio was in the servants quarters watching television, accusedappellant came in. He asked Demetrio how long he wanted to work for him. Demetrio
178

replied that he was willing to work for him forever, and expressed his full trust in him.
Upon hearing this, accused-appellant shed tears and embraced Demetrio. Then
accused-appellant said, "May problema ako, Rio." Demetrio asked what it was, and
accused-appellant told him that Elsa was dead. Demetrio asked, "Bakit mo siya
pinatay?" Accused-appellant answered that he did not kill Elsa, rather she died of
"bangungot".
Demetrio suggested that Elsas body be autopsied, but accused-appellant said that he
had already beheaded her. He asked Demetrio if he wanted to see the decapitated
body, but the latter refused. The two of them went to Shoppesville at the Greenhills
Shopping Center and bought a big bag with a zipper and rollers, colored black and
gray. Demetrio noticed that accused-appellant seemed nervous and his eyes were teary
and bloodshot.
When they returned to the condominium, accused-appellant asked Demetrio to help him
wrap the body in the black garbage bags. Demetrio entered accused-appellants
bathroom and found the dismembered hands, feet, trunk and head of a woman. He
lifted the severed head by the hair and, when he lifted it, he saw Elsas face. He placed
this in a black trash bag. He helped accused-appellant place the other body parts in
three separate garbage bags. They packed all the garbage bags in the bag with the
zipper and rollers, which they had bought in Shoppesville. Then, they brought the bag
down and loaded it in the trunk of accused-appellants car. After that, they boarded the
car. Demetrio took the wheel and accused-appellant sat beside him in front.
ISSUE: Whether or not cruelty can be appreciated in the case at bar.
HELD: YES.The circumstance of outraging and scoffing at the corpse of the victim was
correctly appreciated by the trial court. The mere decapitation of the victims head
constitutes outraging or scoffing at the corpse of the victim, thus qualifying the killing to
murder. In this case, accused-appellant not only beheaded Elsa. He further cut up her
body like pieces of meat. Then, he strewed the dismembered parts of her body in a
deserted road in the countryside, leaving them to rot on the ground. The sight of Elsas
severed body parts on the ground, vividly depicted in the photographs offered in
evidence, is both revolting and horrifying. At the same time, the viewer cannot help but
feel utter pity for the sub-human manner of disposing of her remains.

PEOPLE OF THE PHILIPPINES, plaintiff-appellee, vs.


ALFREDO TARUC y SAPAN and ANTONIO EFE, JR. y LEYCE.
G.R. Nos. L-69337-38 March 8, 1989
First Division, Cruz, J.

179

FACTS: Sanchez, who is also known as Manuela, says he had just left a "gay" dinnerdance and was talking to two women in a street corner at about two-thirty o'clock in the
morning of May 10, 1981, when Taruc approached him and asked him to come with him
to his house. When they arrived there, he was shocked to find Efe strangling Martinez
with a nylon cord with wooden handles at each end and also forcing banana peelings
into the latter's mouth. Martinez looked helpless, with his tongue hanging out, and was
offering not much of a struggle although he was resisting as best as he could. Efe was
demanding to know the whereabouts of a certain Boy Baba, who had stabbed Taruc
some weeks before, but Martinez would not or could not give any answer. Before
Sanchez realized it, a length of wire had also been drawn around his own neck by
Taruc, who also wanted to know where Boy Baba was. By way of further persuasion,
Taruc stabbed Sanchez in the thigh three times with a balisong and continued to
strangle him. He also slashed his dress and warned that both he and Martinez would
die that night if they did not tell him where to find Boy Baba. Efe and Taruc then
changed places, and Efe boxed Sanchez in the mouth and poked a gun at him, then
took over strangling him. But as Efe noticed that Taruc needed assistance because he
was apparently still weak from his wounds, Efe released Sanchez to help Taruc.
Together, they again forced banana peelings into Martinez's mouth and continued
strangling him. They also stabbed him in the chest. The two being thus occupied,
Sanchez seized the opportunity to escape, running out of the house and thereafter
immediately going into hiding in the police outpost until he reappeared the following day
to dennounce his assailants.
ISSUE:
Whether or not the crime committed was attended by aggravating
circumstance of cruelty.
HELD: YES.
The crime was attended by the generic aggravating circumstance of cruelty as
manifested by the stab wounds inflicted on Martinez and the perversion of forcing
banana peelings into his mouth while he was being strangled.

(ALTERNATIVE CISCRUMSTANCES)
PEOPLE OF THE PHILIPPINES, Plaintiff-Appellee, vs.
ROSENDO REBUCAN y LAMSIN, Accused-Appellant.
G.R. No. 182551
July 27, 2011
First Division, Leonardo-De Castro, J.
FACTS: Carmela Tagpis testified as an eyewitness to the incident in question. She
pointed to the accused-appellant as the "Bata Endong" 10 (Uncle Endong) who hacked
her grandfather and brother. She stated that Ranil was hit in the forehead, while Felipe
180

was hit on the face, the left shoulder and the right shoulder. After Felipe was hacked by
the accused-appellant, the former was still able to walk outside of his house, to the
direction of the coconut tree and thereafter fell to the ground. Carmela said that she saw
that a long bolo was used in the killing of Felipe and Ranil. She related that Felipe also
owned a bolo but he was not able to use the same when he was attacked. She was
then inside the house with Felipe and her two younger brothers, Jericho and Bitoy
(Ranil). She was sitting about four meters away when the hacking incident occurred
indoors.11
ISSUE: Whether or not the court a quo gravely erred in failing to appreciate intoxication
as a mitigating circumstance in favor of accused-appellant.
HELD: NO.The third paragraph of Article 15 of the Revised Penal Code provides that
the intoxication of the offender shall be taken into consideration as a mitigating
circumstance when the offender has committed a felony in a state of intoxication, if the
same is not habitual or subsequent to the plan to commit said felony; but when the
intoxication is habitual or intentional, it shall be considered as an aggravating
circumstance. The Court finds that the accused-appellant is not entitled to the mitigating
circumstance of intoxication since his own testimony failed to substantiate his claim of
drunkenness during the incident in question. During his cross-examination, the
accused-appellant himself positively stated that he was only a bit tipsy but not drunk
when he proceeded to the house of Felipe. 65He cannot, therefore, be allowed to make a
contrary assertion on appeal and pray for the mitigation of the crimes he committed on
the basis thereof.
PEOPLE OF THE PHILIPPINES, Appellee,
vs.
EFREN PATELAN LAMBERTE @ "KALBO" and MARCELINO RUIZ NIMUAN @
"CELINE," Accused, MARCELINO RUIZ NIMUAN, Appellant.
G.R. No. 182918
June 6, 2011
Third Division, Brion J.
FACTS: At about 6:00 p.m. of September 22, 2004, Eulalia Garcia was tending her sarisari store along the National Highway in San Eugenio, Aringay, La Union when the
appellant and Lamberte came to borrow her gas lamp. She noticed that both were drunk
and armed. They said they were looking for a bullet that fell on the ground. After finding
the bullet, she asked them where they were going and they answered, "We are going to
kill the doctor." The two then waited under a mango tree. Shortly thereafter, the victim
(Dr. Jose Villanueva), on board a truck, passed by Garcias store on the way to his
poultry farm. The appellant and Lamberte followed on foot. Ten (10) minutes later,
Garcia heard two (2) gunshots coming from the direction of the poultry farm.
ISSUE: Whether or not mitigating circumstance of intoxication can be appreciated in the
case at bar.

181

HELD: NO. We find that the CA erred in crediting the appellant with the mitigating
circumstance of intoxication simply because Garcia testified that "the accused were
both drunk." For intoxication to be considered as a mitigating circumstance, it must be
shown that the intoxication impaired the willpower of the accused that he did not know
what he was doing or could not comprehend the wrongfulness of his acts.
In this case, there is no convincing proof of the nature and effect of the appellants
intoxication. The mitigating circumstance of intoxication cannot be appreciated in the
appellants favor merely on the testimony of a prosecution witness that he was drunk
during the incident. Such testimony does not warrant a conclusion that the degree of the
accuseds intoxication had affected his faculties.
ROBERTO LICYAYO, Petitioner, -versus
PEOPLE OF THE PHILIPPINES,Respondent.
G.R. No. 169425 March 4, 2008
Third Division, Chico-Nazario, J.
FACTS: After the wedding reception, Rufino, Jeffrey and Joel went to Natamas Store at
the Kiangan Public Market and ordered two bottles of gin. While the three were drinking
gin at the said store, petitioner, Paul and Oliver arrived and likewise ordered bottles of
gin. Later, petitioner, Paul and Oliver left the store. Subsequently, Rufino, Jeffrey and
Joel likewise adjourned their drinking session and left the store.
Rufino, Jeffrey and Joel dropped by at Famorcas Store. Petitioner and his brother,
Aron, as well as Paul and Oliver, were also present therein. While Jeffrey was talking to
the stores owner, Larry Famorca (Larry), a brawl suddenly occurred between Rufino
and Aron. As a consequence thereof, Rufino fell to the ground. Aron thereafter placed
himself on top of Rufino and punched the latter several times. Jeffrey approached the
two and tried to pacify them. Paul entered the scene and punched Jeffrey on the
head. Thereupon, a scuffle followed
The officers introduced themselves to petitioner as policemen and pleaded with him to
put down the knife. Petitioner ignored the officers pleas.
Afterwards, petitioner approached Rufino, who was then wrestling with Paul, and
stabbed Rufino in different parts of the body. Officer Baguilat fired a warning shot while
Officer Danglay immediately pounced on petitioner and disarmed the latter. Petitioner
was brought to the Kiangan Police Station while Rufino was taken to a nearby hospital
where he later died due to stab wounds.
ISSUE: Whether or not petitioner is entitled to the mitigating circumstance of
intoxication.

182

HELD: NO. For intoxication to be considered as a mitigating circumstance, it must be


shown that the intoxication impaired the willpower of the accused and that he did not
know what he was doing or could not comprehend the wrongfulness of his acts. The
person pleading intoxication must prove that he took such quantity of alcoholic
beverage, prior to the commission of the crime, as would blur his reason.
In the case at bar, there is no plausible evidence showing that the quantity of liquor
taken by petitioner was of such quantity as to affect his mental faculties. On the
contrary, the fact that petitioner could recall the details that transpired during and after
his drinking session with friends is the best proof that he knew what he was doing
during the incident. His vivid narration that he had a confrontation with Rufino, Jeffrey
and Joel during the drinking session; that Daniel approached and told him that Aron was
being mauled; that he immediately went to the scene and saw Aron being beaten by
Rufino and Jeffrey; that he pushed Jeffrey away from Aron; that he was allegedly beaten
by the companions of Jeffrey; and that he fought back but was allegedly overpowered
--- all point to the conclusion that petitioner had complete control of his mind during the
incident.
Petitioner cannot avail himself of the mitigating circumstance of intoxication merely on
the testimonies of the prosecution witnesses that he was drunk during the
incident. Such testimonies do not warrant a conclusion that the degree of petitioners
intoxication had affected his faculties. There must be convincing proof of the nature and
effect of his intoxication which petitioner failed to adduce in the present case.
PEOPLE OF THE PHILIPPINES, Appellee, vs.
ERNESTO FRAGANTE y AYUDA, Appellant.
G.R. No. 182521
February 9, 2011
Second Division, Carpio, J.
FACTS: Ten (10) Informations filed on 14 July 1998, appellant was charged with nine
(9) counts of acts of lasciviousness and one (1) count of rape all committed against his
own minor daughter AAA.
Ernesto A. Fragante (Ernesto hereafter) married CCC on October 6, 1975, in Sta. Cruz
Manila, and such marriage was ratified on December 7, 1995 celebrated in San
Sebastian Parish Church. That union, produced three offsprings. [AAA], the victim
herein, is their third child. She was born on August 23, 1982.
Sometime in April 1993 to May 1993, three or four months before her eleventh (11)
birthday, [AAA] woke up one early morning to prepare for the driving lessons which her
father Ernesto, promised to teach them that day. [AAA] was the first to wake up. She
was in her room when her father entered and lay on her bed. He then asked [AAA] to lie
beside him to which [AAA] obeyed. While lying beside her, Ernesto was talking to her
about a lot of things, and as he talked he started to fondle her breast and suck her
nipples. The incident was repeated on, June 1993 and August 1993, two occasions,
between October 1993 and December 1993, at Shaolin Chinese Restaurant located in
183

Sucat, Paraaque, December 1993, January 1994, August-September 1994, she was
around twelve (12) years old, September 1995, at the age of thirteen (13) and October
25, 1997.
ISSUE: Whether or not relationship can be appreciated as aggravating circumstance in
the instant case.
HELD: YES. In this case, AAA's testimony clearly showed how appellant took
advantage of his relationship with and his moral ascendancy over his minor daughter
when he had carnal knowledge of her. As found by the Court of Appeals, appellant
instilled fear on AAA's mind every time he sexually molested her, thus:
[AAA] also admitted that after accused-appellant has started sexually molesting her until
she was raped, she was so frightened of him. In fact she could not tell her mother of her
ordeal, mindful of the serious threats on her life and of the chaos it would cause their
family.

PEOPLE OF THE PHILIPPINES, Appellee, versus


ALEJANDRO CALONGUI y LOPEZ, Appellant.
G.R. No. 170566
March 3, 2006
First Division, Ynares-Santiago, J.
FACTS: Marinel testified that she was 5 years old when her family took in appellant,
who is her first cousin and who was then 21 years old, to live with them and help in the
upkeep of the familys farm.
On January 1, 1998, at 2:00 a.m., then 13-year-old Marinel, who slept in the same room
as the appellant and her three siblings, awoke to find that appellant had removed her
shorts and panties. The latter threatened to kill her and her siblings if she resisted his
sexual advances. She tried to repel the sexual assault by moving her body and kicking
the appellants thighs but appellant succeeded in having sexual congress with her. The
next morning, she learned that her 12-year old brother, Noel, witnessed the incident but
pretended to be asleep because the appellant might harm him. Marinel told Noel not to
tell anyone about what he saw. She also did not report the matter to her parents for fear
that appellant would make good his threats. The incident was repeated on September
26, 1998 at 3:00 a.m.
ISSUE: Whether or not relationship can be appreciated as aggravating circumstance in
the instant case
HELD: NO. Relationship is not aggravating because the relationship between Marinel
and the appellant as first cousins is not within the concept contemplated in Article 15[41]
of the Revised Penal Code.[42] Abuse of confidence is likewise absent because the
prosecution did not establish that it facilitated the attainment of the rape.

184

PEOPLE OF THE PHILIPPINES, plaintiff-appellee, vs.


CESAR MARCOS Y MON, accused-appellant.
G.R. No. 132392
January 18, 2001
En banc, Puno, J.
FACTS: Accused-appellant Cesar Marcos (Cesar) and the victim Virgilio Marcos
(Virgilio) are brothers and they live in the same house at Bayambang, Infanta
Pangasinan.
On August 19, 1996 at about 12:00 noon, Fernando Marcos, Jr. (Fernando) was resting
under a mango tree a few meters away from the house of the Marcoses. After a while,
his uncle Virgilio arrived and proceeded to the artesian well (jetmatic) located just at the
back of the house. Virgilio bent down to put on the ground the tools he was carrying. It
was at this precise moment that Fernando saw his uncle Cesar come out of the kitchen
door with a bolo in hand and suddenly hacked the unsuspecting Virgilio from behind.
Virgilio was hit on the nape of the neck which caused him to fall to the ground. Then
Cesar hacked him again and this time Virgilio was hit on the right side of the head.
Fernando rushed to his uncle Cesar and asked why he did that, to which Cesar replied
"You go away if you do not want to get involved." Out of fear, Fernando could only watch
helplessly at Virgilio as the latter was asking him for help. Then Fernando heard Cesar
tell Virgilio "Your life is not enough to pay the money you squandered."
ISSUE: Whether or not relationship can be appreciated as aggravating circumstance in
the instant case
HELD: YES. In order that the alternative circumstance of relationship may be taken into
consideration in the imposition of the proper penalty, the offended party must either be
the (a) spouse, (b) ascendant, (c) descendant, (d) legitimate, natural or adopted brother
or sister, or (e) relative by affinity in the same degree, of the offender. In the case at bar,
prosecution eyewitness Fernando Marcos, Jr. testified that Cesar and Virgilio Marcos
are brothers. Accused likewise declared that Virgilio is his brother.That the victim is the
elder brother of Cesar is likewise alleged in the Information. The rule is that relationship
is aggravating in crimes against persons as when the offender and the offended party
are relatives of the same level such as killing a brother. Thus, relationship was correctly
appreciated as an aggravating circumstance.

(PERSONS CRIMINALLY LIABLE)


(PRINCIPALS)
PEOPLE OF THE PHILIPPINES, appellee, vs.
DOMINGO VASQUEZ y PACHECO and RAMON VASQUEZ y PACHECO, accused.
DOMINGO VASQUEZ y PACHECO, appellant.
185

G.R. No. 123939


May 28, 2004
Second Division, Callejo, J.
FACTS: Luis and Geronimo walked side by side on the right side of Lapu-Lapu Street at
Urduja Village. With them were their cousins, Raymund Luable, Angelo Luable and
Orlando Desca. As they were nearing a Meralco lamp post at the corner of Lapu-Lapu
and Magat Salamat Streets,4 he saw a blue-colored passenger jeep with a whitecolored rear door and with its front lights on, driven by Roels uncle, Domingo Vasquez,
who was with Roels brother, Ramon, and five others. The jeep, which was coming from
the opposite direction, going towards the Vicas supermarket, sped towards them. They
dived to the ground near a grassy area, to avoid being hit. The vehicle sped past Luis
and Geronimo and stopped in front of the lamp post on the left side of the street.
Domingo and Ramon Vasquez, each armed with a bolo, with five others, alighted from
the jeepney and proceeded to where Luis and Geronimo were. Afraid for their lives, the
two fled towards the direction of Mary Homes at North Olympus Street. Luis ran ahead,
and when he looked back towards Geronimo, he saw the latter fleeing towards the
direction of Sumakwel Street5 with three persons, including Domingo and Ramon, in hot
pursuit. By the time he reached Datu Puti Street, only one man was pursuing him. Luis
finally arrived at their house. He then mounted his bicycle and pedaled to the police
station to report the incident, only to learn that his half-brother, Geronimo, was already
dead. He and some policemen proceeded to Bagong Silang Funeral Parlor where they
saw Geronimos body.
Whether Domingo Vasquez chased the deceased with a bolo was averred by Luis
Luable or whether the accused merely incited his companions in the jeepney to kill the
deceased as averred by Luisa Abellanosa, is immaterial in the determination of his
liability because a conspiracy among the occupants of the jeepney has been
established.
ISSUE: Whether or not Domingo Vasquez is a principal in the case at bar.
HELD: YES. Article 8 of the Revised Penal Code provides that there is conspiracy when
two or more person agree to commit a felony and decide to commit it. In People vs.
Pagalasan, this Court held that conspiracy need not be proven by direct evidence. It
may be inferred from the conduct of the accused before, during and after the
commission of the crime, showing that they had acted with a common purpose and
design. Conspiracy may be implied if it is proved that two or more persons aimed by
their acts towards the accomplishment of the same unlawful object, each doing a part
so that their combined acts, though apparently independent of each other were, in fact,
connected and cooperative, indicting a closeness of personal association and a
concurrence of sentiment. Conspiracy once found, continues until the object of it has
been accomplished and unless abandoned or broken up. To hold an accused guilty as a
co-principal by reason of conspiracy, he must be shown to have performed an overt act
in pursuance or furtherance of the complicity. There must be intentional participation in
the transaction with a view to the furtherance of the common design and purpose.
186

Each conspirator is responsible for everything done by his confederates which follows
incidentally in the execution of a common design as one of its probable and natural
consequences even though it was not intended as part of the original design.
Responsibility of a conspirator is not confined to the accomplishment of a particular
purpose of conspiracy but extends to collateral acts and offenses incident to and
growing out of the purpose intended. Conspirators are held to have intended the
consequences of their acts and by purposely engaging in conspiracy which necessarily
and directly produces a prohibited result, they are, in contemplation of law, chargeable
with intending that result. Conspirators are necessarily liable for the acts of another
conspirator unless such act differs radically and substantively from that which they
intended to commit. As Judge Learned Hand put it in Unites States vs. Andolscheck,
"when a conspirator embarks upon a criminal venture of indefinite outline, he takes his
chances as to its content and membership, so be it that they fall within the common
purposes as he understands them."
In the case at bar, the appellant drove the passenger jeepney with his cohorts on board
looking for Luable and Geronimo. When the appellant saw the two going in the opposite
direction, the appellant drove the vehicle and sideswiped Geronimo. And when
Geronimo fled, the appellant, armed with a bolo, pursued him. When the appellant failed
to overtake the victim, he returned to the passenger jeepney and drove it to where his
cohorts ganged up on the victim. The appellant urged them on to kill Geronimo.
Thereafter, he left the scene along with his cohorts, leaving the hapless Geronimo
mortally wounded. All the foregoing constitutes evidence beyond cavil of conspiracy
between the appellant and the principals by direct participation. The appellant is, thus,
criminally liable for the death of the victim, although there is no evidence that he did not
actually stab the latter.

PEOPLE OF THE PHILIPPINES, appellee, vs.


FRANCISCO DACILLO alias DODOY AND JOSELITO PACOT y IBARRA (case
provisionally dismissed),accused,
FRANCISCO DACILLO alias DODOY, appellant.
G.R. No. 149368
April 14, 2004
En banc, Corona, J.
FACTS: Around 7:45 p.m. that evening, witness Jovelyn Dagmil, who was living with
her aunt in the house adjacent to appellants, was looking for her cousin when she saw
the victim Rosemarie on the bridge. Because it was drizzling, she invited Rosemarie
inside their house but the latter declined and told her she was waiting for someone.
After a while, Jovelyn saw Rosemarie walking towards and entering appellants house.
Not long after Rosemarie went inside the house, a struggle was heard therein.
Witnesses Roche and Resna Abregon, who were in the adjacent house singing with a
187

karaoke machine, suddenly felt the floor shaking as if a scuffle was going on at the
other side of the wall. The houses were built on stilts above the seashore, adjoining one
another with mere wooden partitions in between. Roche Abregon peeped through a hole
on the wall and saw appellant and another man grappling with a woman who was
gagged with a handkerchief. When Roche saw appellant choking the woman, she
informed her aunt about the commotion in appellants house but the aunt brushed it
aside as a simple family quarrel. For a while they heard the sound of a woman being
beaten up. Then everything became quiet. Later that evening, they saw appellant
leaving his house.
In his defense, appellant admitted complicity in the crime but minimized his
participation. Appellant alleged that he only held down Rosemaries legs to prevent her
from struggling and, after the latter was killed by another man he identified as Joselito
Pacot, he encased the corpse in cement.
He claimed that Pacot, a co-worker at Davao Union Cement Corporation (DUCC), was
looking for a house where he and his girlfriend Rosemarie could spend the night. He
offered his brothers house which was under his care. In the evening of February 6,
2000, he and Joselito Pacot brought Rosemarie to the house at Purok No. 3, New
Society Village, Ilang, Davao City.
ISSUE: Whether or not appellant was a principal by direct participation.
HELD: YES. Despite appellants self-serving, exculpatory statement limiting his
involvement in the crime, all circumstances pointed to his guilt. His declaration faltered
in the face of the testimonies of eyewitnesses positively identifying him as one of the
two men who were with Rosemarie when she was killed. Witness Roche Abregon
pointed to appellant as the one who strangled Rosemarie. He was established to be
inside the house at the time the witnesses heard a woman being battered. Thus,
assuming for the sake of argument that Pacot was the mastermind, appellants
admission that he participated in its commission by holding Rosemaries legs made him
a principal by direct participation.
Two or more persons taking part in the commission of a crime are considered
principals by direct participation if the following requisites are present:
1. they participated in the criminal resolution and
2. they carried out their plan and personally took part in its execution by acts
which directly tended to the same end.
Both requisites were met in this case. Two or more persons are said to have
participated in the criminal resolution when they were in conspiracy at the time of the
commission of the crime. To establish conspiracy, it is not essential that there be proof
of the previous agreement and decision to commit the crime, it being sufficient that the
malefactors acted in concert pursuant to the same objective.

188

The prosecution was able to prove appellants participation in the criminal resolve by his
own admission that, right after he was told by Pacot to close the door, he held down
Rosemaries legs. He was pinpointed as the one who throttled the victim. He admitted
that they only stopped when they were sure that Rosemarie was already dead. The two
men planned how to dispose of the victims body; it was in fact appellants idea to pour
concrete on the body, prevailing over Pacots suggestion to just dump the body into the
sea. It was appellant himself who encased the body in cement and made sure that there
were no leaks from which foul odor could emanate. He was a conspirator in the killing
and, whether or not he himself did the strangling or the stabbing, he was also liable for
the acts of the other accused.
It is well-settled that a person may be convicted for the criminal act of another where,
between them, there is conspiracy or unity of purpose and intention in the commission
of the crime charged. Conspiracy need not be proved by direct evidence of prior
agreement on the commission of the crime as the same can be inferred from the
conduct of the accused before, during, and after the commission of the crime showing
that they acted in unison with each other pursuant to a common purpose or design.

PEOPLE OF THE PHILIPPINES, appellee, vs.


BUTCHOY DE LA TORRE and FE DE LA TORRE, appellants.
G.R. Nos. 121213 and 121216-23. January 13, 2004
Second Division, Tinga, J.
FACTS: The initial rape incident occurred in the first week of September 1992. She
could not remember the exact date but in that fateful night, as in previous nights, Baby
Jane and the appellant-spouses were asleep on the floor of the same bedroom. At
around 12:00 oclock midnight, appellant Fe de la Torre woke Baby Jane and her
husband Butchoy. Baby Jane was surprised to see that Fe was holding a lighted
kerosene lamp and a scythe. Fe ordered her husband to transfer and lie beside Baby
Jane. As appellant Butchoy did not comply, Fe herself transferred so that Baby Jane
was between her and Butchoy. Fe put down the scythe and the lamp and proceeded to
take Butchoys clothes off and then Baby Janes. Butchoy offered no resistance but
Baby Jane objected and cried to no avail. Fe then ordered Butchoy to have sex with
Baby Jane. Baby Jane, fearful of the spouses and the dawning realization of what
would happen to her, could not ward off his advances. Butchoy placed himself on top of
Baby Jane, inserted his penis into her vagina and did a push and pull motion. Baby
Jane felt pain. All the while, Fe was standing beside them, holding the lamp and the
scythe. After the sexual intercourse, Butchoy kissed her on the neck and fondled her
breasts. Baby Jane found it revolting but could not do much to refuse him, as she was
afraid of Fe. When Butchoy was finished, he threw her clothes to her and got
dressed. Baby Jane immediately put on her clothes. She wanted to leave the room but
Fe prevented her from doing so. They all went back to sleep. Baby Jane could not
sleep. The following morning, Baby Jane saw that there was blood on her panty.
ISSUE: Whether or not spouses appellants are principals in the case at bar.
189

HELD: YES. Under the Revised Penal Code, however, an accused may be considered
a principal by direct participation, by inducement, or by indispensable cooperation. This
is true in a charge of rape against a woman, provided of course a man is charged
together with her. Thus, in two cases this Court convicted the woman as a principal by
direct participation since it was proven that she held down the complainant in order to
help her co-accused spouse consummate the offense.
In People v. Villamala, the Court found the husband and wife guilty for raping their
neighbor and kumare in this factual setting, viz: the wife visited the victim at her home
on the pretext of inquiring as to the whereabouts of her husband. Once inside, she
whistled for her husband and he immediately appeared at the doorstep. The wife then
suddenly pinned her kumare to the floor. The husband forcefully removed the victims
skirt and panties, removed his shorts, placed himself on top of the victim and
consummated the rape. In the more recent People v. Saba, the accused married
couple victimized a fourteen (14) year-old epileptic who stayed at their home for
treatment by the wife who was a reputed healer. On the pretext of conducting a healing
session, the wife ordered the victim to lie down on the floor then pinned the victims
hands to the floor and covered her mouth while her husband removed his pants and
briefs and the victims panties and raped the young girl. These two cases show not only
the possibility but the reality of rape committed by a woman together with a man.
A close look at the cited cases reveals a common thread that not only links them but
also explains why the offenses were committed in the fashion they were. This consists
of the close relationship between the parties and the attendant conducive
environment. The victim and the felons were familiar with each other and there was a
certain bond of trust between them. The same kindred relationship and suitable setting
are extant in the present case. Indeed, the proximity of the victim to the accused
spouses was established by the particular circumstances of their relationship. The
backdrop presented the offenders with a tempting opportunity to satisfy their twisted
desires upon a conveniently placed victim.

(ACCOMPLICES)
PEOPLE OF THE PHILIPPINES, Plaintiff-Appellee, vs.
BARANGAY CAPTAIN TONY TOMAS, SR., BENEDICTO DOCTOR, and NESTOR
GATCHALIAN, Accused-Appellants.
G.R. No. 192251
February 16, 2011
First Division, Velasco, Jr. J
FACTS: Estrella was walking slightly ahead of her mother and Angelita when appellants
Tomas, Sr., Doctor and Gatchalian suddenly came out from the side of the road. Tomas,
Sr. and Doctor are cousins of Estrella. Thereupon, without saying anything, Tomas, Sr.
drew a gun and shot Estrella twice at a distance of about 1.5 meters away. Gatchalian,
without a gun, allegedly supported Tomas, Sr. by standing in a blocking position along
190

the road, while Doctor positioned himself at the back of Damiana and Angelita and
poked a handgun at them, telling them to lie face down on the ground, though they did
not totally drop on the road but were in a kneeling position.
When Tomas, Sr. fired the first two shots at Estrella, the latter fell down but the former
still followed it with three more shots when she was already prone on the ground. After
the five shots, the three accused fled towards the house of Tomas, Sr. Liezl, who was
standing about four meters away from Estrella, shouted, "Saklulu, tulungan ninyo kami
(Help, help us)," then ran to her house.
ISSUE: Whether or not Nestor Gatchalian was an accomplice.
HELD: YES. We note that the evidence adduced and the records would show that
Gatchalian did not do overt acts for the furtherance of the shooting of Estrella. As
mentioned above, mere presence at the scene of the crime at the time of its
commission without proof of cooperation or agreement to cooperate is not enough to
constitute one a party to a conspiracy.
It appears that Gatchalian is a party to the conspiracy as found by the courts a quo.
Gatchalian appeared in the company of Tomas, Sr. and Doctor. He also fled together
with them. However, Gatchalian was unarmed and did not say anything or commit any
overt act to externally manifest his cooperation with the shooting of Estrella. On the
other hand, Gatchalian never attempted to stop the shooting, which tends to show that
he was aware of the plan and intent to kill Estrella or, at the very least, that he
acquiesced to the shooting of Estrella.
Thus, with his lack of overt acts manifestly contributing to the accomplishment of the
common design to shoot Estrella, there is some doubt if he indeed conspired with
Tomas, Sr. and Doctor. This, however, does not exculpate him from criminal liability
absent proof that he merely tagged along or just happened to meet his employer
(Tomas, Sr.) shortly before the incident or was merely taken along without being told
about the other accused-appellants plan. The fact that Gatchalian appeared together
with the other accused-appellants and fled with them, while not constitutive of proof
beyond reasonable doubt of conspiracy, still proves a certain degree of participation and
cooperation in the execution of the crime. Consequently, in line with the principle that
whatever is favorable to an accused must be accorded him, Gatchalian is guilty as an
accomplice only. As We aptly explained in People v. Ballesta:
Mere presence at the scene of the incident, knowledge of the plan and acquiescence
thereto are not sufficient grounds to hold a person as a conspirator. x x x Lacking
sufficient evidence of conspiracy and there being doubt as to whether appellant acted
as a principal or just a mere accomplice, the doubt should be resolved in his favor and
is thus held liable only as an accomplice.

191

COVERDALE ABARQUEZ, y EVANGELISTA, Petitioner, vs.


THE PEOPLE OF THE PHILIPPINES, Respondent.
G.R. No. 150762
January 20, 2006
Third Division, Carpio, J.
FACTS: On 21 November 1993 at 2:00 p.m., Jose Buenjijo Paz ("Paz"), Ricardo
Quejong ("Quejong") and their friends were in the house of one Boyet at 3342 San Jose
St., Sta. Mesa, Manila. They were drinking liquor in celebration of the birthday of Boyets
son. About 7:45 p.m., Paz and Quejong decided to go home. Boyet Tong, Abarquezs
son Bardie and Sonito Masula ("Masula") joined Paz and Quejong. They proceeded
towards the exit of San Jose St.
Meanwhile, about six or seven meters away from Boyets house, Alberto Almojuela also
known as Bitoy ("Almojuela"), a certain Ising and Abarquez also known as Dale, were
likewise drinking liquor in front of Almojuelas house. As the group of Paz was passing
towards the main road, Almojuela and his companions blocked their path.
Almojuela asked Paz, "Are you brave?" Paz replied, "Why?" Almojuela got angry and
attacked Paz with a knife. Paz parried the attack with his left arm but sustained an
injury. Abarquez held Paz on both shoulders while Bardie pacified Almojuela. Paz asked
Abarquez, "What is our atraso, we were going home, why did you block our way?"
Abarquez answered, "Masyado kang matapang. Tumigil ka na, tumigil ka na."
Almojuela then confronted Quejong and they had an altercation, followed by a scuffle.
Paz tried to get away from Abarquez who continued restraining him. Upon seeing
Almojuela and Quejong fall on the ground, Paz struggled to free himself from Abarquez.
Paz approached Quejong and found him already bloodied. It turned out the Almojuela
stabbed Quejong with a knife. Paz tried to pull up Quejong but failed. Paz left Quejong
and ran instead towards the exit of San Jose St. to ask for help. While Paz was running
away, he heard Abarquez shout, "You left your companion already wounded!"
ISSUE: Whether or not Coverdale Abarquez was an accomplice in the instant case.
HELD: NO.Article 18 of the Revised Penal Code defines accomplices as "those persons
who, not being included in Article 17, cooperate in the execution of the offense by
previous or simultaneous acts."
Two elements must concur before a person becomes liable as an accomplice: (1)
community of design, which means that the accomplice knows of, and concurs with, the
criminal design of the principal by direct participation; and (2) the performance by the
accomplice of previous or simultaneous acts that are not indispensable to the
commission of the crime. Mere commission of an act, which aids the perpetrator, is not
enough. Thus: The cooperation that the law punishes is the assistance knowingly
rendered, which cannot exist without the previous cognizance of the criminal act
intended to be executed. It is therefore required in order to be liable as an accomplice,
192

that the accused must unite with the criminal design of the principal by direct
participation.
Pazs testimony does not show that Abarquez concurred with Almojuelas criminal
design. "Tumigil" literally means "stop." Clearly, Abarquez was trying to stop Paz from
joining the fray, not from helping Quejong. Paz claims that he was only trying to talk to
Almojuela. However, Paz could not have been merely talking to Almojuela, as he tried to
portray, because Almojuela was already grappling with Quejong at that time. Paz
interpreted Abarquezs action as an attempt to prevent him from helping Quejong. His
interpretation was adopted by the trial court and sustained by the Court of Appeals. Yet,
in his testimony, Paz admitted that while restraining him, Abarquez was scolding or
reprimanding him and telling him to stop. It was not shown that Abarquez was stopping
Paz from helping Almojuela. It is more likely that Abarquez was trying to stop Paz from
joining the fight. Abarquezs act of trying to stop Paz does not translate to assistance to
Almojuela.
In People v. Fabros, the Court explained:
To be deemed an accomplice, one needs to have had both knowledge of and
participation in the criminal act. In other words, the principal and the accomplice must
have acted in conjunction and directed their efforts to the same end. Thus, it is essential
that both were united in their criminal design.
xxx. The mere fact that the (accused) had prior knowledge of the (principals) criminal
design did not automatically make him an accomplice. This circumstance, by itself, did
not show his concurrence in the principals criminal intent.
Paz stated that Abarquez did not do anything to stop Almojuela. However, Paz testified
that Abarquezs son Bardie, who was one of Pazs companions, was the one trying to
pacify Almojuela. The trial court in its factual findings confirmed this when it stated that
while Abarquez was holding Paz, his son Bardie was pacifying Almojuela.
The prosecution argues that Abarquez was remiss in his duties as a barangay
kagawad in not extending assistance to the then wounded Quejong. This, however,
does not necessarily show concurrence in Almojuelas criminal act. When Paz ran away,
Abarquez shouted at him that he left his wounded companion. Apparently, Abarquez
was not aware of the extent of Quejongs injury and he expected Paz to look after his
own companion.

PEOPLE OF THE PHILIPPINES, Appellee, vs.


EDMAR AGUILOS, ODILON LAGLIBA Y ABREGON and RENE GAYOT PILOLA,
accused, RENE GAYOT PILOLA, Appellant.
G.R. No. 121828
June 27, 2003
Second Division, Callejo, Sr. J.

193

FACTS: On February 5, 1988, at around 11:30 p.m., Elisa Rolan was inside their store
at 613 Nueve de Pebrero Street, Mandaluyong City, waiting for her husband to arrive.
Joselito Capa and Julian Azul, Jr. were drinking beer. Edmar Aguilos and Odilon Lagliba
arrived at the store. Joselito and Julian invited them to join their drinking spree, and
although already inebriated, the two newcomers obliged. In the course of their drinking,
the conversation turned into a heated argument. Edmar nettled Julian, and the latter
was peeved. An altercation between the two ensued. Elisa pacified the protagonists and
advised them to go home as she was already going to close up. Edmar and Odilon left
the store. Joselito and Julian were also about to leave, when Edmar and Odilon
returned, blocking their way. Edmar took off his eyeglasses and punched Julian in the
face. Elisa shouted: "Tama na. Tama na." Edmar and Julian ignored her and traded fist
blows until they reached Aling Soteras store at the end of the street, about twelve to
fifteen meters away from Elisas store. For his part, Odilon positioned himself on top of a
pile of hollow blocks and watched as Edmar and Julian swapped punches. Joselito tried
to placate the protagonists to no avail. Joselitos intervention apparently did not sit well
with Odilon. He pulled out his knife with his right hand and stepped down from his
perch. He placed his left arm around Joselitos neck, and stabbed the latter. Ronnie and
the appellant, who were across the street, saw their gangmate Odilon stabbing the
victim and decided to join the fray. They pulled out their knives, rushed to the scene and
stabbed Joselito. Elisa could not tell how many times the victim was stabbed or what
parts of his body were hit by whom. The victim fell in the canal. Odilon and the appellant
fled, while Ronnie went after Julian and tried to stab him. Julian ran for dear life. When
he noticed that Ronnie was no longer running after him, Julian stopped at E. Rodriguez
Road and looked back. He saw Ronnie pick up a piece of hollow block and with it
bashed Joselitos head. Not content, Ronnie got a piece of broken bottle and struck
Joselito once more. Ronnie then fled from the scene. Joselito died on the spot. Elisa
rushed to Joselitos house and informed his wife and brother of the incident.
ISSUE: Whether or not appellant was an accomplice.
HELD: NO. In this case, Odilon all by himself initially decided to stab the victim. The
appellant and Ronnie were on the side of the street. However, while Odilon was
stabbing the victim, the appellant and Ronnie agreed to join in; they rushed to the scene
and also stabbed the victim with their respective knives. The three men simultaneously
stabbed the hapless victim. Odilon and the appellant fled from the scene together, while
Ronnie went after Julian. When he failed to overtake and collar Julian, Ronnie returned
to where Joselito fell and hit him with a hollow block and a broken bottle. Ronnie then
hurriedly left. All the overt acts of Odilon, Ronnie and the appellant before, during, and
after the stabbing incident indubitably show that they conspired to kill the victim.
The victim died because of multiple stab wounds inflicted by two or more persons.
There is no evidence that before the arrival of Ronnie and the appellant at the situs
criminis, the victim was already dead. It cannot thus be argued that by the time the
appellant and Ronnie joined Odilon in stabbing the victim, the crime was already
consummated.

194

All things considered, we rule that Ronnie and the appellant conspired with Odilon to kill
the victim; hence, all of them are criminally liable for the latters death. The appellant is
not merely an accomplice but is a principal by direct participation.
Even assuming that the appellant did not conspire with Ronnie and Odilon to kill the
victim, the appellant is nevertheless criminally liable as a principal by direct
participation. The stab wounds inflicted by him cooperated in bringing about and
accelerated the death of the victim or contributed materially thereto.

PEOPLE OF THE PHILIPPINES, appellee, vs.


WILFREDO TOLENTINO Y ESPERAT and JONATHAN FABROS Y
CASTRO, accused,
JONATHAN FABROS Y CASTRO, appellant.
G.R. No. 139179
April 3, 2002
Third Division, Panganiban, J.
FACTS: "On February 28, 1996 around 7:30 in the evening, appellant and his cousins,
Sheila Guilayan and Merwin Ledesma, were at their house in Luyahan, Pasonanca,
Zamboanga City when their neighbor Wilfredo Tolentino called them. When asked what
was it all about, Wilfredo simply motioned to them to come to his house located just
across the road. Once they were inside the house, Wilfredo immediately revealed his
plan to kill Hernan Sagario, Sheila's stepfather. Wilfredo explained that it was the only
way to free Sheila's mother - appellant's aunt - of the sufferings being caused by
Hernan. Wilfredo then instructed Merwin to go back to the house and get the bolo of
Hernan. Merwin obliged, got the bolo, and gave it to Wilfredo. Thereafter, they were told
by Wilfredo to go home and wait for Hernan.1wphi1.nt
"Around 8:30 in the evening, Hernan arrived. He went directly to the kitchen and fixed
the bag of rice he was carrying. Meanwhile, appellant, together with Sheila and Merwin,
just stayed quiet in the living room. Moments later, Wilfredo with a 2"x2" piece of wood
in his hand entered the house. He then followed Hernan towards the kitchen. When
about an armslength away from Hernan, Wilfredo, without saying a word, immediately
walloped Hernan on the right side of the neck sending the latter unconscious and falling
face down to the ground. Wilfredo immediately instructed appellant and Merwin to help
him bring Hernan out of the house. Lifting Hernan out of the house, Wilfredo held him by
the neck while both appellant and Merwin grasped his feet. They then carried Hernan
towards the creek about seven meters away from the house.
"Upon reaching the creekside, the three stopped and moved closer to the water. At this
juncture, Wilfredo successively stabbed Hernan on different parts of the body causing
the latter's instant death. After throwing the victim's lifeless body in the creek, the three
immediately left.
ISSUE: Whether or not appellant was an accomplice in the case at bar.
HELD: NO. Neither can appellant be convicted as an accomplice. Article 18 of the
Revised Penal Code defines accomplicesas "those persons who, not being included in
195

Article 17, cooperate in the execution of the offense by previous or simultaneous acts."
To be convicted as an accomplice, it is necessary that the accused be aware of the
criminal intent of the principal and then cooperate knowingly or intentionally by
supplying material or moral aid for the efficacious execution of the crime.
To consider a person an accomplice in the commission of the offense, the following
must concur: (1) community of design -- knowing the criminal design of the principal by
direct participation, one concurs therein; (b) cooperation in the execution of the offense
by previous or simultaneous acts, with the intention of supplying material and moral aid
in the execution of the crime in an efficacious way; and (c) a relation between the acts
done by the principal and those attributed to the person charged as accomplice.
To be deemed an accomplice, one needs to have had both knowledge of and
participation in the criminal act. In other words, the principal and the accomplice must
have acted in conjunction and directed their efforts to the same end. Thus, it is essential
that both were united in their criminal design.
In the case before us, appellant did not concur in or lend support to the nefarious intent
of Tolentino. The mere fact that the former had prior knowledge of the latter's criminal
design did not automatically make him an accomplice. This circumstance, by itself, did
not show his concurrence in the principal's criminal intent.
That appellant helped Tolentino carry the victim from the house to the creek did not
necessarily demonstrate concurrence of wills or unity of purpose or action. Quite the
contrary, the former's attempt to dissuade the latter from killing Sagario was attested to
by the prosecution witness. With the nominal role appellant played in the drama that
had been thrust upon him, we cannot declare that he was an accomplice in the crime
charged.

(ACCESSORIES)
PEOPLE OF THE PHILIPPINES, appellee, vs.
WILFREDO TOLENTINO Y ESPERAT and JONATHAN FABROS Y
CASTRO, accused,
JONATHAN FABROS Y CASTRO, appellant.
G.R. No. 139179
April 3, 2002
Third Division, Panganiban, J.

FACTS: "On February 28, 1996 around 7:30 in the evening, appellant and his cousins,
Sheila Guilayan and Merwin Ledesma, were at their house in Luyahan, Pasonanca,
Zamboanga City when their neighbor Wilfredo Tolentino called them. When asked what
was it all about, Wilfredo simply motioned to them to come to his house located just
across the road. Once they were inside the house, Wilfredo immediately revealed his
plan to kill Hernan Sagario, Sheila's stepfather. Wilfredo explained that it was the only
196

way to free Sheila's mother - appellant's aunt - of the sufferings being caused by
Hernan. Wilfredo then instructed Merwin to go back to the house and get the bolo of
Hernan. Merwin obliged, got the bolo, and gave it to Wilfredo. Thereafter, they were told
by Wilfredo to go home and wait for Hernan.1wphi1.nt
"Around 8:30 in the evening, Hernan arrived. He went directly to the kitchen and fixed
the bag of rice he was carrying. Meanwhile, appellant, together with Sheila and Merwin,
just stayed quiet in the living room. Moments later, Wilfredo with a 2"x2" piece of wood
in his hand entered the house. He then followed Hernan towards the kitchen. When
about an armslength away from Hernan, Wilfredo, without saying a word, immediately
walloped Hernan on the right side of the neck sending the latter unconscious and falling
face down to the ground. Wilfredo immediately instructed appellant and Merwin to help
him bring Hernan out of the house. Lifting Hernan out of the house, Wilfredo held him by
the neck while both appellant and Merwin grasped his feet. They then carried Hernan
towards the creek about seven meters away from the house.
"Upon reaching the creekside, the three stopped and moved closer to the water. At this
juncture, Wilfredo successively stabbed Hernan on different parts of the body causing
the latter's instant death. After throwing the victim's lifeless body in the creek, the three
immediately left.
ISSUE: Whether or not appellant was an accessory in the case at bar.
HELD: NO. Appellant cannot be convicted as an accessory either. Article 19 of the
Revised Penal Code defines an accessory as one who had knowledge of the
commission of the crime and did not participate in its commission as principal or
accomplice, yet took part subsequent to its commission by any of three modes: (1)
profiting oneself or assisting the offender to profit by the effects of the crime; (2)
concealing or destroying the body of the crime, or the effects or instruments thereof, in
order to prevent its discovery; and (3) harboring, concealing, or assisting in the escape
of the principals of the crime, provided the accessory acts with abuse of his public
functions or when the offender is guilty of treason, parricide, murder, or an attempt to
take the life of the Chief Executive, or is known to be habitually guilty of some other
crime. To convict an accused as an accessory, the following elements must be proven:
(1) knowledge of the commission of the crime and (2) subsequent participation in it by
any of the three above-cited modes.
Under paragraph 2 of said codal provision, the concealment or the destruction of the
body of the crime or of the effects or the instruments thereof must have been done in
order to prevent the discovery of the crime. That, precisely, is wanting in the present
case.
In his testimony, appellant stated that because he was afraid his co-accused would hurt
him if he refused, he agreed to assist the latter in carrying the victim towards the river.
The fact that appellant left thereafter likewise indicated his innocence of the
charge. Verily, he adequately explained his conduct prior to the stabbing incident as one
197

born of fear for his own life. It is not incredible for an eyewitness to a crime, especially if
unarmed, to desist from assisting the victim if to do so would put the former's life in peril.
Thus, in People v. Verzola, we explained as follows:
"x x x. It must be noted that Josefina testified that she helped her co-appellant bring the
body of the deceased down the stairs because of fear. Even if she assisted her coappellant without duress, simply assisting Verzola in bringing the body down the house
to the foot of the stairs and leaving said body for anyone to see, cannot be classified as
an attempt to conceal or destroy the body of the crime, the effects or instruments
thereof, must be done to prevent the discovery of the crime."

PEOPLE OF THE PHILIPPINES, plaintiff-appellee, vs.


ALBERTO S. ANTONIO, SPO4 JUANITO N. NIETO and SPO1 HONORIO
CARTALLA, JR., accused-appellants.
G.R. No. 128900
July 14, 2000
First Division, Ynares-Santiago, J.
FACTS: The tragic events began to unravel in the final hours of November 1, 1996.
Antonio, Tuadles, and a certain Danny Debdani, then president of the IBC, had agreed
to meet at the club for another poker session, their third night in a row. Antonio arrived
at the club first, followed by Tuadles at around midnight. Debdani, however, failed to
appear, so after waiting for sometime, Antonio and Tuadles decided to play "pusoy dos",
a game for two (2) players only. They continued playing until morning, pausing only
when either of them had to visit the restroom. They stopped playing at around 9:00
oclock in the morning of November 2, 1996, to eat breakfast.
When it came time to tally their scores and collect the winnings from the loser, an
argument arose. It is at this point where the prosecution and the defense presented two
very different scenarios. The prosecution alleged and sought to prove that in the course
of an argument, without warning or cause, Antonio pulled his gun from behind his back
and shot Tuadles at very close range, thus employing treacherous means to accomplish
the nefarious deed. The pivotal evidence presented by the prosecution was the
testimony of one Jose Jimmy T. Bobis, a security guard who testified as to how the
shooting of Tuadles occurred.
On November 18, 1996, an Information was filed against Antonio for the crime of
murder. Also charged as accessories were SPO4 Nieto and SPO1 Honorio Cartalla, Jr.
The Information alleged that:
The accused Nieto, without having participated in said crime of murder, either as
principal or accomplice, did then and there wilfully, unlawfully and feloniously take part
subsequent to its commission, with abuse of his public functions and position as a
public officer, by harboring or assisting the accused Antonio, by then and there failing to
arrest and surrender immediately the said accused Antonio to the authorities and by
giving false information which tended to deceive the investigating authorities
198

ISSUE: Whether or not SPO4 JUANITO N. NIETO was an accessory in the case at bar.
HELD: YES. Under paragraph 3 of Article 19 of the Revised Penal Code, there are two
(2) classes of accessories, one of which is a public officer who harbors, conceals or
assists in the escape of the principal. Such public officer must have acted with abuse of
his public functions, and the crime committed by the principal is any crime, provided it is
not a light felony. Appellant SPO4 Nieto is one such public officer, and he abused his
public function when he failed to effect the immediate arrest of accused Antonio and to
conduct a speedy investigation of the crime committed.
The evidence in the case at bar, insofar as appellant Nieto's culpability is concerned,
shows that in the middle of the argument between appellant Antonio and the deceased,
Antonio called Nieto by shouting, "Sarge! Sarge!" Hearing this, SG Bobis woke Nieto up
and the latter went upstairs. Immediately thereafter, appellant Antonio shot Tuadles, and
then ordered Nieto to get the scoresheet and the cards from the table, which Nieto did.
Antonio, Nieto and Bobis went downstairs. Antonio told guards Bobis and Ernesto Olac
to go with them, and they all boarded Antonio's Mercedes Benz van, including Nieto.
They arrived at Antonio's residence in Greenmeadows Subdivision at around 11:30
o'clock in the morning. There, they had coffee while Antonio made some telephone
calls. Soon after, a certain Atty. Abaya arrived and talked to the two security guards,
while Nieto was present. Nieto then told Bobis that in his statement, he should say that
the two of them, i.e., Bobis and Nieto, were seated outside the entrance of the Club
when the incident took place. At 5:00 o'clock in the afternoon, Nieto, Bobis and Olac
returned to the Club. They waited outside until members of the San Juan police,
together with Mayor Jinggoy Estrada and Vice Mayor Philip Cezar, arrived at 6:00
o'clock in the evening. After the police investigated the scene, they proceeded to the
police station. There, Nieto reiterated his instruction to Bobis to say that the two of them
were outside the club. While Bobis gave his statement to the police, Nieto remained in
front of him and dictated to him what he should answer to the questions of the police
investigator.
The foregoing facts were culled from the testimony of SG Bobis. Appellant Nieto's
actuations immediately after the commission of the crime demonstrate his liability as an
accessory. Being a police officer in the active service, he had the duty to arrest
appellant Antonio after the latter committed a crime in his presence, and which he
himself witnessed. Unfortunately, he failed to do what was incumbent upon him to do.
Instead, he rode with the offender to the latter's house where they stayed for more than
five (5) hours.
PEOPLE OF THE PHILIPPINES, plaintiff-appellee, vs.
LEONILO CUI y BALADJAY, BEVERLY CUI y CANTUBA, EDUARDO BASINGAN y
SABELLO, WILFREDO GARCIA, alias "TOTO", JOSELITO GARCIA, alias "TATA
GARCIA", EMMANUEL GARCIA, alias "MAWI", a certain SADAM, BEINVENIDO
NACARIO y PARDILLO, alias "REY NACARIO", a certain EDGAR, a certain
BENJIE, LUIS OBESO, alias "LEOS", HILARIA SARTE, alias "LARING", and YUL
199

ALVAREZ, accused, LEONILO CUI y BALADJAY, BEVERLY CUI y CANTUBA, LUIS


OBESO, alias "LEOS", and HILARIA SARTE,alias "LARING", accused-appellants.
G.R. No. 121982 September 10, 1999
First Division, Puno, J.
FACTS: In the evening of December 5, 1990, some ten (10) armed robbers raided the
compound of Johnny and Rose Lim on Edison Street, Lahug, Cebu City. The Lims, their
three (3) children, and the employees of the family-owned business, William's
Educational Supply, were able to see the faces of the leader Wilfredo alias "Toto" Garcia
and two of his men, Mawe Garcia and a certain Edgar. The other robbers could not be
identified as they had flour sacks over their heads.
When the police learned from Lim that his house guard, Eduardo Basingan, hailed from
Quiot, Pardo, Cebu City, they decided to interrogate him.1wphi1.nt
Basingan's interrogation broke the case wide open. He identified Toto Garcia, Mawe
Garcia and Edgar as the three (3) who did not wear masks, Sadam and Rey as the two
(2) who held him and the Lims at gunpoint, and Tata Garcia, Yul Alvarez, a certain
Benjie, a certain Leos and a certain Laring as the look-outs who stayed outside the Lim
compound. He named Toto Garcia as the chief plotter of the crime at bar, and revealed
that his neighbor and close family friends, the spouses Leonilo and Beverly Cui,
participated in the plan. Basingan said he was asked to join the plot and was assured
that he would not be under suspicion because he would be placed at gun point together
with the other members of the Lim household when the crime is committed. However,
he refused to join the plot during the December 2, 1990 meeting of the group at the
residence of the Cuis in Quiot, Pardo, Cebu City. Leonilo Cui even invoked their close
ties as godfathers of each other's children but he was unmoved. At the meeting were
Toto Garcia, Mawi Garcia, Edgar, Rey, Sadam and the Cuis.
ISSUE: Whether or not spouses Cuis are accessories in the case at bar.
HELD: YES. This Court, however, holds that the Cuis profited from the kidnapping of
Stephanie Lim and are liable as accessories.
Art. 19 of the Revised Penal Code, as amended, penalizes as accessories to the crime
those who, subsequent to its commission, take part therein by profiting themselves or
assisting the offenders to profit by the effects of the crime, without having participated
therein, either as principals or accomplices. Conviction of an accused as an accessory
requires the following elements: (1) that he has knowledge of the commission of the
crime; and (2) that he took part in it subsequent to its commission by any of the three
modes enumerated in Article 19 of the Revised Penal Code, as amended. These twin
elements are present in the case of the Cuis, and indubitable proof thereof is extant in
the records of the case.
200

The members of the Metrodiscom Intelligence Security Team (MIST), namely, Lt. John
P. Campos, Lt. Michael Ray B. Aquino, Sgt. Narciso L. Ouano, Jr., Sgt. Felipe
Honoridez, Sgt. Armando Ballon, Sgt. Oscar Dadula, Cpl. Jeremias Canares, and Sgt.
Catalino Ybanez, executed a Joint Affidavit dated December 18, 1990 stating, among
other things, that "the couple Leonilo and Beverly Cui, although denying knowledge of
the kidnapping revealed that Toto Garcia is their Compadre" and that "they also turned
over to us the amount of P10,000.00 representing that given to them by Toto Garcia out
of the ransom money".
This statement charging the Cuis with having partaken of the ransom money was not
denied either in the Counter-Affidavit of Leonilo Cui dated February 15, 1991 or in the
Counter-Affidavit of Beverly Cui of the same date. In his Counter-Affidavit, Leonilo Cui
even admitted that he knew that Toto Garcia and Basingan had held secret meetings in
his house and that he had already become suspicious of their acts, but he did not
confront them because they treated each other as special friends, they being godfather
of each other's children.

THE PEOPLE OF THE PHILIPPINES, plaintiff-appellee, vs.


RICARDO VERZOLA & JOSEFINA MOLINA, accused-appellants.
G.R. No. L-35022 December 21, 1977
Second Division, Antonio, J.
FACTS: At about 10:00 o'clock on the night of September 28, 1969, Bernardo Molina
was clubbed to death by Ricardo Verzola in the presence of appellant Josefina Molina
inside Molina's house at Barrio Lipcan, Bangued, Abra. The body of the victim was
subsequently carried by the two appellee to the ground and left at the foot of the stairs.
Appellant Verzola then went to his house, changed his clothes and threw his
bloodstained sweater undershirt and underwear, including the piece of wood be used in
clubbing the deceased, inside their toilet. Afterwards, he went to the municipal building
and reported to the police authorities that Bernardo had died in an accident.
ISSUE: Whether or not Josefina Molina was an accessory in the case at bar.
HELD: NO. An accessory does not participate in the criminal design, nor cooperate in
the commission of the felony, but, with knows of the commission of the crime, he
subsequently takes part in three (3) ways: (a) by profiting from the effects of the crime;
(b) by concealing the body, effects or instruments of the crime in order to prevent its
discovery; and (c) by assisting in the escape or concealment of the principal of the
crime, provided he acts with abuse of his public functions or the principal is guilty of
treason, parricide, murder, or an attempt to take the life of the Chief Executive or is
known to be habitually guilty of some other crime. The main difference separating
accessories after the fact the responsibility of the accessories is subsequent to the
consummation of the crime and subordinate to that of the principal.
201

According to the trial court, " the bringing down of the body of the victim ... was to
destroy the body of the crime, or its effect that as to make it appear that the death of the
victim was caused by an accident. We disagree. There is no iota of proof that Josefina
Molina ever attempted "to destroy the body of the crime" or to make it appear that death
of the victim was accidental. It must be noted that Josefina testified that she helped her
co- appellant bring the body of the deceased down the stairs because of fear. Even if
she assisted her co-appellant without duress, simply Verzola in bringing the body down
the house to the foot of the stairs and leaving said body for anyone to see, cannot be
classified as an attempt to or destroy the body of the crime the effects or instruments
thereof, must be done to prevent the discovery of the crime. In the case at bar, the body
was left at the foot of the stairs at a place where it was easily visible to the public. Under
such circumstances there could not have been any attempt on the part of Josefina to
conceal or destroy the body of the crime-

PENALTIES IN GENERAL (ART. 21-24)


(RETROACTIVE EFFECTS OF PENAL LAWS)

PEOPLE OF THE PHILIPPINES, Appellee, vs.


GERARDO "GERRY" EVINA Y PADUAL, Appellant.
G.R. Nos. 124830-31
June 27, 2003
Second Division, Callejo, Sr. J.
FACTS: In the evening of November 3, 1991, Luciana, along with the appellants
mother, went to the barangay captain to check out a cash loan transaction. Left in the
house were Basilio, Maritess, the latters older sister, and some visitors. Basilio was at
the balcony whiling away time, and Maritess was busy playing by herself. Meanwhile, a
drinking spree was on-going at the living room. At around 9:00 p.m., Maritess decided to
retire for the night. She proceeded to the bedroom and though it was not lighted, she
saw the appellant inside the room, just sitting there. Unconcerned, Maritess went to bed
to sleep. But before she finally dozed off, she noticed that the appellant had locked the
door. Maritess was suddenly awakened by the appellant, who immediately gagged her
mouth with her red dress. The appellant tied her hands with a big handkerchief and
poked a knife at her. He ordered Maritess to keep quiet. He then took off her
undergarment and undressed himself. He mounted Maritess and thrust his penis into
her vagina. The appellants penis penetrated her vagina by about half an inch. He then
made a push and pull movement. Maritess felt excruciating pain. While pumping, he
ejaculated. Maritess felt something come out from the appellants penis. Satiated, he
dismounted and untied Maritess. He warned her not to breathe a word of what had
happened, otherwise he would kill her and the rest of her family. The appellant then
ordered Maritess to leave the room. Maritess did as she was told, and went to the sala
202

where she watched television. Maritess did not tell her father about the harrowing
incident because she feared that she and her family might indeed be killed. The incident
was repeated on November 7, 1991.
ISSUE: Whether Sections 8 and 9 Rule 110 of the Revised Rules of Court can be given
retroactive affect in the case at bar.
HELD: YES.Although the special aggravating circumstance of the use of a weapon and
the aggravating circumstance of dwelling were proven, these aggravating
circumstances cannot be considered in fixing the penalty because they were not alleged
in the information as mandated by Rule 110, Sections 8 and 9 of the Revised Rules of
Criminal Procedure. Although the crimes charged were committed before the effectivity
of the said rule, nevertheless, the same should be applied retroactively being favorable
to the appellant.

PEOPLE OF THE PHILIPPINES, petitioner, vs.


ARTURO F. PACIFICADOR, respondent.
G.R. No. 139405
March 13, 2001
Second Division, De Leon, Jr. J
FACTS: On October 27, 1988, herein respondent, Arturo Pacificador y Fullon, and his
erstwhile co-accused, Jose T. Marcelo, were charged before the Sandiganbayan with
the crime of violation of Republic Act No. 3019, as amended, otherwise known as the
Anti-Graft and Corrupt Practices Act.
That on or about and during the period from December 6, 1975 to January 6, 1976, in
Metro Manila, Philippines, and within the jurisdiction of this Honorable Court, said
accused, Arturo Pacificador, then Chairman of the Board of the National Shipyard and
Steel Corporation, a government-owned corporation, and therefore, a public officer, and
Jose T. Marcelo, Jr., then President of the Philippine Smelters Corporation, a private
corporation, conspiring and confederating with one another and with other individuals,
did then and there, willfully, unlawfully and knowingly, and with evident bad faith
promote, facilitate, effect and cause the sale, transfer and conveyance by the National
Shipyard and Steel Corporation of its ownership and all its titles, rights and interests
over parcels of land in Jose Panganiban, Camarines Norte where the Jose Panganiban
Smelting Plant is located including all the reclaimed and foreshore areas of about 50
hectares to the Philippine Smelters Corporation by virtue of a contract, the terms and
conditions of which are manifestly and grossly disadvantageous to the Government as
the consideration thereof is only P85,144.50 while the fair market value thereof at that
time was P862,150.00 thereby giving the Philippine Smelters Corporation unwarranted
benefits, advantages and profits and causing undue injury, damage and prejudice to the
government in the amount of P777,005.50.
203

Petitioner contends that, contrary to the ruling of the Sandiganbayan, the provision of
Act No. 3326 on prescription of offenses punishable under special laws is not applicable
to the instant criminal case for the reason that Republic Act No. 3019 provides for its
own prescriptive period. Section 11 thereof provides that offenses committed and
punishable under the said law shall prescribe in fifteen (15) years. However, inasmuch
as Republic Act No. 3019 does not state exactly when the fifteen-year prescriptive
period begins to run, Article 91 of the Revised Penal Code should be applied
suppletorily. Article 91 of the Revised Penal Code, which adopts the "discovery rule" for
the prescription of offenses
ISSUE: Whether or not Section 11 of R.A. No. 3019 may be given retroactive effect in
the case at bar.
HELD: NO.It can be gleaned from the Information in this case that respondent
Pacificador allegedly committed the crime charged "on or about and during the period
from December 6, 1975 to January 6, 1976." Section 11 of R.A. No. 3019, as amended
by B.P. Blg. 195, provides that the offenses committed under the said statute shall
prescribe in fifteen (15) years. It appears however, that prior to the amendment of
Section 11 of R.A. No. 3019 by B.P. Blg. 195 which was approved on March 16, 1982,
the prescriptive period for offenses punishable under the said statute was only ten (10)
years. The longer prescriptive period of fifteen (15) years, as provided in Section 11 of
R.A. No. 3019 as amended by B.P. Blg. 195, does not apply in this case for the reason
that the amendment, not being favorable to the accused (herein private respondent),
cannot be given retroactive effect. Hence the crime prescribed on January 6, 1986 or
ten (10) years from January 6, 1976.
PEOPLE OF THE PHILIPPINES, plaintiff-appellee,vs.
APOLINAR LAZARO y SERVANIA, accused-appellant.
G.R. No. 112090 October 26, 1999
Third Division, Gonzaga-Reyes, J.
FACTS: This is an appeal from the decision dated 1 March 1993 of the Regional Trial
Court of Naga City, Branch 24, finding accused-appellant Apolinar Lazaro y Servania
guilty of the crime of illegal possession of firearms and ammunition under Section 1 of
Presidential Decree No. 1866 and sentencing him to suffer the penalty of reclusion
perpetua and to pay the costs.
Appellant brought an injured person at the hospital for treatment. However, as police
officer was previously informed that the driver (appellant) of the jeep was armed with a
handgun, Major Tuazon ordered the said driver to step out of his jeep. He then saw the
driver with a handgun tucked in his waist, pull out the handgun from its holster and drop
it at the back of the driver's seat. Upon seeing the gun, Major Tuazon pulled the driver
out of the vehicle, got hold of the gun which turned out to be a .38 caliber revolver
bearing Serial Number 1029315 . The gun contained six empty shells.
204

ISSUE: Whether or not RA 8294 can be given retroactive effect.


HELD: YES. In view of the amendments introduced by Republic Act 8294 to
Presidential Decree 1866, separate prosecutions for homicide and illegal possession
are no longer in order. Instead, illegal possession of firearms is merely to be taken as an
aggravating circumstance in the homicide case.
R.A. 8294 took effect on July 6, 1997. The crime involved in the case at bench was
committed on May 5, 1991. As a general rule, penal laws will generally have prospective
application except where the new law will be advantageous to the accused. In this case
R.A. 8294 will spare accused-appellant from a separate conviction for the crime of
illegal possession of firearm. Accordingly, said law should be given retroactive
application.
In People vs. Valdez, this Court stated:
Republic Act No. 8294 took effect on July 6, 1997, fifteen days after its publication on
June 21, 1997. The crimes involved in the case at bar were committed on September
17, 1995. As in the case of any penal law, the provisions of Republic Act No. 8294 will
generally have prospective application. In cases, however, where the new law will be
advantageous to the accused, the law may be given retroactive application (Article 22,
Revised Penal Code). Insofar as it will spare accused-appellant in the case at bar from
a separate conviction for the crime of illegal possession of firearms, Republic Act No.
8294 may be given retroactive application in Criminal Case No. U-8749 (for Illegal
Possession of Firearm) subject of this present review.
(PARDON BY THE OFFENDED PARTY)
ARNOLD STA. CATALINA, Petitioner - versus PEOPLE OF THE PHILIPPINES, Respondent.
G.R. No. 167805
November 14, 2008
Second Division, Quisumbing, J.
FACTS: Petitioner was charged before the Regional Trial Court, Branch 63, Makati City,
with the crime of estafa defined under Article 315, paragraph 1(b) of the Revised Penal
Code.
Sometime in February 1988, Ballecer entered into a joint business venture with
petitioner involving importation of jute sacks from China. Petitioner intimated to Ballecer
that he could secure the jute sacks from China through a company in Hongkong which
would act as his agent. Petitioner also told Ballecer that he had a ready buyer in
the Philippines named Saugus Enterprises which was willing to buy the jute sacks
205

at P12.25 per piece. Convinced, Ballecer ordered through petitioner one container load
of jute sacks with the total cost of P137,000.
While preparing the supporting customs documents for the letter of credit, Ballecer
found that the cost of the jute sacks was not $0.15 but $0.62 or P16.15 per
piece. Realizing that his business venture was a losing proposition, Ballecer cancelled
the importation and asked petitioner to return the P100,000. Petitioner, however, failed
to return the money despite repeated verbal and formal demands.
On April 10, 2000, the public prosecutor filed a Manifestation [15] stating that Ballecer was
no longer interested in pursuing his complaint against petitioner and that the case
should be decided in light of Ballecers Affidavit of Desistance. [16]
ISSUE: Whether or not affidavit of desistance can justify the dismissal of the instant
case.
HELD: NO.Affidavit of Desistance submitted by Ballecer will not justify the dismissal of
the action. By itself, an Affidavit of Desistance is not a ground for the dismissal of an
action, once the action has been instituted in court. Here, Ballecer made the so-called
pardon of the petitioner after the institution of the action. He made the Affidavit of
Desistance only on October 25, 1999 more than two years after the trial court had
rendered its decision. The Court attaches no persuasive value to a desistance
especially when executed as an afterthought. It would be a dangerous rule to reject the
testimony taken before the court of justice simply because the witness who had given it
later on changed his mind for one reason or another. Such a rule will make a solemn
trial a mockery and place the investigation at the mercy of unscrupulous
witnesses. Moreover, if we allow the dismissal of the case in view of Ballecers Affidavit
of Desistance, there is always the probability that it would later on be repudiated, and
criminal prosecution would thus be interminable.
ROLANDO L. BALDERAMA, petitioner, vs.
PEOPLE OF THE PHILIPPINES and JUAN S. ARMAMENTO, respondents.
G.R. Nos. 147578-85
January 28, 2008
First Division, Sandoval-Gutierrez, J.
FACTS: On December 2, 1992, respondent, feeling aggrieved of the malicious
impounding of his vehicle, filed with the Office of the Ombudsman a complaint for
bribery and violation of Section 3(e) of Republic Act (R.A.) No. 3019, as
amended, against herein petitioners as well as Lubrica and de Jesus. He alleged that
prior to the impounding of his taxi, the four LTO officers had been collecting "protection
money" from him. On February 15, 1992, they went to his office and proposed they
would not apprehend his drivers and impound his vehicles for violations of LTO rules,
provided he gives them the amount of P400.00 every 15th and 30th day of the month.
They agreed to the reduced amount of P300.00. On the same day, he started giving
them P300.00 and from then on, every 15th and 30th day of the month until June 15,
206

1992. Thereafter, he failed to give them the agreed amount because his business was
not doing well.
Eventually, the Office of the Ombudsman filed with the Sandiganbayan nine (9)
Informations for violations of Article 210 of the Revised Penal Code against petitioners
and the other members of the team, docketed as Criminal Cases Nos. 20669-20677. All
the Informations were identically worded, except the date of the commission of the
crimes.
Upon conviction, petitioners and Lubrica filed separate motions for reconsideration
arguing that they were not yet grouped as a team on February 15, 1992, hence, there
could be no conspiracy. While the motion was pending resolution, both petitioners filed
separate motions for new trial based on an affidavit dated December 22, 2000 executed
by respondent recanting his previous testimony and pointing to Lubrica and de Jesus as
the only culprits.
ISSUE: Whether or not recantation or an affidavit of desistance may be appreciated in
the instant case.
HELD: NO.A recantation or an affidavit of desistance is viewed with suspicion and
reservation. The Court looks with disfavor upon retractions of testimonies previously
given in court. It is settled that an affidavit of desistance made by a witness after
conviction of the accused is not reliable, and deserves only scant
attention. The rationale for the rule is obvious: affidavits of retraction can easily be
secured from witnesses, usually through intimidation or for a monetary consideration.
Recanted testimony is exceedingly unreliable. There is always the probability that it will
later be repudiated. Only when there exist special circumstances in the case which
when coupled with the retraction raise doubts as to the truth of the testimony or
statement given, can retractions be considered and upheld. As found by the
Sandiganbayan, "(t)here is indubitably nothing in the affidavit which creates doubts on
the guilt of accused Balderama and Nagal."
PEOPLE OF THE PHILIPPINES, Appellee, vs.
EDGARDO DIMAANO, Appellant.
G.R. No. 168168
September 14, 2005
En banc, Per curiam
FACTS: Complainant was born on August 26, 1983, and was 10 years old when she
was first sexually abused in the morning of September 1993. While inside their house in
Sucat, Paraaque, appellant entered her room and laid down beside her. He removed
her clothes and asked her to lie face down then inserted his penis into her anus.
Complainant cried and felt so much pain, but she kept the incident to herself as her
father might hurt her. The incident repeated a few days later. It was only in November
207

1995 that she confided the sexual abuses to her mother. However, the incident was
repeated on December 29, 1995 and on January 1, 1996.
On January 26, 1996, Maricar Dimaano charged her father, Edgardo Dimaano
with two (2) counts of rape and one (1) count of attempted rape in the complaints.
ISSUE: Whether or not the voluntary and due execution of the affidavit of desistance by
the private complainant should have been duly considered as a factor which put to
doubt the reasons behind the filing of the criminal charges of rape against herein
accused.
HELD: NO.Appellant's reliance on complainant's affidavit of desistance deserves scant
consideration. A survey of our jurisprudence reveals that the court attaches no
persuasive value to a desistance, especially when executed as an afterthought. The
unreliable character of this document is shown by the fact that it is quite incredible that a
victim, after going through the trouble of having the appellant arrested by the police,
positively identifying him as the person who raped her, enduring the humiliation of a
physical examination of her private parts, repeating her accusations in open court and
recounting her anguish in detail, will suddenly turn around and declare that she is no
longer interested in pursuing the case.
Too, complainant repudiated the affidavit of desistance in open court by stating that no
lawyer assisted her when she affixed her signature and had shown her resolve to
continue with the prosecution of the cases. Besides, the trial court is not bound to
dismiss the cases, as it is still within its discretion whether or not to proceed with the
prosecution, considering that the compromise agreement and the affidavit of desistance
were executed long after the cases have been filed in court.
Moreover, a criminal offense is an outrage to the sovereign State and to the State
belongs the power to prosecute and punish crimes. By itself, an affidavit of desistance is
not a ground for the dismissal of an action, once it has been instituted in court. A private
complainant loses the right or absolute privilege to decide whether the rape charge
should proceed, because the case was already filed and must therefore continue to be
heard by the trial court.
In addition, a careful scrutiny of the affidavit of desistance reveals that complainant
never retracted her allegation that she was raped by her father. Neither did she give any
exculpatory fact that would raise doubts about the rape. All she stated in the affidavit
was that she had decided to withdraw the complaints after the appellant agreed not to
disturb the complainant; to consent to annul his marriage; allow his wife to solely
manage the conjugal properties; and entrust the custody of his children to his wife.
Rather than contradict, this affidavit reinforces complainant's testimony that appellant
raped her on several occasions.
PENALTIES: CLASSIFICATION, DURATION, EFFECTS (ART. 25-45)
(RECLUSION PERPETUA)
208

PEOPLE OF THE PHILIPPINES, Appellee, vs.


NOLI NOVIO y AYASO, Appellant.
G.R. No. 139332
June 20, 2003
Second Division, Callejo, Sr. J.
FACTS: In the evening of September 23, 1994, Maricel, together with her brothers Jun, aged 11, and Joey, aged 9 years - slept side by side in the store while their parents
slept in the beach house. Maricel was wearing a red duster and panties.
The next day, at about 3:00 a.m., Maricel felt some ticklish sensation, as if somebody
was kissing her. But she also felt stabs of pain in her vagina. She opened her eyes and
was horrified to see Noli completely naked on top of her. Immediately, Noli covered her
mouth with his right hand and held both her hands with his left. He warned her not to
make any noise. She noticed, too, that her red duster had been rolled up to her neck
and her legs were being separated by Noli with the use of his legs. She struggled, but to
no avail. Noli then inserted his penis in Maricels vagina.
Court finds the accused Noli Novio guilty beyond reasonable doubt of the crime of
consummated rape provided for in paragraph one (1), Article 355 (sic) of the Revised
Penal Code, as amended by Section 11 of Republic Act No. 7659 and without the
Indeterminate Sentence Law applicable and neither any mitigating nor aggravating
circumstance present, this court hereby sentences accused Noli Novio to indivisible
penalty of reclusion perpetua with imprisonment of thirty (30) years.
ISSUE: Whether or not the penalty given was commensurate in the case at bar.
HELD: NO.Under Article 335 of the Revised Penal Code, as amended by Republic Act
7659, the prescribed penalty for simple rape is reclusion perpetua. However, the trial
court sentenced the appellant to thirty years of reclusion perpetua. The penalty imposed
by the trial court is void. Although under Article 27 of the Revised Penal Code as
amended by Republic 7659, reclusion perpetua has a range of twenty years and one
day to forty years, by nature, the penalty remains a single and indivisible penalty. It
cannot be divided into periods or equal portions. If the law prescribes reclusion perpetua
as a single and indivisible penalty for a felony, the trial court is mandated to impose said
penalty, absent any privileged mitigating circumstances conformably with Article 63 of
the Revised Penal Code. The trial court is not authorized to vary the penalty provided
for by law either in the character or the extent of punishment inflicted.
There was no need for the trial court to specify the duration of thirty years of reclusion
perpetua whenever it is imposed as a penalty in any proper case. The Court is not
impervious to Article 70 of the Revised Penal Code which pertinently provides that, in
applying the so-called "three-fold rule," i.e., that "(w)hen the culprit has to serve two or
more penalties, . . . the maximum duration of the convicts sentence shall not be more
than three-fold the length of time corresponding to the most severe of the penalties
imposed upon him" "the duration of perpetual penalties (penal perpetua) shall be
209

computed at thirty years." The imputation of a thirty-year duration to reclusion perpetua


in Article 70 is, as this Court recently held, "only to serve as the basis for determining
the convicts eligibility for pardon or for the application of the three-fold rule in the
service of multiple penalties.

PEOPLE OF THE PHILIPPINES, plaintiff-appellee, vs.


SAMMY ZACARIAS (At Large); RODEL ZACARIAS (At Large); WALLY TICALO;
RENE MATUGAS (Acquitted), accused,
WALLY TICALO, accused-appellant.
G.R. No. 138990
January 30, 2002
Third Division, Vitug, J.
FACTS: Pelicano testified that on 25 June 1993, about 12:30 A.M., while waiting for his
son to come home, he heard a commotion just outside his abode. He opened a window
overlooking the road and saw a teen-aged boy being chased by Sammy Zacarias and
Rodel Zacarias. The boy, Christopher Sacay, was the son of his long-time friend
Alejandro Sacay. Seconds later, his two other neighbors, Wally Ticalo and Rene
Matugas, also went after the teen-ager who was by then heading towards the Seventh
Day Adventist Church, about 20 meters from Pelicanos house. Pelicano rushed down
from his house and followed the group until he was only about 10 meters away from
where the four men finally caught up with the boy. He saw Rodel Zacarias hold the
young man while the rest took turns in stabbing and hacking the victim.
The Court finds the accused Wally Ticalo guilty beyond reasonable doubt of the
crime of murder and hereby sentences him to imprisonment of forty (40) years reclusion
perpetua and to pay the aggrieved party the sum of P50,000.00 as indemnity.
ISSUE: Whether or not the Court erred in giving the above penalty for the instant case.
HELD: NO.A word, in passing, about the manner the trial court imposed the penalty. In
the scales of penalties under the Revised Penal Code, reclusion perpetua is the penalty
immediately higher than reclusion temporal which has a duration of twelve years and
one day to twenty years. The minimum range of reclusion perpetua should then, by
necessary implication, start at 20 years and 1 day while the maximum thereunder could
be co-extensive with the rest of the natural life of the offender. Article 70, however,
provides that the maximum period in regard to the service of sentence shall not exceed
40 years. Reclusion perpetua remains to be an indivisible penalty and, when it is the
prescribed penalty, should be imposed in its entirety, i.e., reclusion perpetua sans a
fixed period for its duration, regardless of any mitigating or aggravating circumstance
that may have attended the commission of the crime. In prescribing the penalty
of reclusion perpetua, its duration in years, in fine, need not be specified.

210

Penalties: Classification, duration, effects:: Reclusion Perpetua

G.R. No. 138261


April 17, 2001
PEOPLE OF THE PHILIPPINES, appellee, vs. PEDRO RAMIREZ, appellant.
Third Division, Panganiban, J.:
Facts: Accused-appellant Pedro Ramirez was charged with murder for allegedly
stabbing Jonathan Jojo Alkuino. RTC-Ormoc City, Branch 35, found him guilty of the
crime of Murder and sentenced him to suffer imprisonment of forty years reclusion
perpetua and to pay the aggrieved party the sum of P50,000.00 as indemnity and
another sum of P50,000.00 as moral damages. In view of the penalty imposed, the
appeal was lodged directly with the Supreme Court.
Issue: Whether the lower court imposed the proper penalty
Held: NO. Reclusion perpetua is an indivisible penalty; hence, it is imposed in its
entirety.
There was no justification or need for the trial court to specify the length of
imprisonment, because reclusion perpetua is an indivisible penalty. The significance of
this fundamental principle was laid down by the Court in People v. Diquit:
"Since reclusion perpetua is an indivisible penalty, it has no minimum, medium or
maximum periods. It is imposed in its entirety regardless of any mitigating or
aggravating circumstances that may have attended the commission of the crime. (Art.
63, Revised Penal Code) Reclusion perpetua is imprisonment for life but the person
sentenced to suffer it shall be pardoned after undergoing the penalty for thirty (30)
years, unless by reason of his conduct or some other serious cause, he shall be
considered by the Chief Executive as unworthy of pardon (Art. 27, Revised Penal
Code)."
Penalties: Classification, duration, effects:: Reclusion Perpetua vs. Life
Imprisonment
G.R. Nos. 112453-56. June 28, 2001
PEOPLE OF THE PHILIPPINES, plaintiff-appellee, vs. GERARDO LATUPAN y
SIBAL, alias JERRY, accused-appellant.
211

First Division, Pardo, J.:


Facts: Accused-appellant Gerardo Latupan y Sibal alias Jerry was charged in four
separate informations with 2 counts of frustrated murder for attacking and wounding
Jaime and Leo Asuncion and 2 counts of murder for stabbing and killing Lilian and Jose
Asuncion. Upon arraignment, accused pleaded not guilty.
During the pre-trial
conference of the four cases, accused offered to change his plea of not guilty to guilty of
the complex crime of double murder and frustrated murder. The prosecution did not
interpose any objection. The trial court re-arraigned the accused. He withdrew his plea
of not guilty and instead pleaded guilty to the single offense of multiple murder with
multiple frustrated murder. Thereafter, the trial court ordered the prosecution to present
evidence to establish the culpability of the accused. The trial court rendered a decision,
the dispositive portion of which reads:
WHEREFORE, finding the accused GERARDO LATUPAN alias JERRY GUILTY
beyond reasonable doubt of the complex offense of Double Murder, the Court hereby
sentences him to suffer life imprisonment and to indemnify the heirs of the two victims in
the amount of P50,000.00 each or a total of P100,000.00.
For the physical injuries suffered by Jaime Asuncion, the accused is sentenced to suffer
ten (10) days imprisonment. Likewise, for the physical injuries suffered by Leon
Asuncion, the accused is also sentenced to suffer ten (10) days imprisonment, both to
be suffered simultaneously with the more grievous sentence of life imprisonment, plus
P200.00 indemnity to each of the two victims.
SO ORDERED.
Issue: Whether the trial court imposed the proper penalty
Held: No. Accused-appellant is liable for the deaths of Lilia and Jose Asuncion, and the
physical injuries of Jaime and Leo Asuncion. From the manner accused attacked the
family, he left them with no means of defense or escape. Considering the treacherous
manner by which the victims were killed, the accused-appellant is liable for murder and
physical injuries.
The trial court, however, erred in convicting accused-appellant of the complex crime of
double murder and separate offenses of serious physical injuries. The instant case
does not fall under any of the two mentioned instances when a complex crime is
committed, under Article 48 of the Revised Penal Code. The killing of Lilia Asuncion and
Jose Asuncion and the wounding of Jaime and Leo Asuncion resulted not from a single
act but from several and distinct acts of stabbing. "Where the death of two persons
does not result from a single act but from two different shots, two separate murders, and
not a complex crime, are committed."
Thus, accused-appellant is liable, not for a complex crime of double murder, but for two
separate counts of murder, and separate counts of physical injuries.
212

Under Article 248 of the Revised Penal Code, the penalty for murder at the time of the
commission of the crime in April 1991 was reclusion temporal maximum to death. The
trial court convicted accused-appellant of murder and sentenced him to life
imprisonment. The proper imposable penalty is reclusion perpetua, not life
imprisonment. Obviously, the trial court intended to impose reclusion perpetua.
However, the penalty of life imprisonment is not the same as reclusion perpetua. They
are distinct in nature, in duration and in accessory penalties. First, life imprisonment is
imposed for serious offenses penalized by special laws, while reclusion perpetua is
prescribed under the Revised Penal Code. Second, life imprisonment does not carry
with it any accessory penalty. Reclusion perpetua has accessory penalties. Third, life
imprisonment does not appear to have any definite extent or duration, while reclusion
perpetua entails imprisonment for at least thirty (30) years after which the convict
becomes eligible for pardon, although the maximum period thereof shall in no case
exceed forty (40) years.
SC also noted that the trial court sentenced accused to ten days of imprisonment for
each count of slight physical injuries. The rule is that it is necessary for the courts to
employ the proper legal terminology in the imposition of penalties because of the
substantial difference in their corresponding legal effects and accessory penalties. The
appropriate name of the penalty must be specified inasmuch as under the scheme of
penalties in the Revised Penal Code, the principal penalty for a felony has its own
specific duration and corresponding accessory penalties. Thus, the courts must employ
the proper nomenclature specified in the Revised Penal Code, such as reclusion
perpetua, not life imprisonment or ten days of arresto menor, not ten days of
imprisonment.
Hence, the proper penalty for each murder committed in April 1991, considering the
absence of aggravating and mitigating circumstances, is reclusion perpetua, with its
accessory penalties. Further, accused-appellant is liable for two counts of slight
physical injuries and must be sentenced to twenty (20) days of arresto
menor, each, likewise with its accessory penalties under the Revised Penal Code.
G.R. No. 96092. August 17, 1999
PEOPLE OF THE PHILIPPINES, plaintiff-appellee, vs. ALEXANDER BAUTISTA,
accused-appellant.
Second Division, Mendoza, J.:

Facts: On January 12, 1987, while the victim Allan Jone Clemente was having drinks
with Orlando Ocares, accused-appellant Alexander Bautista arrived and asked
Clemente to accompany him home. Clemente obliged and the two walked down F.
Aguilar Street with accused-appellant placing his left arm around Clementes
213

shoulder. When they passed in front of the house of Danilo Enrique Cancio, they
stopped. Suddenly accused-appellant pulled out a balisong (fan knife) with his right
hand and stabbed Clemente with it, hitting the latter on the lower right
abdomen. Accused-appellant afterwards fled to an alley leaving his victim in critical
condition. Clemente managed to go home, but he collapsed upon reaching their
house. He was rushed to the Chinese General Hospital in Sta. Cruz, Manila, where he
later expired.
The Regional Trial Court found accused-appellant guilty of murder and sentenced him
to suffer the penalty of life imprisonment.
Issue: Whether the lower court imposed the proper penalty
Held: NO. The penalty for murder is reclusion perpetua to death. As there was neither
aggravating nor mitigating circumstance, the imposable penalty, following Art. 63(2) of
the Revised Penal Code, is reclusion perpetua. But reclusion perpetua is not the same
as life imprisonment as the trial court seems to think. The two have important
consequential differences. As held in People v. Ballabare:
While life imprisonment may appear to be the English translation of reclusion
perpetua, in reality, it goes deeper than that. First, life imprisonment is invariably
imposed for serious offenses penalized by special laws, while reclusion perpetua is
prescribed under The Revised Penal Code. Second, life imprisonment,
unlike reclusion perpetua, does not carry with it any accessory penalty. Third, life
imprisonment does not appear to have any definite extent or duration, while reclusion
perpetua entails imprisonment for at least thirty (30) years after which the convict
becomes eligible for pardon, although the maximum period thereof shall in no case
exceed forty (40) years.
Application of Penalties:: Complex Crimes
G.R. No. 182551, July 27, 2011
PEOPLE OF THE PHILIPPINES, Plaintiff-Appellee, vs. ROSENDO REBUCAN y
LAMSIN, Accused-Appellant.
First Division, LeonardoDe Castro, J.:
Facts: On November 6, 2002, accused went inside the house of Felipe Lagera.
Accused suddenly hacked Felipe with the bolo he was carrying. He also hit Felipes 1
year old grandson, Ranil Tagpis, Jr., whom he was carrying at that time. Felipe
sustained 3 hack wounds, while Ranil sustained 1. Accused claimed that Felipe was
feeding chickens when he came to the latters store. They had an altercation when
accused confronted Felipe about his and his sons previous attempts to sexually assault
his wife. Felipe hurled the cover of the chicken cage at the accused, who then hacked
the former. Felipe went inside the house and accused followed. As the accused was
about to hack Felipe for the second time, the latter used Ranil as a shield and the
214

accused was no longer able to withdraw his bolo, thus hitting the child. Accused then
got even more mad and hacked Felipe once more. The latter was still able to go
outside, where the accused hacked him again.
Accused-appellant Rosendo Rebucan was charged with the crime of double murder for
the hacking and killing of Felipe Lagera and his grandson, Ranil Tagpis, Jr. RTC found
him guilty of the crime of double murder as charged in the Information.
CA modified the decision and adopted the position of the Office of the Solicitor General
(OSG) that the felonious acts of the accused-appellant resulted in two separate crimes
of murder as the evidence of the prosecution failed to prove the existence of a complex
crime of double murder. CA adjudged him guilty for two (2) counts of murder for the
deaths of Felipe Lagera and Ramil Tagpis, Jr., and sentenced him to suffer the penalty
of reclusion perpetua for each count of murder.
Issue: Whether the accused-appellant should be held guilty for the complex crime of
double murder
Held: No, accused-appellant should be held liable for 2 separate counts of murder
SC sustained the ruling of the RTC and CA that the circumstance of treachery qualified
the killings of Felipe and Ranil to murder. However, it found erroneous the appreciation
of the aggravating circumstances of evident premeditation, abuse of superior strength,
dwelling, minority and intoxication.
Article 48 of the Revised Penal Code provides that "[w]hen a single act constitutes two
or more grave or less grave felonies, or when an offense is a necessary means for
committing the other, the penalty for the most serious crime shall be imposed, the same
to be applied in its maximum period." There are, thus, two kinds of complex crimes. The
first is known as compound crime, or when a single act constitutes two or more grave or
less grave felonies. The second is known as complex crime proper, or when an offense
is a necessary means for committing the other.
SC found that there is a paucity of evidence to prove that the instant case falls under
any of the two classes of complex crimes. The evidence of the prosecution failed to
clearly and indubitably establish the fact that Felipe and Ranil were killed by a single
fatal hacking blow from the accused-appellant. The eyewitness testimony of Carmela
Tagpis did not contain any detail as to this material fact. To a greater degree, it was
neither proven that the murder of Felipe was committed as a necessary means for
committing and/or facilitating the murder of Ranil and vice versa. As the factual milieu of
the case at bar excludes the application of Article 48 of the Revised Penal Code, the
accused-appellant should be made liable for two separate and distinct acts of murder. In
the past, when two crimes have been improperly designated as a complex crime, SC
has affirmed the conviction of the accused for the component crimes separately instead
of the complex crime.

215

G.R. No. 178321, October 5, 2011


PEOPLE OF THE PHILIPPINES, Plaintiff-Appellee, vs. CONRADO LAOG y
RAMIN, Accused-Appellant.
First Division, LeonardoDe Castro, J.:
Facts: On June 6, 2000, AAA and her friend Jennifer Patawaran-Rosal were walking
along the along the rice paddies on their way to apply for work at a canteen near the
National Highway in Sampaloc, San Rafael, Bulacan. Suddenly, accused-appellant
Conrado Laog, who was holding an ice pick and a lead pipe, waylaid them and forcibly
brought them to a grassy area at the back of a concrete wall. Without warning, accused
struck AAA in the head with the lead pipe and she fell down. Jennifer cried out for help
but accused also hit her on the head with the lead pipe, knocking her down. Accused
stabbed Jennifer several times with the ice pick and thereafter covered her body with
thick grass. Accused then hit AAA in the head several times more with the lead pipe and
stabbed her on the face. Accused then raped AAA. Thereafter, he covered her with
grass and she passed out. When she regained consciousness, she was able to crawl to
her grandfathers farm and ask for help.
Accused was charged with murder for killing Jennifer. In a separate Information, he was
charged with the rape of AAA.
In a Joint Decision, RTC found the accused guilty of both crimes. CA affirmed with
modification, as to actual and exemplary damages.
Issue: Whether the lower court properly found him guilty of rape and murder
Held: NO. While accused indeed was the one who raped AAA and killed Jennifer, he
should not have been convicted of the separate crimes of murder and rape. The facts
alleged and proven clearly show that the crime committed by appellant is rape with
homicide, a special complex crime provided under Article 266-B, paragraph 5 of
the Revised Penal Code, as amended by Republic Act (R.A.) No. 8353.
In People v. Larraaga, this Court explained the concept of a special complex crime, as
follows:
A discussion on the nature of special complex crime is imperative. Where the law
provides a single penalty for two or more component offenses, the resulting crime is
called a special complex crime. Some of the special complex crimes under the Revised
Penal Code are (1) robbery with homicide, (2) robbery with rape, (3) kidnapping with
216

serious physical injuries, (4) kidnapping with murder or homicide, and (5) rape with
homicide. In a special complex crime, the prosecution must necessarily prove
each of the component offenses with the same precision that would be necessary
if they were made the subject of separate complaints. As earlier mentioned, R.A.
No. 7659 amended Article 267 of the Revised Penal Code by adding thereto this
provision: "When the victim is killed or dies as a consequence of the detention, or is
raped, or is subjected to torture or dehumanizing acts, the maximum penalty shall be
imposed;["] and that this provision gives rise to a special complex crime.
A special complex crime, or more properly, a composite crime, has its own definition
and special penalty in the Revised Penal Code, as amended. Justice Regalado, in his
Separate Opinion in the case of People v. Barros, explained that composite crimes are
"neither of the same legal basis as nor subject to the rules on complex crimes in Article
48 [of the Revised Penal Code], since they do not consist of a single act giving rise to
two or more grave or less grave felonies [compound crimes] nor do they involve an
offense being a necessary means to commit another [complex crime proper]. However,
just like the regular complex crimes and the present case of aggravated illegal
possession of firearms, only a single penalty is imposed for each of such composite
crimes although composed of two or more offenses."
Article 266-B of the Revised Penal Code, as amended, provides only a single penalty
for the composite acts of rape and the killing committed by reason or on the occasion of
the rape.
Considering that the prosecution in this case was able to prove both the rape of AAA
and the killing of Jennifer both perpetrated by appellant, he is liable for rape with
homicide under this provision. There is no doubt that appellant killed Jennifer to prevent
her from aiding AAA or calling for help once she is able to run away, and also to silence
her completely so she may not witness the rape of AAA, the original intent of appellant.
His carnal desire having been satiated, appellant purposely covered AAAs body with
grass, as he did earlier with Jennifers body, so that it may not be easily noticed or seen
by passersby. Appellant indeed thought that the savage blows he had inflicted on AAA
were enough to cause her death as with Jennifer. But AAA survived and appellants
barbaric deeds were soon enough discovered.
The facts established showed that the constitutive elements of rape with homicide were
consummated, and it is immaterial that the person killed in this case is someone other
than the woman victim of the rape.
In the special complex crime of rape with homicide, the term "homicide" is to be
understood in its generic sense, and includes murder and slight physical injuries
committed by reason or on occasion of the rape. Hence, even if any or all of the
circumstances (treachery, abuse of superior strength and evident premeditation) alleged
in the information have been duly established by the prosecution, the same would not
qualify the killing to murder and the crime committed by appellant is still rape with
homicide.
217

218

G.R. No. 153559, June 8, 2004


PEOPLE OF THE PHILIPPINES, appellee, vs. ANTONIO COMADRE, GEORGE
COMADRE and DANILO LOZANO, appellants.
EN BANC, PER CURIAM:

Facts: At around 7:00 in the evening of August 6, 1995, Robert Agbanlog, Jimmy Wabe,
Gerry Bullanday,3 Rey Camat and Lorenzo Eugenio were having a drinking spree on the
terrace of the house of Roberts father, Barangay Councilman Jaime Agbanlog, situated
in Barangay San Pedro, Lupao, Nueva Ecija. Jaime Agbanlog was seated on the
banister of the terrace listening to the conversation of the companions of his son. As the
drinking session went on, Robert and the others noticed appellants Antonio Comadre,
George Comadre and Danilo Lozano walking. The three stopped in front of the house.
While his companions looked on, Antonio suddenly lobbed an object which fell on the
roof of the terrace. Appellants immediately fled by scaling the fence of a nearby school.
The object, which turned out to be a hand grenade, exploded ripping a hole in the roof
of the house. Robert Agbanlog, Jimmy Wabe, Gerry Bullanday, Rey Camat and Lorenzo
Eugenio were hit by shrapnel and slumped unconscious on the floor. They were all
rushed to the San Jose General Hospital in Lupao, Nueva Ecija for medical treatment.
However, Robert Agbanlog died before reaching the hospital.

Appellants Antonio Comadre, George Comadre and Danilo Lozano were charged with
Murder with Multiple Frustrated Murder. The lower court convicted appellants of the
complex crime of Murder with Multiple Attempted Murder and sentenced them to suffer
the penalty of death.

Issue: Whether the lower correctly found them liable for the complex crime of Murder
with Multiple Attempted Murder

219

Held: Yes. SC found that the lower court erred in finding that there was conspiracy
between the 3 accused. When Antonio Comadre was in the act of throwing the hand
grenade, George Comadre and Danilo Lozano merely looked on without uttering a
single word of encouragement or performed any act to assist him. The trial court held
that the mere presence of George Comadre and Danilo Lozano provided
encouragement and a sense of security to Antonio Comadre, thus proving the existence
of conspiracy. Contrary to the lower courts assessment, George Comadre and Danilo
Lozano did not have any participation in the commission of the crime and must
therefore be set free. Their mere presence at the scene of the crime as well as their
close relationship with Antonio are insufficient to establish conspiracy considering that
they performed no positive act in furtherance of the crime. Neither was it proven that
their act of running away with Antonio was an act of giving moral assistance to his
criminal act.

The trial court found appellant guilty of the complex crime of murder with multiple
attempted murder under Article 48 of the Revised Penal Code.
The underlying philosophy of complex crimes in the Revised Penal Code, which follows
the pro reo principle, is intended to favor the accused by imposing a single penalty
irrespective of the crimes committed. The rationale being, that the accused who
commits two crimes with single criminal impulse demonstrates lesser perversity than
when the crimes are committed by different acts and several criminal resolutions.

The single act by appellant of detonating a hand grenade may quantitatively constitute a
cluster of several separate and distinct offenses, yet these component criminal offenses
should be considered only as a single crime in law on which a single penalty is imposed
because the offender was impelled by a "single criminal impulse" which shows his
lesser degree of perversity.

Under the aforecited article, when a single act constitutes two or more grave or less
grave felonies the penalty for the most serious crime shall be imposed, the same to be
applied in its maximum period irrespective of the presence of modifying circumstances,
including the generic aggravating circumstance of treachery in this case. Applying the
aforesaid provision of law, the maximum penalty for the most serious crime (murder) is
death. The trial court, therefore, correctly imposed the death penalty.
220

G.R. No. 146099, April 30, 2003


PEOPLE OF THE PHILIPPINES, appellee, vs. JIMMEL SANIDAD, PONCE MANUEL
alias PAMBONG, JOHN DOE (at large) and PETER DOE (at large),accused.
JIMMEL SANIDAD and PONCE MANUEL alias PAMBONG, appellants.
EN BANC, PER CURIAM:
Facts: On 16 January 1999, the victims and the accused-appellants were having a
drinking spree while attending a barangay fiesta in Lagangilang, Abra. Marlon Tugadi
and accused Jimmel Sanidad were drinking buddies and members of the CAFGU
before then.
On 17 January 1999 at about four o'clock in the morning Jimmel Sanidad and his
companions finished drinking and left. Shortly after, the group of Marlon Tugadi also
stopped drinking and headed home for Budac, Tagum, Abra, boarding the same
jeepney driven by Delfin Tadeo. As the jeepney approached a plantation, its headlights
beamed at accused-appellants Jimmel Sanidad, Ponce Manuel and two (2) other
unidentified companions who were armed with an armalite, a .45 caliber pistol and
shotguns with buckshots.
As the jeepney moved closer, the accused suddenly and without warning unleashed a
volley of shots at the jeepney. Delfin stepped on the gas in a vain effort to elude their
assailants, but they continued firing at the hapless victims. Bullets plowed the side of
the vehicle and all the passengers sitting at the back instinctively ducked on the floor to
avoid being hit. The accused pursued the vehicle on foot and fired at it incessantly until
it finally stalled a few meters away.
The jeepney was left in shambles. Miraculously, almost all of its passengers, with the
exception of Rolando Tugadi, survived the ambush and suffered only minor injuries.
Marlon Tugadi tried to pull his brother Rolando Tugadi from the vehicle to safety only to
realize that he was not only too heavy, he was already dead. As the pursuing gunmen
drew near, Marlon decided to abandon Rolando and scampered away with the other
victims until they reached a bushy area about fifteen (15) meters away from the vehicle.
Meanwhile, the accused caught up with the jeepney. Moments later, fire engulfed it.
Marlon Tugadi and Pepito Tugadi later heard one of the unidentified companions of
accused-appellant Sanidad say to him: "My gosh, we were not able to kill all of
them." Thereafter, the accused left the scene, firing their guns indiscriminately into the
air as they walked away.
An Information for murder with multiple attempted murder and malicious mischief was
filed against Jimmel Sanidad, Ponce Manuel alias Pambong, John Doe and Peter Doe.
The defense of the accused rested on bare denial and alibi.
221

The trial court convicted them of the complex crime of murder and multiple attempted
murder, and sentenced them to death.
Issue: Whether the lower court correctly held them liable for the complex crime of
murder with multiple attempted murder
Held: Yes. Accused-appellants had murder in their hearts when they waylaid their
unwary victims. They must consequently be held liable for their acts. Insofar as victims
Marlon Tugadi, Jun Quipay, Raymund Fontanilla, Pepito Tugadi, Delfin Tadeo, Ricardo
Tadeo, Edwin Tumalip, Bobby Velasquez and Dennis Balueg are concerned, although
they barely escaped the ambush with superficial injuries does not alter the nature of
accused-appellants' participation in the crime of murder except that not one of them
having suffered fatal injuries which could have resulted in their death, accusedappellants should only be held guilty of attempted murder. Accused-appellants had
commenced their criminal scheme to liquidate all the victims directly by overt acts, but
were unable to perform all the acts of execution that would have brought about their
death by reason of some cause other than their own spontaneous desistance, that is,
the victims successfully dodged the hail of gunfire and escaped.
The case comes within the purview of Art. 48 of The Revised Penal Code which,
speaking of complex crimes, provides that when "a single act constitutes two or more
grave or less grave felonies, or when an offense is a necessary means for committing
the other, the penalty for the most serious crime shall be imposed in its maximum
period." In a complex crime, although two or more crimes are actually committed, they
constitute only one crime in the eyes of the law as well as in the conscience of the
offender.
Although several independent acts were performed by the accused in firing separate
shots from their individual firearms, it was not possible to determine who among them
actually killed victim Rolando Tugadi. Moreover, there is no evidence that accusedappellants intended to fire at each and every one of the victims separately and distinctly
from each other. On the contrary, the evidence clearly shows a single criminal impulse
to kill Marlon Tugadi's group as a whole. Thus, one of accused-appellants exclaimed in
frustration after the ambush: "My gosh, we were not able to kill all of them." Where a
conspiracy animates several persons with a single purpose, their individual acts done in
pursuance of that purpose are looked upon as a single act, the act of execution, giving
rise to a single complex offense.
G.R. No. 141125, February 28, 2002
PEOPLE OF THE PHILIPPINES, plaintiff-appellee, vs. JEFFREY GARCIA y
CARAGAY and THREE JOHN DOES, accused. JEFFREY GARCIA y
CARAGAY, accused-appellant.
EN BANC, PER CURIAM:

222

Facts: The victim, Cleopatra Changlapon, was nineteen years old and a sophomore
student of B.S. Physical Therapy at the Baguio Central University. On July 14, 1998,
while crossing a street on her way home from school, a white van stopped in front of
her. The rear door slid open and Cleopatra was pulled by the arms into the van.
Something was sprayed on her face which made her eyes sting and feel dizzy. She
shouted, then she felt a fist blow on her stomach and she fell unconscious.
She woke up inside a room, where she was raped, boxed, and burned with cigarettes
by 4 men, including accused-apellant Jeffrey Garcia. One of the men again sprayed
something on Cleopatras face which made her vision blurred. She heard somebody say
that it was 1:30. After that, she blacked out. When she regained consciousness, she
was lying by the roadside somewhere between Tam-awan and Longlong. It was still
dark. She already had her clothes on. She felt pain all over her body and was unable to
move. A taxi passed by and picked her up. Although she was afraid to ride the taxi, she
boarded it just to get home. The taxi brought her to her house.
On July 17, 1998, Cleopatra went back to the police station and gave a description of
the four rapists to the cartographer. She likewise executed another sworn statement to
the police.
Meanwhile, accused-appellant was arrested at 4:30 p.m. of July 17, 1998 in connection
with another rape charge against him filed by a certain Gilda Mangyo.
The cartographic sketches were published in the Sun-Star newspaper. Police Officers
Gilbert Bulalit and Archibald Diaz saw the sketches and noticed that one of the suspects
depicted in the cartographic sketch bore a striking resemblance to accused-appellant,
who was in their custody. On July 26, 1998, Cleopatra was summoned to the police
station to identify accused-appellant. She was brought to the upper floor of the police
building and asked to look below on the basketball court of the city jail and see if any of
the inmates looked familiar to her. Cleopatra recognized accused-appellant among
those watching the basketball game.
On July 27, 1998, formal charges for forcible abduction with rape were brought against
accused-appellant and three John Does. The trial court rendered its decision convicting
accused-appellant of one count of forcible abduction with rape and three counts of rape.
Issue: Whether the lower court correctly held him liable for the complex crime of forcible
abduction with rape and three counts of rape
Held: Yes. The trial court did not err in convicting accused-appellant of the complex
crime of forcible abduction with rape. The two elements of forcible abduction, as defined
in Article 342 of the Revised Penal Code, are: (1) the taking of a woman against her will
and (2) with lewd designs. The crime of forcible abduction with rape is a complex crime
that occurs when there is carnal knowledge with the abducted woman under the
following circumstances: (1) by using force or intimidation; (2) when the woman is
223

deprived of reason or otherwise unconscious; and (3) when the woman is under twelve
years of age or is demented.
In the case at bar, the information sufficiently alleged the elements of forcible
abduction, i.e., the taking of complainant against her against her will and with lewd
design. It was likewise alleged that accused-appellant and his three co-accused
conspired, confederated and mutually aided one another in having carnal knowledge of
complainant by means of force and intimidation and against her will.
Aside from alleging the necessary elements of the crimes, the prosecution convincingly
established that the carnal knowledge was committed through force and intimidation.
Moreover, the prosecution sufficiently proved beyond reasonable doubt that accusedappellant succeeded in forcibly abducting the complainant with lewd designs,
established by the actual rape.
Hence, accused-appellant is guilty of the complex crime of forcible abduction with rape.
He should also be held liable for the other three counts of rape committed by his three
co-accused, considering the clear conspiracy among them shown by their obvious
concerted efforts to perpetrate, one after the other, the crime. As borne by the records,
all the four accused helped one another in consummating the rape of complainant.
While one of them mounted her, the other three held her arms and legs. They also
burned her face and extremities with lighted cigarettes to stop her from warding off her
aggressor. Each of them, therefore, is responsible not only for the rape committed
personally by him but for the rape committed by the others as well.
However, as correctly held by the trial court, there can only be one complex crime of
forcible abduction with rape. The crime of forcible abduction was only necessary for the
first rape. Thus, the subsequent acts of rape can no longer be considered as separate
complex crimes of forcible abduction with rape. They should be detached from and
considered independently of the forcible abduction. Therefore, accused-appellant
should be convicted of one complex crime of forcible abduction with rape and three
separate acts of rape.
The penalty for complex crimes is the penalty for the most serious crime which shall be
imposed in its maximum period. Rape is the more serious of the two crimes and, when
committed by more than two persons, is punishable with reclusion perpetua to death
under Article 266-B of the Revised Penal Code, as amended by Republic Act No. 8353.
Thus, accused-appellant should be sentenced to the maximum penalty of death for
forcible abduction with rape.
G.R. No. L-26222, July 21, 1967
THE PEOPLE OF THE PHILIPPINES, petitioner, vs. HONORABLE JUDGE
HERNANDO PINEDA of the Court of First Instance of Lanao del Norte; and
TOMAS NARBASA, TAMBAC ALINDO and RUFINO BORRES, respondents.
EN BANC, SANCHEZ, J.:
224

Facts: On the night of July 29, 1965, the occupants of the home of the spouses Teofilo
Mendoza and Valeriana Bontilao de Mendoza in Pugaan City of Iligan, were asleep. It
was then that guns (rifle, caliber 22) and paliuntod (homemade gun) were fired in rapid
succession from outside the house. Teofilo Mendoza fell dead. Thereafter, defendants
below destroyed the door of the house, entered therein, and let loose several shots
killing Neceforo Mendoza, all minor children of the couple and wounding Valeriana
Bontilao de Mendoza.
Respondents/ Accused Tomas Narbasa, Tambac Alindo and Rufino Borres were
indicted before the Court of First Instance of Lanao del Norte, as principals, in five (5)
separate cases, four for murder and one for frustrated murder.
Accused Tomas Narbasa and Tambak Alindo moved for a consolidation into one (1)
criminal case, considering that said cases arose out of the same incident and motivated
by one impulse.
Respondent Judge, in an order dated May 13, 1966, directed the City Fiscal to unify all
the five criminal cases, and to file one single information in Case 1246. He also ordered
that the other four cases, Nos. 1247, 1248, 1249 and 1250 be dropped from the docket.
The City Fiscal sought reconsideration thereof, upon the ground that "more than one
gun was used, more than one shot was fired and more than one victim was killed."
Respondent Judge denied the motion to reconsider. He took the position that the acts
complained of "stemmed out of a series of continuing acts on the part of the accused,
not by different and separate sets of shots, moved by one impulse and should therefore
be treated as one crime though the series of shots killed more than one victim;" and that
only one information for multiple murder should be filed, to obviate the necessity of
trying five cases instead of one."
Primarily to annul respondent Judge's orders as having been issued without or in
excess of jurisdiction and/or with grave abuse of discretion, the People came to the
Supreme Court on certiorari with a prayer for a writ of preliminary injunction, and for
other reliefs.
Issue: Whether only one information, either for the complex crime of murder and
frustrated murder or for the complex crime of robbery with multiple homicide and
frustrated homicide should be filed.
Held: NO. The case calls into question the applicability of Article 48 of the Revised
Penal Code, as amended. Article 48 provides for two classes of crimes where a single
penalty is to be imposed: first, where a single act constitutes two or more grave or less
grave felonies (delito compuesto); and, second, when an offense is a necessary means
for committing the other (delito complejo).
Best exemplified by the first of the two cases is where one shot from a gun results in the
death of two or more persons. Jurisprudence teaches that, in this factual setting, the
complex crime defined in the first part of Article 48 finds application. A similar rule
225

obtains where one stabbed another and the weapon pierced the latter's body through
and wounded another. The first died instantaneously; the second, seven days later. This
Court convicted the assailant of double murder. So where a person plants a bomb in an
airplane and the bomb explodes, with the result that a number of persons are killed, that
single act again produces a complex crime.
A different rule governs where separate and distinct acts result in a number killed.
Deeply rooted is the doctrine that when various victims expire from separate shots, such
acts constitute separate and distinct crimes. Thus, where the six defendants, with others
(armed with pistols, carbines and also a submachine gun and Garand rifles), fired
volleys into a house killing eleven and wounding several others, each of the said
accused is "guilty of as many crimes of murder as there were deaths (eleven). Again,
eleven persons were indicted for quadruple murder with the use of bolos, a pistol, a
barbed arrow and a piece of bamboo of a man, his common-law wife, and their two
children in cold blood. The accused were found guilty by the trial court of such offense.
SC, in reversing this ruling below, held that "[t]he four victims were not killed by a single
act but by various acts committed on different occasions and by different parties"; that
such acts "may not be regarded as constituting one single crime"; and that "[t]hey
should be held as separate and distinct crimes. And a third. At the commencement
exercises of an elementary school, "a shot suddenly rang out" followed by a "series of
shots" from a pistol. Two persons lay dead and a third seriously wounded but who
later on also died. SC ruled that there were "three distinct and separate murders"
committed by appellant Juan Mones. And finally, in People vs. Gatbunton, the spouses
Mariano Sebastian and Maxima Capule who were asleep were killed by one burst
of machinegun fire; and then, by a second burst of machinegun fire, two of the couple's
children also asleep were killed. The accused, Tomas Gatbunton, was found guilty
by the trial court of quadruple murder. On appeal, SC declared that "appellant must be
declared guilty of four murders."
The present case is to be differentiated from People vs. Lawas. There, on a single
occasion, about fifty Maranaos were killed by a group of home guards. It was held that
there was only one complex crime. In that case, however, there was no conspiracy to
perpetuate the killing. In the case at bar, defendants performed several acts. And the
informations charge conspiracy amongst them. Needless to state, the act of one is the
act of all. Not material here, therefore is the finding in Lawas that "it is impossible to
ascertain the individual deaths caused by each and everyone" of the accused. It is to be
borne in mind, at this point, that to apply the first half of Article 48, heretofore quoted,
there must be singularity of criminal act; singularity of criminal impulse is not written into
the law.
Application of penalties:: Delito continuado

G.R. No. 109266, December 2, 1993


226

MIRIAM DEFENSOR SANTIAGO, petitioner, vs. HON. JUSTICE FRANCIS


GARCHITORENA, SANDIGANBAYAN (First Division) and PEOPLE OF THE
PHILIPPINES, respondents.
EN BANC, QUIASON, J.:

Facts: Petitioner was charged in Criminal Case No. 16698 of the Sandiganbayan with
violation of Section 3(e) of R.A. No. 3019, as amended, otherwise known as the AntiGraft and Corrupt Practices Act, allegedly committed by her favoring "unqualified" aliens
with the benefits of the Alien Legalization Program.

Petitioner filed with SC a petition for certiorari and prohibition to enjoin the
Sandiganbayan from proceeding with Criminal Case No. 16698 on the ground that said
case was intended solely to harass her as she was then a presidential candidate. She
alleged that this was in violation of Section 10, Article IX-C of the Constitution which
provides that "(b)ona fide candidates for any public office shall be free from any form of
harassment and discrimination." The petition was dismissed.

She then filed a motion for inhibition of Presiding Justice Garchitorena.


The Sandiganbayan (First Division), of which Presiding Justice Garchitorena is a
member, set the criminal case for arraignment. Petitioner moved to defer the
arraignment on the grounds that there was a pending motion for inhibition, and that
petitioner intended to file a motion for a bill of particulars. The Sandiganbayan (First
Division) denied the motion to defer the arraignment.

Petitioner filed a motion for a bill of particulars, stating that while the information alleged
that petitioner had approved the application or legalization of "aliens" and gave them
indirect benefits and advantages, it lacked a list of the favored aliens. According to
petitioner, unless she was furnished with the names and identities of the aliens, she
could not properly plead and prepare for trial.
227

Upon motion of petitioner in G.R. No. 107598 (Miriam Defensor Santiago v.


Sandiganbayan, et al.), SC directed the Sandiganbayan (First Division) to reset the
arraignment to a later date and to dispose of the two incidents pending before it (Re:
disqualification of Presiding Justice Garchitorena and the motion for the bill of
particulars).

At the hearing on November 13, 1992 on the motion for a bill of particulars, the
prosecution stated categorically that they would file only one amended information
against petitioner. However, on December 8, 1992, the prosecution filed a motion to
admit the 32 Amended Informations (Criminal Cases Nos. 18371 to 18402).

The Sandiganbayan (First Division) promulgated a resolution, admitting the 32


Amended Informations and ordering petitioner to post the corresponding bail bonds
within ten days from notice. Petitioner's arraignment on the 32 Amended Informations
was set for April 12, 1993 at 8:00 A.M.

Hence, petitioner filed this petition for certiorari under Rule 65 of the Revised Rules of
Court to set aside: (a) the Resolution dated March 3, 1993 in Criminal Case No. 16698
of the Sandiganbayan (First Division) and to declare Presiding Justice Francis
Garchitorena of the Sandiganbayan, disqualified from acting in said criminal case; and
(b) the Resolution of said court promulgated on March 14, 1993, which deemed as
"filed" the 32 Amended Informations against petitioner.

Issue: Whether the filing of 32 Amended Infromations against the petitioner was proper

Held: No.Technically, there was only one crime that was committed in petitioner's case,
and hence, there should only be one information to be file against her.

228

The 32 Amended Informations charge what is known as delito continuado or "continued


crime" and sometimes referred to as "continuous crime."

According to Cuello Calon, for delito continuado to exist there should be a plurality of
acts performed during a period of time; unity of penal provision violated; and unity of
criminal intent or purpose, which means that two or more violations of the same penal
provisions are united in one and same instant or resolution leading to the perpetration of
the same criminal purpose or aim.

According to Guevarra, in appearance, a delito continuado consists of several crimes


but in reality there is only one crime in the mind of the perpetrator.

Padilla views such offense as consisting of a series of acts arising from one criminal
intent or resolution.

Applying the concept of delito continuado, SC treated as constituting only one offense
the following cases:

(1) The theft of 13 cows belonging to two different owners committed by the accused at
the same time and at the same period of time
(2) The theft of six roosters belonging to two different owners from the same coop and
at the same period of time
(3) The theft of two roosters in the same place and on the same occasion
(4) The illegal charging of fees for services rendered by a lawyer every time he collects
veteran's benefits on behalf of a client, who agreed that the attorney's fees shall be paid
out of said benefits. The collection of the legal fees were impelled by the same motive,
that of collecting fees for services rendered, and all acts of collection were made under
the same criminal impulse.
229

On the other hand, SC declined to apply the concept to the following cases:
(1) Two estafa cases, one of which was committed during the period from January 19 to
December 1955 and the other from January 1956 to July 1956. The said acts were
committed on two different occasions.
(2) Several malversations committed in May, June and July, 1936, and falsifications to
conceal said offenses committed in August and October 1936. The malversations and
falsifications "were not the result of only one purpose or of only one resolution to
embezzle and falsify . . ."
(3) Two estafa cases, one committed in December 1963 involving the failure of the
collector to turn over the installments for a radio and the other in June 1964 involving
the pocketing of the installments for a sewing machine.
(4) 75 estafa cases committed by the conversion by the agent of collections from
customers of the employer made on different dates.

In the case at bench, the original information charged petitioner with performing a single
criminal act that of her approving the application for legalization of aliens not qualified
under the law to enjoy such privilege.

The original information also averred that the criminal act : (i) committed by petitioner
was in violation of a law Executive Order No. 324 dated April 13, 1988, (ii) caused an
undue injury to one offended party, the Government, and (iii) was done on a single
day, i.e., on or about October 17, 1988.

The 32 Amended Informations reproduced verbatim the allegation of the original


information, except that instead of the word "aliens" in the original information each
amended information states the name of the individual whose stay was legalized.

230

At the hearing of the motion for a bill of particulars, the public prosecutors manifested
that they would file only one amended information embodying the legalization of stay of
the 32 aliens.

The 32 Amended Informations aver that the offenses were committed on the same
period of time, i.e., on or about October 17, 1988. The strong probability even exists that
the approval of the application or the legalization of the stay of the 32 aliens was done
by a single stroke of the pen, as when the approval was embodied in the same
document.

Likewise, the public prosecutors manifested at the hearing the motion for a bill of
particulars that the Government suffered a single harm or injury.

G.R. Nos. 169727-28, August 18, 2006


BRIG. GEN. (Ret.) JOSE S. RAMISCAL, JR., Petitioner, vs. SANDIGANBAYAN
(4th Division) and PEOPLE OF THE PHILIPPINES, Respondents.
First Division, Callejo, Sr., J.:
Facts: In 1998, the Senate Committees on Accountability of Public Officers and
Investigation (Blue Ribbon) and on National Defense and Security (collectively, Senate
Blue Ribbon Committee) carried out an extensive joint inquiry into the coup rumors and
the alleged anomalies in the Armed Forces of the Philippines-Philippine Retirement
Benefits Systems (AFP-RSBS). In its Report, the Senate Blue Ribbon Committee
outlined, among others, the anomalies in the acquisition of lots in Tanauan, Batangas,
Calamba, Laguna and Iloilo City by the AFP-RSBS. According to the report, the modus
operandi involve the signing to two deeds of sale covering the same lots. One deed of
sale would be signed only by the seller or sellers (unilateral deed). Another deed of sale
would be signed by the seller or seller and the buyer, AFP-RSBS (bilateral deed). The
unilateral deeds were the one registered with the register of deeds and recorded
lower consideration paid by the System to the buyers than those stated in the
Bilateral Deeds.
Pursuant to the recommendation of the Senate Blue Ribbon Committee to prosecute
and/or cause the prosecution of Gen. Jose Ramiscal Jr. (Ret), past AFP-RSBS
President, who had signed the unregistered deeds of sale covering the acquisition of
certain parcels of land, Ombudsman Investigators of the Office of the Deputy
231

Ombudsman for the Military conducted a fact-finding investigation. They executed a


Joint Affidavit-Complaint, stating that based on their findings, the following may be
charged with falsification of public documents and violation of Section 3(e) and (g) of
Republic Act (R.A.) No. 3019: petitioner B/Gen. Jose Ramiscal, Jr., former AFP-RSBS
president; Atty. Meinrado Enrique Bello, Head of the AFP-RSBS Legal Department in
charge of Land Acquisition; Capt. Perfecto Enrique Quilicot, AFP-RSBS Project Officer,
Tanauan, Batangas, Land Acquisition; and Notaries Public Alfredo Nasser and Manuel
Satuito.
A panel of Ombudsman Investigators issued on March 30, 2001 a Joint Resolution
finding probable cause to file the corresponding Informations for 148 counts of violation
of Article 315, in relation to Article 171, paragraph 4 of the Revised Penal Code, and
Section 3 (e) of R.A. No. 3019 against Meinrado Bello and Atty. Manuel Satuito.
However, it recommended that the complaint against petitioner be dismissed, without
prejudice to a thorough fact-finding investigation on his liability in light of this Courts
ruling in Arias v. Sandiganbayan.
The Ombudsman did not act on this recommendation. Instead, another panel of
prosecutors was directed to review the Joint Resolution and conduct a thorough
investigation of the case. After conducting clarificatory hearings, the investigating panel
issued a Memorandum, recommending to the Ombudsman that petitioner be charged
with 148 counts of estafa through falsification of public documents, and one count
violation of Section 3(e) of R.A. No. 3019.
The Ombudsman approved the recommendation of the Panel of Prosecutors without
prejudice to the liability of the landowners involved in the transactions.
Petitioner and his co-accused filed their respective Motions for Reconsideration of the
investigating
panels Memorandum. The
Panel
of
Prosecutors
issued
a
Memorandum to the Ombudsman recommending that the motion be denied, which the
latter duly approved.
Thereafter, the panel of Prosecutors and the Special Prosecutors had a series of
meetings with the Ombudsman, where it was agreed upon that only five Informations for
estafa through falsification of public documents and five Informations for violation of
Section 3(e) of R.A. No. 3019 would be initially filed with the Sandiganbayan instead of
the 148 counts previously recommended by the Ombudsman. This was due to the lack
of prosecutors who would handle the voluminous cases.
Of the Informations filed, two were raffled to the Fourth Division of the Sandiganbayan,
one of which was docketed as Criminal Case No. 28022 for violation of Section 3(e) of
R.A. No. 3019. The other, for estafa thru falsification of public documents, was docketed
as Criminal Case No. 28023. Raffled to the First Division of the anti-graft court were two
other cases docketed as Criminal Case No. 28026 for violation of Section 3(e) of R.A.
3019, and Criminal Case No. 28027 for estafa through falsification of public documents.
Criminal Case No. 28028 for violation of Section 3(e), R.A. No. 3019 and Criminal Case
232

No. 28029 for estafa through falsification of public documents were raffled to the
Second Division, while Criminal Case No. 28021 for estafa through falsification of public
documents was raffled to the Third Division. Criminal Case No. 28024 for violation of
Section 3(e) of R.A. No. 3019 and Criminal Case No. 28025 for estafa through
falsification of public documents were raffled to the Fifth Division.
Petitioner filed in the Fourth Division of the anti-graft court (in Criminal Case Nos. 28022
and 28023) an Urgent Motion for Hearing to Determine Probable Cause and
Consolidate All Cases in One Information with Prayer to Defer Issuance of An Arrest
Warrant Pending Resolution Hereof. The Sandiganbayan denied the motion, holding
that the judicial determination of probable cause is not an adversarial proceeding but
summary in nature. While it ordered the issuance of warrants of arrest against the
accused, it resolved to hold in abeyance the resolution on the matter of consolidation of
all the cases until after it had acquired jurisdiction over their persons. After petitioner
posted bail for his provisional release, the Sandiganbayan denied the motion for the
consolidation of the cases, considering that the other cases filed were pending in its
other divisions.
Petitioner filed a motion for reconsideration of the resolution and sought to have the
cases dismissed for lack of probable cause. The Sandiganbayan again denied the
motion.
Petitioner filed a Motion to Quash in Criminal Cases Nos. 28022 and 28023, which the
Sandiganbayan denied. When arraigned, petitioner refused to enter a plea, prompting
the anti-graft court to enter a plea of not guilty in both cases.
Thus, petitioner filed the instant petition for certiorari under Rule 65.
Petitioner asserts that the charges filed against him constitute only one crime of estafa
through falsification of public document, in the nature of delito continuado, or a series of
repetition of the same acts arising from one and the same criminal intent. He maintains
that while there are 148 bilateral deeds of sale signed by him and 145 unilateral deeds
of sale signed by the sellers, it cannot thereby be concluded that he is criminally liable
for each deed executed. The number of transactions purportedly entered into is not a
gauge in ascertaining criminal intent for the several transactions. The best test should
be the presence of clear, convincing and positive evidence showing distinct criminal
intent for each sales transaction, which in any event, is wanting in this case. Petitioner
further alleges that for multiple transactions to be considered as separate and distinct
crimes, there must be a series of acts with individual sellers such as (a) negotiations; (b)
discussion of the terms of the sale; (c) finalizing the terms thereof; and (d) instruction to
prepare payment and (e) actual payment. He points out that there is no
evidence that he and the other accused involved ever met with any of the sellers. While
he admits the possibility that he could have signed the bilateral deeds of sale in one
sitting, he insists that these documents were notarized separately; there is even no
evidence on record that the sellers of the property transacted separately with him. He
points out that the corporate officers of AFP-RSBS, especially its President, do not
233

personally deal with any of the sellers. The bare fact that he executed the bilateral
deeds of sale and that the project was approved by the higher level of the management,
cannot lead to the conclusion that he took part in the implementation of the transactions.
Issue: Whether petitioner may be charged and prosecuted for five (5) counts of estafa
thru falsification of public documents
Held: Yes. On petitioners claim that he should be charged with only one count of estafa
through falsification of public document instead of five (5) charges, respondents counter
that the criminal acts petitioner and his co-accused are not continuous crimes.
Respondents argue that a continuous crime may exist only if there is only a single
criminal intent and the commission of diverse acts is merely a partial execution of said
single criminal resolution. In the instant cases, the requirement of singularity of criminal
intent does not exist because there are as many criminal intents as there are
anomalous transactions, causing grave damage to the government at each instance.
There was no need for the accused to perform another or other delictual acts to
consummate the felony. Respondents maintain that petitioner was motivated by
separate intents as he signed each document, all of which are criminal in character;
hence, it is but proper that corresponding Informations be filed against him for each and
every act of falsification committed.
The Sandiganbayan, for its part, sustained the contention of respondents and ruled that
the determination of (a) the charge/s and the person/s against whom the charge is filed
are addressed to the sound discretion of the Prosecutors based on the facts before
them; and (b) the crimes committed by petitioner are separate, and not a single crime
consisting of series of acts arising from a single criminal resolution. It stated that it is not
persuaded that what is involved in these cases is a continuous crime, that is to say, a
single crime consisting of a series of acts arising from a single criminal resolution or
intent not susceptible of division, with each act in that series being merely the partial
execution of a single delict. On the contrary, it is of the view that what is involved herein
are several completed and distinct purported criminal acts which should be prosecuted
as multiple counts of the same type of offense. Thus, there are as many alleged
offenses as there are alleged anomalous transactions involved in these cases.
SC held however, that the determination of what charges to file and who are to be
charged are matters addressed to the discretion of the Ombudsman, including the
matter of whether the crime perpetrated by petitioner and his co-accused under the
Informations pending in the Divisions of the Sandiganbayan constitute delito
continuado or classified as concurso de delitos; or involve separate crimes under the
category of concurso real delito involve factual issues. Such factual issues should be
resolved after trial on the merits, and not in this case.
Effects of the attendance of mitigating and aggravating circumstances, habitual
delinquency
234

G.R. No. L-43556


December 18, 1935
THE PEOPLE OF THE PHILIPPINE ISLANDS, plaintiff-appellee, vs. HONORATO
ESPINA Y REAL, defendant-appellant.
EN BANC, AVANCEA, C.J.:
Facts: The appellant was charged in the lower court with the crime of theft of articles
valued at P585.15 and, having pleaded guilty, was sentenced to six months and one
day of prision correccional and, being a habitual delinquent, to an additional penalty of
two years, four months and one day of prision correccional.
Issue: Whether the penalty imposed by the lower court is correct
Held: No. The principal penalty imposed by the court is not correct. The amount stolen
is more than P200 but does not exceed P6,000 and, under article 309, subsection 3, of
the Revised Penal Code, the penalty to be imposed should be prision correccional in its
minimum and medium periods. Being a recidivist and having pleaded guilty, both
circumstances should compensate each other and the penalty should he imposed in its
medium period, that is, one year, eight months and twenty-one days. As the appellant is
a habitual delinquent, this being his third conviction, the additional penalty of two years,
four months and one day of prision correccional should also be imposed upon him.
The question whether, in imposing the additional penalty on the appellant as habitual
delinquent, recidivism, as an aggravating circumstance inherent of habitual delinquency,
should still be taken into consideration in filing the principal penalty, has already been
expressly decided in the affirmative in People vs. Melendrez (59 Phil., 154).
There is no doubt that the purpose of the law in imposing additional penalty on a
habitual delinquent is to punish him more severely. However, the result would be
otherwise if, for imposing the additional penalty, recidivism could not be considered as
aggravating circumstance in fixing the principal penalty.
G.R. No. 45198, October 31, 1936
THE PEOPLE OF THE PHILIPPINE ISLANDS, plaintiff-appellee, vs. BASILIO DE
JESUS Y JAVIER, defendant-appellant.
EN BANC, DIAZ, J.:
Facts: Basilio de Jesus y Javier was convicted by the Court of First Instance of Manila
in criminal case No. 52270 of the theft of an umbrella and a buri hat valued at P2.65 on
April 28, 1936. He was sentenced to one month and one day of arresto mayor with the
accessory penalties, to indemnify Francisco Liwanag in the sum of P2.50 representing
the value of the umbrella which was not recovered, and being a habitual delinquent, the
additional penalty of two years, four months and one day of prision correccional with the
corresponding accessory penalties was also imposed upon him in conformity with the
provisions of subsection 5, paragraph (a), of article 62 of the Revised Penal Code.
235

Held: Due to the amount involved, the theft imputed to the appellant is punishable with
arresto mayor in its minimum and medium periods the duration of which is from one
month and one day to four months (art. 309, subsection 6, Revised Penal Code); and
the minimum period of said penalty is from one month and one day to two months. It
appears therefrom that the penalty questioned by the appellant is the minimum period,
as no other less penalty could have been imposed upon him because said penalty in
itself already constitutes the minimum limit under the law. The reasons which prompted
the lower court to be lenient with the appellant were undoubtedly his voluntary
confession before the prosecution presented its evidence, which constitutes a mitigating
circumstance (art. 13, subsection 7, Revised Penal Code), and the apparent absence of
all allegation in the information of some aggravating circumstance that may compensate
said mitigating circumstance (art. 63, rule 1,Revised Penal Code).
The imposition of the additional penalty of two years, four months and one day upon the
appellant is justified by his own admission of guilt because the rule is well settled in this
jurisdiction that when one pleads guilty of the crime imputed to him in an information, it
is understood that he admits all the material facts alleged therein, not excluding those
alleging his former convictions of other crimes; and in the information filed against the
appellant, it was alleged that the accused is a habitual delinquent within the purview of
rule 5 of article 62 of the Revised Penal Code.
The Solicitor-General, taking advantage of the allegation in the information that the
appellant is a habitual delinquent, recommends that instead of affirming his principal
penalty of one month and one day of arresto mayor, it be increased to the minimum of
the medium period of that prescribed by law for his crime, or two months and one day
of arresto mayor, considering the aggravating circumstance of recidivism established
but compensated by the mitigating circumstance of voluntary confession. His argument
appears to be as follows: Habitual delinquency necessarily implies recidivism or former
conviction, at least more than once. The appellant having admitted in his confession
that he is a habitual delinquent for having committed theft for the third time within the
period prescribed by law, he must necessarily be considered a recidivist. This naturally
raises the question whether or not in this case the circumstance of recidivism can be
and must be twice taken into consideration against the appellant, first as an aggravating
circumstance although compensated by another mitigating circumstance, and second
as a qualifying circumstance or one inherent, as the case may be, in habitual
delinquency. If such an opinion were sustained, would not an injustice be committed
against the appellant by imposing two penalties upon him, the principal and the
additional, in a period which must be determined by taking into consideration one and
the same fact or circumstance, which is recidivism? There is no express provision of law
prohibiting it.
On the contrary, as to the principal penalty, there is the rule that in cases in which the
penalty prescribed by law contains three periods, the courts must take into
consideration, in the application of said penalty, the aggravating mitigating
circumstances established at the trial if they do not appear to be compensated by other
circumstances; and there is also the rule that when only an aggravating circumstance is
present the former, that is the principal penalty, must be imposed in its maximum period
236

(art. 64, Revised Penal Code); and in People vs. Aguinaldo (47 Phil., 728), this court
has stated, and it is reiterated in People vs. Melendrez (59 Phil., 154), that the
aggravating circumstance of recidivism, even in cases of habitual delinquency, should
be taken into consideration in the application of the principal penalty in the
corresponding period.
As to the additional penalty, if we must rely upon the spirit and letter of the law, we
would say that the purpose of the latter in establishing it was to prevent those who for
the second time or more commit the crimes enumerated in the last paragraph of article
62 of the Revised Penal Code from relapsing thereafter at least during the period fixed
thereby, as if to tell them: "If you relapse, the penalty corresponding to your last offense
will be imposed upon you plus another additional penalty ranging from prision
correccional in its medium and maximum periods to prision mayor in its maximum
period and reclusion temporal in its minimum period, according to your recidivism, that
is, the third, fourth, fifth or more times."
When the law has prescribed the additional penalty for habitual delinquency in a
manner susceptible of division into periods and has enumerated it among the penalties
that may be imposed by incorporating it into the Revised Penal Code, it was for no other
purpose than that all the circumstances present in every case be taken into
consideration in order to avoid arbitrariness in the determination of the period in which
said penalty should be imposed. It would be arbitrary, in the absence of any
circumstance, to impose the maximum of the additional penalty upon a habitual
delinquent, as it would also be arbitrary to impose the minimum thereof upon him when
there are circumstances justifying its application in the maximum period.
We should not lose sight of the fact that when the Legislature incorporated the provision
relative to habitual delinquency into the Revised Penal Code, it was aware this, at
least, is the presumption of law that recidivism was, as it continues to be in the
majority of cases to this date, an aggravating circumstance the effect of which, as the
name itself implies, is to aggravate the criminal responsibility of the delinquent. But
unlike other circumstances, as treachery, evident premeditation, sex, craft, relationship,
public position, dwelling, not to mention several others so as not to be tedious, which
may be aggravating, qualifying and inherent as the case may be, recidivism is and can
be nothing else but an aggravating circumstance. This is the general rule; but as such it
certainly is not without its exception as other general rules. The exception is found in the
case of habitual delinquency, as recidivism is precisely one of those that constitute and
give it existence, the other being former conviction, but it is not necessary that both be
present at the same time.
Treachery, evident premeditation and relationship are aggravating circumstances in
crimes against persons and when one of them is present, for instance, in a case of
homicide, the crime committed ceases to be homicide and becomes murder or
parricide, as the case may be. In such cases, that of the said three circumstances which
has raised the crime committed from the category of homicide to that of murder or
parricide, ceases to be an aggravating circumstance to become a qualifying
circumstance and, once accepted as such, it cannot, by virtue of the legal maxim non
237

bis in idem be considered as an aggravating circumstance at the same time (U.


S. vs.Estopia, 28 Phil., 97; U. S. vs. Vitug, 17 Phil., 1; Decision of the Supreme Court of
Spain of November 13, 1871). So must recidivism be considered in habitual
delinquency. We have taken it into consideration in imposing the principal penalty and
we cannot again take it into consideration in imposing the additional penalty because
inasmuch as recidivism is a qualifying or inherent circumstance in habitual delinquency,
it cannot be considered an aggravating circumstance at the same time. Consequently,
the additional penalty to be imposed upon the appellant must be the minimum of the
prescribed by law as, with the exception of recidivism, no other circumstance or fact
justifying the imposition of said penalty in a higher period has been present.
The proposition based on rules 1 and 2 of article 62 of the Revised Penal Code, that if
recidivism is considered an inherent or qualifying circumstance of habitual delinquency
it should not be taken into account in the imposition of the principal penalty, seems to us
untenable because it is based upon the erroneous assumption that the habitual
delinquency is a crime. It is simply a fact or circumstance which, if present in a given
case with the other circumstances enumerated in rule 5 of said article, gives rise to the
imposition of the additional penalties therein prescribed. This is all the more true
because the law itself clearly provides that the habitual delinquent must be sentenced to
the penalty provided by law for his last crime in addition to the additional penalty he
deserves.
In view of the foregoing facts and considerations and furthermore taking into account
the provisions of article 62, rule 5, paragraph (a), of the Revised Penal Code, we deem
it clear that the appellant deserves the additional penalty imposed by the lower court
upon him. The penalty prescribed by said rule is prision correccional in its medium and
maximum periods, or from two years, four months and one day to six years. What was
imposed upon the appellant is the minimum of said penalty and he has absolutely no
reason to complain because after all he can not be exempt from the additional penalty
by reason of his admission at the trial that he is a habitual delinquent, having committed
the crime of theft for the third time before the expiration of ten years from the
commission of his former crime.
In resume we hold that the principal penalty of the appellant must be two months and
one day of arresto mayor. We therefore modify the appealed sentence in this sense and
so modified it is affirmed in all other respects, with the costs to the appellant. So
ordered.

238

Total extinction of criminal liability:: Death of accused


G.R. No. 154579, October 8, 2003
MA. LOURDES R. DE GUZMAN, petitioner, vs. PEOPLE OF THE
PHILIPPINES, respondent.
Second Division, Callejo, Sr., J.:
Facts: Petitioner /Accused Ma. Lourdes de Guzman was convicted by the trial court of
the crime of theft. On appeal, the CA affirmed the conviction but reduced the award of
damages.
Petitioner filed the instant petition for review under Rule 45 assailing the decision of CA.
The Court required the Office of the Solicitor General (OSG) to comment.1awphi1.nt
On January 30, 2003, counsel for the petitioner filed a Manifestation informing the Court
that the petitioner passed away on January 13, 2003. The death of the petitioner
resulted from a vehicular accident, as indicated in the Certificate of Death attached
thereto.7
Issue: Whether the civil liability can survive and proceed against the estate of the
deceased accused
Held: No. Article 89 (1) of the Revised Penal Code clearly provides that:
Art. 89. How criminal liability is totally extinguished. -Criminal liability is totally
extinguished;
1. By the death of the convict, as to the personal penalties; and as to pecuniary
penalties, liability therefore is extinguished only when the death of the offender occurs
before final judgment;

The issue as to whether an action on the civil liability can survive and proceed against
the estate of the deceased has been settled in the case of People v. Bayotas where it
was held that:
Upon death of the accused pending appeal of his conviction, the criminal action is
extinguished inasmuch as there is no longer a defendant to stand as the accused; the
civil action instituted therein for recovery of civil liability ex delicto is ipso facto
extinguished, grounded as it is on the criminal.
The pecuniary liabilities adjudged against the petitioner are undeniably ex delicto. The
petitioner was ordered to pay actual damages, which is the value of the pieces of
jewelry allegedly taken from the private complainant in the amount of P1,500,000, as
modified by the Court of Appeals; and moral damages of P100,000 for the fear and
239

trauma caused to the complainant because of the petitioners intrusion into her
bedroom. These civil liabilities arose from the crime of Theft and are based solely on
said delict.
Although both the trial and the appellate courts found petitioner guilty beyond
reasonable doubt, she had the right to appeal her case to this Court of last resort and
challenge the findings of the two courts below. The judgment of conviction was pending
review until her untimely demise. It has, therefore, not yet attained finality. Thus,
pursuant to Article 89 of the Revised Penal Code, it is incumbent upon the Court to
dismiss the instant petition for review. The Court is dismissing the case because there is
no longer a need to continue with the review of the appeal. The lower courts decision
has thus become ineffectual.
The civil liability attendant to the crime which includes the restitution of personal or real
property is also extinguished. A substitution of heirs in petitioners stead is no longer
necessary.
G.R. Nos. 97472-73, November 20, 2000
PEOPLE OF THE PHILIPPINES, plaintiff-appellee, vs. VICENTE PACAA y
SENARLO, BERNARDO PACAA, VIRGILIO PACAA and VICTORIANO
PACAA,accused-appellants.
First Division, Ynares-Santiago, J.
Facts: For the death and near death of Raul Leyson and Felizardo del Solo,
respectively, accused-appellants were charged with murder and frustrated murder.
On September 10, 1979, accused-appellant Vicente was charged with homicide for the
killing of Raul Leyson. The Information was later amended, wherein the three other
accused-appellants were included and the crime charged was elevated to murder.
Another Information was filed on the same date against the four accused-appellants for
the frustrated murder of Felizardo Del Solo.
After trial, the Regional Trial Court of Cebu, Branch 11, rendered judgment finding
accused VICENTE PACAA, BERNARDO PACAA, VICTORIANO PACAA AND
VIRGILIO PACAA GUILTY beyond reasonable doubt of the crimes of MURDER and
FRUSTRATED MURDER and sentenced each to a penalty of imprisonment of
RECLUSION PERPETUA for the murder and to indemnify, jointly and severally, the
heirs of Raul Leyson, the sum of P30,000.00. In the case of the Frustrated Murder, and
applying the Indeterminate Sentence Law, each of the accused is hereby sentenced to
suffer a penalty of imprisonment of 10 years of prision mayor as minimum and 17 years
of reclusion temporal as maximum, and to indemnify, jointly and severally, Felizardo del
Solo the sum of P10,000.00 in the form of damages.
All the accused appealed.
240

During the pendency of their appeal, appellants Vicente, Bernardo and Virgilio, who are
confined at the National Bilibid Prison, filed a motion to withdraw their appeal. In a
Resolution dated August 30, 1999, the Court, after confirming the voluntariness of their
withdrawal of appeal, granted the motion insofar as Vicente and Virgilio were
concerned. Accused-appellant Bernardo, on the other hand, was required to confirm the
voluntariness of his motion to withdraw appeal. However, a return from the Bureau of
Corrections shows that he died on April 5, 1999.
Held: The death of an accused extinguishes his criminal liability even if his death should
occur during the pendency of his appeal. Accused-appellant Bernardos death not only
extinguished his criminal liability concerning the personal penalties but also whatever
pecuniary penalties have been imposed on him, considering that he died before final
judgment, as provided in Article 89 (1) of the Revised Penal Code.
Both Bernardos civil and criminal liability were extinguished by his death.
Where a person is charged with homicide, for instance, the civil liability for indemnity is
based solely on the finding of guilt. If he is acquitted because of self-defense, the heirs
of the deceased have no right to indemnity. Should the offender die before final
judgment, their right to indemnity is likewise extinguished as there is no basis for the
civil liability. Civil liability exists only when the accused is convicted by final judgment.
Therefore, the appeal of accused-appellant Bernardo Pacaa should be dismissed.
Only the appeal of Victoriano Pacaa is left for adjudication.
G.R. No. 136843, September 28, 2000
PEOPLE OF THE PHILIPPINES, appellee, vs. PEDRO ABUNGAN alias "Pedring,"
RANDY PASCUA and ERNESTO RAGONTON Jr., accused, PEDRO ABUNGAN
alias "Pedring," appellant.
Third Division, Panganiban, J.:
Facts: Accused-appellant Pedro Abungan was convicted by the Regional Trial Court of
Villasis, Pangasinan, Branch 50 of the crime of murder and sentenced to reclusion
perpetua and ordered to pay P50,000 as indemnity to the heirs of the deceased.
Appellant, through counsel, filed the Notice of Appeal on September 14, 1998. On
January 9, 1999, he was committed to the New Bilibid Prison (NBP) in Muntinlupa. He
filed the Appellant's Brief and the OSG submitted the Appellee's Brief. The case was
deemed submitted for resolution on June 5, 2000, when the Court received the
Manifestation of appellant stating that he would not file a reply brief.
In a letter dated August 7, 2000, however, Joselito A. Fajardo, assistant director of the
Bureau of Corrections, informed the Court that Appellant Abungan had died on July 19,
2000 at the NBP Hospital. Attached to the letter was Abungan's Death Certificate.
241

Issue: What is the effect of Appellant Abungan's death on the case and on the appeal?
Held: The death of the appellant pending appeal and prior to the finality of conviction
extinguished his criminal and civil liabilities arising from the delict or crime. Hence, the
criminal case against him, not the appeal, should be dismissed.
The death of appellant on July 19, 2000 during the pendency of his appeal extinguished
his criminal as well as his civil liability, based solely on delict (civil liability ex delicto).
The consequences of appellant's death are provided for in Article 89 (1) of the Revised
Penal Code.
Applying this provision, the Court in People v. Bayotas made the following
pronouncements:
"1. Death of the accused pending appeal of his conviction extinguishes his criminal
liability as well as the civil liability based solely thereon. As opined by Justice Regalado,
in this regard, 'the death of the accused prior to final judgment terminates his criminal
liability and only the civil liability directly arising from and based solely on the offense
committed, i.e., civil liability ex delicto in senso strictiore.'"
"2. Corollarily, the claim for civil liability survives notwithstanding the death of (the)
accused, if the same may also be predicated on a source of obligation other than delict.
Article 1157 of the Civil Code enumerates these other sources of obligation from which
the civil liability may arise as a result of the same act or omission:
a) Law
b) Contracts
c) Quasi-contracts
d) x x x x x x x x x
e) Quasi-delicts
"3. Where the civil liability survives, as explained in Number 2 above, an action for
recovery therefor may be pursued but only by way of filing a separate civil action and
subject to Section 1, Rule 111 of the 1985 Rules on Criminal Procedure as amended.
This separate civil action may be enforced either against the executor/administrator or
the estate of the accused, depending on the source of obligation upon which the same
is based as explained above.
"4. Finally, the private offended party need not fear a forfeiture of his right to file this
separate civil action by prescription, in cases where during the prosecution of the
criminal action and prior to its extinction, the private offended party instituted together
therewith the civil action. In such case, the statute of limitations on the civil liability is
deemed interrupted during the pendency of the criminal case, conformably with the
provisions of Article 1155 of the Civil Code, that should thereby avoid any apprehension
on a possible privation of right by prescription."
In the present case, it is clear that, following the above disquisition in Bayotas, the death
of appellant extinguished his criminal liability. Moreover, because he died during the
pendency of the appeal and before the finality of the judgment against him, his civil
242

liability arising from the crime or delict (civil liability ex delicto) was also extinguished. It
must be added, though, that his civil liability may be based on sources of obligation
other than delict. For this reason, the victims may file a separate civil action against his
estate, as may be warranted by law and procedural rules.
The death of Appellant Abungan would result in the dismissal of the criminal case
against him. Necessarily, the lower court's Decision -- finding him guilty and sentencing
him to suffer reclusion perpetua and to indemnify the heirs of the deceased -- becomes
ineffectual.
G.R. No. 102007, September 2, 1994
PEOPLE OF THE PHILIPPINES, plaintiff-appellee, vs. ROGELIO BAYOTAS y
CORDOVA, accused-appellant.
EN BANC, ROMERO, J.:

Facts: In Criminal Case No. C-3217 filed before Branch 16, RTC Roxas City, Rogelio
Bayotas y Cordova was charged with Rape and eventually convicted thereof on June
19, 1991. Pending appeal of his conviction, Bayotas died on February 4, 1992 at the
National Bilibid Hospital. Consequently, the Supreme Court in its Resolution of May 20,
1992 dismissed the criminal aspect of the appeal. However, it required the Solicitor
General to file its comment with regard to Bayotas' civil liability arising from his
commission of the offense charged.

The Solicitor General expressed his view that the death of accused-appellant did not
extinguish his civil liability as a result of his commission of the offense charged. The
Solicitor General, relying on the case ofPeople v. Sendaydiego insists that the appeal
should still be resolved for the purpose of reviewing his conviction by the lower court on
which the civil liability is based.

Counsel for the accused-appellant, on the other hand, opposed the view of the Solicitor
General arguing that the death of the accused while judgment of conviction is pending
appeal extinguishes both his criminal and civil penalties. In support of his position, said
243

counsel invoked the ruling of the Court of Appeals in People v. Castillo and
Ocfemia which held that the civil obligation in a criminal case takes root in the criminal
liability and, therefore, civil liability is extinguished if accused should die before final
judgment is rendered.

Issue: Whether the death of the accused pending appeal of his conviction extinguish his
civil liability

Held: Yes. As summarized by SC:

1. Death of the accused pending appeal of his conviction extinguishes his criminal
liability as well as the civil liability based solely thereon. As opined by Justice Regalado,
in this regard, "the death of the accused prior to final judgment terminates his criminal
liability and only the civil liability directly arising from and based solely on the offense
committed, i.e., civil liability ex delicto in senso strictiore."

2. Corollarily, the claim for civil liability survives notwithstanding the death of accused, if
the same may also be predicated on a source of obligation other than delict. Article
1157 of the Civil Code enumerates these other sources of obligation from which the civil
liability may arise as a result of the same act or omission:
a) Law
b) Contracts
c) Quasi-contracts
d) . . .
e) Quasi-delicts

244

3. Where the civil liability survives, as explained in Number 2 above, an action for
recovery therefor may be pursued but only by way of filing a separate civil action and
subject to Section 1, Rule 111 of the 1985 Rules on Criminal Procedure as amended.
This separate civil action may be enforced either against the executor/administrator or
the estate of the accused, depending on the source of obligation upon which the same
is based as explained above.

4. Finally, the private offended party need not fear a forfeiture of his right to file this
separate civil action by prescription, in cases where during the prosecution of the
criminal action and prior to its extinction, the private-offended party instituted together
therewith the civil action. In such case, the statute of limitations on the civil liability is
deemed interrupted during the pendency of the criminal case, conformably with
provisions of Article 1155 of the Civil Code, that should thereby avoid any apprehension
on a possible privation of right by prescription.

Applying this set of rules to the case at bench, we hold that the death of appellant
Bayotas extinguished his criminal liability and the civil liability based solely on the act
complained of, i.e., rape. Consequently, the appeal is hereby dismissed without
qualification.

245

Total extinction of criminal liability:: Prescription of offenses


G.R. No. 167571, November 25, 2008
LUIS PANAGUITON, JR., petitioner vs. DEPARTMENT OF JUSTICE, RAMON C.
TONGSON and RODRIGO G. CAWILI, respondents.
Second Division, Tinga, J.
Facts: In 1992, Rodrigo Cawili borrowed various sums of money from petitioner. On 8
January 1993, Cawili and his business associate, Ramon C. Tongson, jointly issued in
favor of petitioner 3 checks in payment of the said loans. All 3 checks bore the
signatures of both Cawili and Tongson. Upon presentment for payment on 18 March
1993, the checks were dishonored, either for insufficiency of funds or by the closure of
the account. Petitioner made formal demands to pay the amounts of the checks upon
Cawili on 23 May 1995 and upon Tongson on 26 June 1995, but to no avail.
On 24 August 1995, petitioner filed a complaint against Cawili and Tongson for violating
B.P. Blg. 22 before the Quezon City Prosecutor's Office. During the preliminary
investigation, only Tongson appeared and filed his counter-affidavit. In a resolution
dated 6 December 1995, City Prosecutor III Eliodoro V. Lara found probable cause only
against Cawili and dismissed the charges against Tongson. Petitioner filed a partial
appeal before the DOJ even while the case against Cawili was filed before the proper
court. In a letter-resolution dated 11 July 1997, after finding that it was possible for
Tongson to co-sign the bounced checks and that he had deliberately altered his
signature in the pleadings submitted during the preliminary investigation, Chief State
Prosecutor Jovencito R. Zuo directed the City Prosecutor of Quezon City to conduct a
reinvestigation of the case against Tongson and to refer the questioned signatures to
the NBI. Tongson moved for the reconsideration of the resolution, but his motion was
denied for lack of merit.
On 15 March 1999, Assistant City Prosecutor Sampaga dismissed the complaint against
Tongson without referring the matter to the NBI. In her resolution, ACP Sampaga held
that the case had already prescribed pursuant to Act No. 3326, as amended. In this
case, the four (4)-year period started on the date the checks were dishonored, or on 20
January 1993 and 18 March 1993. The filing of the complaint before the Quezon City
Prosecutor on 24 August 1995 did not interrupt the running of the prescriptive period, as
the law contemplates judicial, and not administrative proceedings. Thus, considering
that from 1993 to 1998, more than four (4) years had already elapsed and no
information had as yet been filed against Tongson, the alleged violation of B.P. Blg. 22
imputed to him had already prescribed and that Tongson had no dealings with petitioner.
Petitioner appealed to the DOJ. But the DOJ dismissed the same, stating that the
offense had already prescribed pursuant to Act No. 3326. Petitioner filed a motion for
reconsideration of the DOJ resolution. On 3 April 2003, the DOJ ruled in his favor and
declared that the offense had not prescribed and that the filing of the complaint with the
prosecutor's office interrupted the running of the prescriptive period citing Ingco v.
246

Sandiganbayan. Thus, the Office of the City Prosecutor of Quezon City was directed to
file three (3) separate informations against Tongson for violation of B.P. Blg. 22. On 8
July 2003, the City Prosecutor's Office filed an information charging petitioner with three
(3) counts of violation of B.P. 22.
However, in a resolution dated 9 August 2004, the DOJ, presumably acting on a motion
for reconsideration filed by Tongson, ruled that the subject offense had already
prescribed and ordered "the withdrawal of the three (3) informations for violation of B.P.
Blg. 22" against Tongson.
Petitioner filed a petition for certiorari before the Court of Appeals assailing the 9 August
2004 resolution of the DOJ. The petition was dismissed by the Court of Appeals and
denied petitioner's motion for reconsideration.
Issue: Whether the offense of violation of B.P. Blg. 22 has already prescribed per Act
No. 3326.
Held: Yes. Act No. 3326, entitled An Act to Establish Prescription for Violations of
Special Acts and Municipal Ordinances and to Provide When Prescription Shall
Begin, is the law applicable to offenses under special laws which do not provide their
own prescriptive periods. The pertinent provisions read:
Section 1. Violations penalized by special acts shall, unless otherwise provided in such
acts, prescribe in accordance with the following rules: (a) x x x; (b) after four years for
those punished by imprisonment for more than one month, but less than two years; (c) x
xx
Sec. 2. Prescription shall begin to run from the day of the commission of the violation of
the law, and if the same be not known at the time, from the discovery thereof and the
institution of judicial proceedings for its investigation and punishment.
The prescription shall be interrupted when proceedings are instituted against the guilty
person, and shall begin to run again if the proceedings are dismissed for reasons not
constituting jeopardy.
Act. No. 3326 applies to offenses under B.P. Blg. 22. An offense under B.P. Blg. 22
merits the penalty of imprisonment of not less than thirty (30) days but not more than
one year or by a fine, hence, under Act No. 3326, a violation of B.P. Blg. 22 prescribes
in four (4) years from the commission of the offense or, if the same be not known at the
time, from the discovery thereof. Nevertheless, SC cannot uphold the position that only
the filing of a case in court can toll the running of the prescriptive period.
In Securities and Exchange Commission v. Interport Resources Corporation, et al., the
Court ruled:

247

While it may be observed that the term "judicial proceedings" in Sec. 2 of Act No. 3326
appears before "investigation and punishment" in the old law, with the subsequent
change in set-up whereby the investigation of the charge for purposes of prosecution
has become the exclusive function of the executive branch, the term "proceedings"
should now be understood either executive or judicial in character: executive when it
involves the investigation phase and judicial when it refers to the trial and judgment
stage. With this clarification, any kind of investigative proceeding instituted against the
guilty person which may ultimately lead to his prosecution should be sufficient to toll
prescription.
Indeed, to rule otherwise would deprive the injured party the right to obtain vindication
on account of delays that are not under his control. A clear example would be this case,
wherein petitioner filed his complaint-affidavit on 24 August 1995, well within the four
(4)-year prescriptive period. He likewise timely filed his appeals and his motions for
reconsideration on the dismissal of the charges against Tongson. He went through the
proper channels, within the prescribed periods. However, from the time petitioner filed
his complaint-affidavit with the Office of the City Prosecutor (24 August 1995) up to the
time the DOJ issued the assailed resolution, an aggregate period of nine (9) years had
elapsed. Clearly, the delay was beyond petitioner's control. After all, he had already
initiated the active prosecution of the case as early as 24 August 1995, only to suffer
setbacks because of the DOJ's flip-flopping resolutions and its misapplication of Act No.
3326. Aggrieved parties, especially those who do not sleep on their rights and actively
pursue their causes, should not be allowed to suffer unnecessarily further simply
because of circumstances beyond their control, like the accused's delaying tactics or the
delay and inefficiency of the investigating agencies.
SC held that the offense has not yet prescribed. Petitioner 's filing of his complaintaffidavit before the Office of the City Prosecutor on 24 August 1995 signified the
commencement of the proceedings for the prosecution of the accused and thus
effectively interrupted the prescriptive period for the offenses they had been charged
under B.P. Blg. 22. Moreover, since there is a definite finding of probable cause, with the
debunking of the claim of prescription there is no longer any impediment to the filing of
the information against petitioner.

248

G.R. Nos. 118757 & 121571, October 19, 2004


ROBERTO BRILLANTE, petitioner, vs. COURT OF APPEALS and THE PEOPLE OF
THE PHILIPPINES, respondents.
Second Division, Tinga, J.

Facts: On January 7, 1988, Brillante, then a candidate for the position of Councilor in
Makati, held a press conference at the Makati Sports Club which was attended by some
50 journalists. In the course of the press conference, Brillante accused Binay of plotting
the assassination of Syjuco. He further accused Binay of terrorism, intimidation and
harassment of the Makati electorate. Brillante also circulated among the journalists
copies of an open letter to President Aquino which discussed in detail his charges
against Binay.

Several journalists who attended the press conference wrote news articles about the
same. The open letter was subsequently published under the title "Plea to Cory--Save
Makati" in newspapers such as thePeoples Journal, Balita, Malaya and Philippine Daily
Inquirer.

As a result of the publication of the open letter, Binay filed with the Makati fiscals office
four complaints for libel against Brillante, as the author of the letter; Gonong, Buan and
Camino for writing and publishing the news article on Brillantes accusations against him
in the Peoples Journal; Hernandez, Villanueva and Manuel for writing and publishing a
similar news article in the News Today; and for publishing the open letter, Buan and
Camino of thePeoples Journal; and Arcadio A. Sison (Sison) as President of A. Sison
and Associates, an advertising agency.

Francisco Baloloy, who was identified in the open letter as among the persons who
attended the meeting organized by Binay and Prudente to plan the assassination of
Syjuco, likewise filed a criminal complaint for libel against Brillante, Domingo Quimlat ,
249

Publisher and Editor-in-Chief of Balita, and Sison as President of A. Sison and


Associates.

Subsequently, five Informations for libel against Brillante were filed with the RTC of
Makati.

Similarly, on January 15, 1988, Prudente filed four complaints for libel against Brillante
and the editors and publishers of the newspapers where the open letter was published.
On January 16, 1989, four Informations for libel were filed against Brillante and several
co-accused with the RTC of Manila.

RTC-Manila acquitted Sison but found Brillante guilty of libel on four counts. The Court
of Appeals affirmed the decision of the RTC-Manila.

Brillante was likewise convicted for libel on five counts by the RTC-Makati, which the
Court of Appeals affirmed.

Issue: Whether the offense of libel had already prescribed when the Informations were
filed with the RTC-Manila and RTC-Makati

Held: NO. With respect to the issue of prescription, the fourth paragraph of Article 90 of
the Revised Penal Code provides that the "crime of libel or other similar offenses shall
prescribe in one year." In determining when the one-year prescriptive period should be
reckoned, reference must be made to Article 91 of the same code which sets forth the
rule on the computation of prescriptive periods of offenses:

Computation of prescription of offenses.The period of prescription shall commence to


run from the day on which the crime is discovered by the offended party, the authorities,
250

or their agents, and shall be interrupted by the filing of the complaint or information, and
shall commence to run again when such proceedings terminate without the accused
being convicted or acquitted, or are unjustifiably stopped for any reason not imputable
to him.

The aforequoted provision expressly states that prescriptive period shall be interrupted
by the filing of the complaint or information. The meaning of the phrase "shall be
interrupted by the filing of the complaint or information" in Article 91 has been settled in
the landmark case of People v. Olarte, where the Court settled divergent views as to the
effect of filing a complaint with the Municipal Trial Court for purposes of preliminary
investigation on the prescriptive period of the offense. The Court therein held that the
filing of the complaint for purposes of preliminary investigation interrupts the period of
prescription of criminal responsibility.

Thereafter, the Court in Francisco v. Court of Appeals clarified that the filing of the
complaint with the fiscals office also suspends the running of the prescriptive period of
a crime:

As is a well-known fact, like the proceedings in the court conducting a preliminary


investigation, a proceeding in the Fiscal's Office may terminate without conviction or
acquittal.

There is no conflict in the pronouncements of the Court in Olarte and Francisco as


Brillante erroneously suggests.Olarte laid down the doctrine that a complaint filed for
purposes of preliminary investigation tolls the running of the prescriptive period of a
criminal offense. The criminal complaint for libel in that case was filed, for the purpose of
preliminary investigation, with the Justice of the Peace Court in Pozorrubio,
Pangasinan. Hence, in setting the doctrine, the Court referred to the "filing of the
complaint in the Municipal Court." The question of whether the doctrine laid down
in Olarte also applies to criminal complaints filed with the prosecutors office was settled
inFrancisco. Specifically, the Court in Francisco amplified the Olarte doctrine when it
categorically ruled that the filing of a complaint with the fiscals office suspends the
running of the prescriptive period of a criminal offense.
251

Thus, the Court of Appeals committed no reversible error in ruling that the offense of
libel had not yet prescribed when the informations against Brillante and his co-accused
were filed in the RTC-Manila and RTC-Makati.

G.R. No. 139405, March 13, 2001


PEOPLE OF THE PHILIPPINES, petitioner, vs. ARTURO F.
PACIFICADOR, respondent.
Second Division, De Leon, Jr., J.:
Facts: On October 27, 1988, respondent, Arturo Pacificador y Fullon, and his erstwhile
co-accused, Jose T. Marcelo, were charged before the Sandiganbayan with the crime of
violation of Republic Act No. 3019, as amended, otherwise known as the Anti-Graft and
Corrupt Practices Act, in an Information that reads:
That on or about and during the period from December 6, 1975 to January 6, 1976, in
Metro Manila, Philippines, and within the jurisdiction of this Honorable Court, said
accused, Arturo Pacificador, then Chairman of the Board of the National Shipyard and
Steel Corporation, a government-owned corporation, and therefore, a public officer, and
Jose T. Marcelo, Jr., then President of the Philippine Smelters Corporation, a private
corporation, conspiring and confederating with one another and with other individuals,
did then and there, willfully, unlawfully and knowingly, and with evident bad faith
promote, facilitate, effect and cause the sale, transfer and conveyance by the National
Shipyard and Steel Corporation of its ownership and all its titles, rights and interests
over parcels of land in Jose Panganiban, Camarines Norte where the Jose Panganiban
Smelting Plant is located including all the reclaimed and foreshore areas of about 50
hectares to the Philippine Smelters Corporation by virtue of a contract, the terms and
conditions of which are manifestly and grossly disadvantageous to the Government as
the consideration thereof is only P85,144.50 while the fair market value thereof at that
time was P862,150.00 thereby giving the Philippine Smelters Corporation unwarranted
benefits, advantages and profits and causing undue injury, damage and prejudice to the
government in the amount of P777,005.50.
After his arraignment, the respondent filed a Motion to Dismiss on the following
grounds:
1) The court has no jurisdiction since the crime charged had been extinguished
by prescription; and
2) The information does not charge an offense in view of the decision of the
Supreme Court in the case of San Mauricio Mining Corporation, et al. vs. Hon.
Constante A. Ancheta, et al., G.R. No. L-47859 and L-57132 dated July 10, 1981.
252

Petitioner filed an Opposition to the Motion to Dismiss. Sandiganbayan denied the


Motion to Dismiss.
Respondent Pacificador moved for the reconsideration of the Resolution of the
Sandiganbayan denying his Motion to Dismiss.
On February 3, 1999, the Sandiganbayan reconsidered its Resolution of November 10,
1998 and dismissed the Information in Criminal Case No. 139405 against the
respondent on the ground of prescription. The Urgent Motion for Reconsideration of
petitioner was denied by the Sandiganbayan.
Issue: Whether the offense has prescribed
Held: Yes. It has been settled that Section 2 of Act No. 3326 governs the computation of
prescription of offenses defined and penalized by special laws. In the case of People v.
Sandiganbayan, SC ruled that Section 2 of Act No. 3326 was correctly applied by the
anti-graft court in determining the reckoning period for prescription in a case involving
the crime of violation of Republic Act No. 3019, as amended. In the fairly recent case
of Presidential Ad Hoc Fact-Finding Committee on Behest Loans v. Desierto, SC ruled
that:
Since the law alleged to have been violated, i.e., paragraphs (e) and (g) of Section 3,
R.A. No. 3019, as amended, is a special law, the applicable rule in the computation of
the prescriptive period is Section 2 of Act No. 3326, as amended, which provides:
SECTION 2. Prescription should begin to run from the day of the commission of the
violation of the law, and if the same be not known at the time, from the discovery thereof
and institution of judicial proceedings for its investigation and punishment. (Emphasis
ours)
The prescription shall be interrupted when the proceedings are instituted against the
guilty person and shall begin to run again if the proceedings are dismissed for reasons
not constituting double jeopardy.
This simply means that if the commission of the crime is known, the prescriptive period
shall commence to run on the day it was committed
It can be gleaned from the Information in this case that respondent Pacificador allegedly
committed the crime charged "on or about and during the period from December 6,
1975 to January 6, 1976." Section 11 of R.A. No. 3019, as amended by B.P. Blg. 195,
provides that the offenses committed under the said statute shall prescribe in fifteen
(15) years. It appears however, that prior to the amendment of Section 11 of R.A. No.
3019 by B.P. Blg. 195 which was approved on March 16, 1982, the prescriptive period
for offenses punishable under the said statute was only ten (10) years. The longer
prescriptive period of fifteen (15) years, as provided in Section 11 of R.A. No. 3019 as
amended by B.P. Blg. 195, does not apply in this case for the reason that the
253

amendment, not being favorable to the accused (herein private respondent), cannot be
given retroactive effect. Hence the crime prescribed on January 6, 1986 or ten (10)
years from January 6, 1976.
The petitioner, however, vehemently denies having any knowledge of the crime at the
time it was allegedly committed by the respondent. It claims that the crime charged in
the Information should be deemed as discovered only on May 13, 1987 when the then
Solicitor General, Francisco Chavez, filed a complaint with the Presidential Commission
on Good Government (PCGG) against the respondent, for violation of the provision of
R.A. No. 3019, as amended.
While petitioner may not have knowledge of the alleged crime at the time of its
commission, the registration of the subject Deed of Sale with the Registry of Deeds
constitutes constructive notice thereof to the whole world including the petitioner. Well
entrenched is the jurisprudential rule that registration of deeds in the public real estate
registry is a notice thereof to the whole world. The registration is a constructive notice of
its contents as well as all interests, legal and equitable, included therein. All persons are
charged with the knowledge of what it contains. Hence, even if the period of prescription
is reckoned from February 28, 1977, the crime had already prescribed when the
Information in this case was filed with the Sandiganbayan on October 27, 1988.
It bears emphasis, as held in a number of cases, that in the interpretation of the law on
prescription of crimes, that which is more favorable to the accused is to be adopted. The
said legal principle takes into account the nature of the law on prescription of crimes
which is an act of amnesty and liberality on the part of the state in favor of the offender.
G.R. No. 141931, December 4, 2000
ANICETO RECEBIDO, petitioner, vs. PEOPLE OF THE PHILIPPINES, respondent.
First Division, Kapunan, J.

Facts: On September 9, 1990, private complainant Caridad Dorol went to the house of
her cousin, petitioner Aniceto Recebido, at San Isidro, Bacon, Sorsogon to redeem her
property, an agricultural land located at San Isidro, Bacon, Sorsogon, which Caridad
Dorol mortgaged to petitioner sometime in April of 1985. Petitioner and Caridad Dorol
did not execute a document on the mortgage but Caridad Dorol instead gave petitioner
a copy of the Deed of Sale dated June 16, 1973 executed in her favor by her father,
Juan Dorol.
254

In said confrontation, petitioner refused to allow Caridad Dorol to redeem her property
on his claim that she had sold her property to him in 1979. Caridad Dorol maintained
and insisted that the transaction between them involving her property was a mortgage.

Caridad Dorol verified from the Office of the Assessor in Sorsogon that there exists on
its file a Deed of Sale dated August 13, 1979, allegedly executed by Caridad Dorol in
favor of petitioner and that the property was registered in the latter's name. After
comparison of the specimen signatures of Caridad Dorol in other documents with that of
the signature of Caridad Dorol on the questioned Deed of Sale, NBI Document
Examiner Antonio Magbojas, found that the latter signature was falsified.

Caridad Dorol filed her complaint against petitioner Aniceto Recebido with the NBI,
Legaspi City and its Questioned Documents Division conducted an examination in the
original copy of the Deed of Sale in question allegedly signed by Caridad, particularly
her signature affixed thereon.

Mr. Magboja's report was approved by the Chief of the Questioned Documents Division
and the Deputy Director of Technical Services of the NBI.

Thus, the Office of the Provincial Prosecutor of Sorsogon filed the information indicting
petitioner for Falsification of Public Document with the Regional Trial Court, 5th Judicial
Region, Branch 51, Sorsogon, Sorsogon.

The trial court rendered the decision on December 2, 1996, convicting petitioner of the
crime charged. The Court of Appeals affirmed with modification the decision of the trial
court.

255

Issue: Whether the crime charged had already prescribed at the time the information
was filed

Held: NO. While the defense of prescription of the crime was raised only during the
motion for reconsideration of the decision of the Court of Appeals, there was no waiver
of the defense. Under the Rules of Court, the failure of the accused to assert the ground
of extinction of the offense, inter alia, in a motion to quash shall not be deemed a waiver
of such ground. The reason is that by prescription, the State or the People loses the
right to prosecute the crime or to demand the service of the penalty
imposed. Accordingly, prescription, although not invoked in the trial, may, as in this
case, be invoked on appeal.

Nonetheless, the crime charged has not prescribed. The petitioner is correct in stating
that whether or not the offense charged has already prescribed when the information
was filed would depend on the penalty imposable therefor, which in this case is "prision
correccional in its medium and maximum periods and a fine of not more than 5,000.00
pesos." Under the Revised Penal Code, said penalty is a correctional penalty in the
same way that the fine imposed is categorized as correctional. Both the penalty and fine
being correctional, the offense shall prescribe in ten years.

The issue that the petitioner has missed, however, is the reckoning point of the
prescriptive period. The petitioner is of the impression that the ten-year prescriptive
period necessarily started at the time the crime was committed. This is inaccurate.
Under Article 91 of the Revised Penal Code, the period of prescription shall "commence
to run from the day on which the crime is discovered by the offended party, the
authorities, or their agents," As held in People v. Reyes, registration in public
registry is a notice to the whole world. The record is constructive notice of its contents
as well as all interests, legal and equitable, included therein. All persons are charged
with knowledge of what it contains.

The prosecution has established that private complainant Dorol did not sell the subject
land to the petitioner-accused at anytime and that sometime in 1983 the private
complainant mortgaged the agricultural land to petitioner Recebido. It was only on
256

September 9, 1990, when she went to petitioner to redeem the land that she came to
know of the falsification committed by the petitioner. On the other hand, petitioner
contends that the land in question was mortgaged to him by Juan Dorol, the father of
private complainant, and was subsequently sold to him on August 13, 1983. This Court
notes that the private offended party had no actual knowledge of the falsification prior to
September 9, 1990. Meanwhile, assuming arguendo that the version of the petitioner is
believable, the alleged sale could not have been registered before 1983, the year the
alleged deed of sale was executed by the private complainant. Considering the
foregoing, it is logical and in consonance with human experience to infer that the crime
committed was not discovered, nor could have been discovered, by the offended party
before 1983. Neither could constructive notice by registration of the forged deed of sale,
which is favorable to the petitioner since the running of the prescriptive period of the
crime shall have to be reckoned earlier, have been done before 1983 as it is impossible
for the petitioner to have registered the deed of sale prior thereto. Even
granting arguendo that the deed of sale was executed by the private complainant,
delivered to the petitioner-accused in August 13, 1983 and registered on the same day,
the ten-year prescriptive period of the crime had not yet elapsed at the time the
information was filed in 1991. The inevitable conclusion, therefore, is that the crime had
not prescribed at the time of the filing of the information.

Total extinction of criminal liability:: Prescription of penalties


G.R. No. 141718, January 21, 2005
BENJAMIN PANGAN y RIVERA, petitioner, vs. HON. LOURDES F. GATBALITE, as
the Presiding Judge, Regional Trial Court of Angeles City, Branch 56, and COL.
JAMES D. LABORDO, as the City Jail Warden of Angeles City, respondents.
First Division, Azcuna, J.:
Facts: The petitioner was indicted for simple seduction at the Municipal Trial Court of
Angeles City, Branch 3. During trial, counsel for petitioner, submitted the case for
decision without offering any evidence, due to the petitioners constant absence at
hearings. On September 16, 1987, the petitioner was convicted of the offense charged
and was sentenced to serve a penalty of two months and one day of arresto mayor.
257

On appeal, the Regional Trial Court, on October 24, 1988, affirmed in toto the decision
of the MTC.
On August 9, 1991, the case was called for promulgation of the decision in the court of
origin. Despite due notice, counsel for the petitioner did not appear. Notice to petitioner
was returned unserved with the notation that he no longer resided at the given address.
As a consequence, he also failed to appear at the scheduled promulgation. The court of
origin issued an order directing the recording of the decision in the criminal docket of the
court and an order of arrest against the petitioner.
Pursuant to the order of arrest, on January 20, 2000, the petitioner was apprehended
and detained at the Mabalacat Detention Cell. On January 24, 2000, petitioner filed a
Petition for a Writ of Habeas Corpus at the Regional Trial Court of Angeles City.
Petitioner contended that his arrest was illegal and unjustified on the grounds that: (a)
the straight penalty of two months and one day of arresto mayor prescribes in five years
under No. 3, Article 93 [of the] Revised Penal Code, and (b) having been able to
continuously evade service of sentence for almost nine years, his criminal liability has
long been totally extinguished under No. 6, Article 89 [of the] Revised Penal Code.
After his transfer to the City Jail of Angeles City, petitioner filed an Amended Petition
with the Regional Trial Court, impleading herein respondent Col. James D. Labordo, the
Jail Warden of Angeles City.
The Jail Warden alleged that petitioners detention was pursuant to the order of
commitment (mittimus), issued by the Clerk of Court of the Municipal Trial Court of
Angeles City, Branch 3, dated January 25, 2000.
On January 31, 2000, respondent Judge rendered the assailed decision, denying the
petition for habeas corpus, ruling that:
The elements of prescription are:
1. That the penalty is imposed by final judgment;
2. That convict evaded the service of the sentence by escaping during the term of his
sentence;
3. That the convict who had escaped from prison has not given himself up, or been
captured, or gone to a foreign country with which we have no extradition treaty, or
committed another crime;
4. The penalty has prescribed, because of the lapse of time from the date of the evasion
of the service of the sentence by the convict.
In this case, the essential element of prescription which is the evasion of the service of
sentence is absent. Admittedly, the petitioner herein has not served the penalty imposed
on him in prison and that during the service of the sentence, he escaped therefrom.

258

"There was no evasion of the service of the sentence in this case, because such
evasion presupposes escaping during the service of the sentence consisting in
deprivation of liberty." (Infante vs. Warden)
Issue: Whether the penalty imposed upon the accused has prescribed
Held: NO. There is no dispute that the duty of government to compel the service of
sentence sets in when the judgment of conviction becomes final.
The dispute, however, is in the construction of the phrase "should evade the service of
sentence." When does the period of prescription of penalties begin to run? The ruling in
Infante vs. Warden construes this to mean that the convict must escape from jail
"because such evasion presupposes escaping during the service of the sentence
consisting in deprivation of liberty." A perusal of the facts in Infante v. Warden reveals
that it is not on all fours with the present case. In Infante, the convict was on conditional
pardon when he was re-arrested. Hence, he had started serving sentence but the State
released him. In the present case, the convict evaded service of sentence from the
start, and was arrested eight years later.
The RTC decision, however, must stand, since it is in accord with applicable decisions
of this Court. Article 93 of the Revised Penal Code has been interpreted several times
by the Court.
In Tanega v. Masakayan, SC held that the under the Article 93, the period of prescription
of penalties shall commence to run from the date when the culprit should evade the
service of his sentence. What then is the concept of evasion of service of sentence?
Article 157 of the Revised Penal Code defines and penalizes evasion of service of
sentence. Elements of evasion of service of sentence are: (1) the offender is a convict
by final judgment; (2) he "is serving his sentence which consists in deprivation of
liberty"; and (3) he evades service of sentence by escaping during the term of his
sentence. By the express terms of the statute, a convict evades "service of his
sentence" by "escaping during the term of his imprisonment by reason of final
judgment." That escape should take place while serving sentence, is emphasized by the
provisions of the second sentence of Article 157 which provides for a higher penalty if
such "evasion or escape shall have taken place by means of unlawful entry, by breaking
doors, windows, gates, walls, roofs, or floors, or by using picklocks, false keys, disguise,
deceit, violence or intimidation, or through connivance with other convicts or employees
of the penal institution, . . ." Indeed, evasion of sentence is but another expression of
the term "jail breaking." Thus, for prescription of penalty of imprisonment imposed by
final sentence to commence to run, the culprit should escape during the term of such
imprisonment.
In Del Castillo v. Torrecampo, the Court cited and reiterated Tanega, where SC declared
that, for prescription of penalty imposed by final sentence to commence to run, the
culprit should escape during the term of such imprisonment.
259

Consistent with the two cases, SC pronounced that the prescription of penalties found in
Article 93 of the Revised Penal Code, applies only to those who are convicted by final
judgment and are serving sentence which consists in deprivation of liberty. The period
for prescription of penalties begins only when the convict evades service of sentence by
escaping during the term of his sentence. Since petitioner never suffered deprivation of
liberty before his arrest on January 20, 2000 and as a consequence never evaded
sentence by escaping during the term of his service, the period for prescription never
began.
G.R. No. 139033, December 18, 2002
JOVENDO DEL CASTILLO, petitioner, vs. HON. ROSARIO TORRECAMPO,
Presiding Judge, RTC of Camarines Sur, Branch 33 and PEOPLE OF THE
PHILIPPINES, respondents.
Third Division, Corona, J.:
Facts: On January 14, 1985, the trial court rendered judgment and declared petitioner
guilty beyond reasonable doubt of violating Section 178 (nn) of PD 1296, otherwise
known as the 1978 Election Code, as amended, and sentenced petitioner to suffer the
indeterminate penalty of imprisonment of 1 year as minimum to 3 years as maximum.
The Court of Appeals affirmed the decision of the trial court in toto. Said decision
became final and executory. Thus, the execution of judgment was scheduled on
October 14, 1987.
On October 12, 1987, an urgent motion to reset the execution of judgment was
submitted by petitioner through his counsel. But it was denied for lack of merit. During
the execution of judgment, petitioner failed to appear which prompted the presiding
judge to issue an order of arrest of petitioner and the confiscation of his bond. However,
petitioner was never apprehended. He remained at large.
Ten years later, on October 24, 1997, petitioner filed before the trial court a motion to
quash the warrant issued for his arrest on the ground of prescription of the penalty
imposed upon him. However, it was denied. His motion for reconsideration was likewise
denied.
Dissatisfied, petitioner filed with the Court of Appeals a Petition for Certiorari assailing
the orders of the trial court denying both his motion to quash the warrant of arrest and
motion for reconsideration.
CA dismissed the petition for lack of merit.
Issue: Whether the penalty imposed upon the petitioner has already prescribed
Held: NO. The elements in order that the penalty imposed has prescribed are as
follows:
260

1. That the penalty is imposed by final sentence.


2. That the convict evaded the service of the sentence by escaping during the term of
his sentence.
3. That the convict who escaped from prison has not given himself up, or been
captured, or gone to a foreign country with which we have no extradition treaty or
committed another crime.
4. That the penalty has prescribed, because of the lapse of time form the date of the
evasion of the service of the sentence by the convict.
The conclusion of the Court of Appeals is consistent with the ruling of this Court
in Tanega vs. Masakayan, et. al., where SC declared that, for prescription of penalty
imposed by final sentence to commence to run, the culprit should escape during the
term of such imprisonment.
Article 93 of the Revised Penal Code provides when the prescription of penalties shall
commence to run. Under said provision, it shall commence to run from the date the
felon evades the service of his sentence. Pursuant to Article 157 of the same Code,
evasion of service of sentence can be committed only by those who have been
convicted by final judgment by escaping during the term of his sentence.
"Escape" in legal parlance and for purposes of Articles 93 and 157 of the RPC means
unlawful departure of prisoner from the limits of his custody. Clearly, one who has not
been committed to prison cannot be said to have escaped therefrom.
In the instant case, petitioner was never brought to prison. In fact, even before the
execution of the judgment for his conviction, he was already in hiding. Now petitioner
begs for the compassion of the Court because he has ceased to live a life of peace and
tranquility after he failed to appear in court for the execution of his sentence. But it was
petitioner who chose to become a fugitive. The Court accords compassion only to those
who are deserving. Petitioners guilt was proven beyond reasonable doubt but he
refused to answer for the wrong he committed. He is therefore not to be rewarded
therefor.
Because petitioner was never placed in confinement, prescription never started to run in
his favor.
Total extinction of criminal liability:: Amnesty
G.R. No. 135457, September 29, 2000
PEOPLE OF THE PHILIPPINES, plaintiff-appellee, vs. JOSE PATRIARCA, JR., alias
"KA DJANGO," CARLOS NARRA, alias "KA JESSIE" and TEN (10) JOHN
DOES,accused-appellant.
Second Division, Buena, J.:

261

Facts: On August 16, 1990, an information for murder was filed against Jose Patriarca,
Jr., alias "Ka Django," "Carlos Narra", "Ka Jessie," et al., charging them of murder for
the killing of Alfredo Arevalo. He was also charged with Murder for the killing of one
Rudy de Borja and a certain Elmer Cadag under Informations docketed as Criminal
Cases Nos. 2665 and 2672, respectively.
On January 20, 1998, a decision was rendered convicting the accused of the crime of
Murder for the death of Alfredo Arevalo and acquitting him as regards Crim. Case No.
2665 and Crim. Case No. 2672, for failure of the prosecution to prove the guilt of the
accused beyond reasonable doubt.
Accused filed this appeal.
Accused-appellant applied for amnesty under Proclamation No. 724 amending
Proclamation No. 347, dated March 25, 1994, entitled "Granting Amnesty to Rebels,
Insurgents, and All Other Persons Who Have or May Have Committed Crimes Against
Public Order, Other Crimes Committed in Furtherance of Political Ends, and Violations
of the Article of War, and Creating a National Amnesty Commission." His application
was favorably granted by the National Amnesty Board. The Commission, in its
deliberation on the application on 22 October 1999, resolved to approve the
recommendation of the Local Amnesty Board.
Accused-appellant Jose N. Patriarca, Jr. was granted amnesty under Proclamation No.
724 dated May 17, 1996. It amended Proclamation No. 347 dated March 25, 1994.
Section 1 of Proclamation No. 724 reads thus:
"Section 1. Grant of Amnesty. - Amnesty is hereby granted to all persons who shall
apply therefor and who have or may have committed crimes, on or before June 1, 1995,
in pursuit of their political beliefs, whether punishable under the Revised Penal Code or
special laws, including but not limited to the following: rebellion or insurrection; coup
d'etat; conspiracy and proposal to commit rebellion, insurrection, or coup d'etat;
disloyalty of public officers or employees; inciting to rebellion or insurrection; sedition;
conspiracy to commit sedition; inciting to sedition; illegal assembly; illegal association;
direct assault; indirect assault; resistance and disobedience to a person in authority or
agents of such person; tumults and other disturbances of public order; unlawful use of
means of publication and unlawful utterances; alarms and scandals; illegal possession
of firearms, ammunitions, and explosives, committed in furtherance of, incident to, or in
connection with the crimes of rebellion and insurrection; and violations of Articles 59
(desertion), 62 (absence without leave), 67 (mutiny or sedition), 68 (failure to suppress
mutiny or sedition), 94 (various crimes), 96 (conduct unbecoming an officer and
gentleman), and 97 (general article) of the Articles of War; Provided, That the amnesty
shall not cover crimes against chastity and other crimes for personal ends."
Issue: What is the effect of amnesty on the liability of the accused?
262

Held: Amnesty commonly denotes a general pardon to rebels for their treason or other
high political offenses, or the forgiveness which one sovereign grants to the subjects of
another, who have offended, by some breach, the law of nations. Amnesty looks
backward, and abolishes and puts into oblivion, the offense itself; it so overlooks and
obliterates the offense with which he is charged, that the person released by amnesty
stands before the law precisely as though he had committed no offense.
Paragraph 3 of Article 89 of the Revised Penal Code provides that criminal liability is
totally extinguished by amnesty, which completely extinguishes the penalty and all its
effects.
In the case of People vs. Casido, the difference between pardon and amnesty is given:
"Pardon is granted by the Chief Executive and as such it is a private act which must be
pleaded and proved by the person pardoned, because the courts take no notice thereof;
while amnesty by Proclamation of the Chief Executive with the concurrence of
Congress, is a public act of which the courts should take judicial notice. Pardon is
granted to one after conviction; while amnesty is granted to classes of persons or
communities who may be guilty of political offenses, generally before or after the
institution of the criminal prosecution and sometimes after conviction. Pardon looks
forward and relieves the offender from the consequences of an offense of which he has
been convicted, that is, it abolishes or forgives the punishment, and for that reason it
does 'not work the restoration of the rights to hold public office, or the right of suffrage,
unless such rights be expressly restored by the terms of the pardon,' and it 'in no case
exempts the culprit from the payment of the civil indemnity imposed upon him by the
sentence' (Article 36, Revised Penal Code). While amnesty looks backward and
abolishes and puts into oblivion the offense itself, it so overlooks and obliterates the
offense with which he is charged that the person released by amnesty stands before the
law precisely as though he had committed no offense."
This Court takes judicial notice of the grant of amnesty upon accused-appellant Jose N.
Patriarca, Jr. Once granted, it is binding and effective. It serves to put an end to the
appeal.

Civil Liability:: Subsidiary liability of employers, etc.


G.R. No. 163351, June 21, 2005
ANTONIO V. NUEVA ESPAA petitioner, vs. PEOPLE OF THE
PHILIPPINES, respondent.
Third Division, Corona, J.:
Facts: At about 11:15 a.m. on May 15, 1998, petitioner Antonio E. Nueva Espaa was
driving a passenger bus owned by Vallacar Transit, Inc. He was then traversing the
national highway of Calag-Calag, Ayungon, Negros Oriental and was southbound going
263

to Dumaguete City. While negotiating a curve, the passenger bus collided with a
northbound Honda motorcycle. As a result, Reynard So, the driver of the motorcycle,
and Nilo Castro, the person riding in tandem with him, were killed. An information for
reckless imprudence resulting in double homicide 1 was filed against petitioner who was
also accused of failing to extend aid or assistance to the victims.
The trial court convicted petitioner of the crime charged. The Court of Appeals modified
the judgment and provided for the subsidiary liability of petitioners employer under
Article 103 of the Revised Penal Code.
Held: SC modified the award of damages given by the lower courts.
When death occurs due to a crime, the following damages may be recovered: (1) a civil
indemnity ex delicto for the death of the victim; (2) actual or compensatory damages; (3)
moral damages; (4) exemplary damages; (5) attorneys fees and expenses of litigation,
and (6) interest, in proper cases.
SC adopted the pronouncement of the Court of Appeals regarding the subsidiary liability
of petitioners employer, Vallacar Transit Inc., under Article 103 of the Revised Penal
Code. An employer may be subsidiarily liable for the employees civil liability in the
criminal action if it can be shown that: (1) the employer is engaged in any kind of
industry; (2) the employee committed the offense in the discharge of his duties and (3)
the accused is insolvent. However, subject to prevailing jurisprudence, the subsidiary
liability may be enforced only upon a motion for subsidiary writ of execution against
Vallacar Transit, Inc. and upon proof that petitioner is insolvent.

G.R. No. 155791, March 16, 2005


MELBA QUINTO, Petitioners, vs. DANTE ANDRES and RANDYVER
PACHECO, Respondents.
Second Division, Callejo, Sr., J.:
Facts: At around 7:30 a.m. on November 13, 1995, eleven-year-old Edison Garcia, a
Grade 4 elementary school pupil, and his playmate, Wilson Quinto, who was also about
eleven years old, were at Barangay San Rafael, Tarlac, Tarlac. They saw respondents
Dante Andres and Randyver Pacheco by the mouth of a drainage culvert. Andres and
Pacheco invited Wilson to go fishing with them inside the drainage culvert. Wilson
assented. When Garcia saw that it was dark inside, he opted to remain seated in a
grassy area about two meters from the entrance of the drainage system.
Respondent Pacheco had a flashlight. He, along with respondent Andres and Wilson,
entered the drainage system which was covered by concrete culvert about a meter high
and a meter wide, with water about a foot deep. After a while, respondent Pacheco, who
was holding a fish, came out of the drainage system and left without saying a word.
Respondent Andres also came out, went back inside, and emerged again, this time,
264

carrying Wilson who was already dead. Respondent Andres laid the boys lifeless body
down in the grassy area. Shocked at the sudden turn of events, Garcia fled from the
scene. For his part, respondent Andres went to the house of petitioner Melba Quinto,
Wilsons mother, and informed her that her son had died. Melba Quinto rushed to the
drainage culvert while respondent Andres followed her.
The cadaver of Wilson was buried without any autopsy thereon having been conducted.
The police authorities of Tarlac, Tarlac, did not file any criminal complaint against the
respondents for Wilsons death.
Two weeks thereafter, NBI investigators took the sworn statements of respondent
Pacheco, Garcia and petitioner Quinto. Respondent Pacheco alleged that he had never
been to the drainage system catching fish with respondent Andres and Wilson. He also
declared that he saw Wilson already dead when he passed by the drainage system
while riding on his carabao.
On February 29, 1996, the cadaver of Wilson was exhumed. Dr. Dominic Aguda of the
NBI performed an autopsy thereon at the cemetery and submitted his autopsy report,
showing the cause of death as Asphyxia by drowning; traumatic head injuries,
contributory.
The NBI filed a criminal complaint for homicide against respondents Andres and
Pacheco in the Office of the Provincial Prosecutor, which found probable cause for
homicide by dolo against the two.
An Information was later filed with the Regional Trial Court (RTC) of Tarlac, Tarlac,
charging the respondents with homicide.
After the prosecution rested its case, the respondents filed a demurer to evidence which
the trial court granted on the ground of insufficiency of evidence. It also held that it could
not hold the respondents liable for damages because of the absence of preponderant
evidence to prove their liability for Wilsons death.
The petitioner appealed the order to the Court of Appeals (CA) insofar as the civil
aspect of the case was concerned. The CA rendered judgment affirming the assailed
order of the RTC.
Issue: Whether respondents may be held civilly liable
Held: NO. Every person criminally liable for a felony is also civilly liable. The civil liability
of such person established in Articles 100, 102 and 103 of the Revised Penal Code
includes restitution, reparation of the damage caused, and indemnification for
consequential damages. When a criminal action is instituted, the civil action for the
recovery of civil liability arising from the offense charged shall be deemed instituted with
the criminal action unless the offended party waives the civil action, reserves the right to
institute it separately or institutes the civil action prior to the criminal action. With the
265

implied institution of the civil action in the criminal action, the two actions are merged
into one composite proceeding, with the criminal action predominating the civil.
The prime purpose of the criminal action is to punish the offender in order to deter him
and others from committing the same or similar offense, to isolate him from society, to
reform and rehabilitate him or, in general, to maintain social order. The sole purpose of
the civil action is the restitution, reparation or indemnification of the private offended
party for the damage or injury he sustained by reason of the delictual or felonious act of
the accused. While the prosecution must prove the guilt of the accused beyond
reasonable doubt for the crime charged, it is required to prove the cause of action of the
private complainant against the accused for damages and/or restitution.
The extinction of the penal action does not carry with it the extinction of the civil action.
However, the civil action based on delict shall be deemed extinguished if there is a
finding in a final judgment in the civil action that the act or omission from where the civil
liability may arise does not exist.
Insofar as the civil aspect of the case is concerned, the prosecution or the private
complainant is burdened to adduce preponderance of evidence or superior weight of
evidence. Although the evidence adduced by the plaintiff is stronger than that presented
by the defendant, he is not entitled to a judgment if his evidence is not sufficient to
sustain his cause of action. The plaintiff must rely on the strength of his own evidence
and not upon the weakness of that of the defendants.
In the present case, as held by the trial court and the CA, the prosecution failed to
adduce preponderant evidence to prove the facts on which the civil liability of the
respondents rest, i.e., that the petitioner has a cause of action against the respondents
for damages.
The petitioner even failed to adduce preponderance of evidence that either or both the
respondents hit the deceased with a blunt object or instrument, and, consequently, any
blunt object or instrument that might have been used by any or both of the respondents
in hitting the deceased.
In sum, the petitioner failed to adduce preponderance of evidence to prove a cause of
action for damages based on the deliberate acts alleged in the Information.
G.R. No. 143380, April 11, 2005
OLIMPIO PANGONOROM and METRO MANILA TRANSIT
CORPORATION, Petitioners, vs. PEOPLE OF THE PHILIPPINES, Respondents.
First Division, Carpio, J.:

266

Facts: Olimpio Pangonorom was charged with reckless imprudence resulting in


damage to property with multiple slight physical injuries. RTC rendered its Decision with
the following dispositive portion:
PREMISES CONSIDERED, the Court finds accused Olimpio Pangonorom guilty
beyond reasonable doubt of the crime of reckless imprudence resulting in multiple slight
physical injuries and sentences him to suffer an imprisonment of thirty (30) days
of arresto menor, to indemnify the offended parties of the damages incurred by their
Isuzu Gemini car in the sum of P42,600.00 and to reimburse the medical expenses of
Carlos R. Berba in the sum of P182.50, Amelia Berba in the sum of P217.50 and Mary
Berba in the sum of P45.00.
SO ORDERED.
Petitioners appealed the trial courts decision to the Court of Appeals, which affirmed the
Decision in toto.
Petitioners filed a motion for reconsideration of the assailed decision, stating, among
others that the Court of Appeals erred in not holding that the MMTC was not subsidiarily
liable for Olimpios civil liability in the instant case.
CA denied the motion for reconsideration. With the assailed decision having "amply
discussed, considered and ruled upon" the issues that petitioners raised in their motion
for reconsideration, the Court of Appeals held that there was no cogent reason for it to
reverse the assailed decision. The Court of Appeals also held that the MMTC was
already estopped in assailing the trial courts decision considering that the MMTC never
appealed the decision within the reglementary period.
Held: Petitioners fault the CA for having ruled that the MMTC is already estopped from
assailing the trial courts decision considering that the MMTC "never appealed the same
within the reglementary period."
SC noted that when petitioners filed their Notice of Appeal with the trial court on 8 March
1993, the MMTC already appealed the civil aspect of this case. Petitioners Notice of
Appeal states:
The ACCUSED and his employer, Metro Manila Transit Corporation, by their
undersigned counsel, unto this Honorable Court, most respectfully give notice that they
are appealing, as they hereby appeal, the Decision dated February 5, 1993, which was
received on February 23, 1993, to the Court of Appeals on the ground that the Decision
is contrary to the facts, law and settled jurisprudence.
Metro Manila Transit Corporation likewise interposes an appeal with respect to the civil
aspect of this case because of its subsidiary liability as employer of the accused under
the Revised Penal Code.
267

It is therefore not correct for the Court of Appeals to state that the MMTC failed to
appeal seasonably the issue of its alleged "non-subsidiary liability" as Olimpios
employer.
However, due diligence in the selection and supervision of employees is not a defense
in the present case. The law involved in the present case is Article 103 of the Revised
Penal Code, in relation to Articles 100 and 102 of the same Code, which reads thus:
Art. 103. Subsidiary civil liability of other persons. The subsidiary liability established
in the next preceding article shall also apply to employers, teachers, persons, and
corporations engaged in any kind of industry for felonies committed by their servants,
pupils, workmen, apprentices, or employees in the discharge of their duties.
Pursuant to Article 103, an employer may be subsidiarily liable for the employees civil
liability in a criminal action when there is adequate evidence establishing (1) that he is
indeed the employer of the convicted employee; (2) that he is engaged in some kind of
industry; (3) that the employee committed the offense in the discharge of his duties; and
(4) that the execution against the employee has not been satisfied due to insolvency.
The provisions of the Revised Penal Code on subsidiary liability Articles 102 and 103
are deemed written into the judgments in cases to which they are applicable. Thus, in
the dispositive portion of its decision, the trial court need not expressly pronounce the
subsidiary liability of the employer.
The subsidiary liability of the employer arises only after conviction of the employee in
the criminal action. In the present case, there exists an employer-employee relationship
between petitioners, the MMTC is engaged in the transportation industry, and Olimpio
has been adjudged guilty of a wrongful act and found to have committed the offense in
the discharge of his duties. However, there is no proof here of Olimpios insolvency. The
judgment of conviction against Olimpio has not attained finality. This being so, no writ of
execution can issue against him to satisfy his civil liability. Only after proof of the
accused-employees insolvency may the subsidiary liability of his employer be enforced.
In short, there is as yet no occasion to speak of enforcing the employers subsidiary civil
liability unless it appears that the accused-employees primary liability cannot in the first
instance be satisfied because of insolvency. This fact cannot be known until some time
after the verdict of conviction shall have become final. And even if it appearsprima
facie that execution against the employee cannot be satisfied, execution against the
employer will not issue as a matter of course. The procedure for the enforcement of a
judgment will have to be followed. Once the judgment of conviction against Olimpio
becomes final and executory, and after the writ of execution issued against him is
returned unsatisfied because of his insolvency, only then can a subsidiary writ of
execution be issued against the MMTC after a hearing set for that precise purpose. It is
still too early to hold the MMTC subsidiarily liable with its accused-employee
considering that there is no proof yet of Olimpios insolvency.
268

G.R. No. 147703, April 14, 2004


PHILIPPINE RABBIT BUS LINES, INC., petitioner, vs. PEOPLE OF THE
PHILIPPINES, respondent.
First Division, Panganiban, J.
Facts: On July 27, 1994, accused Napoleon Roman y Macadangdang] was found guilty
and convicted of the crime of reckless imprudence resulting to triple homicide, multiple
physical injuries and damage to property and was sentenced to suffer the penalty of
four (4) years, nine (9) months and eleven (11) days to six (6) years, and to pay
damages. The court further ruled that petitioner, in the event of the insolvency of
accused, shall be liable for the civil liabilities of the accused. Evidently, the judgment
against accused had become final and executory.
Accused jumped bail and remained at-large. Counsel for accused, hired and provided
by petitioner, filed a notice of appeal which was denied by the trial court. CA affirmed the
denial of the notice of appeal filed in behalf of accused. Simultaneously, on August 6,
1994, petitioner filed its notice of appeal from the judgment of the trial court. The trial
court gave due course thereto. On March 2, 1999, respondent/private prosecutor filed a
motion to dismiss. CA granted the motion to dismiss.
Issue: Whether or not an employer, who dutifully participated in the defense of its
accused-employee, may appeal the judgment of conviction independently of the
accused.
Held: NO. When the accused-employee absconds or jumps bail, the judgment meted
out becomes final and executory. The employer cannot defeat the finality of the
judgment by filing a notice of appeal on its own behalf in the guise of asking for a review
of its subsidiary civil liability. Both the primary civil liability of the accused-employee and
the subsidiary civil liability of the employer are carried in one single decision that has
become final and executory.
Both the accused and the prosecution may appeal a criminal case, but the government
may do so only if the accused would not thereby be placed in double jeopardy.
Well-established in our jurisdiction is the principle that the appellate court may, upon
motion or motu proprio, dismiss an appeal during its pendency if the accused jumps
bail. This rule is based on the rationale that appellants lose their standing in court when
they abscond. Unless they surrender or submit to the courts jurisdiction, they are
deemed to have waived their right to seek judicial relief. This doctrine applies not only to
the accused who jumps bail during the appeal, but also to one who does so during the
trial. The accused cannot be accorded the right to appeal unless they voluntarily submit
to the jurisdiction of the court or are otherwise arrested within 15 days from notice of the
judgment against them.

269

In this case, the accused-employee has escaped and refused to surrender to the proper
authorities; thus, he is deemed to have abandoned his appeal. Consequently, the
judgment against him has become final and executory.
Article 102 of the Revised Penal Code states the subsidiary civil liabilities of innkeepers.
The subsidiary liability applies to employers, according to Article 103.
Only the civil liability of the accused arising from the crime charged is deemed impliedly
instituted in a criminal action; that is, unless the offended party waives the civil action,
reserves the right to institute it separately, or institutes it prior to the criminal
action. Hence, the subsidiary civil liability of the employer under Article 103 of the
Revised Penal Code may be enforced by execution on the basis of the judgment of
conviction meted out to the employee.
It is clear that the 2000 Rules deleted the requirement of reserving independent civil
actions and allowed these to proceed separately from criminal actions. Thus, the civil
actions referred to in Articles 32, 33, 34 and 2176of the Civil Code shall remain
"separate, distinct and independent" of any criminal prosecution based on the same act.
What is deemed instituted in every criminal prosecution is the civil liability arising from
the crime or delict per se (civil liability ex delicto), but not those liabilities arising from
quasi-delicts, contracts or quasi-contracts.
Petitioner argues that, as an employer, it is considered a party to the criminal case and
is conclusively bound by the outcome thereof. Consequently, petitioner must be
accorded the right to pursue the case to its logical conclusion -- including the appeal.
The argument has no merit. Undisputedly, petitioner is not a direct party to the criminal
case, which was filed solely against Napoleon M. Roman, its employee.
The cases dealing with the subsidiary liability of employers uniformly declare that,
strictly speaking, they are not parties to the criminal cases instituted against their
employees. Although in substance and in effect, they have an interest therein, this fact
should be viewed in the light of their subsidiary liability. While they may assist their
employees to the extent of supplying the latters lawyers, as in the present case, the
former cannot act independently on their own behalf, but can only defend the accused.
If the present appeal is given course, the whole case against the accused-employee
becomes open to review. It thus follows that a penalty higher than that which has
already been imposed by the trial court may be meted out to him. Petitioners appeal
would thus violate his right against double jeopardy, since the judgment against him
could become subject to modification without his consent.
Moreover, within the meaning of the principles governing the prevailing criminal
procedure, the accused impliedly withdrew his appeal by jumping bail and thereby made
the judgment of the court below final.
270

In sum, petitioner has no right to appeal the criminal case against the accusedemployee; that by jumping bail, he has waived his right to appeal; and that the judgment
in the criminal case against him is now final.
As a matter of law, the subsidiary liability of petitioner now accrues.
Under Article 103 of the Revised Penal Code, employers are subsidiarily liable for the
adjudicated civil liabilities of their employees in the event of the latters insolvency. The
provisions of the Revised Penal Code on subsidiary liability -- Articles 102 and 103 -are deemed written into the judgments in the cases to which they are applicable. Thus,
in the dispositive portion of its decision, the trial court need not expressly pronounce the
subsidiary liability of the employer.
In the absence of any collusion between the accused-employee and the offended party,
the judgment of conviction should bind the person who is subsidiarily liable. In effect and
implication, the stigma of a criminal conviction surpasses mere civil liability.
To allow employers to dispute the civil liability fixed in a criminal case would enable
them to amend, nullify or defeat a final judgment rendered by a competent court. By the
same token, to allow them to appeal the final criminal conviction of their employees
without the latters consent would also result in improperly amending, nullifying or
defeating the judgment.
The decision convicting an employee in a criminal case is binding and conclusive upon
the employer not only with regard to the formers civil liability, but also with regard to its
amount. The liability of an employer cannot be separated from that of the employee.
Before the employers subsidiary liability is exacted, however, there must be adequate
evidence establishing that (1) they are indeed the employers of the convicted
employees; (2) that the former are engaged in some kind of industry; (3) that the crime
was committed by the employees in the discharge of their duties; and (4) that the
execution against the latter has not been satisfied due to insolvency.
The resolution of these issues need not be done in a separate civil action. But the
determination must be based on the evidence that the offended party and the employer
may fully and freely present. Such determination may be done in the same criminal
action in which the employees liability, criminal and civil, has been pronounced; and in a
hearing set for that precise purpose, with due notice to the employer, as part of the
proceedings for the execution of the judgment.
Just because the present petitioner participated in the defense of its accused-employee
does not mean that its liability has transformed its nature; its liability remains subsidiary.
Neither will its participation erase its subsidiary liability. The fact remains that since the
accused-employees conviction has attained finality, then the subsidiary liability of the
employer ipso facto attaches.
271

There is only one criminal case against the accused-employee. A finding of guilt has
both criminal and civil aspects. It is the height of absurdity for this single case to be final
as to the accused who jumped bail, but not as to an entity whose liability is dependent
upon the conviction of the former.
The subsidiary liability of petitioner is incidental to and dependent on the pecuniary civil
liability of the accused-employee. Since the civil liability of the latter has become final
and enforceable by reason of his flight, then the formers subsidiary civil liability has also
become immediately enforceable.
The employer becomes ipso facto subsidiarily liable upon the conviction of the
employee and upon proof of the latters insolvency, in the same way that acquittal wipes
out not only his primary civil liability, but also his employers subsidiary liability for his
criminal negligence.
It should be clear that only after proof of his insolvency may the subsidiary liability of
petitioner be enforced. It has been sufficiently proven that there exists an employeremployee relationship; that the employer is engaged in some kind of industry; and that
the employee has been adjudged guilty of the wrongful act and found to have
committed
the
offense
in
the
discharge
of
his
duties.
G.R. No. 113433, March 17, 2000
LUISITO P. BASILIO, petitioner, vs. THE COURT OF APPEALS, HON. JESUS G.
BERSAMIRA, and FE ADVINCULA, respondents.
Second Division, Quisumbing, J.
Facts: On July 23, 1987, Simplicio Pronebo was charged with the crime of reckless
imprudence resulting in damage to property with double homicide and double physical
injuries. The trial court rendered its judgment, which reads:
WHEREFORE, the court finds accused Simplicio Pronebo y Cruz guilty beyond
reasonable doubt of Reckless Imprudence resulting in the death of Danilo Advincula
and is hereby sentenced to suffer the indeterminate penalty of two (2) years and four (4)
months, as minimum to six (6) years of prision correccional, as maximum, and to
indemnify the heirs of Danilo Advincula P30,000.00 for the latter's death, P31,614.00, as
actual and compensatory damages. P2,000,000.00 for the loss of his earning capacity.
P150,000.00, as moral damages, and P30,000.00 as attorney's fees, plus the costs of
suit.
Thereafter, the accused filed an application for probation, so that the above judgment
became final and executory.

272

Pertinently, the trial court also found that at the time of the vehicular accident accused
Simplicio Pronebo was employed as the driver of the dump truck owned by petitioner
Luisito Basilio.
On March 27, 1991, petitioner Luisito Basilio filed with the trial court a "Special
Appearance and Motion for Reconsideration" praying that the judgment dated February
4, 1991, be reconsidered and set aside insofar as it affected him and subjected him to a
subsidiary liability for the civil aspect of the criminal case. The motion was denied for
lack of merit. Petitioner filed a Notice of Appeal on September 25, 1991.
On September 23, 1991, private respondent filed a Motion for Execution of the
subsidiary civil liability of petitioner Basilio.
On April 7, 1992, the trial court issued two separate Orders. One denied due course and
dismissed Basilio's appeal for having been filed beyond the reglementary period. The
other directed the issuance of a writ of execution against him for the enforcement and
satisfaction of the award of civil indemnity decreed in judgment on February 4, 1991.
Petitioner filed a petition for certiorari under Rule 65 with the Court of Appeals. The
Court of Appeals dismissed the petition. His motion for reconsideration was likewise
denied.
Issue: Whether the petitioner as employer may file a Motion for Reconsideration
concerning civil liability decreed in the judgment if he is not a party to the criminal case
Held: NO. The statutory basis for an employer's subsidiary liability is found in Article 103
of the Revised Penal Code. This liability is enforceable in the same criminal proceeding
where the award is made. However, before execution against an employer ensues,
there must be a determination, in a hearing set for the purpose of 1) the existence of an
employer-employee relationship; 2) that the employer is engaged in some kind of
industry; 3) that the employee is adjudged guilty of the wrongful act and found to have
committed the offense in the discharge of his duties (not necessarily any offense he
commits "while" in the discharge of such duties; and 4) that said employee is insolvent.
There are two instances when the existence of an employer-employee relationship of an
accused driver and the alleged vehicle owner may be determined. One, during the
criminal proceeding, and the other, during the proceeding for the execution of the
judgment. In both instances, petitioner should be given the opportunity to be heard,
which is the essence of due process.
Petitioner knew of the criminal case that was filed against accused because it was his
truck that was involved in the incident. Further, it was the insurance company, with
which his truck was insured, that provided the counsel for the accused, pursuant to the
stipulations in their contract. Petitioner did not intervene in the criminal proceedings,
despite knowledge, through counsel, that the prosecution adduced evidence to show
employer-employee relationship. With the convict's application for probation, the trial
273

court's judgment became final and executory. All told, it is our view that the lower court
did not err when it found that petitioner was not denied due process. He had all his
chances to intervene in the criminal proceedings, and prove that he was not the
employer of the accused, but he chooses not to intervene at the appropriate time.
Petitioner was also given the opportunity during the proceedings for the enforcement of
judgment. Even assuming that he was not properly notified of the hearing on the motion
for execution of subsidiary liability, he was asked by the trial court to make an opposition
thereto, which he did, where he properly alleged that there was no employer-employee
relationship between him and accused and that the latter was not discharging any
function in relation to his work at the time of the incident. In addition, counsel for private
respondent filed and duly served a manifestation praying for the grant of the motion for
execution. This was set for hearing on December 13, 1991. However, counsel for
petitioner did not appear. Consequently, the court ordered in open court that the matter
be submitted for resolution. It was only on January 6, 1992, that the petitioner's counsel
filed a counter-manifestation that belatedly attempted to contest the move of the private
prosecutor for the execution of the civil liability. Thus, on April 7, 1992, the trial court
issued the Order granting the motion for execution of the subsidiary liability.

Anti-fencing Law
G.R. No. 146584, July 12, 2004
ERNESTO FRANCISCO y SPENOCILLA, petitioner, vs. PEOPLE OF THE
PHILIPPINES, respondent.
Second Division, Callejo, Sr., J.:
Facts: The petitioner was charged of violating P.D. No. 1612 under the Information,
which alleged that he, with intent to gain for himself, bought, received, possessed and
acquired from Pacita Linghon y Liza, not the owner, several pieces of jewelry with the
total value of P655,000.00, belonging to Jovita Rodriguez y Cruz, which he knows, or
should be known to him, to have been derived from the proceeds of the crime of
robbery or theft.
Jovita Rodriguez was engaged in business as a general contractor. Macario Linghon
was one of her workers. She had several pieces of jewelry which were placed inside a
locked cabinet in a locked room in their main house. They hired Pacita Linghon,
Macarios sister, as one of their household helpers in February 1989. Sometime in May
1991, she left the employ of the Rodriguez family.
Pacita contacted her brother Macario and asked him to sell some pieces of jewelry. She
told Macario that a friend of hers owned the jewelry. Macario agreed. He went to the
shop of petitioner Ernesto "Erning" Francisco, which had a poster outside that said, "We
buy gold." In several instances, Macario sold the petitioner several pieces of jewelry.
274

On August 19, 1992, Jovita filed a complaint for theft against Pacita and her mother
Adoracion Linghon with the Counter-Intelligence Group of the PNP. She averred that
Pacita had stolen the pieces of jewelry, and that she and her mother Adoracion
disposed of the same. When invited by the police, Pacita gave a sworn statement
pointing to a certain Mang Erning as the person to whom she sold Jovitas jewelry.
Pacita accompanied the police to the shop and pointed to the petitioner as the "Mang
Erning".
Pacita was charged with qualified theft in the Regional Trial Court of San Mateo, Rizal,
Branch 76. Adoracion was also charged with violating P.D. No. 1612 (Anti-Fencing
Law). The cases were consolidated and jointly tried.
Meanwhile, Jovita succeeded in convincing Macario to testify against the petitioner,
assuring him that he would not be prosecuted for violation of P.D. No. 1612. Macario
agreed to testify against the petitioner.
Petitioner was then charged with a violation of P.D. No. 1612.
On August 20, 1993, judgment was rendered by the RTC of San Mateo, Rizal, Branch
76 finding Pacita guilty of theft and Adoracion guilty of fencing under P.D. No. 1612,
beyond reasonable doubt.
The petitioner denied transacting with Pacita regarding Jovitas missing jewels. He also
averred that he had no transaction with Macario of whatever nature. The court rendered
judgment finding the petitioner guilty beyond reasonable doubt of violating P.D. No.
1612. On appeal, the Court of Appeals affirmed his conviction.
Issue: Whether petitioner (accused) should be held liable for violating PD 1612
Held: NO. The essential elements of the crime of fencing are as follows: (1) a crime of
robbery or theft has been committed; (2) the accused, who is not a principal or
accomplice in the commission of the crime of robbery or theft, buys, receives,
possesses, keeps, acquires, conceals, sells or disposes, or buys and sells, or in any
manner deals in any article, item, object or anything of value, which has been derived
from the proceeds of the crime of robbery or theft; (3) the accused knew or should have
shown that the said article, item, object or anything of value has been derived from the
proceeds of the crime of robbery or theft; and, (4) there is, on the part of the accused,
intent to gain for himself or for another.
Fencing is malum prohibitum, and P.D. No. 1612 creates a prima facie presumption of
fencing from evidence of possession by the accused of any good, article, item, object or
anything of value which has been the subject of robbery or theft, and prescribes a
higher penalty based on the value of the property. The stolen property subject of the
charge is not indispensable to prove fencing. It is merely corroborative of the
testimonies and other evidence adduced by the prosecution to prove the crime of
fencing.
275

The prosecution mustered the requisite quantum of evidence, on the basis of the
testimony of Jovita, that Pacita stole the subject jewelry from the locked cabinet in the
main house of her then employer. Jovita testified on her ownership of the jewelry and
the loss thereof, and narrated that Pacita had access to the cabinet containing the
pieces of jewelry. However, the decision of the RTC of Rizal, Branch 76, in Criminal
Case No. 2005 convicting Pacita of theft does not constitute proof against him in this
case, that Pacita had, indeed, stolen the jewelry. There is no showing that the said
decision in Criminal Case No. 2005 was already final and executory when the trial court
rendered its decision in the instant case.
On the second element of the crime, the only evidence of the prosecution to prove that
the petitioner purchased the jewelry from Macario and Pacita are the following: the
testimony and affidavit of PO1 Roldan, Jr.; and, the testimony of Macario during the
preliminary investigation and trial in the court a quo. However, testimony of Macario is
dubious, hence, barren of probative weight. Macario admitted when he testified in the
court a quo that his testimony during the preliminary investigation in Criminal Case No.
92-13841 and his testimony in the court a quo were inconsistent.
On the third element, SC stated that assuming, for the nonce, that the petitioner
purchased the said jewelry from Macario, there is no evidence on record that the
petitioner knew that they were stolen. Significantly, even Macario did not know that the
jewelry was stolen. He testified that his sister Pacita told him before he sold the jewelry
to the petitioner that they belonged to a friend of hers.
The prosecution cannot even validly argue that the petitioner should have known which
pieces of jewelry were stolen, considering that Macario was selling the same
for P50,000 when the said pieces stolen from Jovita were alleged to be worth P655,000.
This is so because the prosecution failed to adduce sufficient competent evidence to
prove the value of the said stolen articles. The prosecution relied solely on the bare and
uncorroborated testimony of Jovita, that they were worth P655,000. When required by
the petitioner, through counsel, to bring to the court any receipts reflecting the price of
the pieces of jewelry to show that she purchased the same, Jovita answered that she
had no such receipts. An ordinary witness cannot establish the value of jewelry, nor may
the courts take judicial notice of the value of the same.
It bears stressing that, in the absence of direct evidence that the accused had
knowledge that the jewelry was stolen, the prosecution is burdened to prove facts and
circumstances from which it can be concluded that the accused should have known that
the property sold to him were stolen. This requirement serves two basic purposes: (a) to
prove one of the elements of the crime of fencing; and, (b) to enable the trial court to
determine the imposable penalty for the crime, since the penalty depends on the value
of the property; otherwise, the court will fix the value of the property at P5.00,
conformably to the ruling in People v. Dator.

276

G.R. No. 134298, August 26, 1999


RAMON C. TAN, petitioner, vs. PEOPLE OF THE PHILIPPINES, respondent.
First Division, Pardo, J.
Facts: Complainant Rosita Lim is the proprietor of Bueno Metal Industries, located at
301 Jose Abad Santos St., Tondo, Manila, engaged in the business of manufacturing
propellers or spare parts for boats. Manuelito Mendez was one of the employees
working for her. Sometime in February 1991, Manuelito Mendez left the employ of the
company. Complainant Lim noticed that some of the welding rods, propellers and boat
spare parts, such as bronze and stainless propellers and brass screws were missing.
She conducted an inventory and discovered that propellers and stocks valued at
P48,000.00, more or less, were missing. Complainant Rosita Lim informed Victor Sy,
uncle of Manuelito Mendez, of the loss. Subsequently, Manuelito Mendez was arrested
in the Visayas and he admitted that he and his companion Gaudencio Dayop stole from
the complainant's warehouse some boat spare parts such as bronze and stainless
propellers and brass screws. Manuelito Mendez asked the complainant's forgiveness.
He pointed to petitioner Ramon C. Tan as the one who bought the stolen items and who
paid the amount of P13,000.00, in cash to Mendez and Dayop, and they split the
amount with one another. Complainant did not file a case against Manuelito Mendez
and Gaudencio Dayop.
On relation of complainant Lim, an Assistant City Prosecutor of Manila filed with the
Regional Trial Court, Manila, Branch 19, an information against petitioner charging him
with violation of Presidential Decree No. 1612 (Anti-Fencing Law).
The accused denied having bought the stolen spare parts worth P48,000.00 for he
never talked nor met Manuelito Mendez, the confessed thief. The two (2) receipts
presented by Mrs. Lim are not under her name and the other two (2) are under the
name of William Tan, the husband, all in all amounting to P18,000.00. Besides, the
incident was not reported to the police. He likewise denied having talked to Manuelito
Mendez over the phone on the day of the delivery of the stolen items and could not
have accepted the said items personally.
The trial court rendered decision finding the accused guilty beyond reasonable doubt of
violating the Anti-Fencing Law of 1979, otherwise known as Presidential Decree No.
1612. On appeal, CA affirmed.
Issue: Whether the prosecution has successfully established the elements of fencing as
against petitioner
Held: NO. "Fencing, as defined in Section 2 of P.D. No. 1612 is "the act of any person
who, with intent to gain for himself or for another, shall buy, receive, possess, keep,
acquire, conceal, sell or dispose of, or shall buy and sell, or in any manner deal in any
article, item, object or anything of value which he knows, or should be known to him, to
have been derived from the proceeds of the crime of robbery or theft."
277

"Robbery is the taking of personal property belonging to another, with intent to gain, by
means of violence against or intimidation of any person, or using force upon things."
The crime of theft is committed if the taking is without violence against or intimidation of
persons nor force upon things.
"The law on fencing does not require the accused to have participated in the criminal
design to commit, or to have been in any wise involved in the commission of, the crime
of robbery or theft."
Before the enactment of P.D. No. 1612 in 1979, the fence could only be prosecuted as
an accessory after the fact of robbery or theft, as the term is defined in Article 19 of the
Revised Penal Code, but the penalty was light as it was two (2) degrees lower than that
prescribed for the principal.
P.D. No. 1612 was enacted to "impose heavy penalties on persons who profit by the
effects of the crimes of robbery and theft." Evidently, the accessory in the crimes of
robbery and theft could be prosecuted as such under the Revised Penal Code or under
P.D. No. 1612. However, in the latter case, the accused ceases to be a mere accessory
but becomes a principal in the crime of fencing. Otherwise stated, the crimes of robbery
and theft, on the one hand, and fencing, on the other, are separate and distinct
offenses. The State may thus choose to prosecute him either under the Revised Penal
Code or P.D. No. 1612, although the preference for the latter would seem inevitable
considering that fencing is malum prohibitum, and P.D. No. 1612 creates a presumption
of fencing and prescribes a higher penalty based on the value of the property.
The essential elements of the crime of fencing are as follows:
1. A crime of robbery or theft has been committed;
2. The accused, who is not a principal or accomplice in the commission of the
crime of robbery or theft, buys, receives, possesses, keeps, acquires, conceals,
sells or disposes, or buys and sells, or in any manner deals in any article, item,
object or anything of value, which has been derived from the proceeds of the said
crime;
3. The accused knows or should have known that the said article, item, object or
anything of value has been derived from the proceeds of the crime of robbery or
theft; and
4. There is on the part of the accused, intent to gain for himself or for another.
Complainant Rosita Lim testified that she lost certain items and Manuelito Mendez
confessed that he stole those items and sold them to the accused. However, Rosita Lim
never reported the theft or even loss to the police. She admitted that after Manuelito
Mendez, her former employee, confessed to the unlawful taking of the items, she
forgave him, and did not prosecute him. Theft is a public crime. It can be prosecuted de
oficio, or even without a private complainant, but it cannot be without a victim. As
complainant Rosita Lim reported no loss, we cannot hold for certain that there was
278

committed a crime of theft. Thus, the first element of the crime of fencing is absent, that
is, crime of robbery or theft has been committed.
There was no sufficient proof of the unlawful taking of another's property. True, witness
Mendez admitted in an extra-judicial confession that he sold the boat parts he had
pilfered from complainant to petitioner. However, an admission or confession
acknowledging guilt of an offense may be given in evidence only against the person
admitting or confessing. Even on this, if given extra-judicially, the confessant must have
the assistance of counsel; otherwise, the admission would be inadmissible in evidence
against the person so admitting. Here, the extra-judicial confession of witness Mendez
was not given with the assistance of counsel, hence, inadmissible against the witness.
Neither may such extra-judicial confession be considered evidence against
accused. There must be corroboration by evidence of corpus delicti to sustain a finding
of guilt. Corpus delicti means the "body or substance of the crime, and, in its primary
sense, refers to the fact that the crime has been actually committed." In theft, corpus
delicti has two elements, namely: (1) that the property was lost by the owner, and (2)
that it was lost by felonious taking. In this case, the theft was not proved because
complainant Rosita Lim did not complain to the public authorities of the felonious taking
of her property. She sought out her former employee Manuelito Mendez, who confessed
that he stole certain articles from the warehouse of the complainant and sold them to
petitioner. Such confession is insufficient to convict, without evidence of corpus delicti.
What is more, there was no showing at all that the accused knew or should have known
that the very stolen articles were the ones sold him. "One is deemed to know a
particular fact if he has the cognizance, consciousness or awareness thereof, or is
aware of the existence of something, or has the acquaintance with facts, or if he has
something within the mind's grasp with certitude and clarity. When knowledge of the
existence of a particular fact is an element of an offense, such knowledge is established
if a person is aware of a high probability of its existence unless he actually believes that
it does not exist. On the other hand, the words "should know" denote the fact that a
person of reasonable prudence and intelligence would ascertain the fact in performance
of his duty to another or would govern his conduct upon assumption that such fact
exists. Knowledge refers to a mental state of awareness about a fact. Since the court
cannot penetrate the mind of an accused and state with certainty what is contained
therein, it must determine such knowledge with care from the overt acts of that
person. And given two equally plausible states of cognition or mental awareness, the
court should choose the one which sustains the constitutional presumption of
innocence."
Without petitioner knowing that he acquired stolen articles, he can not be guilty of
"fencing".
Indeterminate Sentence Law (Act No. 4103)

279

PEOPLE OF THE PHILIPPINES, appellee,


vs. BETH TEMPORADA, appellant.
G.R. No. 173473| December 17, 2008 EN BANC
YNARES-SANTIAGO, J.:

FACTS: From September 2001 to January 2002, accused Rosemarie "Baby" Robles,
Bernadette Miranda, Nenita Catacotan and Jojo Resco and appellant Beth Temporada,
all employees of the Alternative Travel and Tours Corporation (ATTC), recruited and
promised overseas employment, for a fee, to complainants Rogelio Legaspi, Jr. as
technician in Singapore, and Soledad Atle, Luz Minkay, Evelyn Estacio and Dennis
Dimaano as factory workers in Hongkong. The accused and appellant were then
holding office at Dela Rosa Street, Makati City but eventually transferred business to
Discovery Plaza, Ermita, Manila. After complainants had submitted all the requirements
consisting of their respective application forms, passports, NBI clearances and medical
certificates, the accused and appellant, on different dates, collected and received from
them placement fees in various amounts, viz: a) from Rogelio Legaspi, Jr. 57,600.00;
b) from Dennis Dimaano P66,520.00; c) from Evelyn Estacio P88,520.00; d) from
Soledad Atle P69,520.00 and e) from Luz Minkay P69,520.00. As none of them was
able to leave nor recover the amounts they had paid, complainant lodged separate
criminal complaints against accused and appellant before the City Prosecutor of Manila.
On November 29, 2002, Assistant City Prosecutor Restituto Mangalindan, Jr. filed six
(6) Informations against the accused and appellant, one for Illegal Recruitment in Large
Scale under Article 38 (a) of the Labor Code as amended, and the rest for five (5)
counts of estafa under Article 315 paragraph 2 (a) of the Revised Penal Code.
RTC Ruling:
WHEREFORE, the prosecution having established the GUILT of accused Beth
Temporada BEYOND REASONABLE DOUBT, judgment is hereby rendered
CONVICTING the said accused, as principal of the offenses charged and she is
sentenced to suffer the penalty of LIFE IMPRISONMENT and a fine of Five Hundred
Thousand Pesos (P500,000.00) for illegal recruitment; and the indeterminate penalty of
four (4) years and two (2) months of prision correctional as minimum, to nine (9) years
and one (1) day of prision mayor, as maximum for the estafa committed against
complainant Rogelio A. Legaspi, Jr.; the indeterminate penalty of four (4) years and two
(2) months of prision correctional as minimum to ten (10) years and one day of prision
mayor as maximum each for the estafas committed against complainants, Dennis
Dimaano, Soledad B. Atte and Luz T. Minkay; and the indeterminate penalty of four (4)
years and two (2) months of prision correctional as minimum, to eleven (11) years and
one (1) day of prision mayor as maximum for theestafa committed against Evelyn
Estacio.

280

In accordance with the Courts ruling in People v. Mateo, this case was referred to the
CA for intermediate review. On February 24, 2006, the CA affirmed with modification the
Decision of the RTC:
WHEREFORE, with MODIFICATION to the effect that in Criminal Cases Nos. 02208373, 02-208375, & 02-208376, appellant is sentenced to suffer the indeterminate
penalty of six (6) years of prision correccional maximum, as minimum, to ten (10) years
and one (1) day of prision mayor maximum, as maximum; and in Criminal Case No. 02208374, she is sentenced to suffer the indeterminate penalty of eight (8) years and one
(1) day of prision mayor medium, as minimum, to twelve (12) years and one (1) day
of reclusion temporal minimum, as maximum, the appealed decision is AFFIRMED in all
other respects.
ISSUE: Whether CA erroneously computed the indeterminate penalties.
HELD: YES. While we affirm the conviction for the five (5) counts of estafa, we find,
however, that the CA erroneously computed the indeterminate penalties therefor. The
CA deviated from the doctrine laid down in People v. Gabres; hence its decision should
be reversed with respect to the indeterminate penalties it imposed. The reversal of the
appellate courts Decision on this point does not, however, wholly reinstate the
indeterminate penalties imposed by the trial court because the maximum terms, as
determined by the latter, were erroneously computed and must necessarily be rectified.
The prescribed penalty for estafa under Article 315, par. 2(d) of the RPC, when the
amount defrauded exceeds P22,000.00, is prisin correccional maximum to prisin
mayor minimum. The minimum term is taken from the penalty next lower or anywhere
within prisin correccional minimum and medium (i.e., from 6 months and 1 day to 4
years and 2 months). Consequently, the RTC correctly fixed the minimum term for the
five estafa cases at 4 years and 2 months of prisin correccional since this is within the
range of prisin correccional minimum and medium.
On the other hand, the maximum term is taken from the prescribed penalty of prisin
correccional maximum toprisin mayor minimum in its maximum period, adding 1 year
of imprisonment for every P10,000.00 in excess of P22,000.00, provided that the total
penalty shall not exceed 20 years. However, the maximum period of the prescribed
penalty of prisin correccional maximum to prisin mayor minimum is not prisin
mayor minimum as apparently assumed by the RTC. To compute the maximum period
of the prescribed penalty, prisin correccionalmaximum to prisin mayor minimum
should be divided into three equal portions of time each of which portion shall be
deemed to form one period in accordance with Article 65 of the RPC. Following this
procedure, the maximum period of prisin correccional maximum to prisin
mayor minimum is from 6 years, 8 months and 21 days to 8 years. The incremental
penalty, when proper, shall thus be added to anywhere from 6 years, 8 months and 21
days to 8 years, at the discretion of the court.

281

G.R. No. 158797, July 29, 2005


PEOPLE OF THE PHILIPPINES, Appellee, vs. ELPIDIO ENRIQUEZ, JR. and
EMILIANO ENRIQUEZ, Appellants.
Second Division, Puno, J.:

Facts: Elpidio Enriquez, Jr. and Emiliano Enriquez were convicted of kidnapping by the
Regional Trial Court (RTC) of Cavite City, Branch 16, and each was sentenced to suffer
an indeterminate prison term of seventeen (17) years, four (4) months and one (1) day
of reclusion temporal as minimum to reclusion perpetua as maximum. They appealed to
the Court of Appeals which not only affirmed their convictions but imposed upon each of
the appellants the penalty of reclusion perpetua. The Court of Appeals refrained from
entering judgment and certified the case to the Supreme Court pursuant to the second
paragraph of Sec. 13 of the Revised Rules of Criminal Procedure.

Held: (as to the penalty)

The 1987 Constitution prohibits the imposition of the death penalty unless for
compelling reasons involving heinous crimes, Congress provides for it. Republic Act No.
7659 which classified kidnapping as a heinous crime punishable by death took effect on
December 31, 1993. The crime at bar was committed in 1985. We have ruled that R.A.
No. 7659 cannot be applied to a crime that transpired prior to its effectivity under the
principle of non-retroactivity of penal laws which are unfavorable to the
accused. Consequently, reclusion perpetua is the only penalty that can be imposed
against the appellants. As correctly argued by the Solicitor General, Act No. 4103,
otherwise known as the Indeterminate Sentence Law, cannot be applied in the case of
appellants considering the proscription in Sec. 2 thereof, viz:

282

This Act shall not apply to persons convicted of offenses punished with death penalty or
life-imprisonment x x x x

Indeed, in People v. Asturias, Serrano v. Court of Appeals, People v. Lampaza


and People v. Tan, SC in effect equated the penalty of reclusion perpetua as
synonymous to life-imprisonment for purposes of the Indeterminate Sentence Law, and
ruled that the latter law does not apply to persons convicted of offenses punishable with
the said penalty.

283

G.R. No. 132376, April 11, 2002


PEOPLE OF THE PHILIPPINES, plaintiff-appellee, vs. SAMINA ANGELES Y
CALMA, accused-appellant.
First Division, Ynares-Santiago, J.
Facts: Accused-appellant Samina Angeles y Calma was charged with four (4) counts of
estafa and one (1) count of illegal recruitment.
After trial on the merits, the trial court found accused-appellant guilty of illegal
recruitment and four (4) counts of estafa and correspondingly sentenced her as follows:
WHEREFORE, in view of the aforementioned premises the accused SAMINA
ANGELES is hereby declared:
In Criminal Case No. 94-140489 for the crime of Illegal Recruitment, GUILTY (Art. 38
Labor Code) and is hereby sentenced to suffer the penalty of life imprisonment and a
fine of One Hundred Thousand Pesos (P100,000.00).
In Criminal Case No. 94-140485 for the crime of Estafa the accused is hereby declared
GUILTY and is hereby sentenced to suffer the penalty of from twelve (12) years and one
(1) day to twenty (20) years. In addition the accused is ordered to reimburse the amount
of One hundred seven thousand pesos (P107,000.00) to complainant Maria Tolosa de
Sardea. With costs.
In Criminal Case No. 94-140486 for the crime of Estafa the accused is hereby declared
GUILTY and is hereby sentenced to suffer the penalty of from twelve (12) years and one
(1) day to twenty (20) years. In addition the accused is ordered to reimburse the amount
of One hundred ninety thousand pesos (P190,000.00) to complainant Marceliano T.
Tolosa. With costs.
In Criminal Case No. 94-140487 for the crime of Estafa the accused is hereby declared
GUILTY and is hereby sentenced to suffer the penalty of from twelve (12) years and one
(1) day to twenty (20) years. In addition the accused is ordered to reimburse the amount
of Two thousand five hundred fifty dollars (US$2,550.00) or its equivalent in Philippine
currency of Sixty one thousand two hundred pesos (P61,200.00), to complainant Precila
P. Olpindo. With Costs.
In Criminal Case No. 94-140488 for the crime of Estafa the accused is hereby declared
GUILTY and is hereby sentenced to suffer the penalty of from twelve (12) years and one
(1) day to twenty (20) years. In addition the accused is ordered to reimburse the amount
of Two thousand five hundred fifty dollars (US$2,550.00) or its equivalent in Philippine
Currency of Sixty one thousand two hundred pesos (P61,200.00) to complainant Vilma
S. Brina. With costs.

284

Accused-appellant appealed to the Supreme Court, arguing that the prosecution failed
to prove her guilt for estafa and illegal recruitment by proof beyond reasonable doubt.
Held: SC ruled that accused-appellant Samina Angeles cannot be lawfully convicted of
illegal recruitment, however, she is liable for 4 counts of Estafa.
Article 315 of the Revised Penal Code imposes the penalty of prision correccional in its
maximum period to prision mayor in its minimum period, if the amount of the fraud is
over P12,000.00 but does not exceed P22,000.00; if the amount exceeds P22,000.00,
the penalty provided shall be imposed in its maximum period, adding one year for each
additional P10,000.00. However, the total penalty which may be imposed shall not
exceed twenty years.
In People v. Ordono, it was held:
Under the Indeterminate Sentence Law, the maximum term of the penalty shall be "that
which, in view of the attending circumstances, could be properly imposed" under the
Revised Penal Code, and the minimum shall be "within the range of the penalty next
lower to that prescribed for the offense." The penalty next lower should be based on the
penalty prescribed by the Code for the offense, without first considering any modifying
circumstance attendant to the commission of the crime. The determination of the
minimum penalty is left by law to the sound discretion of the court and it can be
anywhere within the range of the penalty next lower without any reference to the periods
into which it might be subdivided. The modifying circumstances are considered only in
the imposition of the maximum term of the indeterminate sentence.
Thus, in Criminal Case No. 94-140485, Maria Tolosa testified that she gave
P107,000.00, P46,000.00 and US$1,500.00 to Samina Angeles. The Information,
however, alleged that Maria gave only P107,000.00. Samina Angeles could therefore be
held accountable for only that amount.
In Criminal Case No. 94-140486, Marceliano testified that he gave P100,000.00,
P46,000.00 and US$1,500.00 to Samina Angeles. The Information however alleged that
Marceliano gave only a total of P190,000.00; hence that is the only amount that Samina
Angeles could be held accountable for.
In Criminal Case No. 94-140487, Precila testified that she gave US$2,550.00 to Samina
Angeles. The Information alleged that the equivalent amount thereof in Philippine
Currency is P61,200.00. Samina Angeles is therefore criminally liable for
P61,200.00.Complainant Vilma Brina did not appear in court to testify. Thus, the
damage in the amount of $2,550.00 alleged in Criminal Case No. 94-140488 was not
proved.

G.R. No. 128513, December 27, 2000


285

EMMA OFFEMARIA MARCELO, petitioner, vs. COURT OF APPEALS and THE


PEOPLE OF THE PHILIPPINES, respondents.
Second Division, De Leon, Jr., J.
Facts: An information for the crime of Estafa was filed against Angelica C.J. Offemaria,
Emma Offemaria Marcelo, Nemia Magalit Diu, Leila Briones and Flor Nocete. When
arraigned, petitioner Emma Marcelo and one of her co-accused, Nemia Magalit Diu,
pleaded "Not guilty" to the crime charged. The other accused, Angelica C.J. Offemaria,
Leila Briones and Flor Nocete, remained at-large.
Following trial, the lower court found petitioner Marcelo and Diu guilty beyond
reasonable doubt of the crime of estafa as charged and applying the Indeterminate
Sentence Law, sentenced them to suffer an imprisonment each of FOUR (4) YEARS
and TWO (2) MONTHS, as minimum to SIX (6) YEARS, EIGHT (8) MONTHS AND
TWENTY-ONE (21) DAYS as maximum, with the accessory penalties of the law; to pay
private complainant CLARITA MOSQUERA the amount of P27,925.00 in solidum with
legal rate of interest per annum reckoned from date of Information until fully paid, plus
25% of the sum of P27,925.00 for and as Attorneys fees, without subsidiary
imprisonment in case of insolvency; and finally to pay their proportionate share of the
costs of the proceedings.
From the judgment of conviction, only Marcelo appealed to the Court of Appeals, which
affirmed the trial courts decision.
Held: The penalty for the crime of estafa is prescribed by paragraphs 1 to 4 of Article
315 of the Revised Penal Code as follows:
"1st. The penalty of prision correccional in its maximum period to prision mayor in its
minimum period, if the amount of the fraud is over 12,000 pesos but does not exceed
22,000 pesos, and if such amount exceeds the latter sum, the penalty provided in this
paragraph shall be imposed in its maximum period, adding one year for each additional
10,000 pesos; but the total penalty which may be imposed shall not exceed twenty
years. In such cases, and in connection with the accessory penalties which may be
imposed and for the purpose of the other provisions of this Code, the penalty shall be
termed prision mayor or reclusion temporal, as the case may be;
"2nd. The penalty of prision correccional in its minimum and medium periods, if the
amount of the fraud is over 6,000 pesos but does not exceed 12,000 pesos;
"3rd. The penalty of arresto mayor in its maximum period to prision correccional in its
minimum period if such amount is over 200 pesos but does not exceed 6,000 pesos;
and

286

"4th. By arresto mayor in its maximum period, if such amount does not exceed 200
pesos, provides that in the four cases mentioned, the fraud be committed by any of the
following means: x x x.1wphi1
In the case of People v. Gabres, the Court had occasion to so state that
Under the Indeterminate Sentence Law, the maximum term of the penalty shall be "that
which, in view of the attending circumstances, could be properly imposed" under the
Revised Penal Code, and the minimum shall be "within the range of the penalty next
lower to that prescribed" for the offense. The penalty next lower should be based on the
penalty prescribed by the Code for the offense, without first considering any modifying
circumstance attendant to the commission of the crime. The determination of the
minimum penalty is left by law to the sound discretion of the court and it can be
anywhere within the range of the penalty next lower without any reference to the periods
into which it might be subdivided. The modifying circumstances are considered only in
the imposition of the maximum term of the indeterminate sentence.
"The fact that the amounts involved in the instant case exceed P22,000.00 should not
be considered in the initial determination of the indeterminate penalty; instead, the
matter should be so taken as analogous to modifying circumstances in the imposition of
the maximum term of the full indeterminate sentence. This interpretation of the law
accords with the rule that penal laws should be construed in favor of the accused. Since
the penalty is prision correccional maximum to prision mayor minimum, the penalty next
lower would then be prision correccionalminimum to medium. Thus, the minimum term
of the indeterminate sentence should be anywhere within six (6) months and one
(1) day to four (4) years and two (2) months x x x.
Probation Law (PD 968, as amended)
G.R. No. 182748, December 13, 2011
ARNEL COLINARES, Petitioner, vs. PEOPLE OF THE PHILIPPINES, Respondent.
EN BANC, ABAD, J.
Facts: Accused Arnel Colinares was charged with frustrated homicide before the
Regional Trial Court (RTC) of San Jose, Camarines Sur, in Criminal Case T-2213.
On July 1, 2005 the RTC rendered judgment, finding Arnel guilty beyond reasonable
doubt of frustrated homicide and sentenced him to suffer imprisonment from two years
and four months of prision correccional, as minimum, to six years and one day of prision
mayor, as maximum. Since the maximum probationable imprisonment under the law
was only up to six years, Arnel did not qualify for probation.
Arnel appealed to the Court of Appeals, invoking self-defense and, alternatively, seeking
conviction for the lesser crime of attempted homicide with the consequent reduction of
287

the penalty imposed on him. The CA entirely affirmed the RTC decision but deleted the
award for lost income in the absence of evidence to support it. Not satisfied, Arnel came
to the Supreme Court on petition for review.
In the course of its deliberation on the case, the Supreme Court required Arnel and the
Solicitor General to submit their respective positions on whether or not, assuming Arnel
committed only the lesser crime of attempted homicide with its imposable penalty of
imprisonment of four months of arresto mayor, as minimum, to two years and four
months of prision correccional, as maximum, he could still apply for probation upon
remand of the case to the trial court.
Both complied with Arnel taking the position that he should be entitled to apply for
probation in case the Court metes out a new penalty on him that makes his offense
probationable. The language and spirit of the probation law warrants such a stand. The
Solicitor General, on the other hand, argues that under the Probation Law no application
for probation can be entertained once the accused has perfected his appeal from the
judgment of conviction.
Issue: Given a finding that Arnel is entitled to conviction for a lower offense and a
reduced probationable penalty, whether or not he may still apply for probation on
remand of the case to the trial court.
Held: Yes. Ordinarily, Arnel would no longer be entitled to apply for probation, he having
appealed from the judgment of the RTC convicting him for frustrated homicide.
But, the Court found Arnel guilty only of the lesser crime of attempted homicide and held
that the maximum of the penalty imposed on him should be lowered to imprisonment of
four months of arresto mayor, as minimum, to two years and four months of prision
correccional, as maximum. With this new penalty, it would be but fair to allow him the
right to apply for probation upon remand of the case to the RTC.
While it is true that probation is a mere privilege, the point is not that Arnel has the right
to such privilege; he certainly does not have. What he has is the right to apply for that
privilege. If the Court allows him to apply for probation because of the lowered penalty, it
is still up to the trial judge to decide whether or not to grant him the privilege of
probation, taking into account the full circumstances of his case.
Under the probation law the accused who appeals "from the judgment of conviction" is
disqualified from availing himself of the benefits of probation. But, as it happens, two
judgments of conviction have been meted out to Arnel: one, a conviction for frustrated
homicide by the regional trial court, now set aside; and,two, a conviction for attempted
homicide by the Supreme Court.
If the Court chooses to go by the dissenting opinions hard position, it will apply the
probation law on Arnel based on the trial courts annulled judgment against him. He will
not be entitled to probation because of the severe penalty that such judgment imposed
288

on him. More, the Supreme Courts judgment of conviction for a lesser offense and a
lighter penalty will also have to bend over to the trial courts judgmenteven if this has
been found in error. And, worse, Arnel will now also be made to pay for the trial courts
erroneous judgment with the forfeiture of his right to apply for probation.
Arnel did not appeal from a judgment that would have allowed him to apply for
probation. He did not have a choice between appeal and probation. The stiff penalty that
the trial court imposed on him denied him that choice. Thus, a ruling that would allow
Arnel to now seek probation under this Courts greatly diminished penalty will not dilute
the sound ruling in Francisco vs. CA. It remains that those who will appeal from
judgments of conviction, when they have the option to try for probation, forfeit their right
to apply for that privilege.
Besides, in appealing his case, Arnel raised the issue of correctness of the penalty
imposed on him. He claimed that the evidence at best warranted his conviction only for
attempted, not frustrated, homicide, which crime called for a probationable penalty. In a
way, therefore, Arnel sought from the beginning to bring down the penalty to the level
where the law would allow him to apply for probation.
In a real sense, the Courts finding that Arnel was guilty, not of frustrated homicide, but
only of attempted homicide, is an original conviction that for the first time imposes on
him a probationable penalty.
The Probation Law never intended to deny an accused his right to probation through no
fault of his. The underlying philosophy of probation is one of liberality towards the
accused.
URBANO M. MORENO, Petitioner,
vs. COMMISSION ON ELECTIONS and NORMA L. MEJES, CHICONAZARIO, Respondents.
G.R. No. 168550 August 10, 2006 EN BANC
TINGA, J.:
FACTS: Norma L. Mejes (Mejes) filed a petition to disqualify Moreno from running for
Punong Barangay on the ground that the latter was convicted by final judgment of the
crime of Arbitrary Detention and was sentenced to suffer imprisonment of Four (4)
Months and One (1) Day to Two (2) Years and Four (4) Months by the Regional Trial
Court, Branch 28 of Catbalogan, Samar on August 27, 1998.
Moreno filed an answer averring that the petition states no cause of action because he
was already granted probation. Allegedly, following the case of Baclayon v. Mutia, 4 the
imposition of the sentence of imprisonment, as well as the accessory penalties, was
thereby suspended. Moreno also argued that under Sec. 16 of the Probation Law of
1976 (Probation Law), the final discharge of the probation shall operate to restore to him
all civil rights lost or suspended as a result of his conviction and to fully discharge his
289

liability for any fine imposed. The order of the trial court dated December 18, 2000
allegedly terminated his probation and restored to him all the civil rights he lost as a
result of his conviction, including the right to vote and be voted for in the July 15, 2002
elections.
In this petition, Moreno argues that the disqualification under the Local Government
Code applies only to those who have served their sentence and not to probationers
because the latter do not serve the adjudged sentence. The Probation Law should
allegedly be read as an exception to the Local Government Code because it is a special
law which applies only to probationers. Further, even assuming that he is disqualified,
his subsequent election as Punong Barangay allegedly constitutes an implied pardon of
his previous misconduct.
ISSUE: Whether petitioner is disqualified from running for Punong Barangay on the
ground that the latter
was convicted by final judgment.
HELD: NO. It is regrettable that the Comelec and the OSG have misapprehended the
real issue in this case. They focused on the fact that Morenos judgment of conviction
attained finality upon his application for probation instead of the question of whether his
sentence had been served.
The Comelec could have correctly resolved this case by simply applying the law to the
letter. Sec. 40(a) of the Local Government Code unequivocally disqualifies only those
who have been sentenced by final judgment for an offense punishable by imprisonment
of one (1) year or more, within two (2) years after serving sentence.
This is as good a time as any to clarify that those who have not served their sentence
by reason of the grant of probation which, we reiterate, should not be equated with
service of sentence, should not likewise be disqualified from running for a local elective
office because the two (2)-year period of ineligibility under Sec. 40(a) of the Local
Government Code does not even begin to run.
The fact that the trial court already issued an order finally discharging Moreno fortifies
his position. Sec. 16 of the Probation Law provides that "[t]he final discharge of the
probationer shall operate to restore to him all civil rights lost or suspended as a result of
his conviction and to fully discharge his liability for any fine imposed as to the offense for
which probation was granted." Thus, when Moreno was finally discharged upon the
courts finding that he has fulfilled the terms and conditions of his probation, his case
was deemed terminated and all civil rights lost or suspended as a result of his
conviction were restored to him, including the right to run for public office.
Even assuming that there is an ambiguity in Sec. 40(a) of the Local Government Code
which gives room for judicial interpretation, our conclusion will remain the same.
It is unfortunate that the deliberations on the Local Government Code afford us no clue
as to the intended meaning of the phrase "service of sentence," i.e., whether the
290

legislature also meant to disqualify those who have been granted probation. The Courts
function, in the face of this seeming dissonance, is to interpret and harmonize the
Probation Law and the Local Government Code. Interpretare et concordare legis
legibus est optimus interpretandi.
On this score, we agree with Moreno that the Probation Law should be construed as an
exception to the Local Government Code. While the Local Government Code is a later
law which sets forth the qualifications and disqualifications of local elective officials, the
Probation Law is a special legislation which applies only to probationers. It is a canon of
statutory construction that a later statute, general in its terms and not expressly
repealing a prior special statute, will ordinarily not affect the special provisions of such
earlier statute.
G.R. No. 108747, April 6, 1995
PABLO C. FRANCISCO, petitioner, vs. COURT OF APPEALS AND THE
HONORABLE MAXIMO C. CONTRERAS, respondents.
EN BANC, BELLOSILLO, J.:

Facts: Petitioner was charged with multiple grave oral defamation in five (5) separate
Informations instituted by five (5) of his employees. On 2 January 1990, after nearly ten
(10) years, the Metropolitan Trial Court of Makati, Br. 61, found petitioner guilty of grave
oral defamation in four (4) of the five (5) cases filed against him, sentenced him to a
prison term of one (1) year and one (l) day to one (1) year and eight (8) months
of prision correccional "in each crime committed on each date of each case, as alleqed
in the information(s)," ordered him to indemnify each of the offended parties, Victoria
Gatchalian, Rowena Ruiz, Linda Marie Ayala Pigar and Marie Solis, P10,000.00 as
exemplary damages, and P5,000.00 for attorney's fees, plus costs of suit. He was
however acquitted in Crim. Case No. 105208 for persistent failure of the offended party,
Edgar Colindres, to appear and testify.

Not satisfied with the Decision of the MeTC, and insisting on his innocence, petitioner
elevated his case to the Regional Trial Court. The Regional Trial Court of Makati, Br. 59,
affirmed his conviction but appreciated in his favor a mitigating circumstance analogous
to passion or obfuscation. Accordingly, petitioner was sentenced "in each case to a
STRAIGHT penalty of EIGHT (8) MONTHS imprisonment".

291

After he failed to interpose an appeal therefrom the decision of the RTC became final.
The case was then set for execution of judgment by the MeTC which, as a
consequence, issued a warrant of arrest. But before he could be arrested petitioner filed
an application for probation which the MeTC denied "in the light of the ruling of the
Supreme Court in Llamado v. Court of Appeals. He went to the Court of Appeals
on certiorari which dismissed his petition. The motion for reconsideration was likewise
denied.

Issue: whether petitioneris still qualified to avail of probation even after appealing his
conviction to the RTC which affirmed the MeTC except with regard to the duration of the
penalties imposed.

Held: NO First. Probation is a mere privilege, not a right. Its benefits cannot extend to
those not expressly included. Probation is not a right of an accused, but rather an act of
grace and clemency or immunity conferred by the state which may be granted by the
court to a seemingly deserving defendant who thereby escapes the extreme rigors of
the penalty imposed by law for the offense of which he stands convicted. It is a special
prerogative granted by law to a person or group of persons not enjoyed by others or by
all. Accordingly, the grant of probation rests solely upon the discretion of the court which
is to be exercised primarily for the benefit of organized society, and only incidentally for
the benefit of the accused. The Probation Law should not therefore be permitted to
divest the state or its government of any of the latter's prerogatives, rights or remedies,
unless the intention of the legislature to this end is clearly expressed, and no person
should benefit from the terms of the law who is not clearly within them.

That an appeal should notbar the accused from applying for probation if the appeal is
taken solely to reduce the penalty is simply contrary to the clear and express mandate
of Sec, 4 of the Probation Law, as amended, which opens with a negativeclause, "no
application for probation shall be entertained or granted if the defendant has perfected
the appeal from the judgment of conviction." And where the law does not distinguish the
courts should not distinguish; where the law does not make exception the court should
not except.

292

Second. At the outset, the penalties imposed by the MeTC were already probationable.
Hence, there was no need to appeal if only to reduce the penalties to within the
probationable period. Multiple prison terms imposed against an accused found guilty of
several offenses in one decision are not, and should not be, added up. And, the sum of
the multiple prison terms imposed against an applicant should not be determinative of
his eligibility for, nay his disqualification from, probation. The number of offenses is
immaterial as long as all the penalties imposed, taken separately, are within the
probationable period. For, Sec. 9, par. (a), P.D. 968, as amended, uses the
word maximum not total when it says that "[t]he benefits of this Decree shall not be
extended to those . . . . sentenced to serve a maximum term of imprisonment of more
than six years."

Fixing the cut-off point at a maximum term of six (6) years imprisonment for probation is
based on the assumption that those sentenced to higher penalties pose too great a risk
to society, not just because of their demonstrated capability for serious wrong doing but
because of the gravity and serious consequences of the offense they might further
commit. The Probation Law, as amended, disqualifies only those who have been
convicted of grave felonies as defined in Art. 9 in relation to Art. 25 of The Revised
Penal Code, and not necessarily those who have been convicted of multiple offenses in
a single proceeding who are deemed to be less perverse.

Considering that the multiple prison terms should not be summed up but taken
separately as the totality of all the penalties is not the test, petitioner should have
immediately filed an application for probation as he was already qualified after being
convicted by the MeTC, if indeed thereafter he felt humbled, was ready to
unconditionally accept the verdict of the court and admit his liability. Consequently, in
appealing the Decision of the MeTC to the RTC, petitioner lost his right to probation.
For, plainly, the law considers appeal and probation mutually exclusive remedies.

Third. Petitioner appealed to the RTC not to reduce or even correct the penalties
imposed by the MeTC, but to assert his innocence. By perfecting his appeal,
petitioner ipso facto relinquished his alternative remedy of availing of the Probation Law
the purpose of which is simply to prevent speculation or opportunism on the part of an
accused who although already eligible does not at once apply for probation, but doing
so only after failing in his appeal.
293

Fourth. The application for probation was filed way beyond the period allowed by law.
The law, simply, does not allow probation after an appeal has been perfected.

294

S-ar putea să vă placă și